Exam 1 & 2 Microbiology

Ace your homework & exams now with Quizwiz!

All of the following substances are effective against nonenveloped viruses EXCEPT A) alcohol. B) chlorine. C) ethylene oxide. D) ozone. E) None of the answers is correct; all of these are equally effective against nonenveloped viruses.

A) alcohol.

Botulism is caused by ingestion of a proteinaceous exotoxin; therefore, it can easily be prevented by A) boiling food prior to consumption. B) administering antibiotics to patients. C) not eating canned food. D) preventing fecal contamination of food. E) filtering food.

A) boiling food prior to consumption.

In bacteria, photosynthetic pigments are found in A) chromatophores. B) ribosomes. C) cytoplasm. D) chloroplasts. E) mesosomes.

A) chromatophores.

Which of the following is found normally in serum? A) complement B) interferon C) histamine D) leukocytosis-promoting factor E) TLRs

A) complement

Haemophilus influenzae b capsular polysaccharide with a protein is a(n) A) conjugated vaccine. B) subunit vaccine. C) nucleic acid vaccine. D) inactivated whole-agent vaccine. E) toxoid vaccine.

A) conjugated vaccine.

Table 15.1: - S. aureus: wound: ID50 = <10 - S. aureus: wound + Ampicillin: ID50 = 300 Table 15.1 shows the ID50 for Staphylococcus aureus in wounds with and without the administration of ampicillin before surgery. Based on the data, the administration of ampicillin before surgery A) decreases the risk of staphylococcal infection. B) increases the risk of staphylococcal infection. C) has no effect on risk of infection. D) replaces tetracycline. E) The answer cannot be determined based on the information provided.

A) decreases the risk of staphylococcal infection.

Which of the following uses red blood cells as the indicator? A) Agglutination B) Complement fixation C) Precipitation D) Flow cytometry E) Neutralization

B) Complement fixation

In an immunodiffusion test to diagnose histoplasmosis, a patient's serum is placed in a well in an agar plate. In a positive test, a line forms as the serum diffuses from the well and meets material diffusing from a second well. What type of test is this? A) an agglutination reaction B) a precipitation reaction C) a complement-fixation test D) an indirect ELISA test E) a direct ELISA test

B) a precipitation reaction

For each of the following statements, determine whether it describes a benefit or a limitation of Koch's postulates.

Benefit: - Laboratory findings provide experimental evidence that support the germ theory of disease. - Healthy, susceptible animals can be used as model organisms for testing many types of infections, thus avoiding the unethical inoculation of healthy human hosts. - The steps are designed to systematically link a pathogen to a specific infectious disease. Limitation: - Different pathogens can produce the same signs and symptoms, making it difficult to determine which microorganism is causing a disease. - Some pathogens cause several different diseases, which makes it difficult to link one pathogen to one disease using Koch's postulates. - Some infectious agents have specific growth requirements that prevent them from being artificially cultured in the laboratory.

The biosafety level (BSL) for a clinical microbiology laboratory working with potentially airborne pathogens, such as tuberculosis bacteria, is A) BSL-1. B) BSL-2. C) BSL-3. D) BSL-4.

C) BSL-3.

Which of the following is a fungal zoonosis that can be transmitted by direct contact? A. tapeworm B. West Nile encephalitis C. ringworm D. Lyme disease

C. ringworm

Physical Requirement: pH: - Most bacteria grow between a pH of ________ to _______. - Molds and yeasts grow between _______ and _____. - _________ grow in acidic environments. - Most living cells have a very ________ range of pH. - pH is __________ in all biological organisms. - Each pH unit represents a factor of ______ difference in pH. - pH results in ions, which create ___________.

- 6.5 - 7.5 - 5 - 6 - acidophiles - narrow - controlled - 10 - concentration gradients

Monoclonal Antibodies: - _________: immortal cancerous B cell (myeloma) combined with an antibody-producing normal B cell. * Hybridoma produces _________.

- Hybridoma * monoclonal antibodies (Mabs)

Hematopoiesis: LOOK AT PICTURE

- Pluripotent Stem Cells: 1. Myeloid stem cell 2. Lymphoid stem cell 1. Myeloid stem cell: * RBC * platelets * mast cells * eosinophils * basophils * neutrophils * dendritic cells * monocytes --> macrophages 2. Lymphoid stem cell: * T cell * B cell ---> plasma cell * natural killer cells (NK cells)

- _______ infects E coli. - Jim Humble: came up with a miracle mineral solution, which was basically ________.

- T4 virus - industrial bleach (ClO2-)

Fluorescence Microscopy: - Uses ____ (_____ wavelength) light - Fluorescent substances absorb _______ and emit longer wavelength (visible) light. - Cells can be stained with fluorescent dyes (__________) if they do not naturally fluoresce. - Used a lot to identify _________ like _______.

- UV (short) - UV light - fluorochromes - microorganisms/ strep

The Specific Immune System: - _________: defenses that target a specific pathogen. * acquired through ______ or _____. * _________: first time the immune system combats a particular foreign substance. * _________: later interactions with the same foreign substance; faster and more effective due to memory.

- adaptive immunity * infection or vaccination * primary response * secondary response

Fever: - Abnormally _____ body temperature. - __________ is normal set at 37 degrees C - part of the brain that controls temperature. - Cytokines cause the hypothalamus to release _______ that reset the hypothalamus to a higher temperature, which slows down the growth of bacteria. - Body constricts the blood vessels, and ________ occurs (which raises temperature). - As body temperature rises (______), vasodilation and sweating occurs.

- high - hypothalamus = prostaglandins - shivering - (crisis)

Development of Disease: - ____________; interval between initial infection and first signs and symptoms. - ___________; short period after incubation; early, mild symptoms. - ___________; disease is most severe. - ___________; signs and symptoms subside. - ___________; body returns to its prediseased state

- incubation period - prodromal period - period of illness - period of decline - period of convalescence

Select strategies that are likely to interfere with viral infection and replication but will NOT damage the normal functioning of host cells. Select all that apply.

- inhibiting the release of viruses from the host cell - inhibiting viral assembly and maturation processes - blocking attachment sites on the virus surface - blocking host cell receptors that serve as sites for viruses to attach - inhibiting viral-encoded DNA polymerase

Dual Nature of the Adaptive Immune System: - It is a coordinated attack! - Cellular immunity attacks antigens found ______ cells. * examples: _______; some _______ and _______. - Humoral immunity fights invaders _______ cells. * examples: _______ and ________.

- inside * viruses/ fungi and parasites - outside * bacteria and toxins

Phase-Contrast Microscopy: - Allows examination of ________ and ________. - Brings together two sets of light rays; ______ and ______; to form an image. - ________ = microorganism can move around.

- living organisms and internal cell structures - direct rays and diffracted rays - hanging drop sample

The Rate of Microbial Death: - Effectiveness of treatments depends on what 4 things? - Bacteria die in an ___________ way (what percent dies with one log decrease?)

- number of microbes, environment (organic matter, temperature, biofilms), time of exposure, and microbial characteristics - exponential (90%)

- Prokaryote comes from the Greek words for ________. - Eukaryote comes from the Greek words for ________.

- prenucleus - true nucleus

Which of the following elements is NOT correctly matched with its cellular function? A) nitrogen — needed for amino acid synthesis B) phosphorus — incorporated into nucleic acids C) sulfur — used for synthesis of thiamin and biotin D) magnesium and potassium — required as cofactors for enzymes E) phosphorus — used for production of carbohydrates.

E) phosphorus — used for production of carbohydrates.

Which of the following cytopathic effects is cytocidal? A) inclusion bodies B) giant cells C) antigenic changes D) transformation E) release of enzymes from lysosomes

E) release of enzymes from lysosomes

In Figure 17.2, which areas are similar for all IgG antibodies? A) a and b B) a and c C) b and c D) c and d E) b and d

D) c and d

Fungal infections are studied by A) virologists. B) bacteriologists. C) parasitologists. D) mycologists. E) herpetologists.

D) mycologists.

In microscopy, the term resolution A) is only observed in stained specimens. B) is improved when longer wavelengths of light are employed. C) refers to magnification when using the electron microscope. D) refers to the ability to distinguish fine structure and detail in a specimen. E) is the same as the total magnification of a specimen.

D) refers to the ability to distinguish fine structure and detail in a specimen.

Which enzyme catalyzes the reaction: O2- + O2- + 2H+ → H2O2 + O2? A) catalase B) oxidase C) peroxidase D) superoxide dismutase

D) superoxide dismutase

Which one of the following steps does NOT occur during multiplication of a picornavirus? A) synthesis of + strands of RNA B) synthesis of - strands of RNA C) synthesis of viral proteins D) synthesis of DNA E) None of the answers is correct.

D) synthesis of DNA

Which of the following is considered a major category of transmission of disease? A. Vector transmission B. Contact transmission C. Vehicle transmission D. Contact, vehicle, and vector transmission

D. Contact, vehicle, and vector transmission

The antibodies that can bind to large parasites are A) IgG. B) IgM. C) IgA. D) IgD. E) IgE.

E) IgE.

Which of the following is NOT utilized to culture viruses? a. animal cell cultures b. culture media c. bacterial cultures d. embryonated eggs e. laboratory animals

b. culture media

In which of the following situations would Koch's postulates be utilized? a. whenever the scientific method is used to investigate a microbiological problem b. determination of the cause of a new emerging disease by scientists studying disease transmission c. development of a new antibiotic in a pharmaceutical lab d. formulation of a vaccine against a new pathogen in a genetic engineering lab e. determination of the cause of cancer in a patient

b. determination of the cause of a new emerging disease by scientists studying disease transmission

Viruses possess genetic material comprised of DNA or __________. a. plasmids b. ATP c. RNA d. NADPH e. guanine

c. RNA

Which of the following T cell is a component of both the cellular and humoral immune response? a. Natural killer cells b. T regulatory cell c. T helper cells d. Cytotoxic T cells

c. T helper cells

Which terms regarding components of adaptive immunity are mismatched? a. activated macrophage - membrane ruffling. b. M cell - microfolds. c. TH cells - MHCI interaction. d. Dendritic cells- Langerhans cell. e. TH-17 cells - recruit neutrophils.

c. TH cells - MHCI interaction.

Consider a helminthic infection in which an individual is colonized by a parasitic worm. The worm is too big to be engulfed by a phagocytic cell. How does the immune system respond? a. Blood flow is reduced to the area, which starves the worm of nutrients. b. Antibodies bind to the worm and disrupt its plasma membrane. c. The worm gets coated with antibodies, which activate other cells in the immune system to secrete chemicals that kill it. d. Numerous phagocytes work cooperatively to ingest the worm.

c. The worm gets coated with antibodies, which activate other cells in the immune system to secrete chemicals that kill it.

Which of the following is NOT a structure of prokaryotic cells? a. flagellum b. capsule c. chloroplast d. chromosome e. ribosome

c. chloroplast

Infections in which the pathogen is distributed throughout the body are referred to as generalized infections or __________. a. emerging infections b. zoonoses c. systemic infections d. focal infections e. local infections

c. systemic infections

CHAPTER 14: Principles of Disease and Epidemiology

STARTS HERE

Contaminated hospital equipment represents a difficult environment to control harmful microbial growth. Why is this?

The cell wall of gram-negative bacteria is especially resistant to many chemicals that are used destroy microbes.

What is the name given to the viral DNA incorporated into a lysogenic cell? a. Prophage b. Oncogenic virus c. Latent phage d. Bacteriophage

a prophage

An exotoxin that has the ability to kill or damage host cells is referred to as a(n) a) cytotoxin. b) superantigen. c) enterotoxin. d) neurotoxin.

a) cytotoxin.

Which complement protein directly forms the membrane attack complex (MAC)? a. C9 b. C5 c. C3 d. C1

a. C9

Which type of leukocyte is the most abundant in blood? a. Neutrophils b. Eosinophils c. Monocytes d. Lymphocytes

a. Neutrophils

Which of these substances does NOT protect a bacterium from phagocytosis? a. Siderophore b. Capsule c. Leukocidin d. M protein

a. Siderophore

An infectious protein is a a. prion. b. viroid. c. bacteriophage. d. papovavirus. e. retrovirus.

a. prion.

Figure 6.1, which line best depicts a facultative anaerobe in the absence of O2? a b c

b

A human host can prevent a pathogen from getting enough iron by: a. Reducing dietary intake if iron. b. Binding iron with transferrin. c. Binding iron with hemoglobin. d. Excreting excess iron. e. Binding iron with siderophores.

b. Binding iron with transferrin.

Which of the following is the least abundant Ig? a. IgM b. IgE c. IgG d. IgD

b. IgE

Which answer is true of the anamnestic response? a. IgM predominates. b. IgG predominates. c. IgG and IgM are present in equal proportions. d. IgM is not produced. e. IgG is not produced.

b. IgG predominates.

The most abundant class of antibodies in serum is a. IgE. b. IgG. c. IgM. d. IgD. e. IgA.

b. IgG.

The presence of which of the following indicates a current infection rather than a previous infection or vaccination? a. IgD b. IgM c. IgA d. IgE e. IgG

b. IgM

Which of the following is not a characteristic of quaternary ammonium compounds? a. Bactericidal against gram positive bacteria b. Sporicidal c. Amebicidal d. Fungicidal e. Kills enveloped viruses.

b. Sporicidal

Which of the following are NOT major targets for action of antimicrobials? a. Proteins b. Enzymes c. Nucleic acids d. Flagella e. Cell membranes

d. Flagella

Which of these disinfectants does not act by disrupting the plasma membrane? a. Phenolics b. Phenol c. Quats d. Halogens e. Biguanides

d. Halogens

The protection from disease provided by vaccination is called __________. a. asepsis b. antibiotic therapy c. gene therapy d. immunity

d. immunity

Which of the following are possible strategies for treating viral infections? a. Blocking viral attachment to host cell receptors b. Blocking uncoating of the virus after entry c. Blocking insertion of viral DNA into the host cell chromosomes d. Blocking biosynthesis of viral nucleic acids e. All of the listed strategies are correct.

e. All of the listed strategies are correct.

Which of the following is NOT a predisposing factor of disease? a. genetic background b. lifestyle c. occupation d. climate e. All of these are predisposing factors of disease.

e. All of these are predisposing factors of disease.

Transient microbiota differ from normal microbiota in that transient microbiota a. are always acquired by direct contact. b. cause diseases. c. are found in a certain location on the host. d. never cause disease. e. are present for a relatively short time.

e. are present for a relatively short time.

In a healthy human, resident microorganisms would be found in all of the following areas EXCEPT the __________. a. nasal passages b. lower urethra c. tooth and gum surfaces d. conjunctiva e. bloodstream

e. bloodstream

The major food producers for other living organisms is/are ______ A) higher plants. B) cyanobacteria. C) algae. D) higher plants and algae. E) higher plants, cyanobacteria, and algae.

e. higher plants, cyanobacteria, and algae.

___________ is the study of protozoa. a. bacteriology b. mycology c. immunology d. virology e. parasitology

e. parasitology

Development of emerging infectious disease can be a result of all of the following EXCEPT ______ A) modern transportation. B) changes in the environment. C) microbial mutation. D) overuse of antibiotics. E) use of genetically modified foods.

e. use of genetically modified foods

A positive complement-fixation test is indicated by the lysis of the sheep red blood cells added in the indicator phase of the test. true or false

false

Haptens can bind to antibody molecules only if the haptens are attached to a carrier molecule. true or false

false

In performing a ten-fold dilutions series from a sample containing 10,000 bacteria per milliliter, the fourth tube in the dilution series will have 10 cells per milliliter. true or false

false

Interferons can cure most viral infections, including the common cold. true or false

false

Viruses can be grown ONLY in living animal or plant hosts. true or false

false

When haptens attach to carrier molecules, an epitope forms on hapten which then can be bound to antibody. true or false

false

In a _________, cells are added to melted agar and poured into a Petri dish.

pour plate

Structurally, bacteriophages are complex viruses. true or false

true

The basic mechanism of viral multiplication is similar for all viruses. true or false

true

Cytokines are protein-based chemical messengers that allow for communication between cells of the immune system. true or false

true

Ingestion of certain lactic acid bacteria (LABs) has been shown to be beneficial for function and health of the intestinal tract. true or false

true

A double-stranded, enveloped DNA virus that contains reverse transcriptase belongs to which family? a. Picornaviridae b. Retroviridae c. Rhabdoviridae d. Hepadnaviridae e. Poxviridae

d. Hepadnaviridae

The Biosynthesis of RNA Viruses: - Virus multiplies in the host cell's cytoplasm using ____________ (unique to the virus/ virus brings it with them because its part of their genome). - ssRNA: * _________: viral RNA serves as mRNA for protein synthesis. * _________: viral RNA is transcribed to a + strand to serve as mRNA for protein synthesis. - ________: double stranded RNA - _______: function is to add nucleotides

- RNA-dependent RNA polymerase * ssRNA + (sense) strand * ssRNA - (antisense) strand - dsRNA - polymerase

Chemical Sterilization: - Gaseous sterilants cause __________ - replacing hydrogen atoms of a chemical group with a _________. - Cross links ________ and _________. - Used for _________ material. * example?

- alkylation/ free radical - nucleic acid and proteins - heat-sensitive * ethylene oxide

T Cytotoxic Cells (CD8+ T cells): - ________: programmed cell death. * prevents the spread of infectious viruses into other cells. * cells cut their genome into fragments, causing the membranes to bulge outward via _______.

- apoptosis * blebbing

- What structures do all bacteria have? - What structures do only some bacteria have?

- cell wall, plasma membrane, cytoplasm, 70s ribosomes, nucleoid containing DNA - capsule, flagella, inclusions, plasmid, fimbriae

What microscope allows for the highest resolution and magnification?

- electron microscope

Flagella Staining: - Flagella are structures of _________. - Uses a _______ and ________.

- locomotion - mordant and carbolfuchsin

Atomic Force Microscopy: - Uses a _______ placed onto a specimen. - Produces ______ images

- metal-and-diamond probe - 3D

- _______: is the study of viruses. - _________ in 1892 and ________ in 1935 discovered the cause of mosaic disease of tobacco as a virus. - _______ microscopes have made it possible to study the structure of viruses in detail.

- virology - Dmitri Iwanowski and Wendall Stanley - electron

The Process of Inflammation: A. Tissue Damage B. Vascular Reactions and Phagocytosis: 1. chemicals such as histamines, kinins, prostaglandins, and cytokines are released by damaged cells. 2. blood clot forms 3. abscess starts to form. 4. ________: phagocytes stick to endothelium. 5. _______: phagocytes squeeze between endothelial cells. 6. phagocytosis of invading bacteria occurs.

4. Margination 5. Diapedesis

The use of phenol (carbolic acid) as a wound disinfectant was first practiced by A) Lister. B) Semmelweis. C) Pasteur. D) Holmes. E) Koch.

A) Lister.

A nosocomial infection is A) acquired during the course of hospitalization. B) always caused by medical personnel. C) only a result of surgery. D) always present, but is inapparent at the time of hospitalization. E) always caused by pathogenic bacteria.

A) acquired during the course of hospitalization.

Thymic selection A) destroys T cells that do not recognize self-molecules of MHC. B) destroys B cells that make antibodies against self. C) destroys MHC molecules. D) destroys CD4+cells that attack self. E) activates B cells.

A) destroys T cells that do not recognize self-molecules of MHC.

Which of the following is a test to determine a patient's blood type by mixing the patient's red blood cells with antisera? A) direct agglutination reaction B) passive agglutination reaction C) immunofluorescence D) neutralization reaction E) precipitation reaction

A) direct agglutination reaction

Some viruses, such as human herpesvirus 1, infect a cell without causing symptoms. These are called A) latent viruses. B) lytic viruses. C) phages. D) slow viruses. E) unconventional viruses.

A) latent viruses.

The major significance of Robert Koch'ʹs work is that A) microorganisms cause disease. B ) diseases can be transmitted from one animal to another. C) microorganisms can be cultured. D) microorganisms are the result of disease. E) microorganisms are present in a diseased animal.

A) microorganisms cause disease.

Pseudomonas bacteria colonized the bile duct of a patient following his liver transplant surgery. This is an example of a A) nosocomial infection. B) sporadic disease. C) latent infection. D) communicable disease. E) None of the answers is correct.

A) nosocomial infection.

Which of the following is NOT associated with viruses? A) organelles B) nucleic acid C) envelope D) spikes E) capsid

A) organelles

The definition of lysogeny is A) phage DNA is incorporated into host cell DNA. B) lysis of the host cell due to a phage. C) the period during replication when virions are not present. D) when the burst time takes an unusually long time. E) attachment of a phage to a cell.

A) phage DNA is incorporated into host cell DNA.

Which microscope takes advantage of differences in the refractive indexes of cell structures? A) phase-contrast microscope B) compound light microscope C) darkfield microscope D) electron microscope E) fluorescence microscope

A) phase-contrast microscope

A culture medium consisting of agar, peptone, and beef heart is a A) chemically defined medium. B) complex medium. C) selective medium. D) differential medium. E) reducing medium.

B) complex medium.

A disease that is constantly present in a population is called a(n): A) notifiable disease. B) endemic disease. C) communicable disease. D) epidemic disease.

B) endemic disease.

When aerosols containing pathogens spread disease from a distance of less than one meter, it is considered A. waterborne transmission. B. contact transmission. C. airborne transmission. D. vector transmission.

B. contact transmission.

CHAPTER 3

BEGINS HERE

Which of the following is NOT an advantage of live attenuated vaccine agents? A) They elicit lifelong immunity. B) They stimulate by cell-mediated and humoral immune responses. C) They occasionally revert to virulent forms. D) They require few or no booster immunizations. E) The immune response generated by the vaccine closely mimics a real infection.

C) They occasionally revert to virulent forms.

Each of the following provides protection from phagocytic digestion EXCEPT A) M protein. B) capsules. C) formation of phagolysosomes. D) leukocidins. E) biofilms.

C) formation of phagolysosomes.

If a prodromal period exists for a certain disease, it should occur prior to A) convalescence. B) incubation. C) illness. D) decline.

C) illness.

What type of vaccine involves host synthesis of viral antigens? A) conjugated vaccine B) subunit vaccine C) nucleic acid vaccine D) attenuated whole-agent vaccine E) toxoid vaccine

C) nucleic acid vaccine

Which of the following would be considered a fomite? A. A fly B. Contaminated water C. An infected toy D. A tick

C. An infected toy

An experiment began with 4 cells and ended with 128 cells. How many generations did the cells go through? A) 64 B) 32 C) 6 D) 5 E) 4

D) 5

The structures illustrated in Figure 13.1 are composed of A) DNA. B) RNA. C) DNA or RNA. D) Capsomeres. E) viroids.

D) Capsomeres.

What type of infectious agent causes potato spindle tuber disease? a. Viroid b. Prion c. Virus d. Virino

a. Viroid

In the Gram-stain procedure, a clear oval in the center of a cell could indicate __________. a. the presence of an endospore b. a capsule c. an unstained nucleus d. lack of the central nucleoid e. the unstained bacterial chromosome

a. the presence of an endospore

Each antibody has __________ antigen binding sites. a. two b. zero c. one d. four

a. two

Which of the following is NOT a chemical factor that helps the skin to be relatively resistant to infection? a. Lysozyme b. Complement proteins c. Acidic pH d. Sebum

b. Complement proteins

The antibodies found almost entirely and only on the surface of B cells (not secreted from them), and which always exist as monomers, are a. IgE. b. IgD. c. IgM. d. IgG. e. IgA.

b. IgD.

Which of the following would be the most effective method in preserving bacterial cultures? a. Deep-freezing b. Lyophilization c. Refrigeration d. Autoclaving

b. Lyophilization

Which of these enzymes is necessary for the replication of a + strand RNA virus? a. RNA-dependent DNA polymerase b. RNA-dependent RNA polymerase c. DNA-dependent RNA polymerase d. DNA-dependent DNA polymerase e. Reverse transcriptase

b. RNA-dependent RNA polymerase

Which of the following is NOT a sign or symptom of a Clostridium difficile infection? a. Abdominal pain b. Respiratory distress c. Fever d. Bloody stools

b. Respiratory distress

__________ has little value as an antiseptic but is important in the mechanical removal of microbes. a. Alcohol b. Soap c. Iodine d. Silver

b. Soap

For pathogen below, choose the type of cell that would be used in the adaptive immune response. * regarding to picture! a. B cells b. TH1 cells c. TH2 cells d. TH17 cells

b. TH1 cells

Which of these is NOT a characteristic of the autoclave? a. The use of high temperatures b. The requirement for long (hours) exposure times c. The use of high pressures d. The ability to sterilize solutions with endospores e. The use of moist heat

b. The requirement for long (hours) exposure times

You have isolated a cell with a peptidoglycan cell wall. What other structure can you safely assume the cell has? a. a nucleus b. a plasma membrane c. a mitochondrion d. a chloroplast

b. a plasma membrane

The __________ is a fluid structure that allows membrane proteins to move freely. a. glycocalyx b. cell membrane c. ribosome d. cell wall

b. cell membrane

In the human intestinal tract, E. coli produces vitamins beneficial to the host and can inhibit pathogen growth. In turn, the bacterium is supplied with nutrients and an environment for growth. This symbiotic relationship between E. coli and its host is an example of __________. a. commensalism b. mutualism c. parasitism d. antagonism e. opportunism

b. mutualism

The resistance to reinfection with measles virus following recovery from measles infection is called __________. a. natural selection b. naturally acquired active immunity c. naturally acquired passive immunity d. artificially acquired active immunity e. innate immunity

b. naturally acquired active immunity

During the __________, a person recovers from a disease and the body returns to its pre-disease state. a. period of incubation b. period of convalescence c. period of illness d. period of decline

b. period of convalescence

An envelope is acquired during which of the following steps? a. biosynthesis b. release c. adsorption d. penetration e. uncoating

b. release

Gram-negative cells contain a periplasmic space that is __________. a. the site of protein synthesis b. rich in degradative enzymes c. abundant in teichoic acid d. a site of endocytosis e. filled with lysozyme

b. rich in degradative enzymes

A student is observing an object that displays the internal detail on a virus at great magnification. Which type of microscope is used for this image? a. scanning electron microscope b. transmission electron microscope c. light microscope d. scanning acoustic microscope

b. transmission electron microscope

In Figure 6.1, which line shows the growth of an obligate aerobe incubated anaerobically? a b c

c

In Figure 6.1,which line best depicts an obligate anaerobe in the presence of O2? a b c

c

The molecule serving as mRNA can be incorporated in the newly synthesized virus capsids of all of the following except a. + strand RNA picornaviruses. b. + strand RNA togaviruses. c. - strand RNA rhabdoviruses. d. double-stranded RNA reoviruses. e. Rotavirus.

c. - strand RNA rhabdoviruses.

A nurse is degerming a patient's arm before giving a vaccination. Which of these concentrations of alcohol is most effective? a. 50% b. 100% c. 70% d. 25%

c. 70%

Which of the following fields of study and its example are NOT correctly matched? a. Mycology—study of athlete's foot b. Immunology—study of vaccines c. Parasitology—study of HIV d. Bacteriology—study of E. coli O157:H7

c. Parasitology—study of HIV

The following steps occur during bacteriophage replication. What is the second step? a. Attachment b. Lysis c. Penetration d. Biosynthesis

c. Penetration

For which of the following diseases can you receive a vaccination without the requirement for another booster? a. Tetanus b. Rabies c. Pneumococcal pneumonia d. Influenza

c. Pneumococcal pneumonia

The LD50 of Vibrio cholerae is 108 cells through the oral route. If the bacterial cells are ingested with bicarbonate, the LD50 drops to 104. Which of these explanations is the most likely? a. Sodium bicarbonate inactivates Vibrio cholerae. b. Vibrio cholerae makes toxins only in the presence of stomach acid. c. Stomach acid decreases the virulence of Vibrio cholerae. d. Stomach acid increases the virulence of Vibrio cholerae. e. Sodium bicarbonate decreases the virulence of Vibrio cholerae.

c. Stomach acid decreases the virulence of Vibrio cholerae.

For pathogen below, choose the type of cell that would be used in the adaptive immune response. * regarding to picture! a. B cells b. TH1 cells c. TH17 cells d. TH2 cells

c. TH17 cells

How would you know that viruses were multiplying in a confluent lawn of E. coli on a solid culture medium? a. The bacterial culture would grow faster. b. The bacterial colonies would swell. c. There would be small zones of clearing in the bacterial culture. d. There would be small blue spots on the bacterial culture.

c. There would be small zones of clearing in the bacterial culture.

In the capsule stain using India ink, capsules are distinguished as __________ surrounding cells. a. pink circles b. green shadows c. clear halos d. purple rims e. blue extensions

c. clear halos

When an antibody binds to a toxin, the resulting action is referred to as a. opsonization. b. ADCC. c. neutralization. d. agglutination. e. apoptosis.

c. neutralization.

Disinfection of water is achieved by all of the following EXCEPT a. chlorine. b. ozone. c. peracetic acid. d. copper sulfate. e. UV radiation.

c. peracetic acid.

The toxin production by Corynebacterium diphtheriae carrying a temperate phage is an example of __________. a. specialized transduction b. phage lysis c. phage conversion d. general transduction

c. phage conversion

Which of the following are possible locations for bacterial adhesins? a. Fimbriae b. Flagella c. Pili d. All of the listed choices are possible locations for bacterial adhesins.

d. All of the listed choices are possible locations for bacterial adhesins.

You are about to start a multiyear experiment in a laboratory that works with poliovirus. Although the chance of exposure is low, you are still required to get immunized. What type of immunity now protects you? a. Naturally acquired active immunity b. Naturally acquired passive immunity c. Artificially acquired passive immunity d. Artificially acquired active immunity

d. Artificially acquired active immunity

Which of these cells do NOT have a role in cell-mediated immunity? a. T cytotoxic cells b. T helper cells c. Antigen-presenting cells d. Erythrocytes

d. Erythrocytes

Histamine causes all of the following reactions. Which occurs first? a. Redness b. Pain c. Swelling d. Vasodilation

d. Vasodilation

Which of the following is the correct order of events after tissue damage during the process of inflammation? a. Vasodilation; diapedesis; margination; phagocytosis b. Vasodilation; margination; phagocytosis; diapedesis c. Margination; vasodilation; diapedesis; phagocytosis d. Vasodilation; margination; diapedesis; phagocytosis

d. Vasodilation; margination; diapedesis; phagocytosis

All of the following are eukaryotes EXCEPT __________. a. algae b. helminths c. fungi d. bacteria e. protozoa

d. bacteria

Siderophores are bacterial proteins that compete with the host's a. antibodies. b. red blood cells. c. receptors. d. iron-transport proteins. e. white blood cells.

d. iron-transport proteins.

The secondary (anamnestic) immune response is due to __________. a. long-lived T helper cells b. long-lived antibody-producing plasma cells c. persistent high levels of antibody after the initial exposure to antigen d. long-lived memory cells

d. long-lived memory cells

Macrophages arise from which of the following? a. eosinophils b. lymphocytes c. neutrophils d. monocytes e. basophils

d. monocytes

What metric unit would be most appropriate for expressing the size of most viruses? a. millimeter b. micrometer c. centimeter d. nanometer

d. nanometer

Immunity acquired by transplacental transfer is called __________. a. artificially acquired active immunity b. naturally acquired active immunity c. artificially acquired passive immunity d. naturally acquired passive immunity

d. naturally acquired passive immunity

Oxidizing agents, such as hydrogen peroxide, are useful for irrigating deep wounds, in which released oxygen will inhibit the growth of __________. a. obligate aerobes b. microaerophiles c. facultative anaerobes d. obligate anaerobes

d. obligate anaerobes

T cytotoxic cells __________. a. engulf foreign cells b. respond to viruses free in circulation but not to those in host cells c. recognize antigen presented by class II MHC molecules d. produce perforin e. are distinguished by the CD4 markers on their surface

d. produce perforin

Most RNA viruses carry which of the following enzymes? a. lysozyme b. reverse transcriptase c. ATP synthase d. DNA-dependent DNA polymerase e. RNA-dependent RNA polymerase

e. RNA-dependent RNA polymerase

__________ is a compound found in antimicrobial soaps that targets gram-positive bacteria. a. Sodium nitrate b. Mercuric chloride c. Formaldehyde d. Hydrogen peroxide e. Triclosan

e. Triclosan

All of the following are prion diseases EXCEPT __________. a. kuru b. Gerstmann-Straussler-Scheinker syndrome c. fatal familial insomnia d. Creutzfeldt-Jakob disease e. Wiles-Davidoff syndrome

e. Wiles-Davidoff syndrome

Which of the following is a beneficial activity of microorganisms? a. some microorganisms are used as food for humans b. some microorganisms use carbon dioxide c. some microorganisms provide nitrogen for plant growth d. some microorganisms are used in sewage treatment processes e. all the above

e. all the above

Antibiotics can lead to septic shock if used to treat a. viral infections. b. gram-positive bacterial infections. c. protozoan infections. d. helminth infestations. e. gram-negative bacterial infections.

e. gram-negative bacterial infections.

The respiratory system is protected against harmful microbes by all of the following EXCEPT __________. a. the ciliary escalator b. the epiglottis c. ciliated cells d. mucus-coated hairs e. the lacrimal apparatus

e. the lacrimal apparatus

Apoptosis results in significant leakage of cellular contents. true or false

false

Most host damage resulting from bacterial infection is the result of direct damage by the microbe. true or false

false

Most viruses are highly resistant to disinfectants and antiseptics. true or false

false

Soluble antigens CANNOT be used in agglutination reactions. true or false

false

A disease that has increased in incidence or could increase in incidence in the near future is called an emerging infectious disease: - true and false

true

Histamine and kinins cause increased blood flow and capillary permeability. true or false

true

A test used to detect anti-Rickettsiaantibodies in a patient's serum is the A) direct fluorescent-antibody test. B) indirect fluorescent-antibody test.

B) indirect fluorescent-antibody test.

Purified protein from Bordetella pertussis is used in a(n) A) conjugated vaccine. B) subunit vaccine. C) nucleic acid vaccine. D) attenuated whole-agent vaccine. E) toxoid vaccine.

B) subunit vaccine.

Which of the following is an organic growth factor? A) glucose B) vitamin B1 C) peptone D) Mg+2 E) H2O

B) vitamin B1

In what year did Stanley Prusiner discover prions? A. 1928 B. 1982 C. 1997 D. 1979

B. 1982

A differential cell count is used to determine each of the following EXCEPT A) the total number of white blood cells. B) the numbers of each type of white blood cell. C) the number of red blood cells. D) leukocytosis. E) leukopenia.

C) the number of red blood cells.

5) Which of the following pairs is mismatched? A) alcohol-acetone — decolorizer B) crystal violet — basic dye C) carbolfuchsin — basic dye D) safranin — acid dye E) iodine — mordant

D) safranin — acid dye

Which of the following is NOT useful for observing living cells? A) darkfield microscope B) brightfield microscope C) scanning acoustic microscope D) scanning electron microscope E) phase-contrast microscope

D) scanning electron microscope

All of the following are iron-binding proteins found in humans EXCEPT A) lactoferrin. B) transferrin. C) hemoglobin. D) siderophorin. E) ferritin

D) siderophorin.

Cholera toxin polypeptide A binds to surface gangliosides on target cells. If the gangliosides were removed, A) polypeptide A would bind to target cells. B) polypeptide A would enter the cells. C) polypeptide B would not be able to enter the cells. D) Vibrio would not produce cholera toxin. E) Vibrio would bind to target cells.

C) polypeptide B would not be able to enter the cells.

Neutrophils with defective lysosomes are unable to A) undergo chemotaxis. B) migrate. C) produce toxic oxygen products. D) attach to microorganisms and other foreign material. E) engulf microorganisms and other foreign material.

C) produce toxic oxygen products.

Why is a mordant used in the Gram stain? In the flagella stain?

In a Gram stain, the mordant combines with the basic dye to form a complex that will not wash out of gram-positive cells. In a flagella stain, the mordant accumulates on the flagella so that they can be seen with a light microscope.

Composition and Characteristics: - Peptidoglycan: * polymer of a repeating ______ in rows: _____ and ______. * rows are linked by _______.

* disaccharide/ NAG and NAM * polypeptides

Based on the figure, match each component in the experiment with its purpose in this experiment.

- Diseased or dead animal: original source of the infectious microbial agent. - Healthy laboratory animal: organism injected with a potential infectious agent. - Microscope: tool used to view and identify individual microorganisms. - Growth media: nutrient-rich environment for isolating and culturing microorganisms

- Which graph in Figure 7.2 best depicts the effect of placing the culture in an autoclave for 15 minutes at time x? a b c d e - Which graph in Figure 7.2 best depicts the effect of placing the culture at 7°C at time x? a b c d e

- b - d

Compromised hosts are always suffering from suppressed immune systems. true or false

false

Which of the following statements concerning cellular immunity is FALSE?

Cellular immunity involves cells that recognize antigens and make specific antibodies against them.

Complement proteins in their intact and unactivated form act as opsonins by binding to microorganisms and promoting phagocytosis. true or false

false

What types of microbes would a microbiologist use culture techniques for living host cells?

obligate intracellular organisms, like chlamydia or viruses.

Antigenic stimulation of a particular B cell that results in the production of a large number of plasma and memory cells, all capable of responding to that antigen, is referred to as __________. a. clonal selection b. class switching c. affinity

a. clonal selection

The fluorescent-antibody test, in which labeled antibodies are added to a specimen on a microscope slide, is called the __________ FA test. a. direct b. indirect c. complement d. agglutination

a. direct

Which of the following exhibits the highest phagocytic activity? a. macrophages b. erythrocytes c. eosinophils d. neutrophils e. basophils

a. macrophages

The symptoms of protozoan diseases are usually due to __________. a. metabolic waste products b. capsules c. endotoxins d. exotoxins

a. metabolic waste products

Instead of oxygen, __________ may utilize nitrate or sulfate as their final electron acceptors. a. obligate aerobes b. halophiles c. facultative anaerobes d. microaerophiles e. aerotolerant anaerobes f. obligate anaerobes

f. obligate anaerobes

Bacteria that can grow in the presence or absence of Oxygen are called:

facultative anaerobes

Antibodies must be labeled with a radioactive, enzyme, or fluorescent tag to be detected. true or false

false

Glycoprotein spikes are found on the capsids of all viruses. true or false

false

MMWR is a publication by the CDC that reports on only emerging diseases. true of false

false

Macrophages and dendritic cells are the only cells that can present antigen to T cells. true or false

false

Microaerophiles do NOT require oxygen for growth. true or false

false

Most symptoms of endotoxins can be treated with administration of anti-endotoxin antibodies. true or false

false

The same disease symptoms result, regardless of how a particular microbe enters the body. true or false

false

Ultraviolet light (UV) causes irreversible breaks in DNA strands. true or false

false

What is the correct list, in increasing order, of the resistance of microorganisms to chemical biocides?

gram-positive bacteria, fungi, endospores, prions

Both phase-contrast microscopy and differential interference contrast microscopy are used to view the internal structures of cells without staining. true of false

true

Complement component C3b acts to increase adherence of phagocytes to microbes. true or false

true

Cytokine storms negatively impact human health. true or false

true

Nitrogen-fixing bacteria, such as cyanobacteria, can use atmospheric nitrogen (N2) for their nitrogen source. true or false

true

Scanned probe microscopy is used to examine fine detail of molecular complexes, such as blood clots, or molecules, such as DNA. true or false

true

The Limulus amoebocyte assay is used to detect minute amounts of endotoxin in drugs and medical devices. true or false

true

The plane in which a bacterial cell divides determines the arrangement of cells. true or false

true

The protection from infection received when individuals susceptible to a particular disease live in a population where many individuals are immune is referred to as herd immunity. true or false

true

For control of microbial growth on a surface, which of these conditions is LEAST likely to interfere with the effectiveness of an antiseptic or disinfectant? a. High numbers of microbes b. Presence of biofilms c. Warm environmental conditions d. Presence of organic matter

c. Warm environmental conditions

Campylobacter bacteria are grown with a CampyPak that produces 5% carbon dioxide and 15% oxygen. This bacterium is __________. a. an anaerobe b. a capnophile c. a microaerophile d. an aerotolerant anaerobe

c. a microaerophile

The protein coat of a virus is called the ___________. a. viral membrane b. envelope c. capsid d. capsomere

c. capsid

Sterilization is the __________. a. elimination of all gram-negative bacteria b. elimination of vegetative bacterial cells and fungal spores c. destruction of all microbial life d. destruction of all forms of microbial life except endospores e. reduction of microbial growth

c. destruction of all microbial life

A disease that is constantly present in a population is called a(n) __________. a. epidemic disease b. communicable disease c. endemic disease d. reportable disease

c. endemic disease

Some organisms are capable of orchestrating alterations that are collectively termed antigenic variation. This allows an organism to __________. a. drastically decrease its ID50 b. rapidly move through the host's tissues c. evade the host's immune system d. enter the host's body through more than one portal of entry

c. evade the host's immune system

Which of the following processes uses membrane proteins that act as channels or carriers allowing ions or large molecules to move across the plasma membrane without using energy? a. active transport b. osmosis c. facilitated diffusion d. simple diffusion

c. facilitated diffusion

A cold transmitted by a facial tissue is an example of a. vehicle transmission. b. droplet transmission. c. fomite. d. direct contact. e. vector.

c. fomite.

Which of the following does not kill endospores? a. Autoclaving b. Incineration c. Hot-air sterilization d. Pasteurization e. All of the above kill endospores.

d. Pasteurization

Which of the following diseases CANNOT be prevented by toxoids? a. diphtheria b. tetanus c. botulism d. gram-negative septic shock

d. gram-negative septic shock

Which of the following is the third stage of a disease? a. Incubation period b. Period of decline c. Period of convalescence d. Period of illness e. Prodromal period

d. Period of illness

Complement component C3, in the classical pathway, is split by __________. a. C2bC4a b. C5 c. C2bC4b d. C4bC4a e. C2aC4b

e. C2aC4b

Gluteraldehydes are among the most effective chemical control agents because they______.

are relatively safe, yet considered a sterilizing agent

Vectors: - _________, especially fleas, ticks, and _________ (deadliest animals on the planet). - Transmit disease by two general methods: * ___________: arthropod carries pathogen on its feet (steps on poop). * ___________: pathogen reproduces in the vector; transmitted via bites or feces

arthropods/ mosquitos * mechanical transmission * biological transmission

The Nucleus: * double membrane structure (___________) that contains the cell's DNA. * DNA is complexed with _______ proteins to form _________. * during mitosis and meiosis, chromatin condenses into ________.

* (nuclear envelope) * histone/ chromatin * chromosomes

Exotoxins vs. Endotoxins: - Exotoxins: * bacterial source: mostly from ________ bacteria. * relation to microorganism: metabolic product of growing cell. * chemistry: proteins, usually with two parts (A-B). * pharmacology (effect on body): specific for a particular cell structure or function in the host (mainly affects cell functions, nerves, and GI tract). * heat stability: unstable, can usually be destroyed at 60-80 degrees C (except S. enterotoxin) * toxicity (ability to cause disease): high * fever-producing: no * immunology (relation to antibodies): can be converted to toxoids to immunize against toxin; neutralized by antitoxin. * lethal dose: small * representative diseases: gas gangrene, tetanus, botulism, diphtheria, and scarlet fever. - Endotoxins: * bacterial source: mostly from ________ bacteria. * relation to microorganism: present in LPS of outer membrane of cell wall and released with destruction of cell or during cell division. * chemistry: lipid portion (Lipid A) of LPS of outer membrane (lipopolysaccharide). * pharmacology (effect on body): general, such as fever, weakness, aches, and shock; all produce the same effects. * heat stability: stable, can withstand autoclaving (121 degrees C for 1 hour). * toxicity (ability to cause disease): low * fever-producing: yes * immunology (relation to antibodies): not easily neutralized by antitoxin; therefore, effective toxoids cannot be made to immunize against toxin. * lethal dose: considerably larger. * representative diseases: typhoid fever, urinary tract infections, and meningococcal meningitis.

* Gram + * Gram -

Representative Normal Microbiota: - Skin: * __________: cause ache. * 95% of species on the skin are __________ because of the acidic environment. - Eyes: S. epidermidis and S. aureus - Nose and Throat: S. epidermidis and S. aureus - Mouth: Streptococcus * Staph and strep are both _______ organisms. - Strep. pneumonia = ________. - Once you hit the stomach, the bacteria species are _______. - Large and Small intestines: * E. coli and _______ (probiotics) * ________: organisms found in the digestive tract/ if in water = contaminated. - Urinary and Reproductive Organs: * Staphylococcus

* Propionibacterium * gram + cocci * gram + - strep throat - gram - * Bifidobacterium * coliforms

Fimbriae and Pili: - Fimbriae: * hairlike appendages that allow for ________. * ________ than pili. - Pili: * involved in _____ (______ and _______ motions) * ________ involved in DNA transfer from one cell to another.

* attachment * shorter * motility (gliding and twitching) * conjugation pili

Comparing Prokaryotic and Eukaryotic Cells: - Prokaryote: * one _______ chromosome, not in a _________. * no ______ or _______. * include _____ and ______. * bacteria: _______ cells walls * archaea: ______ cell walls * divides by ________. - Eukaryote: * _______ chromosomes, in __________. * has _______ and _______. * ________ cell walls, when present. * divides by _______ for what three things?

* circular/ membrane * histones or organelles * bacteria and archaea * peptidoglycan * pseudomurein * binary fission * paired linear/ nuclear membrane * histones and organelles * polysaccharide * mitosis/ growth, repair, and replace

Compound Light Microscopy: - Brightfield illumination * _______ objects are visible against a _______ background. * Light reflected off the specimen does not enter the _______ lens. - In a compound microscope the image is _______ and ______ from the actual sample you are looking at.

* dark/ light * objective - reversed and backwards

Monoclonal Antibodies: - Mabs are uniform, highly specific, and produced in large quantities. * used in _______ tools. * used in _____ therapy. - neutralize TNF with _______. - treat allergic asthma by preventing the binding of _____ on mast cells and basophils - Often derived from ______ cells, leading to side effects.

* diagnostic tools * human - rheumatoid arthritis - IgE - mouse

A disk-diffusion test using Staphylococcus gave the following results: Disinfectant/Zone of inhibition (mm) - A/0 - B/2.5 - C/10 - D/5 * In the table, which compound was bactericidal? a. A b. B c. C d. D e. The answer cannot be determined based on the information provided. * In Table 7.1, which compound was the most effective against Staphylococcus? a. A b. B c. C d. D e. The answer cannot be determined based on the information provided. * In Table 7.1, which compound was the most effective against E. coli? a. A b. B c. C d. D e. The answer cannot be determined based on the information provided.

* e. The answer cannot be determined based on the information provided. * c. C * e. The answer cannot be determined based on the information provided.

Formed Elements in Blood: - Cells and cell fragments suspended in plasma: * ________ * _______ * _______ - Created in __________ * from stem cells via __________. - ________: capable of giving rise to several different cell types.

* erythrocytes (RBC) * leukocytes (WBC) * platelets - red bone marrow * hematopoiesis - pluripotent

Atypical Cell Walls: - Acid-fast cell walls: * like _________ cell walls * include _____ and ______ * stain with _________ * _______ (______) bound to peptidoglycan = this interferes with gram staining! - Myoplasmas: * lack _______ (need a controlled environment because it is susceptible to _____________) * ______ in plasma membrane - Archaea: * ______ or * walls of _______ (lack NAM and D-amino acids).

* gram-positive * mycobacterium and nocardia * carbolfuchsin * waxy lipid (mycolic acid) * cell walls/ (osmotic pressure changes) * sterols * wallless * pseudomurein

Gram-Positive Cell Walls: - Teichoic acids: * _______ links cell wall to plasma membrane. * wall ________ links the peptidoglycan. * carry a _______ charge (due to acid) * regulate movements of ______. - _______ and ________ provide antigenic specificity. - What gives gram-positive bacteria strength and rigidity? - What does penicillin target in gram-positive bacteria?

* lipoteichoic acid * teichoic acid * negative * cations - polysaccharides and teichoic acids - carbohydrate backbone - the enzyme that forms the bond between protein and carbohydrate cross-linkage.

In Figure 6.3, which tube shows the expected growth pattern for a facultative anaerobe? a b c d e

b

The Plasma (cytoplasmic) Membrane: - Compare and contrast prokaryotic and eukaryotic plasma membranes: * Similar in structure to prokaryotic cell membranes include: ________ and ________. * Differences in structure: ______ - complex lipids and ________ for attachment and cell-to-cell recognition. *Similar functions include: __________ and all _______/_______ processes. * Differences in function: - __________: phagocytosis and pinocytosis. - __________: pseudopods extend and engulf particles. - __________: membrane folds inward, bringing in fluid and dissolved substances.

* phospholipid bilayer and internal/peripheral proteins * sterols/ carbohydrates * selective permeability and passive/active * endocytosis/ phagocytosis/ pinocytosis

The Cell Wall and Glycocalyx: - Cell Wall: * found in _______ (3) * made of ________ (_____ in plants, _______ in fungi, and ________ in yeasts). - Glycocalyx: * found in _______ * _________ bonded to ________ and _______ in the plasma membrane.

* plants, algae, and fungi * carbohydrates (cellulose/ chitin/ glucan and mannan) * animal cells * carbohydrates/ proteins and lipids

Which statement best describes what happens when a gram-negative bacterium is placed in distilled water and penicillin? a) No change will result; the solution is isotonic b) Water will move into the cell c) Water will move out of the cell d) the cell will undergo osmotic lysis e) Sucrose will move into the cell from an area of higher concentration to one of lower concentration

b) Water will move into the cell

Emerging Infectious Diseases: - Contributing factors: * genetic _________: Escherichia coli O157 and avian influenza (H5N1) --> E. coli: took up portion of Shigella toxin = disease causing part/ normal E. coli is good for stomach. * ______ of new strains: Vibrio cholerae O139 * widespread use of _______ and _______: antibiotic-resistant strains. * changes in ______ patterns: Hantavirus * modern __________: Chikungunya and West Nile virus * ecological disaster, war, and expanding human _________: Coccidioidomycosis. * ______ control measures: Lyme disease. * ______ health failure: Diphtheria.

* recombination * evolution * antibiotics and pesticides * weather * transportation * settlement * animal * public

Outcomes of Complement Activation: - Regulation of Complement: * ___________ readily break down complement proteins, minimizing host cell destruction. - Complement and Disease: * lack of complement proteins causes ________ to infections. - Evading the Complement System: * ________ prevent complement activation to occur.

* regulatory proteins * susceptibility * capsules

The antibiotic amphothericin B disrupts plasma membranes by combining with sterols; it will affect all the following cells except: a) animal cells b) gram-negative bacterial cells c) fungal cells d) Mycoplasma cells e) plant cells

b) gram-negative bacterial cells

Biosynthesis of RNA Viruses That Use DNA: - Single stranded RNA, produce DNA: * Use ___________ (not unique to the virus = the human body has some) to produce DNA from the viral genome. * RNA backwards (reverse transcription): can't use _________. * viral DNA integrates into the host chromosome as a _________ (stabilized in host). - Example comes from the virus family _________. - Retroviridae: * _________: HIV * Oncoviruses

* reverse transcriptase * viral RNA * provirus - Retroviridae * Lentivirus

Which of the following pairs is mismatched? a) glycocalyx -- adherence b) pili -- reproduction c) cell wall -- toxin d) cell wall -- protection e) plasma membrane -- transport

b) pili -- reproduction

all members of a group of ornithologists studying barn owls the wild have had salmonellosis (salmonella gastroenteritis). One birder is experiencing her third infection. What is the most likely source of their infections? a) the ornithologist are eating the same food b) they are contaminating their hands while handling the owl and nests c) one of the workers is a salmonella carrier d) their drinking water is contaminated

b) they are contaminating their hands while handling the owl and nests

In which of the following cases would the Limulus amebocyte lysate (LAL) assay be used? a) to confirm the diagnosis of gas gangrene b) to ensure that a sterilized medical device is free of endotoxin c) to check for enterotoxins d) to detect the presence of the botulinum toxin

b) to ensure that a sterilized medical device is free of endotoxin

- Knowledge of microorganisms allows human to: * prevent food __________ * prevent __________ * understand _______ and ________ of disease to prevent epidemics.

* spoilage * disease * causes and transmission

- Gram-Positive Cell Walls: * _______ peptidoglycan (___________) * ________ acids * _____ in basal body of flagella. * produce _______. * _____ susceptibility to penicillin * disrupted by ________. - Gram-Negative Cell Walls: * ______ peptidoglycan * ______ membrane (main difference) * _________ space * _______ in basal body of flagella * produce ______ and ______. * _____ susceptibility to penicillin. - _____% of bacteria fall into one of these groups.

* thick (outer layer) * teichoic * 2-rings * exotoxins * high * lysozyme * thin * outer * periplasmic * 4-rings * endotoxins and exotoxins * low - 99

Plasma Membrane Structure: - Fluid Mosaic Model: * membrane is as _________ as olive oil. * semipermeable and ________ (______ have a hard time passing) * _____ move freely for various functions. * _______ rotate and move laterally. * self-sealing and self-healing * surprisingly _________. * proteins receive ________ from the world outside or transport ________ and ________.

* viscous * selective/ (charged ions) * proteins * phospholipids * flexible (supple) * signal/ nutrients and wastes

Preparing Smears for Staining: - Live and/or unstained specimens have little contrast with the surrounding medium (________ for the most part). Live specimens are used to study ________. - ________: coloring microorganisms with a dye that emphasizes certain structures. - _______: a thin film of a material containing microorganisms spread over a slide. - Microorganisms are _______(attached) to the slide, which kills the microorganisms.

- (transparent)/ cell behavior - staining - smear - fixed

Big Picture: Immunity: - Innate: * __________; granulocyte/ releases histamines that cause inflammation. * __________; granulocyte/ kills parasites with oxidative bursts. * __________; agranulocyte/ kills infected cells via cytolysis. - Both: * __________; granulocyte/ phagocytizes bacteria and fungi * __________; agranulocyte/ precursor to macrophages. * __________; many surface projections/ in skin and respiratory/ phagocytize bacteria and present antigens to T cells. * __________; agranulocyte (lymphocyte)/ kills cancer cells and virus-infected cells. - Adaptive: * __________; agranulocyte (lymphocyte)/ recognize antigens and produce antibodies. * ___________; agranulocyte (lymphocyte)/ secrete cytokines, recognize and kill non self cells, and destroy cells that do not correctly recognize self cells.

- * basophils * eosinophils * mast cells - * neutrophils * monocytes * dendritic cells * natural kill cells (NK cells) - * plasma cells (B cells) * T cells (include: T helper cell, cytotoxic T lymphocytes, and T regulatory cells)

Physical Factors: - Skin: * _______: inner portion made of connective tissue. * _______: outer portion made of tightly packed epithelial cells containing _______, a protective protein that is waterproof and creates a barrier. * Shedding and dryness of skin _______ microbial growth. - Mucous Membranes: * epithelial layer that lines the _______, ________, and _______ tracts. * _______: viscous glycoproteins that trap microbes and prevent tracts from drying out; secreted by the ______ cells. * _______: drains tears; washes the eyes - cornea is not innervated = transplants are easy - blinking = washing out the eyes. - ________: transports microbes trapped in mucus away from the lungs. - _______: prevents microbes from entering the ear. - _______: cleans the urethra via flow. - _______: move microorganisms out of the vaginal tract. - Other methods of physical: _______, defecation, vomiting, and diarrhea.

- * dermis * epidermis/ keratin * inhibits - * gastrointestinal, respiratory, and genitourinary * mucus/ goblet * lacrimal apparatus - ciliary escalator (cilia) - earwax - urine - vaginal secretions - peristalsis

Other Chemical Used for Sterilization: - Plasma: * ________ state of matter, consisting of electrically excited gas. * _________ destroy microbes * used for ____________. - Supercritical Fluids: * _______ with gaseous and liquid properties. * used for ____________ * demonstrates all _______ of matter. - Peroxygens and Other Forms of Oxygen: * ________ agents (what do they do with electrons?) * used for _________ and _________ * example

- * fourth * free radicals * tubular instruments - * CO2 * medical implants * 3 phases - * oxidizing (lose them) * contaminated surfaces and food packaging * O3 (ozone), H2O2, and peracetic acid

- IgG: * valence of 2 * monomer * ______ percentage of total serum antibodies. * location: blood, lymph, and intestine. * complement fixation: ______ * placental transfer: ______ * functions: enhances phagocytosis, neutralizes toxins and viruses; protects fetus and newborn. - IgM: * valence of 10 * pentamer * percentage of total serum antibodies: 6% * location: blood, lymph, _________ (as monomer) * complement fixation: ______ * function: especially effective against microorganisms and agglutinating antigens; ______ antibodies produced in response to initial infection. - IgA: * valence of 4 * soluble! * has a J chain * dimer with secretory component * 13% of total serum antibodies. * locations: _______ (tears, saliva, mucus, intestine, milk), blood, and lymph. * functions: localized protection on mucosal surfaces. - IgD: * valence of 2 * monomer * 0.02% of total serum antibodies. * location: ________, blood, and lymph. * function: serum function not known; presence on B cells functions in initiation of immune response. - IgE: * valence of 2 * 0.002% of total serum antibodies. * location: bound to ____ and _____ cells throughout the body, and blood. * function: _______ reactions; possibly lysis of _________.

- * highest (80%) * yes * yes (only one that can cross the placenta = in developing embryo = can be a problem) - * B cell surface * yes * first - * secretions - * B cell surface - * mast and basophil cells * allergic/ parasitic worms

Chemical Requirements: - Carbon: * structural backbone of ________ molecules (forms 4 covalent bonds) * __________ use organic molecules as energy (humans) * _________ use CO2 as energy (photosynthesis) * what does -troph mean? - Nitrogen: * component of _________, ________, and _______. * most bacteria decompose _________ material for the nitrogen source. * some bacteria use NH4+ or NO3- from organic material. * a few bacteria use N2 in ______________ (which is what?) * what is the element that be breath in most? - Sulfur: * used in _________, _______, and _______. * most bacteria decompose ________ for the sulfur source. * some bacteria use SO4 2- or H2S. - Phosphorus: * used in ______, ______, and _______. * found in _________. * PO4 3- is a source of phosphorus. - Trace Elements: * _______ elements required in small amounts. * usually as enzyme _______. * include iron, copper, molybdenum, and zinc.

- * organic * chemoheterotrophs * autotrophs * to eat - * proteins, DNA, and ATP * protein * nitrogen fixation (take N2 and fix it into compounds they can use) * nitrogen (79%) - * amino acids, thiamine, and biotin * proteins - * DNA, RNA, and ATP * membranes - * inorganic * cofactors

Selective and Differential Media: - Selective: * _______ unwanted microbes and _______ desired microbes. * contain _________ to suppress growth. - Differential: * allow __________ of colonies of different microbes on the same plate. - What is an example of media that is selective and differential? EXPLAIN

- * suppress/ encourage (selective for certain organisms) * inhibitors - * distinguishing - mannitol salt agar. It is selective for halophiles (like salt) and is differential when phenol red is used. phenol red has a pH of 7, which gives it its red color. A drop in pH turns the phenol red to a yellow color (drop in pH because they can metabolize it).

ssRNA - Difference: - the ________ must first be transcribed from the - viral genome before proteins can be synthesized. - additional - strands are transcribed from mRNA.

- + strand (mRNA)

ssRNA + Difference: - - strand is transcribed from ________ genome. - mRNA is transcribed from the - strand

- + viral

- Cell in isotonic solution: Under these conditions, the solute concentration in cell is equivalent to a solute concentration of _________ (physiological saline). - Plasmolyzed cell in hypertonic solution: If the concentration of solutes such as NaCl is higher in the surrounding medium than in the cell, water tends to _______ the cell and growth of the cell is inhibited (what does not shrink?)

- 0.85% sodium chloride (NaCl) - leave/ (cell wall)

Multiplication of Animal Viruses: - 1. __________: viruses attach to the cell membrane. - 2. _________: by receptor-mediated endocytosis or fusion. - 3. _________: by viral or host enzymes. - 4. ________: production of nucleic acid and proteins. - 5. _________: nucleic acid and capsid proteins assemble. - 6. ________: (enveloped viruses) or rupture. - Animal viruses utilize what type of endocytosis: ________? - The intake of water from outside the cell is known as ________; while the intake of particles is known as ________.

- 1. attachment - 2. entry - 3. uncoating - 4. biosynthesis - 5. maturation - 6. release by budding - receptor-mediated - pinocytosis/ phagocytosis

Evolution of Eukaryotes: - Life arose as simple organisms _____ to _____ billion years ago. - First eukaryotes evolved _____ billion years ago. - Endosymbiotic Theory: * larger bacterial cells engulfed smaller bacterial cells, developing the first ___________. * ingested photosynthetic bacteria became ________. * ingested aerobic bacteria become _________.

- 3.5 to 4 - 2.5 * eukaryotes * chloroplasts * mitochondria

Plate Counts: - Count colonies on plates that have ____ to _____ colonies (CFUs). - To ensure the right number of colonies, the original inoculum must be diluted via ________ (in a line or series). THE MOST COMMON WAY TO DETERMINE # OF BACTERIA. - Counts are performed on bacteria mixed into a dish with agar (___________) or spread on the surface of a plate (___________). - Serial Dilution equation: ? * If there are 54 colonies on a plate of 1:100 dilution, then the count of bacteria/ml in sample is ________.

- 30 - 300 - serial dilution - (pour plate method)/ (spread plate method) - number of colonies on plate x reciprocal of dilution of sample = number of bacteria/ml * 5400

A rabbit was found to have floppy ear disease, which causes long-ear infections. Its etiology (cause) is unknown. Unknown Microorganism X from the infected rabbit's right ear is successfully isolated and cultured on laboratory growth media. The ear of a healthy laboratory mouse is then inoculated with isolated microorganism, and after a period of time, no disease is observed. Which of the following statements accurately describe(s) the conclusion(s) that can be drawn from this experiment? Select all that apply.

- A laboratory mouse might not be an appropriate, susceptible host; an experiment with a laboratory rabbit may be needed. - Microorganism X might not be linked to floppy ear disease, because the inoculated mouse remained healthy.

- In Table 7.3, which disinfectant is the most effective at stopping bacterial growth? A) Doom B) K.O. C) Mortum D) Sterl E) The answer cannot be determined based on the information provided. - In Table 7.3, which disinfectant was bactericidal? A) Doom B) K.O. C) Mortum D) Sterl E) The answer cannot be determined based on the information provided. - In Table 7.3, which disinfectant was most effective against Salmonella? A) Doom B) K.O. C) Mortum D) Sterl E) The answer cannot be determined based on the information provided.

- A) Doom - C) Mortum - E) The answer cannot be determined based on the information provided.

Exotoxins: - ________ contain an ______ component (A part) and a ______ component (B part). * example: _________ * A part: goes and does something = disrupts functioning of the cell. * B part: shaped to bind to a specific target (receptor).

- A-B toxins/ enzyme/ binding * Diphtheria toxin

This activity will explore the overall process of replication of a DNA-containing animal virus in more detail. Drag each of the statements into the box indicating the appropriate infection stage.

- Attachment: requires a physical and chemical interaction between the surface of the virus and host cell surface - Entry and uncoating: * This process disassembles the viral capsid and releases the viral DNA. * may involve fusion of the viral envelope with the host cell plasma membrane * may involve receptor-mediated endocytosis - Biosynthesis * Transcription of viral genes by host cell RNA polymerase occurs during this stage. * New copies of viral DNA genomes are made during this stage. * Translation of viral capsid proteins and other viral proteins occurs during this stage. * The viral DNA must enter the host cell nucleus prior to this stage. - Maturation: * Capsids are assembled from capsomeres. * Viral genomic DNA is packed into newly assembled capsids. - Release: * Nonenveloped viruses often lyse the cell during this process. * Enveloped viruses accomplish this via a budding process.

A suspension of 106 Bacillus cereus endospores was put in a hot-air oven at 170°C. Plate counts were used to determine the number of endospores surviving at the time intervals shown. - In Figure 7.1, what is the thermal death time? A) 150°C B) 60 minutes C) 120 minutes D) 100°C E) The answer cannot be determined based on the information provided. - In Figure 7.1, the thermal death point for this culture is A) 15 minutes. B) 50°C. C) 30 minutes. D) 170°C. E) The answer cannot be determined based on the information provided - In Figure 7.1, the decimal reduction time (D value) for the culture, which is defined as the time to reduce a population by one log, is approximately A) 0 minutes. B) 10 minutes. C) 30 minutes. D) 40 minutes. E) 60 minutes.

- B) 60 minutes - E) The answer cannot be determined based on the information provided - B) 10 minutes.

The Alternative Pathway: - C3 present in the blood combines with factors _____, _____, and _____ on microbe surface. - C3 splits into ______ and _____, functioning the same as in the classical pathway. - The _________ contains the factors B, D, and P to which C3 binds to.

- B, D, and P - C3a and C3b - lipid-carbohydrate complex

Biosafety Levels: - __________: no special precautions; basic teaching labs. - __________: lab coat, gloves, and eye protection. - __________: biosafety cabinets to prevent airborne transmission. - __________: sealed, negative pressure; "hot zone" * exhaust air is filtered twice through _________.

- BSL-1 - BSL-2 - BSL-3 - BSL-4 * HEPA filters

Bacterial Growth Curve: - Lag phase: ? - Log phase: ? - Stationary phase: ? - Death phase: ?

- Bacteria are metabolically very active, but the numbers are not increasing. - Period of most rapid growth. Number of bacteria is increasing exponentially. - Population growth has halted. Number of new cells formed balances the number of cells dying. - Rate of the cell death exceeds the rate at which new cells are being formed. Numbers of cells decline exponentially.

Recombinant DNA Technology cont.: - 1941: ________ and ________ showed that genes encode a cell's enzymes. - 1944: _________, ________, and _________ showed that DNA is the hereditary material. - 1953: ________ and _________ proposed a model of DNA structure. * __________: used her crystal structure to determine the 3D structure of DNA; however she is not credited. - 1961: _________ and _________ discovered the role of mRNA in protein synthesis.

- Beadle and Tatum - Avery, MacLeod, and McCarty - Watson and Crick * Rosalind Franklin - Jacob and Monod

Inflammation Stimulated by Complement: - _____ and _____ attach to mast cells, causing a release of histamines. * this attracts neutrophils and macrophages

- C5a and C3a - PICTURE IS HELPFUL

T Helper Cells: - Also known as _______. - TCR on the TH cell recognize and bind to the antigen fragment and __________ on APCs. - APC or TH secrete a _________ molecule, activating the ____ cell. - TH cells produce _______ and differentiate into: * _______ * ______ * ______ * ______

- CD4+ T cells - MHC Class II - costimulatory/ TH - cytokines * TH1 cells * TH2 cells * TH17 cells * memory cells

Classes of T Cells: - What are the two classes: _____ and _____? - Clusters of differentiation (CD) * CD4+ - ________ cells - _____ signaling with B cells (coordinate _______ response); interact directly with antigens. - bind __________ molecules on B cells and APCs. * CD8+ - __________ cells - bind ________ molecules - surveillance of self vs non-self and cancer cells.

- CD4+ and CD8+ - T helper cells (TH) - cytokine/ (humoral) - MHC Class II - cytotoxic T lymphocytes (CTL) - MHC Class I

T Regulatory Cells: - Treg - Subset of ______ T cells; carry an additional ______ molecule. - Suppress T cells against ______; protect _______ bacteria required for digestion; and protect the growing _______.

- CD4+/ CD25 - self/ intestinal/ fetus

T Cytotoxic Cells: - Also known as ______ T cells. - Activated into _______ with the help of TH cell and costimulatory signals. - CTLs recognize and kill ______ cells altered by infections * Self-cells carry __________ on a surface presented with MHC class ______ molecules. - CTL releases _____ and ______ that induce ______ in the infected cells. - PICTURE: 1. a normal cell will not trigger a response by a cytotoxic T lymphocyte (CTL), but a virus-infected cell or a cancer cell produces abnormal endogenous antigens. 2. the abnormal antigen is presented on the cell surface in association with MHC class I molecules. Binding of a TH1 cell promotes secretion of cytokines. 3. the cytokines activate a precursor CTL, which produces a clone of CTLs. 4. the CTL induces destruction of the virus-infected cell by apoptosis.

- CD8+ - cytotoxic T lymphocytes (CTL) - self * endogenous antigens/ I - perforin and granzymes/ apoptosis

EXTRA VIRUSES FROM CHARTS: Single Stranded RNA, + Strand Enveloped: - Coronaviridae: ___________: associated with upper respiratory tract infections and the common cold; SARS virus, MERS-CoV. Single Stranded RNA, - Strand: - Filoviridae: Filovirus: enveloped, helical virus; ______ and Marburg viruses are filoviruses (hemorrhagic symptoms).

- Coronavirus - Ebola

Fungi Toxins: - ______ are alkaloid toxins that cause hallucinations. - ______ is a carcinogenic toxin produced by Aspergillus. - ________ are produced by mushrooms and are neurotoxic: * ______ and _______ are examples.

- Ergot - Aflatoxin - Mycotoxins * Phalloidin and amanitin

This activity asks you to match each key term related to etiology and infectious disease with its description. Based on the information contained in the introduction, match each of the following concepts with its best description.

- Etiology: the study of the cause of a disease. - Infectious disease: disease directly caused by microorganisms and not by other means, for example, genetic or degenerative diseases. - Germ theory of disease: this theory states that microorganisms cause infectious diseases. - Koch's postulates: experimental requirements for identifying the agent of an infectious disease.

Principal Vaccines for Bacterial Disease: - ________: polysaccharide from Haemophilus influenzae type b conjugated with protein to enhance effectiveness. - ________: purified polysaccharide from Neisseria meningitis - _______: purified polysaccharide from seven strains of Streptococcus pneumoniae. - _______: DTaP: a = acellular = subunit or fragment of pertussis.

- Hib - MenACWY - PPSV/PCV - DTaP (tetanus, diphtheria, and pertussis)

Clonal Selection of Antibody-Producing Cells: - Inactive B cells contain surface _____ that bind to antigens. - B cell internalizes and processes antigen. - Antigen fragments are displayed on ________ molecules. - _________ contacts the displayed antigen fragment and releases cytokines that activate B cells. - B cell undergoes proliferation (___________) * mature into ________, which make antibodies.

- Ig - MHC class II - T helper cells (Th) - (clonal expansion) * plasma cells

- What antibodies have a valence of 2: _____, ______, and ______? - What antibody has a valence of 4: ______? - What antibody has a valence of 10: ________? - What antibody makes up the highest percentage of total serum antibodies: _______? - What two antibodies are a part of complement fixation: _____ and _____? - What antibody can cross the placenta: _______? - What two antibodies are on the B cell surface: _______ and _______? - What is the first antibody produced in response to initial infection: _______? - What antibody is found in secretions like tears, saliva, mucus, intestine, and milk: _______? - What antibody is bound to mast and basophil cells/ and induce allergic reaction and possibly lysis of parasitic worms: _______?

- IgG, IgD, and IgE - IgA - IgM - IgG - IgG and IgM - IgG - IgM and IgD - IgM - IgA - IgE

Epidemiology: - _________; 1848-1849: mapped the occurrence of cholera in London. - ________; 1846-1848: showed that hand washing decreased the incidence of puerperal sepsis. - ________; 1858: showed that improved sanitation decreased the incidence of epidemic typhus.

- John Snow - Ignaz Semmelweis - Florence Nightingale

- ___________: determine etiology of diseases. 1. the same ________ must be present in every case of the disease. 2. the pathogen must be ________ from the diseased host and grown in pure culture. 3. the pathogen from the pure culture must cause the ________ when it's inoculated into a healthy, susceptible laboratory animal. 4. the pathogen must be isolated from the __________ animal and must be shown to be the original organism. ***** all of these steps determine the _____________ of the disease. - PICTURE: 1. Microorganisms are isolated from a diseased or dead animal. 2. The microorganisms are grown in pure culture 3. Microorganisms are inoculated into a healthy laboratory animal 4. Disease is reproduced in a laboratory animal 5. The microorganisms are isolated from this animal and grown in pure culture

- Koch's Postulates 1. pathogen 2. isolated 3. disease 4. inoculated ***** causative agent

Cell Wall Components: - _________ resists phagocytosis. * example: Streptococcus pyogenes - _________ allows attachment to host cells. * example: Neisseria gonorrhoeae - _________ (___________) resist digestion (live and replicate in macrophages) * example: Mycobacterium tuberculosis

- M protein - Opa protein - Waxy lipid (mycolic acid)

The Golden Age of Microbiology: - 1857-1914 - Beginning with ________ work, discoveries included the relationship between microbes and disease, immunity, and antimicrobial drugs.

- Pasteur's

Big Picture: Immunity:

- Pathogens (viruses, bacteria, and fungi) ------> * First line of defense: skin, mucous membranes, antimicrobial substances * Second line of defense: inflammation, fever, phagocytes * Third line of defense: humoral and cellular immunity

Principle Cells that Function in Cell-Mediated Immunity: - _________: activates cells related to cell-mediated immunity: macrophages, Tc cells, and natural killer cells. - ________: stimulates production of eosinophils, IgM, and IgE. - ________: recruits neutrophils; stimulates production of antimicrobial proteins. - ________: destroys target cells on contact; generated from Tc cells. - ________: regulates immune response and helps maintain self-tolerance. - ________: enhanced phagocytic activity; attacks cancer cells. - ________: attacks and destroys target cells; participates in antibody-depedent cell-mediated cytotoxicity.

- T helper (Th1) cells - T helper (Th2) cells - T helper (Th17) cells - cytotoxic T lymphocytes (CTL) - T regulatory cells - activated macrophage - natural killer cells

Clonal Selection of Antibody-Producing Cells: - _________: antigen that requires a Th cell to produce antibodies. - ________: stimulate the B cell without the help of T cells. * provoke a _____ immune response; producing _____. * non memory cells generated.

- T-dependent antigen - T-independent antigens * weak/ IgM

T Helper Cells (CD4+ T cells) - _______ cells produce IL-17 and contribute to inflammation. - ______ cells produce IFN-gamma, which activates macrophages, enhances complement, and stimulates antibody production that promotes phagocytosis. - ______ cells activate B cells to produce IgE; activate eosinophils - TH cells = coordination of the immune system. PICTURE: * TH17 cells secrete cytokines that promote inflammatory responses; recruit neutrophils for protection against extracellular bacteria and fungi. * TH2: IL-4 = important in allergic responses, especially by production of IgE. Activate eosinophils to control extracellular parasites such as helminths. * TH1: IFN-gamma: cytokines activate CD8+ T cells and NK cells, which control intracellular pathogens by killing infected host cells. They also enhance phagocytosis by antigen-presenting cells such as macrophages.

- TH17 - TH1 - TH2

Select each statement that accurately describes what is happening to bacterial cultures during stationary phase. - The number of cells that are dying is balanced by the number of new cells that are being formed. - The cells are dead; therefore, the number is staying constant. - The cells are likely running out of nutrients. - Harmful waste products may be accumulating. - Cells are not increasing in number because they have not yet adjusted to the nutrient availability in the new media.

- The number of cells that are dying is balanced by the number of new cells that are being formed. - The cells are likely running out of nutrients. - Harmful waste products may be accumulating.

This activity asks you to identify Koch's postulates for determining the etiology of infectious diseases. Which statements accurately describe Koch's postulates?

- The same pathogen must be present in every case of the disease. - The pathogen must be isolated from the inoculated animal and must be shown to be the original organism. - The pathogen must be isolated from the diseased host and grown in pure culture. - The pathogen from the pure culture must cause the disease when it is inoculated into a healthy, susceptible laboratory animal.

A virus first interacts with a host cell during the attachment process that precedes viral entry into the cell. Which of the following are possible consequences of the specific interaction between viral attachment sites and host cell surface receptors? Select all that apply.

- Viral infection of a cell may be prevented by blocking the attachment sites on a virus with antibody. - Some people are naturally (innately) resistant to certain viral infections. - Viral infection of a cell may be prevented by blocking the receptor sites on a host cell. - A particular virus may be able to infect humans but not mice. - A particular virus may be able to infect a liver cell but not a lung cell.

Big Picture Immunity: - _________; measure leukocytes in the blood. - ______ WBC counts indicate bacterial infections, autoimmune diseases, or side effects of medications. - _______ WBC counts may indicate viral infections, pneumonia, autoimmune diseases, or cancers. - Normal Range and Percentages: * WBCs = _________ WBCs per cubic millimeter. * __________: 60-70% (highest percentage of WBCs) * _________: 20-25% * __________: 3-8% * __________: 2-4% * __________: 0.5-1%

- WBC counts - high - low * 5000-10000 * neutrophils. * lymphocytes * monocytes * eosinophils * basophils

Nomenclature: - After the first use, scientific names may be ________ with the first letter of the genus and the specific epithet: * Escherichia coli and Staphylococcus aureus are found in the human body. * E. coli is found in the ________, and S. aureus is on the ______.

- abbreviated * large intestine/ skin

Active Processes: - __________: requires a transporter protein and ATP; goes against gradient. - _________; requires a transporter protein and phosphoenolpyruvic acid (PEP); substance is altered as it crosses the membrane.

- active transport - group translocation

Severity or Duration of a Disease: - __________; symptoms develop rapidly but the disease lasts only a short time. - __________; symptoms develop slowly. - __________; intermediate between acute and chronic. - __________; causative agent is inactive for a time but then activates and produces symptoms. - __________; immunity in most of a population.

- acute disease - chronic disease - subacute disease - latent disease - herd immunity

Inflammation: - Inflammation activates ________ by the liver that cause _______ and _______ of the blood vessels. - Release: * ________= broad category of signaling molecules (rest of the 4 are types of these). * _______: associated with allergies. * ______ * _______: increase body temperature (part of the fever response). * ________

- acute-phase proteins/ vasodilation and increased permeability * cytokines * histamines * kinins * prostaglandins * leukotrienes

The Biosynthesis of DNA Viruses: Disease Causing Families: - What are the 5 families of DNA viruses _______? - Adenoviridae: * ________ DNA; & ________ (naked) * cause ________ infections in humans and _______ in animals. - Poxviridae: * _______ DNA; & _________ (coat) * causes skin ________ * example: _______ and _____ viruses (orthopoxvirus) * Molluscipoxvirus - Papovaviridae: * ______ DNA; & ________ (naked) * example: _________; which causes ________ and can transform cells causing _______. * ______, _______, and _______ reduce probability of cervical cancer by 90%. * Polyomavirus - Herpesviridae * ________ DNA; & ________(coat). ---> listed are mutant subspecies that are not the same but have similar _________. * HHV-1 and HHV-2: Simplexvirus: causes ________ * HHV-3: Varicellovirus: causes _______, which can develop into _______. * HHV-4: Lymphocryptovirus: causes __________. * HHV-5: ____________: get huge and is tested for during pregnancy. * HHV-6 and HHV-7: ____________ * HHV-8: Rhadinovirus: causes ___________ (lesions in lower extremities = usually HIV) - Hepadnaviridae: * ________ DNA; & ________ (coat) * Has to do with the hepatocytes, which are in the ________. * ________ virus * Use _________ to make DNA from RNA.

- adenoviridae, poxviridae, papovaviridae, herpesviridae, and hepadnaviridae - * double stranded/ non enveloped * respiratory/ tumors - * double stranded/ enveloped * lesions * vaccinia and smallpox - * double-stranded/ non enveloped * papillomavirus/ warts/ cancer (like cervical) * cervarix, Gardasil 4, and Gardasil 9 - * double-stranded/ enveloped/ genomes * cold sores * chickenpox/ shingles * mononucleosis * Cytomegalovirus * Roseolovirus * Kaposi's sarcoma - * double stranded/ enveloped * liver * Hepatitis B * reverse transcriptase

Adherence: - Almost all pathogens attach to host tissues in a process called ________ (________). - __________ (_________) on the pathogen (mimic human hormones; taken in by endocytosis) bind to receptors on the host cells. Two forms of these on pathogens are: * _________ * _________ - Microbes form _________ (communities that share nutrients). Do NOT need adhesins for this!

- adherence (adhesion) - adhesins (ligands) * glycocalyx * fimbriae - biofilms

- _________ are chemical additives added to vaccines to improve effectiveness. * ______ was the first aluminum salt used as an adjuvant, but has been almost completely replaced by ________ and ________ for commercial vaccines. * improve the _______ immune response (also acts like a self life for the vaccine = longer in the body).

- adjuvants * alum/ aluminum hydroxide and aluminum phosphate * innate

Culture Media: - Solid = ______. - Agar: * complex _________ * used as a ___________ for culture media in Petri plates, slants, and deeps. * generally not ______ by microbes. * liquifies at ______ * solidifies at ______

- agar * polysaccharide * solidifying agent * metabolized * 100 degrees C * about 40 degrees C

- ___________: particulate antigens binding to antibodies to form visible aggregates. * involved in suspensions! - ___________: detect antibodies against large cellular antigens. * measure concentration of serum antibody (known as ______) - rise in titer indicates a greater immunity to disease. * ________ is a significant change in titer as a disease progresses.

- agglutination reactions - direct agglutination tests * (titer) * seroconversion

The Classical Pathway: - _______ bind to antigens, activating ____ (fixing it because it was inactive). - C1 splits and activates ______ and ____. - ______ and ______ combine and activate C3. * C3a functions in ________. * C3b functions in _____ and ______.

- antibodies/ C1 - C2 and C4 - C2a and C4b * inflammation * cytolysis and opsonization

Antigen-Antibody Binding and Its Results: - An ___________ forms when antibodies bind to antigens. * strength of the bond is the _______ (NOT a covalent bond). * protects the host by ______ foreign molecules or cells for destruction. * 5 outcomes can occur: ______, _____, ____, _____, or _______.

- antigen-antibody complex * affinity * tagging * agglutination, opsonization, antibody-dependent cell-mediated cytotoxicity, neutralization, and activation of the complement system.

- ________: substances that cause the production of antibodies. * usually components of invading microbes or foreign substances. * antibodies interact with ________, or _________, on the antigen. - ________: antigens too small to provoke immune responses; attach to carrier molecules. - PICTURE: * polyclonal! * _______ antibodies are the best for diagnostic tests.

- antigens * epitopes/ antigenic determinants - haptens * monoclonal

Heavy Metals and their Compounds: - Heavy metals are ________ agents. - ____________: very small amounts exert antimicrobial activity. - _________ proteins. - Ag, Hg, Cu, and Zn * ________ is used to prevent ophthalmia neonatorum (newborn) * __________ prevents mildew in paint. * _________ is an algicide * ________ is found in mouthwash

- antimicrobial - oligodynamic action - denature * silver nitrate * mercuric chloride * copper sulfate * zinc chloride

Types of Adaptive Immunity: - _________: blood-derived fluids containing antibodies. - _________: the study of reactions between antibodies and antigens (ie they study the 5 outcomes of Abs ..... neutralization .... etc.). - _________: serum proteins. * ________: serum fraction containing antibodies.

- antiserum - serology - globulins * Gamma globulin

- What are the steps of gram staining? - What does a mordant do?

- application of crystal violet (purple dye) - application of iodine (mordant) - alcohol wash (decolonization) - application of safranin (counterstain) - seals color of g+ to the surface of the cells.

Safety of Vaccines: - On rare occasions, vaccines can cause the disease (live attenuated vaccines) - No medical or scientific proof of MMR vaccines being linked to _______. - Safest and most effective means of preventing infectious disease in children.

- autism

Shapes of Bacteria: - Include? * rod-shaped = * spherical = * spiral = * what differentiates spirillum from spirochete? - Most bacteria are spherical because it requires the _________ to maintain the shape.

- bacillus, coccus, spiral, star-shaped, or rectangular * bacillus * coccus * vibrio, spirillum, and spirochete * spirochetes have an endoflagella - least amount of energy

Endotoxins: - Toxins are part of the ________ (like the cell wall). - Associated with _______ bacteria. - _______ portion of lipopolysaccharides (LPS) of gram-negative bacteria (antigen = we make antibodies against this). - Released during bacterial _________ and when gram-negative bacteria ________. * stimulates macrophages to release _________. * cause disseminated intravascular _________. * bacteria is killed = fragments of LPS are released into the body = amplify the immune system = leading to _________ and septic shock. - Example of Gram - bacteria = Salmonella typhimurium.

- bacteria - Gram - - Lipid A - multiplication/ die * cytokines * coagulation * antibody storm

What are the types of microorganisms (kingdoms)?

- bacteria, archaea, fungi, protozoa, algae, viruses, and multicellular animal parasites

Viruses and Bacteria Compared (yes or no): - Bacteria: * intracellular parasite: ______ * plasma membrane: ________ * binary fission: ________ * pass through bacteriological filters: ________ * possess both DNA and RNA: ________ * ATP-Generating metabolism: ________ * ribosomes: ________ * sensitive to antibiotics: ________ * sensitive to interferon: ________ - Viruses: * intracellular parasite: ______ * plasma membrane: ________ * binary fission: ________ * pass through bacteriological filters: ________ * possess both DNA and RNA: ________ * ATP-Generating metabolism: ________ * ribosomes: ________ * sensitive to antibiotics: ________ * sensitive to interferon: ________ - why would a doctor give antibiotics with viral infections (even though they will not work against the virus)? _____________ - Viruses on their own don't have plasma membranes, but can contain an outer plasma membrane known as an __________ from a host cell.

- bacteria: * no * yes * yes * no * yes * yes * yes * yes * no - viruses: * yes * no * no * yes * no * no * no * no * yes - to prevent secondary infections (bacterial) - envelope

Bacterial Division: - Also known as ________ - Increase in _______ of cells, not cell ______. - Uses ________. - Budding (yeasts and molds) - Conidiospores (actinomycetes) - Fragmentation of filaments

- bacterial growth - number/ size - binary fission

Antibiotics: - ___________: proteins produced by one bacterium that inhibits another. - ______ and ________ prevent spoilage of cheese.

- bacteriocins - nisin and natamycin

- _________: is the study of bacteria. - ________: is the study of fungi. - _______: is the study of protozoa and parasitic worms.

- bacteriology - mycology - parasitology

Scanning Electron Microscopy: - An electron gun produces a _________ that scans the surface of an entire specimen. - Secondary electrons emitted from the specimen produce a ______ image. - Magnifies objects _____ to _______; resolution of ______. - Sweep across the entire specimen.

- beam of electrons - 3D - 1000 - 10000x/ 10 nm

Taxonomy of Viruses: - _________ is used to name viruses. - ___% of our genome is viral. Viruses can replicate so fast because they have segments (_____________) in their genome. - Genus names end in _______. - Family names end in ________. - Order names end in ______. - __________: a group of viruses sharing the same genetic information and ecological niche (host). * descriptive common names are used for _______. * subspecies are designated by a ________. - Coronavirus is in the same family as _______ and ______. - Best way to combat viral species is to ________.

- binomial nomenclature - 7/ (transposons) - -virus - -viridae - -ales - viral species * species * numbers - MERS and SARS - wash hands (not hand sanitizers).

Special Culture Techniques: - Used with what type of organisms? - Capnophiles: * microbes that require high _______ conditions. * what type methods achieve these conditions?

- capnophiles * CO2 * CO2 packet or candle jar

dsRNA Difference: - mRNA is produced inside the ______ and released into the cytoplasm of the host. - RNA polymerase initiates production of _________. The mRNA and - strands form the dsRNA that is incorporated as new viral genome.

- capsid - - strands

Indicate the bacterial structures that are likely to be antigens, to which host antibodies bind, marking the invader for phagocytosis: nucleoid capsule flagella cell wall ribosomes plasmids fimbriae

- capsule/ flagella/ cell wall/ fimbriae

- __________: This viscous outer covering found in certain microorganisms helps pathogens evade the host's defenses by impairing phagocytosis. - ________: These structures contain substances that contribute to a pathogen's virulence; for example, M protein mediates microbial attachment to epithelial cells. - _________: These proteins contribute to a pathogen's virulence by, for example, forming and breaking down fibrin clots, breaking down connective proteins, and countering certain types of antibodies. - __________: This process allows pathogens to alter their surface antigens to avoid attack by antibodies produced by the immune system. - __________: These microbial surface proteins rearrange the host cell's actin filaments, allowing pathogens to enter and move in and between cells.

- capsules - cell wall components - enzymes - antigenic variation - invasins

- What are the chemical requirements for microbial growth?

- carbon, nitrogen, sulfur, phosphorus, trace elements, oxygen, and organic growth factors.

Koch's Postulates: - Koch's postulates are used to prove the ______ of an infectious disease. - Exceptions to Koch's postulates: * some pathogens can cause several ________. * some pathogens cause disease only in ________. * some microbes have never been _________.

- cause * disease conditions * humans * cultured

Glycocalyx: - External to the ______. - ______ and gelatinous. - Made of _______ and/or ______. - Two types: * ______: neatly organized and firmly attached. * _____: unorganized and loose. - Contribute to __________: * capsules prevent _________. * extracellular polymeric substance helps form ______.

- cell wall (peptidoglycan) - viscous - polysaccharide/ polypeptide * capsule * slime layer - virulence * phagocytosis (hid from immune system) * biofilms

Culture Media: - ____________: exact chemical composition is known. * ________ organisms are those that require many growth factors provided in this type of media. * used in the growth of ________ and _______; microbiological assays. - __________: made from parts of yeasts, meat, or plants; chemical composition varies batch to batch (on the EXAM what should you look for to tell that it is this type of media?) * examples? * used in the growth of most __________ organisms.

- chemically defined media * fastidious * chemoautotrophs and photoautotrophs - complex media (extracts and digests) * nutrient broth and nutrient agar * chemoheterotrophic

The Mechanisms of Phagocytosis: - _________: chemical signals attract phagocytes to microorganisms * example: cytokines = produce a chemical gradient (signaling) - _________: attachment of a phagocyte to the surface of the microorganism. * exception: ________ bacteria: hard for macrophages to attach too. - _________ (phagocytosis): * __________: microorganism is coated with serum proteins (outcome of complement), making phagocytosis easier (increases ability of macrophages to engulf). - _________: broken down. * microorganism is digested inside a _____________.

- chemotaxis - adherence * capsulated - ingestion * opsonization - digestion * phagolysosome

The Nucleoid: - Bacterial _________ (nucleoid): _____ thread of DNA that contains the cell's genetic information. - ________: extrachromosomal genetic elements; carry non-crucial genes that help with ________ and ________.

- chromosome/ circular, singular - plasmids/ antibiotic resistance and production of toxins

Classifying Infectious Diseases: - __________: a disease that is spread from one host to another. EXAMPLE: flu - __________: diseases that are easily and rapidly spread from one host to another (extremely communicable). EXAMPLE: measles - __________: a disease that is not spread from one host to another. EXAMPLE: cancer.

- communicable disease - contagious disease - noncommunicable disease

Biofilms: - Microbial ____________. - Form ______ or ________ that adhere to surfaces. * bacteria communicate cell-to-cell via __________. - Share ________. - Shelter bacteria from _______ environmental factors. - Found in ________ and sewage treatment systems; can clog pipes. - _______ resistant to microbicides. - Involved in _____% of infections * Examples of places that can get infections?

- communties - slime or hydrogels * quorum sensing - nutrients - harmful - digestive system - 1000x - 70 * catheters, heart valves, contact lenses, and dental caries

- What are the factors that are related to effective disinfection?

- concentration of disinfectant, organic matter, pH, and time

Reservoirs of Infection: - Reservoirs: ________ sources of infection. Reservoir for coronavirus = bats/ increased % of humans are carriers of TB. * Human reservoirs: ________ may have inapparent or latent diseases. * Animal reservoirs: _______ are diseases transmitted from animals to humans (rabies). * Nonliving reservoirs: _____ and ____.

- continual * carriers * zoonoses * soil and water

Aldehydes: - Inactivate proteins by __________ with functional groups (NH2, OH, COOH, SH) = prevents 3D structure = can't function. - Used for preserving specimens and in ___________. - Formaldehyde and ortho-phthalaldehyde - _________ is one of the few liquid chemical sterilizing agents. - LOOK AT PICTURE OF BASIC ALDEHYDE STRUCTURE.

- cross-linking - medical equipment - glutaraldehyde

Cell Walls and the Gram Stain Mechanism: - ________ crystals form inside cell. - Gram-positive: * _______ dehydrates peptidoglycan * CV-I crystals ________. - Gram-negative: * alcohol dissolves _______ and leaves holes in ________. * CV-I _______; cells are colorless. * ______ added to stain cells. - What are the four steps of the staining process?

- crystal violet-iodine * alcohol * do not leave * outer membrane/ peptidoglycan * washes out * safranin - 1. crystal violet (basic purple dye) for 1 minute/ 2. mordant (iodine) = sticks dye to bacteria = 1 minute/ 3. decolorizer (alcohol) = seconds/ 4. counterstain (red safranin) = 1 minute.

Culture Media: - _________: nutrients prepared for microbial growth. - ________: no living microbes - ________: introduction of microbes into a medium (mixing). - _______: microbes growing in or on a culture medium.

- culture medium - sterile - inoculum or inoculation - culture

Inflammation: - Controlled by _______ release. - Four signs and symptoms: ______ (erythema), _______ = due to influx of WBCs, _______, and ________ - _______ injurious agent or limits its effects on the body. - ______ and _______ tissue damaged by the injurious agent. - Inflammation leads to ______ and increased ________.

- cytokine - redness, swelling (edema), pain, and heat - destroys - repairs or replaces - vasodilation/ permeability

- __________: are chemical messengers produced in response to a stimulus. * ___________: cytokines between leukocytes. * ___________: induce migration of leukocytes. * ___________: interfere with viral infections of host cells. * ___________: involved in the inflammation of autoimmune diseases. * ___________: control stem cells (the volume) that develop into red and white blood cells. - Overproduction of cytokines leads to a ___________.

- cytokines * interleukins * chemokines * interferons * tumor necrosis factor (TNF) * hematopoietic cytokines - cytokine storm

Outcomes of Complement Activation: - What are the 3 possible outcomes: ______, _______, and/or _______? - Cytolysis: activated complement proteins create a ________. * ______ inserts into the cell walls of bacteria, creating holes in the bacteria = cytolysis. - ________: promotes attachment of a phagocyte to a microbe. - Inflammation: activated complement proteins bind to _______, releasing histamine.

- cytolysis, opsonization, and/or inflammation - membrane attack complex (MAC). * C9 - opsonization - mast cells

Pathogenic Properties of Viruses: - __________; are visible effects of viral infection on a cell; used to identify a specific virus. - Example: * stopping cell synthesis * causing cell lysosomes to release enzymes * creating ________ in the cell cytoplasm. * fusing cells to create a ________. * changing host cell function or inducing chromosomal changes. * inducing antigenic changes on the cell surface. * loss of _______ in the cell, leading to cancer. * producing _______ to protect unaffected cells (class of cytokines that warns neighboring cells).

- cytopathic effects (CPE) * inclusion bodies * syncytium * contact inhibition * interferons

- _________ substance inside the plasma membrane and outside the nucleus. - ________: fluid portion of cytoplasm - ________: made of microfilaments and intermediate filaments; give _______ and _______. - ________: movement of the cytoplasm throughout a cell.

- cytoplasm - cytosol - cytoskeleton/ shape and support - cytoplasmic streaming

Preserving Bacterial Cultures: - What are two methods for preserving bacteria? * How do these methods preserve the bacteria? * Deep-freezing: ______ to _____ degrees C. * Lyophilization (freeze-drying): frozen (_______ to _______ degrees C) and _______ in a vacuum.

- deep-freezing or lyophilization (freeze-drying) * the lower temperatures cause metabolism and molecular vibrations to slow down. * -50 - -95 * (-54 - -72)/ dehydrated(desiccate)

Surface Active Agents: - Soap = ________; emulsification - Acid-anionic sanitizers = anions react with __________. - Quaternary ammonium compounds (quats) = cations are _________, _______ proteins, and disrupt ___________.

- degerming - plasma membrane - bactericidal, denature, plasma membranes

Why 70% ethanol instead of 95%: - Protein _________ is the mode of action in alcohol sterilization. Since pure ethyl alcohol is a dehydrating agent and water supports the denaturing of proteins, the proteins are denatured more readily by ethanol in the presence of _______. * if the percent is too high, the bacteria still grow because all of the proteins just _________ on the cells = cannot penetrate the plasma membrane to kill it.

- denaturation/ water * clump

Heat: - Heat ________ enzymes. - ______________: lowest TEMPERATURE at which all cells in a liquid culture are killed in 10 minutes (specific). - _____________: minimal TIME for all bacteria in a liquid culture to be killed at a particular temperature. - ______________ minutes to KILL 90% of a population at a given temperature (like in the microbial death curve).

- denatures - thermal death point (TDP) - thermal death time (TDT) - decimal reduction time (DRT)

Moist Heat Sterilization: - Moist heat ________ proteins (allows heat to penetrate the _________). *boiling * free-flowing steam - __________: steam under pressure. * specifications? (most vegetatively growing organisms will be killed at this) * kills all organisms and _________. * ______ must contact the item's surface/ the _________ allows for boiling without overflowing of liquid.

- denatures/ (plasma membrane) - autoclave * 121 C/ 15 psi/ 15 minutes * steam/ pressure

Antigen-Presenting Cells: - Include: _______, _______, and _______. - Dendritic cells (DCs): * engulf and degrade microbes and display them to ______. * found in the ______, genital tract, lymph nodes, spleen, thymus, and blood. - Macrophages: * activated by ________ or the ingestion of antigenic material. * migrate to the lymph tissue, presenting antigen to _______.

- dendritic cells, macrophages, and B cells. * T cells * skin * cytokines * T cells

Epidemiology: - ____________: collection and analysis of data ---> SNOW - ___________: analyzes a particular disease to determine its probable cause ---> NIGHTINGALE - ___________: involves a hypothesis and controlled experiments ---> SEMMELWEIS

- descriptive epidemiology - analytical epidemiology - experimental epidemiology

Arrangement of Coccus Bacterial Cells: - Pairs: - Clusters: - Chains: - Groups of four: - Cube-like groups of eight:

- diplo- - staphylo- - strepto- - tetrads - sarcinae

Enzyme-Linked Immunosorbent Assay (ELISA): - _________: only one antibody is used. * detects _______. * sample containing antigens is mixed with antibody. * enzyme-linked antibodies react with the antigen. * detected by adding a substrate for the linked enzyme; a _____ is produced. - _________: requires two antibodies: * detects _______.

- direct ELISA * antigens * color - indirect ELISA * antibodies

Contact Transmission: - ____________: requires close association between the infected and a susceptible host. - ____________: spreads to a host by a nonliving object called a ________ (like a door handle or medical tool). - ____________: transmission via airborne droplets less than 1 meter.

- direct contact transmission - indirect contact transmission/ fomite - droplet transmission

- ___________ for microbial growth: count microbial cells. * what are the four ways that can achieve this? - What are the three indirect methods for estimating bacterial numbers?

- direct measurements * plate count, filtration, most probable number (MPN) method, and direct microscopic count - turbidity, metabolic activity, and dry weight

- Differential Stains: used to _______ between bacteria. * what are the two types?

- distinguish * gram stain and acid-fast stain

Generation Time: - Time required for a cell to _________: _____ minutes to 24 hours. - Binary fission _______ the number of cells each generation. - Total number of cells = _________ - ________ are represented logarithmically or exponentially.

- divide/ 20 - doubles - 2^ number of generations - growth curves

Mitochondria: - ______ membrane. - Contain inner folds (________) and fluid (_______) - Involved in __________ (ATP production).

- double - (cristae)/ (matrix) - cellular respiration

Phagocytes: - Phago: from the Greek, meaning _____. - Cyte: from the Greek, meaning ______. - _________; are residents in tissues and organs (like in the lymph nodes). - ________; roam tissues and gather at sites of infection.

- eat - cell - fixed macrophages - free (wandering) macrophages

The Disk Diffusion Method: - Evaluates _______ of chemical agents. - Filter paper disks are soaked in a chemical and placed on a culture. - Look for the _________ around the disk, this indicates how well the chemical works against the microbe. * ie the ____________ = the better the chemical substance is. - cannot differentiate between bactericidal or bacteriostatic.

- efficacy - zone of inhibition * bigger halos

Electron Microscopy: - Uses _______ instead of light. - The shorter wavelength of electrons gives _________. - Used for images too small to be seen with light microscopes, such as _________.

- electrons - higher resolution - viruses

Axial Filaments: - Also called _________. - Found in _________. - Anchored at one end of a cell. - Rotation causes cell to move like a _________ because it is wrapped around the actual bacteria.

- endoflagella - spirochetes - corkscrew

- _________: encourages the growth of a desired microbe by increasing very small numbers of a desired organism to detectable levels. * usually a _______.

- enrichment culture * liquid

The Centers for Disease Control and Prevention (CDC): - Collects and analyzes _________ information in the United States. - Publishes ___________. * ________: incidence of a specific notifiable diseases. * ________: deaths from notifiable diseases.

- epidemiological - Morbidity and Mortality Weekly Report (MMWR) * morbidity * mortality

Algae Characteristics: - __________ - _________ cell walls - Found in freshwater, saltwater, and soil - Use _________ for energy - Produce ________ and __________. - ALL LIFE RESULTS FROM THESE GUYS! - __________; are the ancestral precursors for land plants. - A lot of algae are communal, meaning ________. - 6 CO2 + 6 H20 -> (sun) O2 + sugar! - Are green because chlorophyll A and B ______ all colors except green and _______ it.

- eukaryotes - cellulose - photosynthesis - oxygen and carbohydrates - spirogyra - numerous single cells working together in a colony. - absorb/ reflect

Fungi Characteristics: - _________ (distinct nucleus) - ______ cell walls - Absorb ______ chemicals for energy - _______ are unicellular examples - _______ and ________ are multicellular examples * Molds consist of masses of ________, which are composed of filaments called _______. - ________: mold that grows on wheat and bread (green color).

- eukaryotes - chitin - organic - yeasts - molds and mushrooms * mycelia/ hyphae - mucor

Multicellular Animal Parasites Characteristics: - __________ - Multicellular animals - Not strictly _________. - Parasitic flatworms and roundworms are called _________. - Have some microscopic stages in their _________.

- eukaryotes - microorganisms - helminths - life cycles

Protozoa (protists) Characteristics: - ___________ - Absorb or ingest _______ chemicals - May be motile via _________. - _________ or ________(derive nutrients from a living host) - Use _________ to digest food, and use amoeboid action = extensions known as _________. - ________ are major examples.

- eukaryotes - organic - pseudopods, cilia, or flagella - free-living or parasitic - phagocytosis/ pseudopods - amoeba

Passive Processes: - __________: solute combines with a transporter protein in the membrane. * transports _____ and _______ across a membrane with the concentration gradient. - _________; the movement of water across a selectively permeable membrane from an area of high water to an area of low water concentration. * through ______ layer/ ______ (water channels)

- facilitated diffusion * ions and larger molecules - osmosis * lipid/ aquaporins

Cilia are appendages that allow locomotion in bacteria: - true or false

- false

The avian influenza A virus is transmitted only between birds: - true or false

- false

Flagella: - _______ appendages external of the cell. - _______ bacteria. - Made of protein _______. - Movement is _______ driven. - Three parts: * ________: outermost region. * ________: attaches to the filament. * _______: consists of rod an pairs of rings; anchors flagellum to the cell wall and membrane. - Flagella allow bacteria to move toward or away from stimuli (________) - Flagella rotate to _____ or ______. - Flagella proteins are ________ and distinguish among _______ (e.g. Escherichia coli O157:H7) = (ie this is an ___________). - ________: foreign substances that cause the immune system to make antibodies.

- filamentous - propel - flagellin - ATP * filament * hook * basal body - (taxis) - run or tumble - H antigens/ serovars/ (antigenic flagella) - antigens

Filtration: - Solution passed through a ______ that collects bacteria. - Filter is transferred to a _______ and grows as colonies on the surface. - Used for bacteria that are in ________ samples (like E. coli in sewage). - Pore size is 2 micrometers, capturing most bacteria, but what won't be captured in this method?

- filter - Petri dish - water - viruses

1. ____________:used after initial focusing to sharpen the image 2. ____________:controls how much light from the illuminator reaches the specimen 3. _____________:used for initial focusing; should never be used when the high-power objective lens or oil immersion objective lens is in place 4. _____________:used to focus the light from the illuminator onto the slide 5. ____________:lens that is closest to the slide and provides initial magnification of a specimen 6. _____________:lens that you look through 7. ___________:platform on which the slide is placed for viewing

- fine focus knob - diaphragm - coarse focus knob - condenser lens - objective lens - ocular lens - stage

Fluorescent-Antibody Techniques: - ___________: laser beam strikes a droplet containing a cell. * detector determines size and fluorescence of surface molecules * imparts a ________ to the cell, separating cells.

- fluorescence-activated cell sorter (FACS) * charge

Fluorescent-Antibody Techniques: - Combine ________ dyes with antibodies - _________: identify a microorganism in a clinical specimen. - _________: detect a specific antibody in serum. * __________ is added and will react with any antibody in serum with any antibody in serum if the result is positive.

- fluorescent - direct FA tests - indirect FA tests * anti-human immune serum globulin (anti-HISG)

Two-Photon Microscopy: - Cells are stained with __________. - Two photons of __________ light are used to excite the dyes. - Can study living cells up to _____ deep.

- fluorochrome dyes - long-wavelength (red) - 1mm

Confocal Microscopy: - Cells are stained with __________. - ___________ light is used to excite a single plane of a specimen. - Each plane in a specimen is illuminated and a 3D image is constructed with a _________.

- fluorochrome dyes - short-wavelength (blue) - computer

- Psychrotrophs cause __________. - Thermophiles are found in ________ and _______.

- food spoilage - hot springs and organic compost

Direct Damage: - Disrupts host cell _________. - Uses host cell _________. - Produces _______ products. - Multiplies in host cells and causes ________.

- function - nutrients - waste - ruptures

Capsules: - _________ around the cell wall. - Impair __________ (hid from immune system): * Streptococcus pneumoniae: pneumonia * Haemophilus influenzae: pneumonia and meningitis * Bacillus anthracis: anthrax * Yersinia pestis: plague

- glycocalyx - phagocytosis

Gram Stains: - Classifies bacteria into _______ or _______. - Gram positive bacteria have thick ___________ - Gram negative bacteria have thin _________ and a layer of __________. - This staining method allows us to determine and differentiate = put in a category.

- gram-negative or gram-positive - peptidoglycan cell walls - peptidoglycan cell walls/ lipopolysaccharides

- coccus are usually _______. - rods are usually _______. * if flipped then they are pathogenic.

- gram-positive (purple) - gram-negative (red)

Tissue Repair: - Cannot be completed until all _____ substances are removed or neutralized. - ________ is the supporting connective tissue of the dermis that is repaired. - ________ is the functioning part of the epidermal tissue that is repaired.

- harmful - stroma - parenchyma

Healthcare-Associated Infections (HAIs): - Acquired while receiving treatment in a _________ facility. * also known as ____________. - Affect 1 in 25 hospital patients * 2 million per year infected; 20,000 deaths. - HAIs result from: * ________ in the hospital environment. * ________ status of the host * chain of _________ in a hospital. - ___________: an individual whose resistance to infection is impaired by disease, therapy, or burns. - Most __________ stains of bacteria are found in the hospital, & a lot of the species are _________.

- healthcare * nosocomial infections * microorganisms * weakened * transmission - compromised host - antibiotic resistant/ normal microbiota

- ___________: agglutination of RBC surface antigens and complementary antibodies; used for blood typing. * __________; occurs when viruses agglutinate RBCs without an antigen-antibody reaction. EXAMPLES: ______, ______, and ______.

- hemagglutination * viral hemagglutination/ mumps, measles, and influenza

- ___________: the spectrum of host cells a virus can infect. - Most viruses infect only ________ types of cells in one host (specificity). * viruses are identified by specific host ________ sites and cellular factors. EXAMPLE: HIV -> attaches to _________/ or ________ attaches to LDLr on hepatocytes. - __________: viruses that infect bacteria. * range from 20 nm to 1000 nm in length. * _______ for short. - What are the two ways to identify a virus: _______ and _______? - If a virus can't _______, it can't infect.

- host range - specific * attachment/ CD4+ (T helper cells)/ hepatitis C - bacteriophages * phage - specific place it infects/ signs and symptoms - attach

Dual Nature of the Adaptive Immune System: - What are to categories of adaptive immunity: _____ and _____? - Humoral Immunity: produces ________ that combat foreign molecules known as ________. * _______ are lymphocytes that are created for humoral immunity. * mature in the _________. * recognize antigens and make _________. * named for bursa of Fabricius in birds. * _______ are the actual cell that makes the antibodies. - Cellular Immunity (cell-mediated immunity): * produces ________ = lymphocytes for cell-mediated immunity. * recognize antigenic peptides processed by _______ cells. * mature in the ________. * ___________ on the T cell surface contact antigens, causing the T cells to secrete _________ instead of antibodies. - PICTURE SHOW DIFFERENTIATION OF T cells AND B cells!

- humoral and cell-mediated immunity - antibodies/ antigens * B cells * red bone marrow * antibodies * plasma cells * T lymphocytes * phagocytic * thymus * T cell receptors (TCRs)/ cytokines

Physical Requirement: Osmotic Pressure: - __________ environments (higher in solutes than inside the cell) cause ________ due to high osmotic pressure. - _________ or _________ require high osmotic pressure (high salt). - _________ tolerate high osmotic pressure. - _________: restricted to a particular function or mode of life. - _________: capable of but not restricted to a particular function or mode of life.

- hypertonic/ plasmolysis (imploding) - extreme and obligate halophiles - facultative halophiles - obligate - facultative

The Concept of Immunity: - ________: the ability to ward off disease. - ________: lack of resistance to a disease. - ________: defenses against any pathogen; rapid, present at birth. - ________: immunity or resistance to a specific pathogen; slower to respond, has memory component.

- immunity - susceptibility - innate immunity - adaptive immunity

Precipitation Reactions CONT: - ______________ (_________) are precipitation reactions carried out in an agar gel medium. * precipitate develops at the point where the optimal antigen-antibody ratio occurs. - ____________: combines electrophoresis with immunodiffusion. * separates proteins in human serum.

- immunodiffusion tests (Ouchterlony) - immunoelectrophoresis

Antibodies: - Globular proteins called _____________. - ________ is the number of antigen-binding sites on an antibody. * _______ antibodies have two binding sites (antibody in picture is an example).

- immunoglobulins (Ig) - valence * bivalent

Vehicle Transmission: - Transmission by an ______ reservoir. * ________ * _______ * _______ - These are NOT fomites!

- inanimate * waterborne * foodborne * airborne

Occurrence of a Disease: - _________; number of people who develop a disease during a particular time period. - _________; number of people who develop a disease at a specified time, regardless of when it first appeared (day to day). * takes into account both old and new cases.

- incidence - prevalence

Using the Host's Nutrients: - ______ damage. - ______ is required for most pathogenic bacteria. - _______ are proteins secreted by pathogens that bind iron more tightly than host cells (like sponges that suck up the ferric iron). - Bacterial: Fe3+ = ________ (look at picture --> structure of enterobactin, a type of siderophore) - Animal iron-binding molecules: * ________: serum (blood) * ________: cytoplasm * ________: sweat, tears, and milk

- indirect - iron - siderophores - ferric iron * transferrin * ferritin * lactoferrin

Agglutination Tests CONT: - ___________: antibody reacts with the soluble antigen adhering to the particles or vice versa. 1. reaction in a positive indirect test for ________: when particles (latex beads here) are coated with antigens, agglutination indicates the presence of antibodies, such as the IgM shown here. 2. reaction in a positive indirect test for ________: when particles are coated with monoclonal antibodies, agglutination indicates the presence of antigens.

- indirect (passive) agglutination tests 1. antibodies 2. antigens

A broth medium has been inoculated, and microbial numbers will be counted periodically to generate a bacterial growth curve. At 2 hours after inoculation, the culture has progressed through lag phase and is now in log phase. At this point, the population size is 1 million cells. The generation time is 30 minutes. Assuming the continuation of log growth, how many cells would there be at 2 hours of growth in log phase? a. 8 million b. 16 million c. 4 million d. 32 million e. 2 million

b. 16 million

- Numbers of Invading Microbes: - ID50: __________ for 50% of a sample population: * measures ________ of a microbe. - LD50: _________ for 50% of a sample population: * measures _______ of a toxin. - Both measure the virulence! - ______ the number given, the more virulent (ID50) or potent/lethal (LD50) the given microbe is. * example: Bacillus anthracis: skin = 10-50 endospores/ inhalation = 10000-20000 endospores/ ingestion = 250000-1000000 endospores (meaning skin is most virulent or lethal because it takes less of the microbe to cause disease or death) -------> ie PICK THE LITTLE NUMBER ON THE EXAM!

- infectious dose (microbe load that causes infection) * virulence - lethal dose (microbe load that causes death) * potency - lower

Microbial Evasion of Phagocytosis: - What are the six ways: _____? 1. Inhibit adherence: _______, capsules: * Streptococcus pyogenes, S. pneumoniae 2. Kill phagocytes: _________ * Staphylococcus aureus 3. Lyse phagocytes: __________ * Listeria monocytogenes 4. Escape phagosome: * Shigella and Rickettsia 5. Prevent phagosome-lysosome fusion: * _______ and _______. 6. Survive in phagolysosome: * Coxiella burnetii

- inhibit adherence, kill phagocytes, lyse phagocytes, escape phagosome, prevent phagosome-lysosome fusion, and survive in phagolysosome 1. M protein 2. leukocidins 3. membrane attach complex 5. * HIV and Mycobacterium tuberculosis

Identify antibacterial strategies that would likely be selectively toxic for bacteria: inhibition of peptidoglycan synthesis interfering with translation at 70s ribosomes interfering with lysosomal function inhibition of fimbriae synthesis inhibition of microtubule function

- inhibition of peptidoglycan synthesis/ interfering with translation at 70s ribosomes/ inhibition of fimbriae synthesis

An Overview of the Body's Defenses: - _______ immunity (nonspecific immunity): * ___________: intact skin, mucous membranes and their secretions, and normal microbiota. * ___________: phagocytes (such as neutrophils, eosinophils, dendritic cells, and macrophages), inflammation, fever, and antimicrobial substances. - Adaptive immunity: * __________: specialized lymphocytes (T cells and B cells), and antibodies.

- innate * first line of defense * second line of defense * third line of defense

Chlamydia trachomatis elementary bodies measure approximately 0.25 micrometers in diameter. What is this measurement expressed in nanometers (nm)? a. 2500 nm b. 250 nm c. 25,000 nm d. 25 nm e. 2.5 nm

b. 250 nm

Cellular Immunity Response Process: - T cells combat ________ pathogens. * mature in the _______. * __________ eliminates immature T cells. * migrate from the thymus to lymphoid tissue. * attach to antigens via __________.

- intracellular * thymus * thymic selection * T-cell receptors (TCRs)

Viral Multiplication: - For a virus to multiple: it must ________ a host cell and take over the host's ___________. - LOOK AT ONE-STEP GROWTH CURVE PICTURE * _________: the time the virus is inside the cells but cannot detect it (acclimation).

- invade/ metabolic machinery * eclipse period

Penetration into the Host Cell Cytoskeleton: - ________: surface proteins produced by bacteria that rearrange actin filaments of the cytoskeleton*** * cause membrane ________. - Use ______ to move from one cell to the next * examples: ________, __________, and ___________ (Salmonella tyhimurium = typhus is an example). *** cytoskeletal rearrangement is how cells _____ and how ______ cells engulf things ----> bacteria figure out this rearrangement to get into the cells.

- invasins * ruffling - actin * Shigella, Listeria, and Salmonella *** move/ phagocytic

A scientist is using sterilized river water as a growth medium for her bacterial cultures. The nutrient availability is very low in this water, and no additional nutrients have been added. What type of membrane transport are the bacteria growing in this media most likely to be using to obtain nutrients? a. Pinocytosis b. Active transport c. Facilitated diffusion d. Simple diffusion e. Osmosis

b. Active transport

Penicillin was discovered by accident by a. Edward Jenner. b. Alexander Fleming. c. Joseph Lister. d. Robert Koch. e. Paul Ehrlich.

b. Alexander Fleming.

Radiation (kills cells): - ___________: X-rays, gamma rays, and electron beams. * ionizes water to create reactive ____________. * damages ______ by causing lethal mutations. * strips off _______ from the atoms, causing free radical to form. * used to sterilize _______. - __________: UV, 260 nm * damages DNA by creating ___________ = results in damaged transcription and translation. - ___________; kill by heat; not especially ___________ (some microorganisms can survive). * impact covalent bonds in _______ = causes vibrations. - __________ wavelengths = higher energy (explain).

- ionizing radiation * hydroxyl radicals * DNA * charges/electrons * Petri dishes - non ionizing radiation * thymine dimers - microwaves/ antimicrobial * water - shorter/ (gamma rays have shorter wavelength than microwaves = just look at how much more DAMAGE gamma rays cause)

Which of the following statements is NOT true for the indirect ELISA? a. It is important to wash off unbound molecules between steps. b. An antibody sandwich is created. c. It can detect antibody in patient serum. d. Microtiter plate wells are first coated with antigen. e. If testing human serum, labeled antiglobulins directed against human antibodies should be used.

b. An antibody sandwich is created.

Which answer is NOT true for adherence of a phagocyte to a microbe? a. Complement molecules attached to the microbe can enhance adherence. b. Antibody molecules attached to the microbe will limit adherence. c. Adherence is a critical step in phagocytosis. d. The M protein of Streptococcus pyogenes limits adherence. e. A capsule limits adherence.

b. Antibody molecules attached to the microbe will limit adherence.

- __________: solute concentrations equal inside and outside of cell; water is at equilibrium. - _________: solute concentration is lower outside than inside the cell; water moves into cell, causing ___________. - _________: solute concentration is higher outside of cell than inside; water moves out of cell, causing _________. - ________: the pressure needed to stop the movement of water across the membrane. * what environment were meats placed in for preservation before refrigerators?

- isotonic solution - hypotonic solution/ osmotic lysis (cell bursts) - hypertonic solution/ plasmolysis (imploding/ cell wall stay intact) - osmotic pressure * hypertonic

Phase of Growth: - What are the phases of bacterial growth? - _______: intense activity preparing for population growth, but no increase in population. - _______: exponential increases in population. - ______: period of equilibrium; microbial deaths balance production of new cells (why does this occur?). - _______: population is decreasing at a logarithmic rate. - What are the two phases that use logs?

- lag phase, log phase, stationary phase, and death phase - lag phase - log phase - stationary phase/ (nutrients are used up/ no space/ and wastes toxify the environment) - death phase - log growth and log death

- ______ viruses remain in asymptomatic host cells for long periods (not _______). * may reactivate due to changes in ________. * examples: _______, ______, or ______. - A ________ occurs gradually over a long period; is generally _______. * examples: ________ and _________ - Both of these are _______ infections.

- latent/ (fatal) * immunity * cold sores, shingles, and genital herpes - persistent viral infection/ fatal * subacute sclerosing panencephalitis (measles virus) and HIV - chronic

- __________: any kind of microscope that uses visible light to observe specimens. - Types include?

- light microscope - compound light microscope/ dark field microscopy/ phase-contrast microscope/ differential interference contrast (DIC) microscopy/ fluorescence microscopy/ and confocal microscopy

Darkfield Microscopy: - ______ objects are visible against a _______ background. - ________ placed in condenser. - Only light is reflected off the ________ and enters the objective lens.

- light/ dark - opaque disk - specimen

- Which of the lines best depicts the log phase of a thermophile incubated at room temperature? - Which of the lines best depicts the log phase of Listeria monocytogenes growing in a human

- line C - line A

Phenol and Phenolics: - Injure or target the _______ of ____________, causing leakage. - ________: contain two phenol groups connected by a bridge (hexachlorophene and triclosan) * disrupts the ___________.

- lipids of plasma membranes - bisphenol * plasma membranes

You are performing an agglutination test to detect the enzyme streptolysin in a patient's serum. To do this, what would you mix the serum sample with? a. Streptococcus bacteria b. Antibody-coated latex spheres c. Anti-Streptococcus antibodies d. Antigen-coated latex spheres

b. Antibody-coated latex spheres

Growing Bacteriophages in the Laboratory: - Viruses must be grown in ______ cells. - Bacteriophages are grown in ________. * Bacteriophages form _______, which are clearings on a lawn of bacteria on the surface of agar (infects the bacteria -> lyse and kill it) - each plaque corresponds to a single virus, can be expressed as ____________.

- living - bacteria * plaques - plaque-forming units (PFU)

Which of these complement proteins binds to the surface of microbes and enhances phagocytosis in a process termed opsonization? a. C1 b. C3b c. C9 d. C5

b. C3b

Which of the following destroys virus-infected cells? a. TH b. CTL c. dendritic cells d. Treg e. B cells

b. CTL

How can capsules enable bacteria to evade the immune system? a. A capsule is a superantigen that distracts the immune system. b. Capsules block the complement biding sites on the surface of the pathogen. c. Capsules can bind up IgA, rendering it inactive.

b. Capsules block the complement biding sites on the surface of the pathogen.

A chemostat is continuous culture system that is designed to promote and prolong exponential growth and prevent bacteria from entering stationary phase. How might this work? a. Chemostats promote the formation of endospores, which enable the long-term survival of the bacteria. b. Chemostats provide a continued source of fresh nutrients and remove wastes and dead bacterial cells. c. Chemostats include growth-promoting agents that prolong microbial growth. d. Chemostats include detoxifying agents that inactivate growth-inhibiting toxins that develop during continued microbial growth.

b. Chemostats provide a continued source of fresh nutrients and remove wastes and dead bacterial cells.

Which of the following statements about biofilms is FALSE? a. Biofilms in your body protect mucous membranes from harmful microbes. b. Compared to free-living bacteria, biofilms are more sensitive to antibiotics. c. Biofilms in pipes can block the flow of water. d. Biofilms on rocks provide food for animal life. e. Biofilms on medical devices cause infections.

b. Compared to free-living bacteria, biofilms are more sensitive to antibiotics.

Women who have a healthy population of Lactobacillus spp. as part of the normal vaginal microbiota are less likely to get yeast infections. Which of the following terms is used to explain this observation? a. Commensalism b. Competitive exclusion c. Parasitism d. Mutualism e. Synergism

b. Competitive exclusion

Growing Animal Viruses in the Laboratory: - In _______ animals. - In ___________ eggs. * virus injected into the egg. * viral growth is signaled by _______ or ______ of the embryo. - In ____ cultures. * _______ are treated with enzymes to separate cells (have to know what cell type the virus infects to do this though!). * virally infected cells are detached via their deterioration; known as the __________. * ____________ are used. * cancer or viral infected cells make masses and divide rapidly instead of monolayers of cells (looses _________). - Viral Identification: doctors look at ________ effects of the virus. * if a virus affects cytokinesis of a cell, they can't divide = result in ________.

- living - embryonated * changes or death - cell * tissues * cytopathic effect (CPE) * continuous cell lines * (contact inhibition) - cytopathic * multinucleate cells

Extent of Host Involvement: - __________; pathogens are limited to a small area of the body. - __________; an infection throughout the body. - __________; systemic infection that began as a local infection. - __________; toxic inflammatory condition arising from the spread of microbes, especially bacteria or their toxins, from a focus of infection. - __________; bacteria in the blood (just present and circulating). - __________; also known as blood poisoning; growth of bacteria in the blood (actively multiplying/metabolizing and producing toxins).

- local infection - systemic (generalized) infection - focal infection - sepsis - bacteremia - septicemia

Flagella and Cilia: - Projections used for _______ or moving substances along the cell surface. - ________: long projections; few in number - ________: short projections; numerous. - Both consist of _______ made of the protein _______. * microtubules are organized as _____ pairs in a ring, plus ______ microtubules in the center (______ array). - Allow flagella to move in a _______ manner.

- locomotion - flagella - cilia - microtubules/ tubulin * nine/ two/ (9+2) - wavelike

Physical Methods for Microbial Control: - _________ has a bacteriostatic effects. * examples: refrigerator, deep-freezing, and _________ (freeze drying). - _________ denatures proteins. - _________: absence of water prevents metabolism. - __________: uses salts and sugars to create hypertonic environment; causes ________ of the cell.

- low temperatures * lyophilization - high pressure - desiccation - osmotic pressure/ plasmolysis

Terms: - _________: breaking down of the membrane of a cell, often by viral, enzymic, or osmotic mechanisms that compromise its integrity (mechanical). - ________: occurs when a cell bursts due to an osmotic imbalance that has caused excess water to move into the cell. - ________: process in which cells lose water in a hypertonic solution, plasma membrane pulls away from cell wall (plants and bacteria).

- lysis - cytolysis - plasmolysis

Organelles: - __________: vesicles formed in the Golgi complex/ contain digestive enzymes. - __________: cavities in the cell formed from the Golgi complex/ bring food into cells; provide shape and storage. - _________: oxidize fatty acids; destroy H2O2. - _________: networks of protein fibers and centrioles/ form the mitotic spindle; crucial role in cell division.

- lysosomes - vacuoles - peroxisomes - centrosomes

Damage to Cell Wall: - _______ hydrolyzes bonds in peptidoglycan (in tears, sweat, and saliva). - ______ inhibits peptide bridges in peptidoglycan - ________ is a wall-less gram-positive cell. - ________ is a wall-less gram-negative cell. * both other the previous ones are susceptible to ________. - ______ are wall-less cells that swell into irregular shapes.

- lysozyme - penicillin - protoplast - spheroplast * osmotic lysis - L forms

Which of the following is NOT characteristic of the glycocalyx found in bacteria? a. A structure that can be visualized by negative staining b. Creates a slimy, slippery coating that prevents bacteria form attaching to surfaces c. If firmly attached, contributes to bacterial virulence d. A viscous coating surrounding the cell made of polysaccharide, polypeptide, or both

b. Creates a slimy, slippery coating that prevents bacteria form attaching to surfaces

Multiplication of Bacteriophages: - What are the two types it could be ________ and ________? - Lytic Cycle: * phage causes lyse and ________ of the host cell. - Lysogenic Cycle: * phage DNA is incorporated in the host DNA is a process known as __________. * specialized __________ occurs. - VIRUSES CAN CHANGE BACK AND FORTH BETWEEN THE TWO CYCLES. - ***** The lifecycle difference between bacteriophages and animal viruses is the _______ step that animal viruses have because the entire virus enters the cell through _________ (this uncoating utilizes enzymes).

- lytic cycle or lysogenic cycle * death * phage conversion * transduction - uncoating/ endocytosis

Clonal Selection of Antibody-Producing Cells: - ____________ genes encode molecules on the cell surface. * _______ are on the membrane of nucleated animal cells - identify self by presenting ________ protein fragments. * _______ are on the surface of antigen-presenting cells (APCs): including ______, ______, and _______.

- major histocompatibility complex (MHC) * Class I MHC - endogenous * Class II MHC/ B cells, macrophages, and dendritic cells.

Phagocyte Migration and Phagocytosis: - _________ is the sticking of phagocytes to blood vessels in response to cytokines at the site of inflammation ---> results in _________ and increased ________. - Phagocytes squeeze between endothelial cells of blood vessels via _________.

- margination/ vasodilation and increased permeability - diapedesis

Actions (Effects) of Microbial Control Agents on Cellular Structures: - Alteration of ___________. - Damage to ______ (enzymes). - Damage to _________.

- membrane permeability - proteins - nucleic acids

Exotoxins: - ___________: lyse host cells by disrupting plasma membranes. * __________: kill phagocytic leukocytes. * _________: kill erythrocytes by forming protein channels. * _________: hemolysins produced by streptococci.

- membrane-disrupting toxins * leukocidins * hemolysins * streptolysins

Inclusions: - ___________: phosphate reserves. - ___________ (3): energy reserves - ___________: RuBisCO enzyme for CO2 fixation during photosynthesis. - __________: protein-covered cylinders that maintain buoyancy. - __________: iron oxide inclusions; destroy H2O2 * doesn't constitute as a ________.

- metachromatic granules (volutin) - polysaccharide granules, lipid inclusions, and sulfur granules - carboxysomes - gas vacuoles - magnetosomes * organelle

Normal Microbiota and Innate Immunity: - Normal microbiota compete with pathogens via __________. * produce substances _______ to pathogens. * alter conditions that affect pathogen survival. * take all of the ________. - _________: one organisms benefits while the other (host) is unharmed. - ________: live microbial cultures administered to exert a beneficial effect.

- microbial antagonism * harmful * nutrients - commensalism - probiotics

Relationships between the Normal Microbiota and the Host: - ____________: is the competition between microbes. - Normal microbiota protect the host by: * competing for ________. * producing substances _________ to invading microbes. * affecting ______ and available ________.

- microbial antagonism (competitive exclusion) * nutrients * harmful * pH/ oxygen

Recombinant DNA Technology: - __________: the study of how microbes inherit traits. - _________: the study of how DNA directs protein synthesis. - _________: the study of an organism's genes; has provided new tools for classifying microorganisms. - _________: DNA made from two different sources. * In the 1960s, __________ inserted animal DNA into bacterial DNA, and the bacteria produced an animal protein.

- microbial genetics - molecular biology - genomics - recombinant DNA * Paul Berg

Cellular Immunity Response Process: - Pathogens entering the gastrointestinal tract pass through _________ located over _________. * transfer antigens to lymphocytes and antigen presenting cells (APCs).

- microfold cells (M cells)/ peyer's patches

- Microorganisms are measured in ___________ and _________. - List the metric system units!

- micrometers (um) and nanometers (nm) - Exa: E: 10^18/ Peta: P: 10^15/ Tera: T: 10^12/ Giga: G: 10^9/ Mega: M: 10^6/ Kilo: k: 10^3/ Hecto: h: 10^2/ Deka: da: 10^1/ deci: d: 10^-1/ centi: c: 10^-2/ milli: m: 10^-3/ micro: u: 10^-6/ nano: n: 10^-9/ pico: p: 10^-12/ femto: f: 10^-15/ atto: A: 10^-18

Which of the following is NOT a step in bacterial cell division? a. Splitting apart of two new daughter cells b. Disappearance of nuclear envelope c. Cell elongation d. Replication of the genetic material

b. Disappearance of nuclear envelope

Some microbes are very resistant to antimicrobial chemicals. Which of the following would be the easiest to kill? a. Pseudomonas aeruginosa b. E. coli c. Cysts of protozoa d. Clostridium endospores e. Mycobacterium tuberculosis

b. E. coli

You conduct a Limulus amebocyte lysate (LAL) assay on a sample of fluid that should be sterile. The result is positive. What does this indicate? a. The fluid is not sterile. b. Endotoxin is present. c. Gram-negative bacteria are growing. d. Gram-positive bacteria are growing in the sample.

b. Endotoxin is present.

Which of the following is the most effective for sterilization mattresses and plastic petri dishes? a. Chlorine b. Ethylene oxide c. Glutaraldehyde d. Autoclaving e. Nonionization radiation

b. Ethylene oxide

Physical Requirements Temperature: - What are the 3 cardinal temperatures to consider for each bacteria? - ___________: cold-loving bacteria. - __________: grow between 0 and 20-30 degrees C. Cause food spoilage. - _________: moderate-temperature-loving bacteria (what is special about these?) * How does the innate immune system help kill bacteria? - _________: heat-loving/ optimum growth temperature of 50-60 degrees C/ found in hot springs and organic compost. - ________: optimum growth temperature above 80 degrees C.

- minimum growth temperature, optimum growth temperature, and maximum growth temperature - psychrophiles - psychrotrophs - mesophiles (this is around body temperature = bacteria of the body fall in this category = physicians are most concerned with these) * fever = slow down metabolism of bacteria = fight it! - thermophiles - hyperthermophiles

3 Cardinal Temperatures: - First, what are the 3 cardinal temperatures? - Psychophiles = ? - Psychotrophs = ? - Mesophiles = ? - Thermophiles = ? - Hyperthermophiles = ?

- minimum growth temperature, optimum growth temperature, and maximum growth temperature ALL OF THE ANSWERS ARE IN CELSIUS - -7/ 12/ 18 (can survive in when water freezes and in refrigerator) - 0/ 22/ 31 (can survive in refrigerator and at room temperature) - 10/37/ 48 (can survive at room temperature and body temperature) - 40/61/ 72 - 66/ 92/ 110

Monoclonal Antibodies: - ____________: Mabs made from mouse proteins (end in _______). - ___________: Mabs with mouse variable region and human constant region (end in ________). - ___________: Mabs that are mostly human, except for mouse antigen-binding sites (end in _______). - ___________: Mabs produced from a human gene in a mouse (end in _______).

- murine monoclonal antibodies (-omab) - chimeric monoclonal antibodies (-ximab) - humanized antibodies (-zumab) - fully human antibodies (-umab)

Extracellular Killing by the Immune System: - Use _______ cells for extraceullar killing. - Natural Killer (NK) Cells: * granular leukocytes that destroy cells that don't express ________ self-antigens. * kill ________ and ______ cells and attack parasites. * not always stimulated by an ______ (but still attack cells). * form pores in the target cell, leading to _____ or ______.

- natural killer (NK) cells * MHC class I * virus-infected and tumor * antigen * lysis or apoptosis

Before cells are stained, the specimen must be fixed. Fixing a specimen accomplishes all of the following EXCEPT which one? a. Fixation kills the cells that are being stained for observation. b. Fixation differentiates the various cells being observed. c. Fixation attaches cells to the slide so that they will not wash off during the staining process. d. Fixation preserves cellular structure in the natural state, with minimum distortion.

b. Fixation differentiates the various cells being observed.

Types of Adaptive Immunity: - What are the four types: _____, ______, ______, and ______? - Naturally acquired active immunity: * resulting from _______ ---> body induces antibodies and specialized lymphocytes. - Naturally acquired passive immunity: * ________ or via _________ * antibodies pass from mother to fetus via ________ or to infant via the mother's ______. - Artificially acquired active immunity: * injection of _________; known as ________. * antigens are introduced in _______; body produces antibodies and specialized lymphocytes. - Artificially acquired passive immunity: * injection of ________ * ___________ in immune serum are introduced by injection. - PASSIVE: I MAKE IT! - ACTIVE: SOMEONE ELSE MAKES IT!

- naturally acquired active immunity, naturally acquired passive immunity, artificially acquired active immunity, and artificially acquired passive immunity. - * infection - * transplacental/ colostrum * placenta/ milk - * vaccination/ immunization * vaccines - * antibodies * performed antibodies (from someone else)

- ________ staining is used for capsules. - Capsules are important for bacteria because they? - Suspension of _______ or ________ contrasts the background within the capsule, which appears as a _____ around the cell.

- negative - camouflage the bacteria = eludes the immune system (covers proteins) = increases valiance (ability to cause disease) - India ink or nigrosin/ halo

- ___________: antigen-antibody reaction where the harmful effects of an exotoxin or a virus are blocked by antibodies to the toxin (________). - _____________ is used for sub typing viruses. * viruses and RBCs are mixed with the patient's serum. * if the serum contains antibodies to a virus, they neutralize the virus and inhibit hemagglutination.

- neutralization reaction/ (antitoxin) - viral hemagglutination inhibition test

Emerging Infectious Diseases: - Diseases that are ______, increasing in _______, or showing a _________ to increase in the near future. - Most are _______, or viral origin, and likely to be _______-borne.

- new/ incidence/ potential - zoonotic/ vector

Virus Characteristics: - _________ chemicals - ________ (not made of cells) = STRIKE - Consist of _____ or ______ core. - Core is surrounded by a ________ coat. - Coat may be enclosed in a __________. * those without = _______ * those with this, get it from the host cell they infect (take some of its ___________). - Are replicated only when they are in a living host cell (________________) * Some bacteria also have to do this (must reproduce inside the cell) = not differentiating between viruses and bacteria. EX. _______ or ______. - Insert outside living hosts (cannot _________ on their own) = STRIKE

- nonliving - acellular - DNA or RNA - protein - lipid envelope * naked * cell membrane - (obligate intracellular parasite)/ chlamydia or rickettsia - reproduce

Prions: - PrP^C: ______ cellular prion proteins, on the cell surface. - PrP^Sc: _______ protein; accumulates in _____ cells, forming _______ (missing brain tissue). - FIGURE: 1. PrP^C produced by cells is secreted to the cell surface. 2. PrP^Sc may be acquired or produced by an altered PrP^C gene. 3. PrP^Sc reacts with PrP^C on the cell surface. 4. PrP^Sc converts the PrP^C to PrP^Sc 5. The new PrP^Sc converts more PrP^C. 6. The new PrP^Sc is taken in, possibly by receptor-mediated endocytosis 7. PrP^Sc accumulates in endosomes. 8. PrP^Sc continues to accumulate as the endosome contents are transferred to lysosome. The result is cell death.

- normal - scrapie/ brain/ plaques

The Centers for Disease Control and Prevention (CDC): - ____________: diseases in which physicians are required to report occurrence. - ___________: number of people affected in relation to the total population in a given time period. - ___________: number of deaths from a disease in relation to the population in a given time. - There are nationally __________ diseases (have to be reported) = doctors must call CDC right away = population should know quickly!

- notifiable infectious diseases - morbidity rate - mortality rate - notifiable

The Biosynthesis of DNA Viruses: - Viruses can be either DNA or RNA (NOT BOTH). - DNA viruses replicate their DNA in the _______ of the host using viral enzymes. - Synthesize _______ in the cytoplasm using host cell enzymes. - _______ cycle because the cell still lives = ________ infections. - Each step of the replication process would be a target for ________ (example: target attachment = no infection will occur).

- nucleus - capsid - lysogenic/ chronic - antiviral drugs

Normal Microbiota: - Distribution and composition of normal microbiota are determined by many factors: _______, _________, ________, and _________.

- nutrients, physical and chemical factors, host defenses, and mechanical factors

Which of the following is true of microbes? a. 99% of all microbes are pathogenic. b. Gene expression in bacteria is very similar to gene expression in humans, which facilitates the use of bacteria in recombinant biotechnology and gene therapy. c. All bacterial enzymes are harmful to humans and the environment. d. Microbes create pollutants and toxins that harm the environment.

b. Gene expression in bacteria is very similar to gene expression in humans, which facilitates the use of bacteria in recombinant biotechnology and gene therapy.

Helicobacter pylori uses the enzyme urease to counteract a chemical defense in the human organ in which it lives. This chemical defense is: a. Lysozyme. b. Hydrochloric acid. c.Superoxide Radicals. d. Sebum. e. None of the above.

b. Hydrochloric acid.

Chemical Requirement: Oxygen: - ___________; require oxygen (what is the reason why humans need oxygen?) - ___________; grow via fermentation or anaerobic respiration when oxygen is not available. - ___________; unable to use oxygen and are harmed by it. - ___________; tolerate but cannot use oxygen. - ___________; require oxygen concentration lower than air. - GO OVER PRINTED PICTURE!!!!!!!

- obligate aerobes (ETC: oxygen takes up the extra electrons (free radicals) and makes water) - facultative anaerobes - obligate anaerobes - aerotolerant anaerobes - microaerophiles

General Characteristics of Viruses: - _____________: require living host cells to multiply. * Note: there are some bacteria that require living cells to multiply like _______ and ________, which means this overall idea is not a differentiation between bacteria and viruses. - Contain ______ or _______ (NOT BOTH). - Contain a _______ coat. - No _______ or _______ mechanisms.

- obligatory intracellular parasites * chlamydia and rickettsia - DNA or RNA - protein - ribosomes or ATP-generating

Which of these statements is true regarding portals of exit? a. The portal of exit for tuberculosis is the skin. b. In most cases, a microbe uses the same portal for both entry and exit. c. The urinary tract is the most common portal of exit. d. Polioviruses most often use the respiratory portal of exit.

b. In most cases, a microbe uses the same portal for both entry and exit.

- __________: remagnifies the image formed by the objective lens. - _________: transmits the image from the objective lens to the ocular lens. - _________: primary lenses that magnify the specimen. - ________: holds the microscope slides in position. - ________: focuses light through the specimen. - _________: controls the amount of light entering the condenser. - ________: light source.

- ocular lens - body tube - objective lenses - stage - condenser - diaphragm - illuminator

What is the correct name for the fluid that is collected from the body by lymphatic capillaries? a. Plasma b. Interstitial fluid c. Complement d. Blood

b. Interstitial fluid

Which of the following is most likely to be bactericidal? a. Membrane filtration b. Ionizing radiation c. Lyophilization (freeze-drying) d. Deep- freezing e. All of the above.

b. Ionizing radiation

Which of the following statements is NOT of the IgA antibody class? a. It is the most abundant antibody class in body secretions. b. It can trigger the complement cascade. c. It is a dimer in its most effective form. d. It prevents pathogens from attaching to mucosal surfaces. e. It can be found as a monomer in serum.

b. It can trigger the complement cascade.

What is the effect of alpha interferon on an uninfected cell? a. It causes the cell to activate complement. b. It causes the cell to produce antiviral proteins. c. It causes the cell to release histamines. d. It causes the cell to undergo chemotaxis.

b. It causes the cell to produce antiviral proteins.

Which of the following statements is NOT true of nitric oxide (NO)? a. It can be produced by blood vessel endothelial cells. b. It is of little value in killing microbes or tumor cells. c. It can be produced by macrophages that have been induced to produce NO synthase. d. It can cause relaxation of blood vessel smooth muscle. e. Excessive production can cause septic shock.

b. It is of little value in killing microbes or tumor cells.

Compound Light Microscopy: - In a compound microscope, the image from the objective lens is magnified again by the ____________. - _________ is the process of enlarging the appearance, not physical size, of something. - ___________ = objective lens x ocular lens - _________: area seen in the microscope. - _________: is the ability of the lenses to distinguish two points. * A microscope with a resolving power of 0.4 nm can distinguish between two points at least 0.4 nm apart. * __________ of light provide greater resolution (what colors have the greats resolution based on the color spectrum). - The __________ is a measure of the light-bending ability of a medium. * light may refract after passing through a specimen to an extent that it does not pass through the _____________ (effects __________). * _________ is used to keep light from refracting (used at high magnifications). This substance _______ light towards the objective lens.

- ocular lens - magnification - total magnification - field of view - resolution * shorter wavelengths (Y, I, G) - refractive index * objective lens/ (resolution) * immersion oil/ compresses

The Transformation of Normal Cells into Tumor Cells: - _________: transform normal cells into cancerous cells. - _________ become integrated into the host cell's DNA and induce tumors because of loss of contact inhibition. - A transformed cell harbors a __________ (only expressed if cancerous) on the surface and a ________ in the nucleus --> both of which tells the immune system that they need to be removed. - A lot of our genes are junk! * subsets mutate and cause cancer: example: mutations in the ______ genes are an oncogene that cause cancer.

- oncogenes - oncogenic viruses - tumor-specific transplant antigen (TSTA)/ T antigen * cyclin

- A simple microscope has _____ lens and was invented by ________.

- one/ Anton van Leeuwenhoek

Golgi Complex: - Transport ________. - Modifies _______ from the ER. - Transports modified proteins via _________ to the plasma membrane.

- organelle - proteins - secretory vesicles

The Cell Wall: - Prevents _______ and protects the cell membrane. - Made of ______ (in bacteria) - Contributes to ________.

- osmotic lysis - peptidoglycan - pathogenicity

Dry Heat Sterilization: - Kills by direct application of heat. - Kills with _________. - Example?

- oxidation - flaming, incineration, and hot-air sterilization (oven)

Antibody-Dependent Cell-Mediated Cytotoxicity: - Used in _______ infections. - _______ and ______ are too large to be phagocytized. * protozoan or helminth target cell is coated with _________. * immune system cells attach to the ______ regions of antibodies. * target cell is ______ by chemicals secreted by the immune system cells (release granzymes or perforins = digestive enzymes).

- parasitic - protozoans and helminths * antibodies * Fc * lysed

Special Stains: - Used to distinguish ______ of microorganisms. - What are the three types of special stains?

- parts - capsule stain, endospore stain, and flagella stain

The Movement of Materials Across Membranes: - __________: substances move from high concentration to low concentration; _______ expended (DIFFUSION). - _________: substances move from low concentration to high concentration; ______ expended.

- passive processes/ no energy - active processes/ energy

Heat: - __________: reduces spoilage organisms and pathogens, but is NOT a form of _____________. - Equivalent treatments: * ______ degrees C for 30 minutes. * _________________: ______ degrees C for 15 seconds (USA and Canada = 7-10 days). * _______________: ______ degrees C for 4 seconds (Europe = 3 months) - __________ organisms will survive pasteurization. - duric = _______.

- pasteurization/ sterilization * 63 * high-temperature short time (HTST): 72 * ultra-high temperature (UHT): 140 - thermoduric - durable

How Microorganisms Enter a Host: - __________; the ability to cause disease. - __________; the degree of pathogenicity. - Portals of Entry: * ___________ * ___________ * ___________ (other than the alimentary canal): deposited directly into _________ when barriers are penetrated. - Most pathogens have a ________ portal of entry.

- pathogenicity - virulence * mucous membranes * skin * parenteral route/ tissues - preferred

Pathology, Infection, and Disease: - __________: the study of disease. - __________: the cause of a disease. - __________: the development of disease. - __________: invasion or colonization of the body by pathogens. - __________: an abnormal state in which the body is not performing normal functions.

- pathology - etiology - pathogenesis - infection - disease

Gram-Negative Cell Walls: - _______ between the outer membrane and the plasma membrane contains peptidoglycan (______ layer). - Outer membrane made of _______, ________, and ________. - Protect from ______, ______, and _______. - Made of ____________ (is a strong antigen that causes ________). * _________ functions as antigens (antigenic LPS). * _______ is an endotoxin embedded in the top layer. - ________(proteins) form channels through membrane. - What happens if you destroy a single gram-negative bacteria into lots of pieces?

- periplasm/ (thin) - polysaccharides, lipoproteins, and phospholipids - phagocytes, complement, and antibiotics - lipopolysaccharide (LPS)/ (fever) * O polysaccharide * Lipid A - porins - antigenic storm = amplify immune response = shock (sepsis)

Flagella Types: - ___________; multiple flagella everywhere. - ___________; one flagella. - ___________; multiple flagella on polar end. - ___________; multiple flagella on both ends.

- peritrichous - monotrichous - lophotrichous - amphitrichous

The Plasma (cytoplasmic) Membrane: - __________ that encloses the cytoplasm. - ________ on the membrane surfaces. - ______ and ______ proteins penetrate the membrane. - These proteins are what the _________ looks for in order to identify the cells.

- phospholipid bilayer - peripheral proteins - integral and transmembrane - immune system

Chloroplasts: - Locations of __________. - Contain flattened membranes (________) that contain chlorophyll.

- photosynthesis - (thylakoids)

The Biosynthesis of RNA Viruses: - What are the four families of RNA viruses: ________? - Picornaviridae: * _____ stranded RNA, _______, non enveloped (naked) *________: poliovirus and coxsackievirus * _______: common cold (200 different viruses) .... different from flu because flu is a specific virus * ________ virus - Togaviridae: * _____ stranded RNA; _______, enveloped (coat) * _________: transmitted by arthropods; includes chikungunya * _________: rubella - Rhabdoviridae: * ______ stranded RNA; _________, one RNA strand * ________: rabies * numerous animal diseases - Reoviridae: * _________ RNA; non enveloped (few families of viruses contain dsRNA). * _______: respiratory enteric orphan * _______: mild respiratory infections and gastroenteritis (part of common cold).

- picornaviridae, togaviridae, rhabdoviridae, and reoviridae - * single/ + strand * Enterovirus * Rhinovirus * Hepatitis A - * single/ + strand * Alphavirus * Rubivirus - * single/ - strand * Lyssavirus - * double stranded * Reovirus * Rotavirus

Cytoplasm: - The substance inside the _______. - Eighty percent ______ plus proteins, carbohydrates, lipids, and ions. - Rigidity: _________ for eukaryotes and _________ for prokaryotes.

- plasma membrane - water - cytoskeleton/ cell wall

- ________: may carry genes for toxins, production of antibiotics, and enzymes. * NOT part of chromosome/ but create genes for toxins and antibiotic resistance. - _________; changes characteristics of a microbe due to incorporation of a prophage. * take up DNA from environment = new characteristics that they didn't have before. * example: _________.

- plasmids - lysogenic conversion * E. coli O157 H7

Most Probable Number (MPN) Method: - Multiple tube test - Count _______ tubes - Compare with a ________.

- positive - statistical table

Preparing Smears for Staining: - Stains consist of a _______ and _____ ion, one of which is colored (______). - In a ______, the chromophore is a cation (+). * note: the polar head of phospholipids has an intrinsic ________. - In an _______, the chromophore is an anion (-) - Staining the background instead of the cell is called ________. - Using an _____ dye with stain everything but the cell and using a ____ dye with usually stain the cells (like bacteria).

- positive and negative/ (chromophore) - basic dye * negative charge - acidic dye - negative staining - acidic/ basic

- ____________: reaction of soluble antigens with antibodies to form large, interlocking aggregates called _______. * involved in solutions! * _____________ forms, followed by the formation of a lattice that precipitates from solution. - ____________: a cloudy line forms where there is the optimal ratio of antigen and antibody.

- precipitation reaction/ lattices * antigen-antibody complex - precipitin ring test

Principles and Effects of Vaccination: - Provokes a _______ immune response. * leads to the formation of ______ and _______ cells. - Produces a rapid, intense _______ response. - _________: immunity in most of the population. * outbreaks are sporadic due to the lack of susceptible individuals.

- primary * antibodies and memory cells - secondary - herd immunity

- ________: infectious proteins (200-20 nm). - _________: infects E coli. - Viruses don't have name tags, but rather have _______ on their surface that are used to identify them.

- prions - bacteriophage T4 - spikes

Differential Interference Contrast (DIC) Microscopy: - Similar to phase-contrast - Uses two light beams and ________ to split light beams, giving more _____ and _____ to the specimen.

- prisms/ contrast and color

Antibodies: - Four ______ chains that form a Y shape. * two identical _______ chains and two identical ______ chains joined by ________ bonds. - _________ regions are at the ends of the arms; bind epitopes (specificity comes from this region = varies from epitope to epitope). - ________ region is the stem, which is identical for a particular Ig class. * five classes of Ig: ______, ______, ______, ______, and _______.

- protein * light/ heavy/ disulfide - variable (v) - constant (Fc) * IgG, IgM, IgA, IgD, and IgE

Ribosomes: - Sites of ________. - Made of _____ and ribosomal ______. - Prokaryotes = _______ (____S + _____S) - Eukaryotes = _______ (____S + ______S) * sedimentation coefficients in ________ units. * why are the S units important?

- protein synthesis - protein/ RNA - 70S (50/ 30) - 80S (60/ 40) * Svedberg * doctors exploit the difference in S units in order to target cells with protein synthesis inhibitors.

Prions: - _________ infectious particles because they folded wrong. - _______ and _______ by ingestion, transplant, and surgical instruments. - Called ________________ - Types of Spongiform encephalopathies: * ___________ * __________ * Gerstmann-Straussler-Scheinker syndrome * fatal familial insomnia * sheep scrapie - Prions are hard to get rid of = autoclaves even have a difficult time. * autoclave specifications: ______?

- proteinaceous (proteins) - inherited and transmissible - Spongiform encephalopathies * mad cow disease * Creutzfeldt-Jakob disease (CJD) * 134 degrees C/ 15 psi/ 30 minutes + NaOH

Alcohol: - Denature ________ and dissolve _______. - No effect on ________ and _________. - Ethanol and isopropanol: require ________.

- proteins/ lipids - endospores and non enveloped viruses - water

Obtaining Pure Cultures: - A _________ contains only one species or strain. - A _______ is a population of cells arising from a single cell or spore or from a group of attached cells. - A colony is often called a ___________. - The _________ is used to isolate pure cultures.

- pure culture - colony - colony-forming units (CFU) - streak plate method

The Lymphatic System: - Include: lymph, lymphatic vessels, lymphoid tissue, and _________. - Open system (unlike circulatory system) - Contains lymphocytes and ______ cells. - Lymph carries microbes to _______ where lymphocytes and macrophages destroy the pathogen (contain fixed macrophages = stay in these areas). - Contain one-way valves - ________: peyer's patches which filter blood of the small intestines.

- red bone marrow - phagocytic - lymph nodes - M cells

Control of Healthcare-Associated Infections: - ______ number of pathogens: * hand washing * _______ tubs used to bathe patients. * ______ instruments scrupulously. * using __________ bandages and intubation. - Infection control _________

- reduce * disinfecting * cleaning * disposable - committees

Anaerobic Growth Media and Methods: - Uses what? - Reducing media: * used for the cultivation of __________. * contains chemicals (_____________) that combine O2 to deplete it. * _______ to drive off O2.

- reducing media * anaerobic bacteria (obligate) * (sodium thioglycolate) * heated

Endospore Staining: - Endospores are _________, _______ structures inside some cells that cannot be stained by ordinary methods. - Endospores are NOT a _________, they are a ___________ = survival mechanism. - Primary stain: ________, usually with heat (forces color into spores) - Counterstain: ________ - Spores appear ______ within _____ or ______ cells. - What are two examples of endospores? - Endospores can be inside canned food -> bulging from gases trapped -> _______ interferes with germination of endospores.

- resistant, dormant (highly resistant to heat and antibiotics) - reproductive organs, time capsule - malachite green - safranin - green/ red or pink - clostridium and bacillus (anthrax) - nitrates

Portals of Exit: - Specific and preferred exits! - What are the five portals of exits: ________? - Respiratory tract: coughing and sneezing - Gastrointestinal tract: feces and saliva. - Genitourinary tract: urine; secretions from the penis and vagina. - Skin - Blood: arthropods that bite; needles or syringes.

- respiratory tract, gastrointestinal tract, genitourinary tract, skin, and blood

Endospores: - _____ cells; produced when ________ are depleted. - Very _______ to desiccation (_________), heat, chemicals, and radiation and antibiotics. - Produced by _______ and _______ (both soil bacteria). - __________; endospore formation. - _________; endospore returns to vegetative state (growing and dividing = make new bacteria) - NOT a _____________; rather a tie capsule. - Can all bacteria make these? - Important in food production because ________ interfere with germination processes.

- resting/ nutrients - resistant/ (removal of water) - Bacillus and Clostridium - sporulation - germination - reproductive organ - no - nitrates

- __________: are the sites of protein synthesis. - 80S: * consists of the large ______ subunit and the small _______ subunit. * membrane bound: attached to _______. * free: in ________. - 70S; * in _______ and ________.

- ribosomes * 60S/ 40S * ER * cytoplasm * chloroplasts and mitochondria

Select the structures below that are found in all bacterial cells. flagella ribosomes capsule cell wall plasma membrane nucleoid cytoplasm fimbriae pilus (pili)

- ribosomes/ plasma membrane/ nucleoid/ and cytoplasm

Filtration: - Passage of substances through a _________ material. - Used for ________ materials. - ___________: remove microbes > 0.3 micrometers. - _________: remove microbes > 0.22 micrometers/ these will not trap _______.

- screenlike - heat-sensitive - high-efficiency particulate air (HEPA) filters - membrane filters/ viruses

Chemical Factors: - _______ forms a protective film and lowers the pH (3-5) of skin. * most bacteria on the skin are _______ bacteria because they can tolerate the acidic environment. - _______ in perspiration, tears, saliva, and urine destroys bacterial cell walls - Low pH (1.2-3) of ________ destroys most bacteria and toxins. - Low pH (3-5) of __________ in females inhibit microbes.

- sebum * Gram + - lysozyme - gastric juice - vaginal secretions

Exotoxins: - Proteins produced and _______ by bacteria (exiting). * ________ in bodily fluids; destroy host cells and inhibit metabolic functions. * associated with _______ bacteria. - Treatments: * _______: antibodies against specific exotoxins. * _______: inactivated exotoxins used in vaccines (like tetanus shots). - Forms (4): * ________, ________, ________, and ________. - ________: an example of a Gram + bacterium that produces exotoxins.

- secreted * soluble * Gram + * antitoxins * toxoids * A-B toxins, membrane disrupting, superantigens, and genotoxins - Clostridium botulinum

Plasma Membrane Functions: - The plasma membrane's _________ allows the passage of some molecules, but not others. - Contains enzymes for _____ production. - Some membranes have photosynthetic pigments on foldings called __________. - Damage to the membrane by ______, _________, and _______ causes leakage of cell contents (disrupts its integrity). - The plasma membrane creates a barrier which makes ____________ = creates energy (DAM EXAMPLE). - Where do bacteria has ATPase (used to make ATP)?

- selective permeability - ATP - chromatophores - alcohols, quaternary ammonium (detergents), and polymyxin antibiotics - concentration gradients - on outer membrane

Diagnostic Immunology: - __________ (________): probability that a test is reactive if the specimen is a true positive. * extent to which true positives are not overlooked (so false negatives are few). * rarely overlooks the thing it is looking for. - _________ (________): probability that a test will NOT be reactive if a specimen is a true negative. * extent to which true negatives are not counted (so false positives are few). * rarely mistakes anything for that thing. - Immunologic-based diagnostic tests: * interactions of humoral antibodies with antigens. * known antibodies can identify an unknown pathogen. * known pathogen can identify an unknown antibody. * ________ are a great tool for these tests.

- sensitivity (true positive rate) - specificity (true negative rate) * antibodies

Bacillus anthracis can cause infection via three different portals of entry. The ID50 of cutaneous anthrax is 10 to 50 endospores, whereas inhalation anthrax requires 10,000 to 20,000 endospores, and gastrointestinal anthrax requires 250,000 to 1,000,000 endospores. Which statement best describes a conclusion that can be drawn based on this information? a. Exposure to a small number of endospores (fewer than 1000) is most likely to result in gastrointestinal anthrax. b. It is significantly easier to be infected with cutaneous anthrax as compared to other forms of anthrax. c. The portal of exit impacts the infectious dose needed to cause infection at the portal of entry. d. Cutaneous anthrax is rarely observed when cases across the population are monitored. e. Equal numbers of cutaneous, inhalation, and gastrointestinal anthrax infections are observed in human hosts.

b. It is significantly easier to be infected with cutaneous anthrax as compared to other forms of anthrax.

Which of the following does NOT accurately describe innate immunity? a. It includes defenses present at birth. b. It produces strong, long-lasting memory responses. c. It includes inflammation, fever, and phagocytosis. d. It includes the first and second lines of defense.

b. It produces strong, long-lasting memory responses.

The Terminology of Microbial Control: - ________: refers to bacterial contamination. - _______: is the absence of significant contamination. * aseptic surgery techniques prevent the microbial contamination of _______. - _______: removing and destroying all microbial life. - _______: killing C. botulinum endospores from canned goods. * what interferes with endospore formation? - _______: destroying harmful microorganisms (nonliving surfaces). - _______: destroying harmful microorganisms from living tissue. - _______: the mechanical removal of microbes from a limited area (reducing microbial count, not ____________) - ______: lowering microbial counts on eating utensils to safe levels. - ______: treatments that kill microbes. - ______: inhibiting, not killing, microbes. - We want to be able to ________ the growth of microbes!!!! - cide = _______ - stasis = _______

- sepsis - asepsis * wound - sterilization - commercial sterilization * nitrites and nitrates - disinfection - antisepsis - degerming (sterilization) - sanitization - biocide (germicide) - bacteriostasis - control - killing - stationary = interfering but not killing

The Complement System: - Complement: ________ proteins produced by the liver that assist the immune system in destroying microbes: insoluble and roam around in an inactive form. * act in a cascade in a process called _________ = activates the complement. - Proteins are designated with ________ and numbered in order of discovery. * activated fragments are indicated with _______ a or b.

- serum * complement activation - uppercase C * lowercase

- Acute infection: ______ term/ _____ duplication, examples: _____ or common cold. - Chronic: * _______ or ______ infections * ______ term and ________ duplication (can be dormant)

- short/ lytic/ flu * latent or persistant * long/ lysogenic

- __________: These proteins bind up iron obtained from the host cell's iron-transport proteins and transport this iron to bacteria through interactions with cell surface receptors. - __________: This occurs as a result of nutrient depletion, accumulation of waste products, pathogen entry and exit, and ruptured host cells. - __________: These poisonous substances cause most of the damage to host cells; they can be transported by the blood or lymph and may produce far-reaching effects. - __________: This results in a change in microbe characteristics due to the presence of prophage genes that confer new properties. - __________: These describe the visible effects of viral infections that results in host cell damage.

- siderophores - direct damage - toxins - lysogenic conversion - cytopathic effects

Simple Stains: - Simple stain: use of a ________. - Highlights the entire microorganism to visualize _______ and ______. - What are the three common basic dyes? - A _________ may be used to hold the stain or coat the specimen to enlarge it. * _______ is commonly used as a mordant in gram staining.

- simple basic dye - cell shapes and structures - crystal violet, methylene blue, and safranin (red) - mordant * iodine

- __________: movement of a solute from an area of high concentration to an area of low concentration. * continues until molecules reach _________. * with this process, large (polar) ions can't move through, but what can diffuse through the membrane?

- simple diffusion * equilibrium * O2, CO2, and water

A strain of Streptococcus pneumoniae is no longer able to synthesize its capsular polysaccharide. What is a likely outcome? a. It will produce more potent toxins which damage the host b. It will be readily phagocytized upon entering the host c. It will not be able to appropriately transport ions across its plasma membrane d. It will no longer be motile e. It will no longer be able to carry out fermentation

b. It will be readily phagocytized upon entering the host

Oxygen: - ___________: boosted to a higher-energy state and is reactive. - __________: (O2-): * O2- + O2- + 2H+ --(enzyme?)--> H2O2 + O2 - _________: (O2 2-): * 2H2O2 --(enzyme?)--> 2H2O + O2 (hydrogen peroxide) * H2O2 + 2H+ --(enzyme?)--> 2H2O - _______: (OH-) - All of the enzymes on here are found in bacteria in order to get rid of _________.

- singlet oxygen - superoxide radicals: * superoxide dismutase (SOD) - peroxide anion: * catalase * peroxidase - hydroxyl radical - free radicals

Biofilms: - Microbes attach to ______ surfaces and grow into masses (are everywhere!) - They will grow on rocks, pipes, teeth, and medical implants. - Biofilms can cause ________ and are often resistant to _________ (exchange nutrients between each other!)

- solid - infections/ antibiotics

Comparison of Solution and Suspension: - _________; clear/ homogenous/ solute particles do not settle to the bottom/ solute particles of the solution pass through the filter paper. - __________; cloudy (turbidity)/ heterogeneous/ solid particles settle to the bottom when suspension left to stand/ solute particles of the solution cannot pass through the filter paper and becomes residue on the filter paper.

- solution - suspension

Scanning Acoustic Microscopy: - Measures ________ that are reflected back from a specimen. - Used to study cells __________. - Resolution of ______ (things smaller than this we can't see).

- sound waves - attached to surfaces (like biofilms) - 1 um

Occurrence of a Disease: - _________: disease that occurs only occasionally. - ________: disease constantly present in a population. - ________: disease acquired by many people in a given area in a short time (one continent). - _________: worldwide epidemic.

- sporadic disease - endemic disease - epidemic disease - pandemic disease

Arrangement of Bacillus Bacterial Cells: - Include: single bacillus, coccobacillus, diplobacillus, and streptobacillus. - What arrangement can bacillus bacteria not be? Why?

- staphylo-/ because they don't divide along their long axis

Chemical Food Preservatives: - ________ prevents wine spoilage. - Organic acids: * inhibits _________. * sorbic acid, benzoic acid, and calcium propionate prevent molds in __________. - __________ prevent endospore germination.

- sulfur dioxide * metabolism * acidic foods - nitrites and nitrates

Exotoxins: - __________; cause an intense immune response due to release of cytokines from host cells (T cells). * cause symptoms of fever, nausea, vomiting, diarrhea, shock (anaphylactic), and death. - __________; damage DNA (causing mutations, disrupting cell division, and leading to cancer).

- superantigens - genotoxins

Predisposing Factors: - Make the body more _________ to disease: * gender * inherited traits, such as the sickle cell gene * climate and weather * fatigue * age * lifestyle * nutrition * chemotherapy

- susceptible

Relationships between the Normal Microbiota and the Host: - _________: is the relationship between normal microbiota and the host. * _________: one organism benefits, and the other is unaffected. * _________: both organisms benefit. EXAMPLES: S. epidermidis produces lactic acid and provides antagonism on our skin/ E. coli digest food and makes vitamin K and B - complex. * _________: one organism benefits at the expense of the other. EXAMPLE: viruses benefit from the host machinery for replication while host endures acute or chronic disease. - Normal microbiota are __________ (if given the right conditions).

- symbiosis * commensalism * mutualism * parasitism - opportunistic pathogens

Classifying Infectious Diseases: - __________; changes in body function that are felt by the patient as a result of disease. - __________; changes in a body that can be measured or observed as a result of disease. - __________; a specific groups of signs and symptoms that accompany a disease.

- symptoms - signs - syndrome

The Requirements for Growth: - The 3 major physical requirements that are critical when growing bacteria include? - The 5 chemical requirements to consider when growing bacteria include?

- temperature, pH, and osmotic pressure - carbon/ nitrogen, sulfur, and phosphorus/ trace elements/ oxygen/ and organic growth factors

The Concept of Immunity: - _________ on host cells attach to _______________. - TLRs bound to PAMPs induce the release of _______ from the host cell that regulate the intensity and duration of the immune system (recruitment through chemotaxis).

- toll-like receptors (TLRs)/ pathogen-associated molecular patterns (PAMPs) - cytokines

Extent of Host Involvement: - _________: toxins in the blood. - _________: viruses in the blood. - _________; acute infection that causes the initial illness. - ________; opportunistic infection after a primary (predisposing) infection. - ________; no noticeable signs or symptoms (inapparent infection).

- toxemia - viremia - primary infection - secondary infection - subclinical disease

Fungi: - _____ metabolic products. - Provoke an ______ response. - ________ toxins inhibit protein synthesis. - ________ modify host cell membranes. - _______ prevent phagocytosis.

- toxic - allergic - trichothecene - proteases - capsules

Production of Toxins: - ________: poisonous substances produced by microorganisms. * produce fever, cardiovascular problems, diarrhea, and shock (dropped BP). - ________: ability of microorganism to produce a toxin. - ________: presence of toxin in the host's blood. - ________: presence of toxin without microbial growth.

- toxins - toxigenicity - toxemia - intoxications

Iron-Binding Proteins: - ALL limit iron from bacteria = can't survive without iron! - _________: found in blood and tissue fluids. - _________: found in milk, saliva, and mucus. - ________: found in the liver, spleen, and red bone marrow. - ________: located in red blood cells. - Bacteria produce ________ to compete with iron-binding proteins.

- transferrin - lactoferrin - ferritin - hemoglobin - siderophores

- ____________: microbiota that may be present for days, weeks, or months. - ___________: microbiota that permanently colonize the host and do not cause disease under normal conditions. - __________ analyzes relationships between microbial communities on the body and human health. * found that the majority of bacteria are in the digestive system.

- transient microbiota - normal microbiota - Human Microbiome Project

Endoplasmic Reticulum: - Folded ______ network: - _________: studded with ribosomes; sites of protein synthesis. - ________: no ribosomes; synthesizes cell membranes, fats, and hormones.

- transport - rough ER - smooth ER

Scanning Tunneling Microscopy: - Uses a _______ to scan a specimen and reveal details of its surface. - Resolution of _______ of an atom - Drag across surface (like a record player)

- tungsten probe - 1/100

Estimating Bacterial Numbers by Indirect Methods: - _________: measurement of cloudiness with a spectrophotometer. * in the light method, cell _______ the light. So _______ cells = less light through. - ________: amount of metabolic product is proportional to the number of bacteria. * how is this measured? - ________: bacteria are filtered, dried, and weighed; used for ___________ organisms.

- turbidity * reflect/ more - metabolic activity * measure the amount of waste products/ measure the before and after nutrients - dry weight/ filamentous

Transmission Electron Microscopy: - A beam of electrons passes through _______ sections of a specimen, then through an ________ lens, then focused on a ________ lens. - Specimens may be stained with _________ for contrast. - Magnifies objects _______ to _______; resolution of ______. - ____ resolution and magnification. - Do not look directly at sample = look at a _______.

- ultrathin/ electromagnetic/ projector - heavy metal salts - 10000 - 100000x/ 10 pm - high - viewing screen

Vaccines: - _________: inoculation of smallpox (Variola) into the skin. - Edward Jenner inoculated ______(Vaccinia) to prevent smallpox. * termed _______ by Pasteur (vacca = cow). * James Phipps (8 y. o. son of Jenner's gardener) was inoculated with material from cowpox lesions on Sarah Nelmes (milkmaids) hands. - ________ suspension of organisms or fractions of organisms that induce immunity. - Pasteur honored Jenner in 1891 and widened the use of the term vaccination/vaccine to include "the artificial induction of immunity against any infectious disease".

- variolation - cowpox * vaccination - vaccine

Interferons: - Specific responses: released during ______ infections. - Type of _______ produced by cells; have antiviral activity. - ______ and _______: produced by cells in response to viral infections (specifically for viruses) * cause neighboring cells to produce __________ that inhibit viral replication. - ______: causes neutrophils and macrophages to kill bacteria.

- viral - cytokine - IFN-alpha and IFN-beta * antiviral proteins (AVPs) - IFN-gamma

Viral Structure: - _________; complete, fully developed viral particle. * nucleic acid - _______ or _______ can be single or double stranded; linear or circular. * _______: protein coat made of capsomeres (subunits); surrounds nucleic acids. * ______: lipid, protein, and carbohydrate coating on some viruses/ without this = called ________. * _________: projections from outer surface= identify.

- virion * DNA or RNA * capsid * envelope/ naked * spikes

General Morphology: - ________: a complete, active structure of a virus. - ________ viruses: hollow, cylindrical capsid. - ________ viruses: many-sided - ________ viruses: contain outer membrane from the host cell. - ________ viruses: complicated structures (like T4 bacteriophage). - _______: composed solely of a short strand of circular, single-stranded RNA that has no ________ coating. * smallest ________ pathogens known to humans. * influence transcription and translation.

- virion - helical - polyhedral - enveloped - complex - viroid/ protein * infectious

Enzymes: - Increase _________ of microbes. - Utilize circulatory to become systemic. - Type: * __________: coagulate fibrinogen = reduces blood flow = reduces WBC entry = walls itself off basically. * _________: digest fibrin clots...... __________ things. * _________: digests polysaccharides that hold cells together. * _________: breaks down collagen * _________: destroy IgA antibodies.

- virulence * coagulases * kinases/ phosphorylates * hyaluronidase * collagenase * IgA proteases

Viruses and Cancer: - Several types of cancer are caused by ________. * may develop long after a viral infection (example HPV = cervical cancer). * cancers caused by viruses are not _________. - _________: cancer of connective tissue. - ________: cancers of glandular epithelial tissue. - adeno- = _________. - Cancer is ____________. * During interphase, there are ______ used as checkpoints for cell growth/ cancer breaks down these checkpoints = uncontrolled. - Cells affected by cancer or cell affected by viruses both lead to cancer = overlapping.

- viruses * contagious - sarcoma - adenocarcinomas - glands - uncontrolled cell growth * cyclins

Direct Microscopic Count: - _______ of a bacterial suspension placed on a slide. - Average number of bacteria per _______ is calculated. - Uses a special ________ counter. - Number of bacteria = __________.

- volume - viewing field - Petroff-Hausser - number of cells counted/ volume of area counted

Protozoa: - Presence of protozoa and their ______ products causes symptoms. - Avoid host defenses by: * _______ cells and tissue fluids. * growing in ________. * antigenic ______ (changing antigens).

- waste * digesting * phagocytes * variation

Helminths: - Use host tissue for growth. - Produce large masses; cause cellular damage. - Produce ______ products ---> lead to symptoms. Algae: - Some produce a neurotoxin called ________. * example: paralytic ______ poisoning.

- waste - saxitoxin * shellfish

Acid-Fast Stain: - Binds only to bacteria that have a ______ material (_________ = inhibits crystal violet and safranin of gram stain) in their cell walls, which is not decolorized by ________. - Used for the identification of ______ and _______. - What is one common disease that needs this type of stain? - What is the end color of acid-fast bacteria vs. non-acid-fast bacteria. - Does not use what step that gram stains use?

- waxy (mycolic acid)/ acid-alcohol - mycobacterium and nocardia - tuberculosis - red vs. blue - mordant

Protein Analytical Technique: - ____________: protein immunoblotting. * analytical technique used to identify and locate specific proteins via _________ and a blotter, based on their ability to bind to specific antibodies.

- western blotting * electrophoresis

Budding of Enveloped Virus: - _______ usually bud. - Release of viral particles = budding (_______ cycle) - Virus takes plasma membrane from the host cell to make its _______, which is a _______ factor that lets the virus hide from the immune system. * CANNOT create membrane on their own = comes from ______.

- yeast - (lysogenic) - envelope/ virulence * host cell

An example of descriptive epidemiology is __________. a. the first report of a disease occurrence b. John Snow's study of the London cholera outbreak from 1848 to 1849 c. Florence Nightingale's studies of disease transmission in soldiers and civilians during the Crimean War d. a comparison of drug effectiveness in two groups of patients e. a study comparing disease rates in vaccinated and unvaccinated individuals

b. John Snow's study of the London cholera outbreak from 1848 to 1849

Which type of vaccine is most likely to be contaminated with endotoxin? a. Conjugated vaccine b. Killed whole-cell vaccine c. Subunit vaccine d. Toxoid vaccine

b. Killed whole-cell vaccine

Table 15.2: Bacterium/ ID50 - E. coli O157:H7/ 20 - Legionella pneumophila/ 1 - Shigella/ 10 - Treponema pallidum/ 57 Which organism in the table most easily causes an infection? a. E. coli O157:H7 b. Legionella pneumophila c. Shigella d. Treponema pallidum e. The answer cannot be determined based on the information provided. Which organism in Table 15.2 causes the most severe disease? A) coli O157:H7 B) Legionella pneumophila C) Shigella D) Treponema pallidum E) It cannot be determined from the information provided.

b. Legionella pneumophila E) It cannot be determined from the information provided.

Which statement regarding the lymphatic system is true? a. Lymph nodes are sites of activation of neutrophils, which destroy microbes. b. Lymphatic capillaries possess one-way valves. These valves permit the uptake of fluid from the body but do not allow the fluid to flow back out of the capillaries into the intracellular spaces. c. The thymus serves as the site for maturation of B cells. d. The pancreas contains lymphocytes and macrophages that monitor the blood for microbes.

b. Lymphatic capillaries possess one-way valves. These valves permit the uptake of fluid from the body but do not allow the fluid to flow back out of the capillaries into the intracellular spaces.

Which of the following viruses may cause a persistent viral infection? a. Varicellovirus b. Measles virus c. Herpes simplex 1 d. HTLV-1

b. Measles virus

Which of these viruses is known to cause a persistent viral infection? a. Herpes simplex virus b. Measles virus c. Varicellovirus d. Hepatitis A virus

b. Measles virus

Which of the following types of radiation is the LEAST effective control method against microorganisms? a. Gamma rays b. Microwaves c. X rays d. UV light

b. Microwaves

What are leukocidins? a. Molecules that destroy the complement proteins b. Molecules that are capable of destroying phagocytes c. Molecules that can degrade IgA

b. Molecules that are capable of destroying phagocytes

Which of the following statements about adherence is true? a. Most bacteria can adhere to any cell in the host. b. Most bacterial adhesins are glycoproteins or lipoproteins. c. The host cell receptors for bacterial adhesins are usually proteins. d. Adhesins are always located on the bacterium's cell membrane.

b. Most bacterial adhesins are glycoproteins or lipoproteins.

I developed a test for a pathogen that is very specific but NOT very sensitive. What does this mean? a. My test is very accurate and can detect very low levels of the pathogen. b. My test is very accurate, but there must be a lot of pathogen present for it to work. c. My test is not very accurate. d. My test is very likely to give me a false positive result.

b. My test is very accurate, but there must be a lot of pathogen present for it to work.

Capsules play a role in the virulence of all of the following EXCEPT __________. a. Bacillus anthracis b. Mycobacterium tuberculosis c. Klebsiella pneumoniae d. Yersinia pestis e. Haemophilus influenzae

b. Mycobacterium tuberculosis

Which of the following is a likely outcome, based on the unique cell structure of members of the genus Mycoplasma? a. Mycoplasma species can release endotoxins when they lyse. b. Mycoplasma species have very flexible cells that can pass through bacteriologic filters. c. Mycoplasma species usually stain gram-positive. d. Mycoplasma species are susceptible to penicillin. e. Mycoplasma species are susceptible to disruption by lysozymes.

b. Mycoplasma species have very flexible cells that can pass through bacteriologic filters.

Which of the following statements concerning prion diseases is true? a. Prion diseases affect humans but not other animals. b. Normal host cellular prion proteins (PrPC) are converted into scrapie proteins (PrPSc). c. Prion diseases affect brain function but do not affect the morphology (overall appearance) of brain tissues. d. Prion diseases are always inherited.

b. Normal host cellular prion proteins (PrPC) are converted into scrapie proteins (PrPSc).

You get the following antibody titers against West Nile virus in three patients. Which patient probably has a current infection? Patient A: 128 IgG, 0 IgM Patient B: 128 IgG, 256 IgM Patient C: 0 IgG, 0 IgM a. Patient A b. Patient B c. Patient C d. Patients A and C

b. Patient B

Which of these conditions is likely to interfere with the effectiveness of an antiseptic or disinfectant? a. Increased temperature b. Presence of biofilms c. Increased time of exposure d. Absence of endospores

b. Presence of biofilms

Which of the following is a reaction of the adaptive immune response? a. Fever b. Production of antibodies c. Inflammation d. Phagocytosis by macrophages

b. Production of antibodies

Which of the following kinds of microscopy would be most appropriate for viewing the shape and arrangement of pili or fimbriae on the surface of a bacterial cell? a. Brightfield microscopy b. Scanning electron microscopy c. Darkfield microscopy d. Phase-contrast microscopy e. Transmission electron microscopy

b. Scanning electron microscopy

Based on the following LD50 values, which microbe is the most virulent? Assume each bacterium enters through the appropriate portal of entry. a. Cryptosporidium: 50 cells b. Shigella: 10 cells c. E. col O157: 1000 cells d. Vibrio cholerae: 108 cells

b. Shigella: 10 cells

The Results of Antigen-Antibody Binding: 1. __________: clumps bacteria = disrupts duplication. * reduces number of infectious units to be dealt with. 2. _________: coating antigen with antibodies enhances phagocytosis. 3. _________: blocks adhesion of bacteria and viruses to mucous/ blocks attachment of toxins (cover the surface of viruses) 4. ________; causes inflammation and cell lysis. 5. ________: large target cell (parasite): antibodies attached to target cell cause destruction by macrophages, eosinophils, and NK cells.

1. agglutination 2. opsonization 3. neutralization 4. activation of complement 5. antibody-dependent cell-mediated cytotoxicity

use the following choices to answer the next 2 questions a. hemolysis b. hemagglutination c. hemagglutination-inhibition d. no hemolysis e. precipitin ring formed 1. patients serum, influenza virus, sheep red blood cells, and anti-sheep red blood cells are mixed in a tube. what happens if the patient has antibodies against influenza? 2. patients serum, chlamydia, guinea pig complement, sheep red blood cells, and anti-sheep red blood cells are mixed in a tube. What happens if the patient has antibodies against Chlamydia? 3. the examples in the questions are a. direct tests b. indirect tests

1. c. hemagglutination-inhibition 2. d. no hemolysis 3. b. indirect tests

The Phases of Phagocytosis: 1. _________ and _________ of phagocyte to microbe. 2. _______ of microbe by phagocyte. 3. Formation of _______ (phagocytic vesicle). 4. Fusion of phagosome with a lysosome to form a ________. 5. _________ of ingested microbes by enzymes in the phagolysosome. 6. Formation of the residual body containing indigestible material. 7. _______ of waster materials. ****** Macrophages are _______, which present bacterial parts to the adaptive immune system - activate it!.

1. chemotaxis and adherence 2. ingestion 3. phagosome 4. phagolysosome 5. digestion 7. discharge ****** APCs (antigen presenting cells)

Match the following choices to questions 1-4: a. Innate resistance. b. Naturally acquired active immunity. c. Naturally acquired passive immunity. d. Artificially acquired active immunity. e. Artifically acquired passive immunity. 1. The type of protection provided by the injection of diphtheria toxoid. 2. The type of protection provided by the injection of antirabies serum. 3. The type of protection resulting from recovery from an infection. 4. A newborn's immunity to yellow fever.

1. d. Artificially acquired active immunity. 2. e. Artificially acquired passive immunity. 3. b. Naturally acquired active immunity. 4. c. Naturally acquired passive immunity.

Types of Vaccines and Their Characteristics: 1. _____________: * weakened pathogen. * closely mimic an actual infection. * only 1 to 2 confers lifelong cellular and humoral immunity. * downside: people with compromised immune systems should not get this vaccine and they need to be kept cold so not good for areas without refrigeration. 2. ______________: * safer than live vaccines (pathogen is dead). * require repeated booster doses. * induce mostly humoral immunity.

1. live attenuated vaccines 2. inactivated killed vaccines

What is the correct sequence of events for the replication of a DNA virus? Arrange the following statements in chronological order .

1. virions attach to the host cells 2. viral DNA is released into the nucleus of the host cell. 3. Enzymes required for multiplication of viral DNA are produced via transcription and translation 4. A copy of the DNA is made 5. Capsid and other structural proteins are manufactured 6. virions are assembled to form complete viruses and are released from the host cell.

Why may you be asked whether you are allergic to eggs before receiving a vaccination? a. Some viruses used for vaccines are grown and isolated in embryonic eggs. Because of their growth in this environment, you may be allergic to the vaccine. b. Some viruses are grown and isolated in embryonated eggs; especially those used for vaccines. The egg proteins may still be present in the viral vaccine preparations. c. You are asked about allergies as a precaution any time you receive a vaccine. d. This is an incorrect statement. You are usually never asked about allergies before receiving a vaccine.

b. Some viruses are grown and isolated in embryonated eggs; especially those used for vaccines. The egg proteins may still be present in the viral vaccine preparations.

For which of the following can Koch's postulates be used to identify the organism causing the disease? a. Treponema pallidum, the causative agent of syphilis, which cannot be grown in the laboratory b. Streptococcus pyogenes, the organism that causes strep throat, which can be cultured on blood agar in the laboratory c. HIV (human immunodeficiency virus), the agent causing AIDS, a disease of the human immune system d. Allergies

b. Streptococcus pyogenes, the organism that causes strep throat, which can be cultured on blood agar in the laboratory

A drug that binds to mannose on human cells would prevent: a. The entrance of Vibrio enterotoxin. b. The attachment of pathogenic E. coli. c. The action of botulinum toxin. d. Streptococcal pneumonia. e. The action of diphtheria toxin.

b. The attachment of pathogenic E. coli.

A classmate is trying to determine how a disinfectant might kill cells. You observed that when he spilled the disinfectant in your reduced litmus milk, the litmus turned blue again. You suggest to your classmate that. a. The disinfectant might inhibit cell wall synthesis. b. The disinfectant might oxidize molecules c. The disinfectant might inhibit protein synthesis. d. The disinfectant might denature proteins. e. He take his work away from yours.

b. The disinfectant might oxidize molecules

Which of the following statements is INCORRECT about adjuvants? a. Adjuvants are aluminum salts that are combines with many vaccines. b. The exact mechanism by which adjuvants work is well researched and known. c. Alum is the only adjuvant approved for humans in the United States. d. Some adjuvants are approved only for use in animals.

b. The exact mechanism by which adjuvants work is well researched and known.

Endotoxins and the Pyrogenic Response: 1. A macrophage ingests a Gram - bacterium. 2. The bacterium is degraded in a vacuole, releasing endotoxins that induce the macrophage to produce signaling molecules: _______, interleukin-1, and tumor necrosis factor alpha. 3. The cytokines are released into the bloodstream by the macrophages, through which they travel to the _______, the temperature control center of the brain. 4. The cytokines induce the hypothalamus to produce ________ (regulator hormone of temperature), which reset the body's thermostat to a higher temperature, producing fever.

2. cytokines 3. hypothalamus 4. prostaglandins

Types of Vaccines and Their Characteristics: 3. ___________: use antigenic fragments to stimulate an immune response. * __________: subunit vaccines produced by genetic modification. * _________: resemble intact viruses but do not contain viral genetic material. * ________: used for diseases in children with poor immune response to capsular polysaccharides. * ________: injected naked DNA produces the protein antigen encoded in the DNA/ protein antigens carried to the red bone marrow stimulate humoral and cellular immunity. 4. __________: inactivated toxins. * target the harmful substances made by bacteria. * ________: serums containing antibodies against the toxin (target exotoxins that bacteria secrete). * antitoxin neutralizing a toxin examples: _______ toxoid and ______ toxoid.

3. subunit vaccines * recombinant vaccines * virus-like particle (VLP) vaccines * conjugated vaccines * nucleic acid (DNA) vaccines 4. toxoids * antitoxins * tetanus and diphtheria

use the following choices to answer the next 2 questions: a. anti-brucella b. brucella c. substrate for the enzyme 4. which is the third step in the direct ELISA test 5. which item is from the patient in an indirect ELISA test

4. c. substrate for the enzyme 5. a. anti-brucella

Match the following choices to the statements in questions 5-7: a. IgA b. IgD c. IgE d. IgG e. IgM 5. Antibodies that protect the fetus and newborn. 6. The first antibodies synthesized; especially effective against microorganisms. 7. Antibodies that are bound to mast cells and involved in allergic reactions.

5. d. IgG 6. e. IgM 7. c. IgE

use the following choices to answer the next 3 questions a. direct fluorescent antibody b. indirect fluorescent antibody c. rabies immune globulin d. killed rabies virus e. none of the above 7. treatment given to a person bitten by a rabid bat 8. test used to identify rabies virus in the brain of a dog 9. test used to detect the presence of antibodies in a patients serum

7. c. rabies immune globulin 8. a. direct fluorescent antibody 9. b. indirect fluorescent antibody

The nucleus of a eukaryotic cell differs from the nucleoid of a prokaryotic cell in all of the following ways EXCEPT which one? a. The eukaryotic nucleus contains nucleoli. There are no nucleoli in the prokaryotic nucleoid. b. The prokaryotic nucleoid contains DNA combined with histones. Histones are lacking in the eukaryotic nucleus. c. The eukaryotic nucleus is surrounded by a nuclear envelope. There is no envelope surrounding the prokaryotic nucleoid. d. The prokaryotic nucleoid usually contains a single circular chromosome, whereas the eukaryotic nucleus contains multiple linear chromosomes.

b. The prokaryotic nucleoid contains DNA combined with histones. Histones are lacking in the eukaryotic nucleus.

What is the purpose of a counterstain in the acid-fast stain?

A counterstain stains the colorless non-acid-fast cells so that they are easily seen through a microscope (blue color)

8) Which of the following places the steps in the correct sequence? 1-Staining 2-Making a smear 3-Fixing A) 2-3-1 B) 1-3-2 C) 3-2-1 D) 1-2-3 E) The order is unimportant.

A) 2-3-1

Which of the following is NOT a membrane-disrupting toxin? A) A-B toxin B) hemolysin C) leukocidin D) streptolysin O E) streptolysin S

A) A-B toxin

All of the following bacteria release endotoxin EXCEPT A) Clostridium botulinum. B) Salmonella typhi. C) Neisseria meningitidis. D) Proteus vulgaris. E) Haemophilus influenzae.

A) Clostridium botulinum.

Which of the following statements about biological transmission is FALSE? A) Houseflies are an important vector. B) The pathogen may be injected by the bite of the vector. C) The pathogen may require the vector as a host. D) The pathogen reproduces in the vector. E) The pathogen may enter the host in the vector's feces.

A) Houseflies are an important vector.

6) Which of the following is NOT true regarding the acid-fast stain? A) If cells are acid-fast, they are gram-negative. B) It is used to identify members of the genus Mycobacterium. C) Non-acid-fast microbes appear blue in a completed acid-fast stain. D) Acid-fast cells appear red in a completed acid-fast stain. E) Acid-fast cells retain the primary dye after treatment with acid-alcohol.

A) If cells are acid-fast, they are gram-negative.

The arguments supporting spontaneous generation were finally disproved by A) Louis Pasteur. B) Francesco Redi. C) Rudolf Virchow. D) John Needham. E) Lazzaro Spallanzani.

A) Louis Pasteur.

LOOK AT PICTURE: In Table 18.1, who probably has the disease? A) Patients A and B B) Patients B and C C) Patients A and C D) Patients C and D E) Patients A and D In Table 18.1, who is most likely protected from the disease, as observed by the test results over time? A) Patient A B) Patient B C) Patient C D) Patient D In Table 18.1, who showed seroconversion during these observations? A) Patient A B) Patient B C) Patient C D) Patient D

A) Patients A and B D) Patient D A) Patient A

Which statement is CORRECT concerning animal viruses? A) Retroviruses use an enzyme called reverse transcriptase, which synthesizes DNA by copying RNA. B) Capsid proteins are produced in the nucleus. C) The genome of animal viruses is always single-stranded. D) Enveloped viruses are surrounded by a lipid and carbohydrate coat, which is made from the host cell's mitochondria.

A) Retroviruses use an enzyme called reverse transcriptase, which synthesizes DNA by copying RNA.

All of the following organisms produce exotoxins EXCEPT A) Salmonella typhi. B) Clostridium botulinum. C) Corynebacterium diphtheriae. D) Clostridium tetani. E) Staphylococcus aureus.

A) Salmonella typhi.

Which of the following is NOT a verified exception in the use of Koch's postulates? A) Some diseases are noncommunicable. B) Some diseases have poorly defined etiologies. C) Some human diseases have no other known animal host. D) Some pathogens can cause several disease conditions. E) Some diseases are not caused by microbes.

A) Some diseases are noncommunicable.

Which of the following best describes the pattern of microbial death? A) The cells in a population die at a constant rate. B) All the cells in a culture die at once. C) Not all of the cells in a culture are killed. D) The pattern varies depending on the antimicrobial agent. E) The pattern varies depending on the species.

A) The cells in a population die at a constant rate.

What happens to the packaged DNA of a specialized transduced phage when it infects a new recipient cell? A) The host DNA integrates, with the prophage, into the new recipient chromosome. B) The DNA begins to replicate without integrating into the host chromosome. C) The DNA is chewed up by enzymes found in the recipient cell.

A) The host DNA integrates, with the prophage, into the new recipient chromosome.

A lytic virus has infected a patient. Which of the following would best describe what is happening inside the patient? A) The virus is causing the death of the infected cells in the patient. B) The virus is not killing any cells in the host. C) The virus is incorporating its nucleic acid with that of the patient's cells. D) The virus is slowly killing the patient's cells. E) The virus is infecting cells and then releasing only small amounts of virus.

A) The virus is causing the death of the infected cells in the patient.

Which of the following statements is TRUE? A) There are at least thirty complement proteins. B) All of the complement proteins are constantly active in serum. C) Factors B, D, and P cause cytolysis. D) Complement activity is antigen-specific. E) Complement increases after immunization.

A) There are at least thirty complement proteins.

Tim ate raw oysters at a local restaurant and has since discovered he contracted hepatitis A. Which mode of transmission is this? A) Vehicle transmission B) Droplet transmission C) Direct contact D) Mechanical transmission E) Airborne transmission

A) Vehicle transmission

In Figure 17.2, which areas represent antigen-binding sites? A) a and b B) a and c C) b and c D) c and d E) b and d

A) a and b

All of the following pertain to fever EXCEPT that it A) accelerates microbial growth by increasing iron absorption from the digestive tract. B) stimulates T lymphocyte activity. C) is caused by interleukin-1 and TNF-alpha coming into contact with the hypothalamus. D) intensifies the effect of antiviral interferons. E) can be initiated by specific types of pathogens.

A) accelerates microbial growth by increasing iron absorption from the digestive tract.

Cytokines released by TH1 cells A) activate CD8+cells to CTLs. B) convert TH1 cells to TH2 cells. C) convert TH2 cells to TH1 cells. D) kill parasites. E) convert B cells to T cells.

A) activate CD8+cells to CTLs.

Activation of C3a results in A) acute inflammation. B) increased blood vessel permeability. C) opsonization. D) attraction of phagocytes. E) cell lysis.

A) acute inflammation.

Which step in the Gram stain is the critical step in differentiating gram-positive cells from gram-negative cells? A) alcohol-acetone B) iodine C) safranin D) crystal violet

A) alcohol-acetone

The ability of some microbes, such as Trypanosoma or Giardia to alter their surface molecules and evade destruction by the host's antibodies is called A) antigenic variation. B) lysogenic conversion. C) virulence. D) cytopathic effect. E) cytocidal effect.

A) antigenic variation.

Symptoms of disease differ from signs of disease in that symptoms A) are changes felt by the patient. B) always occur as part of a syndrome. C) are changes observed by the physician. D) are specific for a particular disease. E) None of the answers is correct.

A) are changes felt by the patient.

Which of the following pairs of terms is mismatched? A) bacteriostatic — kills vegetative bacterial cells B) germicide — kills microbes C) virucide — inactivates viruses D) sterilant — destroys all living microorganisms E) fungicide — kills yeasts and molds

A) bacteriostatic — kills vegetative bacterial cells

All of the following substances are used to preserve foods EXCEPT A) biguanides. B) nisin. C) potassium sorbate. D) sodium nitrite. E) calcium propionate.

A) biguanides.

Which of the following is NOT used as a criterion to classify viruses? A) biochemical tests B) morphology C) nucleic acid D) size E) number of capsomeres

A) biochemical tests

The addition of which of the following to a culture medium will neutralize acids? A) buffers B) sugars C) pH D) heat E) carbon

A) buffers

Which of the following bacterial components would most likely result in B cell stimulation by T-independent antigens? A) capsule B) flagellum C) pili D) ribosome E) plasmid

A) capsule

Which structure acts like an "invisibility cloak" and protects bacteria from being phagocytized? A) capsule B) slime layer C) cell wall D) fimbriae E) cell membrane

A) capsule

Which of the following is NOT found in mitochondria and prokaryotes? A) cell wall B) binary fission C) circular chromosome D) 70S ribosomes E) ATP-generating mechanism

A) cell wall

After ingesting a pathogen, lysosomal enzymes produce all of the following EXCEPT A) complement. B) O2-. C) H2O2. D) OH. E) HOCl.

A) complement.

An ELISA for Hepatitis C has 95 percent sensitivity and 90 percent specificity. This means that the test A) detects 95 percent of the true positive samples and has 10 percent false positive results. B) detects 5 percent of the true positive samples and has 90 percent false positive results. C) detects 90 percent of the true positive samples and has 5 percent false positive results. D) detects 95 percent of the true positive samples and has 90 percent false positive results. E) detects 5 percent of the true positive samples and has 10 percent false positive results.

A) detects 95 percent of the true positive samples and has 10 percent false positive results.

In which of the following situations would Koch's postulates be utilized? A) determination of the cause of a patient's illness in a hospital microbiology lab B) development of a new antibiotic in a pharmaceutical lab C) determination of the cause of cancer in a patient D) formulation of a vaccine against a new pathogen in a genetic engineering lab E) whenever the scientific method is used to investigate a microbiological problem

A) determination of the cause of a patient's illness in a hospital microbiology lab

A test used to identify Streptococcus pyogenes in a patient's throat swab is the A) direct fluorescent-antibody test. B) indirect fluorescent-antibody test. C) hemagglutination test. D) hemagglutination-inhibition test. E) indirect ELISA test.

A) direct fluorescent-antibody test.

An agent used to reduce the number of bacteria on a toilet would most accurately be called a(n) A) disinfectant. B) antiseptic. C) aseptic. D) fungicide. E) virucide.

A) disinfectant.

The term aerotolerant anaerobe refers to an organism that A) does not use oxygen but tolerates it. B) is killed by oxygen. C) tolerates normal atmospheric nitrogen gas levels. D) requires less oxygen than is present in air. E) requires more oxygen than is present in air.

A) does not use oxygen but tolerates it.

ADCC is a process that is most effective in destroying A) eukaryotic pathogens. B) prions. C) extracellular viruses. D) bacterial pathogens. E) bacterial toxins.

A) eukaryotic pathogens.

The microbial process of converting sugars to alcohol is known as A) fermentation. B) pasteurization. C) tyndallization. D) lyophilization. E) alcoholism.

A) fermentation.

Lysogenic bacteriophages contribute to bacterial virulence because bacteriophages A) give new gene sequences to the host bacteria. B) produce toxins. C) carry plasmids. D) kill the bacteria, causing release of endotoxins. E) kill human cells.

A) give new gene sequences to the host bacteria.

Dead Bordetella pertussis can be used in a(n) A) inactivated whole-agent vaccine. B) attenuated whole-agent vaccine. C) conjugated vaccine. D) subunit vaccine. E) toxoid vaccine.

A) inactivated whole-agent vaccine.

CD4+ T cells are activated by A) interaction between CD4+and MHC II. B) interaction between TCRs and MHC II. C) cytokines released by dendritic cells. D) cytokines released by B cells. E) complement.

A) interaction between CD4+and MHC II.

The alternative pathway for complement activation is initiated by A) lipid-carbohydrate complexes and C3. B) C5-C9. C) antigen-antibody reactions. D) factors released from phagocytes. E) factors released from damaged tissues.

A) lipid-carbohydrate complexes and C3.

Bacteriophages derive all of the following from the host cell EXCEPT A) lysozyme. B) tRNA. C) amino acids. D) nucleotides. E) ATP.

A) lysozyme.

Which microscope is best used for observing the surfaces of intact cells and viruses? A) scanning electron microscope B) phase-contrast microscope C) fluorescence microscope D) brightfield microscope E) darkfield microscope

A) scanning electron microscope

You are studying a cell structure that is approximately 100 nm in size. Which of the following provides the lowest magnification you can use to see this structure? A) scanning electron microscope B) transmission electron microscope C) phase-contrast microscope D) darkfield microscope E) brightfield microscope

A) scanning electron microscope

Koch observed Bacillus anthracis multiplying in the blood of cattle. What is this condition called? A) septicemia B) local infection C) bacteremia D) focal infection E) systemic infection

A) septicemia

2) What structure does light pass through after leaving the condenser in a compound light microscope? A) specimen B) ocular lens C) objective lens D) illuminator

A) specimen

The following steps occur during multiplication of retroviruses. Which is the fourth step? A) synthesis of double-stranded DNA B) synthesis of +RNA C) attachment D) penetration E) uncoating

A) synthesis of double-stranded DNA

Injectable drugs are tested for endotoxins by A) the Limulus amoebocyte lysate test. B) counting the viable bacteria. C) filtering out the cells. D) looking for turbidity. E) culturing bacteria.

A) the Limulus amoebocyte lysate test.

The antimicrobial activity of chlorine is due to which of the following? A) the formation of hypochlorous acid B) the formation of hydrochloric acid C) the formation of ozone D) the formation of a hypochlorite ion E) disruption of the plasma membrane

A) the formation of hypochlorous acid

How might a patient who is not being treated with an antibiotic still be exposed to an antibiotic? A. Antibiotics can be used in aerosols, thereby entering the environment. B. Visitors currently being treated with antibiotics can pass them on to the patient. C. Health care workers being treated with antibiotics may pass the antibiotic on to the patient.

A. Antibiotics can be used in aerosols, thereby entering the environment.

What type of nosocomial infection is likely to arise from intravenous catheterizations? A. Bacteremia B. Cutaneous C. Lower respiratory D. Urinary tract E. Surgical

A. Bacteremia

What is the most common method used to identify viruses A. serological methods B. observation of cytopathic effects C. PCR D. morphology

A. serological methods

How do NK cells recognize the target cells that they will destroy? a. The target cells express foreign antigens. b. The target cells lack MHC I self-antigens. c. The target cells are coated with complement. d. The target cells are coated with antibody.

b. The target cells lack MHC I self-antigens.

1) Which of the following is NOT equal to 1 mm? A) 10-3 m B) 100 μm C) 10 6 nm D) 0.1 cm E) 0.001 m

B) 100 μm

Most fungi grow best at pH A) 1. B) 5. C) 7. D) 9. E) 14.

B) 5.

In a direct ELISA test, what are you looking for in the patient? A) Antibodies B) Antigen C) Either antigen or antibodies D) None of the above

B) Antigen

Who is credited with first observing microorganisms? A) Robert Hooke B) Anton van Leeuwenhoek C) Robert Koch D) Louis Pasteur E) Carolus Linnaeus

B) Anton van Leeuwenhoek

A hybridoma results from the fusion of a(an) A) B cell with a T cell. B) B cell with a myeloma cell. C) antigen with an antibody. D) antigen with a B cell. E) myeloma cell with a virus.

B) B cell with a myeloma cell.

The circumsporozite antigen of Plasmodium can be used for all of the following except to A) Vaccinate healthy people B) Cure infected people C) Produce monoclonal antibodies D) Decrease recurring infections E) All of the above

B) Cure infected people

Which microscope uses two beams of light to produce a three-dimensional color image? A) fluorescence microscope B) DIC microscope C) phase-contrast microscope D) electron microscope E) darkfield microscope

B) DIC microscope

Which of the following is most likely a product of an early gene? A) capsid proteins B) DNA polymerase C) envelope proteins D) spike proteins E) lysozyme

B) DNA polymerase

Large antibodies that agglutinate antigens are A) IgG. B) IgM. C) IgA. D) IgD. E) IgE.

B) IgM.

Any of the following may occur in diseases transmitted by vectors except: A) In biological transmission, vectors can spread pathogens by vomiting. B) In biological transmission, the vector multiplies in the human host. C) In biological transmission, a vector transmits pathogens by defecating while taking a blood meal. D) In biological transmission, pathogens multiply in the vector, which can transmit the pathogens by injecting saliva directly into the host. E) In mechanical transmission, insects contact feces containing pathogenic bacteria and transfer it to food.

B) In biological transmission, the vector multiplies in the human host.

To detect botulinum toxin in food, suspect food is injected into two guinea pigs. The guinea pig that was vaccinated against botulism survives, while the one that was not vaccinated dies. This is an example of A) Agglutination B) Neutralization C) Hemagglutination D) Fluorescent antibodies E) ELISA

B) Neutralization

________ were first identified in cancer-causing viruses and can induce ________ in infected cells. A) Herpes viruses; lesions B) Oncogenes; transformation C) T antigens; lysis D) Glycoprotein spikes; syncytia formation E) Segmented genomes; reassortment

B) Oncogenes; transformation

Which of the following statements about fixed macrophages is FALSE? A) They are found in certain tissues and organs. B) They develop from neutrophils. C) They are cells of the mononuclear phagocytic system. D) They are mature monocytes. E) They gather at sites of infection.

B) They develop from neutrophils.

In an immunodiffusion test to diagnose the fungal disease histoplasmosis, a patient's serum is placed in a well in an agar plate. In a positive test, a precipitate forms as the serum diffuses from the well and meets material diffusing from a second well. In this test process, what is the most likely identity of the material in the second well? A) antibodies B) a purified fungal antigen C) entire fungal cells D) a purified protozoan antigen E) red blood cells

B) a purified fungal antigen

What contributes to antigenic shift in influenza viruses? A) worldwide distribution of the virus B) a segmented genome C) attachment spikes D) ease of virus transmission E) different virus subtypes

B) a segmented genome

Which of the following pairs is mismatched? A) Ag — wound dressings B) alcohols — open wounds C) CuSO4— algicide D) H2O2— open wounds E) organic acids — food preservation

B) alcohols — open wounds

Which item is from the patient in a direct ELISA test? A) substrate for the enzyme B) antigen C) antihuman immune serum D) antibodies against the antigen

B) antigen

Live weakened polio virus can be used directly in a(n) A) inactivated whole-agent vaccine. B) attenuated whole-agent vaccine. C) conjugated vaccine. D) subunit vaccine. E) toxoid vaccine.

B) attenuated whole-agent vaccine.

In Figure 13.1, which structure is a complex virus? A) a B) b C) c D) d E) All of the structures are complex viruses.

B) b

LOOK AT PICTURE: Which component in Figure 18.1 came from the patient in this indirect ELISA test? A) a B) b C) c D) d E) e Figure 18.1 is an illustration of a(an) A) negative indirect ELISA test. B) positive indirect ELISA test. C) complement fixation test. D) hemagglutination test. E) precipitation test.

B) b B) positive indirect ELISA test.

A chill is a sign that A) body temperature is falling. B) body temperature is rising. C) body temperature is not changing. D) the metabolic rate is decreasing. E) blood vessels are dilating.

B) body temperature is rising.

Which of the following diseases is NOT spread by droplet infection? A) diphtheria B) botulism C) the common cold D) measles E) tuberculosis

B) botulism

The mechanism whereby an enveloped virus leaves a host cell is called A) transduction. B) budding. C) abduction. D) lysogeny. E) penetration.

B) budding.

Pathogenic bacteria isolated from the respiratory or intestinal tracts of humans are A) strict aerobes that grow best in candle jars. B) capnophiles that grow best in carbon dioxide incubators. C) facultative anaerobes that require reducing media for growth. D) strict aerobes that grow best in reducing media. E) capnophiles that prefer highly oxygenated growth conditions.

B) capnophiles that grow best in carbon dioxide incubators.

Which of the following is mismatched? A) diapedesis — movement of leukocytes between capillary walls cells out of blood and into tissue B) chemotaxis — chemical degradation inside a phagolysosome C) abcess — a cavity created by tissue damage and filled with pus D) pus — tissue debris and dead phagocytes in a white or yellow fluid E) scab — dried blood clot over injured tissue

B) chemotaxis — chemical degradation inside a phagolysosome

An example of a latent viral infection is A) subacute sclerosing panencephalitis. B) cold sores. C) influenza. D) smallpox. E) mumps.

B) cold sores.

A reaction that uses the absence of hemolysis of red blood cells to indicate an antigen—antibody reaction is called a(n) A) agglutination reaction. B) complement fixation. C) immunofluorescence. D) neutralization reaction. E) precipitation reaction.

B) complement fixation.

Biogenesis refers to the A) spontaneous generation of organisms from nonliving matter. B) development of life forms from preexisting life forms. C) development of aseptic technique. D) germ theory of disease.

B) development of life forms from preexisting life forms.

In using this microscope, the observer does NOT look directly at an image through a lens. A) phase-contrast microscope B) electron microscope C) fluorescence microscope D) compound light microscope E) darkfield microscope

B) electron microscope

In a direct ELISA test to screen for drugs in a patient's urine, what is the third step in the test process? A) substrate for the enzyme is added B) enzyme-labeled antibodies against the drug being tested is added C) the patient's urine sample is diluted D) antibody against the drug being tested is added

B) enzyme-labeled antibodies against the drug being tested is added

Which of the following is involved in resistance to parasitic helminths? A) basophils B) eosinophils C) lymphocytes D) monocytes E) neutrophils

B) eosinophils

All of the following are effects of histamine EXCEPT A) vasodilation. B) fever. C) swelling. D) redness. E) pain.

B) fever.

Which of the following have a cell wall? A) mycoplasmas B) fungi C) protoplasts D) L forms E) animal cells

B) fungi

Oxidizing agents include all of the following EXCEPT A) chlorine. B) glutaraldehyde. C) hydrogen peroxide. D) iodine. E) ozone.

B) glutaraldehyde.

The swelling associated with inflammation decreases when the fluid A) returns to the blood. B) goes into lymph capillaries. C) is excreted in urine. D) is lost as perspiration. E) is transported into macrophages.

B) goes into lymph capillaries.

A viral species is a group of viruses that A) has the same morphology and nucleic acid. B) has the same genetic information and ecological niche. C) infects the same cells and cause the same disease. D) cannot be defined.

B) has the same genetic information and ecological niche.

One effect of washing regularly with antibacterial agents is the removal of normal microbiota. This can result in A) body odor. B) increased susceptibility to disease. C) normal microbiota returning immediately. D) fewer diseases. E) no bacterial growth bc washing removes their food source.

B) increased susceptibility to disease.

Which of the following is a pregnancy test used to find the fetal hormone HCG in a woman's urine using anti-HCG and latex spheres? A) direct agglutination reaction B) indirect agglutination reaction C) immunofluorescence D) neutralization reaction E) precipitation reaction

B) indirect agglutination reaction

A test used to identify antibodies against Treponema pallidum in a patient's serum is the A) direct fluorescent-antibody test. B) indirect fluorescent-antibody test. C) direct agglutination test. D) direct ELISA test. E) hemagglutination-inhibition test.

B) indirect fluorescent-antibody test.

A viroid is a(n) A) complete, infectious virus particle. B) infectious piece of RNA without a capsid. C) capsid without nucleic acid. D) provirus. E) infectious protein.

B) infectious piece of RNA without a capsid.

Innate immunity A) is slower than adaptive immunity in responding to pathogens. B) is nonspecific and present at birth. C) involves a memory component. D) involves T cells and B cells. E) provides increased susceptibility to disease.

B) is nonspecific and present at birth.

During which growth phase will gram-positive bacteria be most susceptible to penicillin? A) lag phase B) log phase C) death phase D) stationary phase E) The culture is equally susceptible during all phases.

B) log phase

An antibody's Fc region can be bound by A) antibodies. B) macrophages. C) T helper cells. D) B cells. E) CTLs.

B) macrophages.

The lectin pathway for complement action is initiated by A) mannose on host membranes. B) mannose on the surface of microbes. C) lectins of the microbe. D) gram-negative cell walls. E) gram-positive cell walls.

B) mannose on the surface of microbes.

Palivizumab is used to treat respiratory syncytial virus disease. This antiviral drug is a(n) A) toxoid. B) monoclonal antibody. C) vaccine. D) immunosuppressive. E) nucleoside analog.

B) monoclonal antibody.

Which non-specific defense mechanism is mismatched with its associated body structure or body fluid? A) lysozyme — tears and saliva B) mucociliary escalator — intestines C) very acidic pH — stomach D) keratin and tightly packed cells — skin E) cerumen and sebum — ear

B) mucociliary escalator — intestines

What type of immunity results from recovery from mumps? A) innate immunity B) naturally acquired active immunity C) naturally acquired passive immunity D) artificially acquired active immunity E) artificially acquired passive immunity

B) naturally acquired active immunity

Which of the following does NOT achieve sterilization? A) dry heat B) pasteurization C) autoclave D) supercritical fluids E) ethylene oxide

B) pasteurization

In a hypertonic solution, a bacterial cell will typically A) burst. B) plasmolyze. C) osmolyze. D) lyse. E) stay the same.

B) plasmolyze.

A virus's ability to infect an animal cell depends primarily upon the A) host cell's ability to phagocytize viral particles. B) presence of receptor sites on the cell membrane. C) type of viral nucleic acid. D) enzymatic activity of a host cell. E) presence of pili on the host cell wall.

B) presence of receptor sites on the cell membrane.

The difference between simple diffusion and facilitated diffusion is that facilitated diffusion A) moves materials from a lower to a higher concentration. B) requires transporter proteins. C) does not require ATP. D) requires ATP. E) moves materials from a higher to a lower concentration.

B) requires transporter proteins.

Endotoxins in sterile injectable drugs could cause A) infection. B) septic shock symptoms. C) giant cell formation. D) nerve damage. E) no damage, because they are sterile.

B) septic shock symptoms.

IL-2, produced by TH cells, A) activates macrophages. B) stimulates THcell maturation. C) causes phagocytosis. D) activates antigen-presenting cells. E) activates TCcells to CTLs.

B) stimulates THcell maturation.

Bacteria that cause periodontal disease have adhesins for receptors on streptococci that colonize on teeth. This indicates that A) streptococci get bacterial infections. B) streptococcal colonization is necessary for periodontal disease. C) bacteria that cause periodontal disease adhere to gums and teeth. D) bacteria that cause periodontal disease adhere to teeth. E) streptococci cause periodontal disease.

B) streptococcal colonization is necessary for periodontal disease.

A vaccine against HIV proteins made by a genetically-engineered vaccinia virus that has infected a eukaryotic cell line is a(n) A) conjugated vaccine. B) subunit vaccine. C) nucleic acid vaccine. D) inactivated whole-agent vaccine. E) toxoid vaccine.

B) subunit vaccine.

Which of the following would be the first step in biosynthesis of a virus with a - (minus) strand of RNA? A) synthesis of DNA from an RNA template B) synthesis of double-stranded RNA from an RNA template C) synthesis of double-stranded RNA from a DNA template D) transcription of mRNA from DNA E) synthesis of DNA from a DNA template

B) synthesis of double-stranded RNA from an RNA template

Which of the following is an example of direct damage due to bacterial infection? A) the uncontrolled muscle contractions in Clostridium tetani infection B) the invasion and lysis of intestinal cells by coli C) the hemolysis of red blood cells in a staphylococcal infection D) the fever, nausea, and low blood pressure in a Salmonella infection E) the excessive secretion of fluids in a Vibrio cholera infection

B) the invasion and lysis of intestinal cells by coli

Which of the following is the best definition of generation time? A) the length of time needed for lag phase B) the length of time needed for a cell to divide C) the minimum rate of doubling D) the duration of log phase E) the time needed for nuclear division

B) the length of time needed for a cell to divide

Which of the following pairs of microbe classification terms and optimal growth temperatures is mismatched? A) psychrotroph — growth at 0°C B) thermophile — growth at 37°C C) mesophile — growth at 25°C D) psychrophile — growth at 15°C E) hyperthermophiles — growth at 85°C

B) thermophile — growth at 37°C

All of the following are part of the mechanism of action of alpha and beta interferons EXCEPT A) they bind to the surface of uninfected cells. B) they are effective for long periods. C) they initiate manufacture of antiviral proteins. D) they disrupt stages of viral multiplication. E) they initiate transcription.

B) they are effective for long periods.

Nonpathogenic Vibrio cholerae can acquire the cholera toxin gene by A) phagocytosis. B) transduction. C) conjugation. D) transformation. E) infecting a pathogenic Vibrio cholerae.

B) transduction.

In which of the following diseases can gender be considered a viable predisposing factor? A) pneumonia B) urinary tract infections C) anthrax D) salmonellosis E) tetanus

B) urinary tract infections

__________________ stimulated with ___________ differentiate into __________, which secrete antibodies into the bloodstream.

B-cells, antigen, plasma cells

Which of the following would be considered a vector? A. Water containing bacteria from fecal matter B. A fly carrying disease from fecal matter to food C. Water droplets that come from a sneeze from an infected individual D. Saliva that is transmitted between individuals during kissing

B. A fly carrying disease from fecal matter to food

Which of the following is an example of the symbiotic relationship known as mutualism? A. saprophytic Mycobacterium of the ear B. E. coli within the large intestine C. a tapeworm in the gastrointestinal tract of a human D. Corynebacterium on the surface of the eye

B. E. coli within the large intestine

Which is an example of vehicle disease transmission? A. The bite of a mosquito containing malaria B. The presence of Listeria on undercooked chicken served for dinner C. The transmission of MRSA from skin to skin contact D. Touching a telephone with cold viruses on its surface

B. The presence of Listeria on undercooked chicken served for dinner

Why are invasive procedures likely to increase the risk of nosocomial infections? A. Invasive procedures require long term hospital stays, thereby increasing the number of visitors seen by the patient. B. These procedures allow microbes from the skin to enter the bloodstream of the patient. C. Invasive procedures must use antibiotics. D. These procedures are carried out by health care workers, who carry resistant microbes.

B. These procedures allow microbes from the skin to enter the bloodstream of the patient.

How are prions different from other infectious agents? A. They lack protein. B. They lack nucleic acid. C. They cannot replicate. D. They cause cardiovascular disease.

B. They lack nucleic acid.

What is the function of the structural elements of a virus? A. To provide a source of energy for the virus B. To package and protect the viral genome C. To use all of the cell proteins

B. To package and protect the viral genome

In commensalism, A. both organisms benefit B. one organism benefits, and the other is unaffected C. both organisms are unaffected D. one organism benefits at the expense of the other

B. one organism benefits, and the other is unaffected

CHAPTER 13: VIRUSES, VIROIDS, AND PRIONS

BEGINS HERE

CHAPTER 4

BEGINS HERE

CHAPTER 6

BEGINS HERE

CHAPTER 6 & 7: QUIZ AND TEST QUESTIONS

BEGINS HERE

CHAPTER 7: THE CONTROL OF MICROBIAL GROWTH

BEGINS HERE

CHAPTERS 3 & 4: QUIZ & TEST QUESTIONS

BEGINS HERE

4) Which of the following places the steps of the Gram stain in the correct order? 1-Alcohol-acetone 2-Crystal violet 3-Safranin 4-Iodine A) 2-1-4-3 B) 1-3-2-4 C) 2-4-1-3 D) 4-3-2-1 E) 1-2-3-4

C) 2-4-1-3

In an agglutination test, eight serial dilutions to determine antibody titer were set up. Tube #1 contained a 1:2 dilution; tube #2, a 1:4, etc. If tube #6 is the last tube showing agglutination, what is the antibody titer? A) 6 B) 1:6 C) 64 D) 1:32 E) 32

C) 64

Three cells with generation times of 60 minutes are inoculated into a culture medium. How many cells are there after 5 hours? A) 900 B) 180 C) 96 D) 32 E) 15

C) 96

All of the following are disadvantages of a live virus vaccine except A) The live vaccine may revert to a more virulent form B) Exogenous protein contaminants may be present C) Antibody response is not as good as with inactivated viruses D) Live viruses generally require refrigeration E) None of the above

C) Antibody response is not as good as with inactivated viruses

What is the third step in a direct ELISA test? A) Substrate for the enzyme B) Antigen C) Antihuman immune serum D) Antibodies against the antigen

C) Antihuman immune serum

The complement protein cascade is the same for the classical pathway, alternative pathway, and lectin pathway after the point in the cascade where the activation of ________ takes place. A) C1 B) C2 C) C3 D) C5 E) C6

C) C3

Which of the following statements about the classical pathway of complement activation is FALSE? A) C1 is the first protein activated in the classical pathway. B) The C1 protein complex is initiated by antigen-antibody complexes. C) C3 is not involved in the classical pathway. D) Cleaved fragments of some of the proteins act to increase inflammation. E) C3b causes opsonization.

C) C3 is not involved in the classical pathway.

Generally, in an infection caused by a DNA-containing virus, the host animal cell supplies all of the following EXCEPT A) RNA polymerase. B) nucleotides. C) DNA polymerase. D) tRNA. E) None of the answers are correct; all of these are supplied by the host animal cell.

C) DNA polymerase.

Which of the following is true concerning a lysogenic viral replication cycle? A) During lysogeny, the viral DNA is present as a circular plasmid. B) Once the lysogenic portion of the cycle has begun, virus is never produced again. C) During lysogeny, the viral genome integrates into the host DNA, becoming a physical part of the chromosome. D) Lysogenic infections are similar to persistent infections in that virus is constantly produced.

C) During lysogeny, the viral genome integrates into the host DNA, becoming a physical part of the chromosome.

Who was the first scientist to pursue a "magic bullet" that could be used to treat infectious disease? A) Jenner B) Pasteur C) Ehrlich D) Lister E) Semmelweis

C) Ehrlich

The following steps are used to produce monoclonal antibodies. What is the fourth step? A) A B cell is activated to produce antibodies. B) Culture the hybridoma in a selective medium. C) Fuse a B cell to a myeloma cell. D) Isolate antibody-producing B cells. E) Vaccinate a mouse.

C) Fuse a B cell to a myeloma cell.

Which of the following statements is TRUE? A) All three types of interferons have the same effect on the body. B) Alpha interferon promotes phagocytosis. C) Gamma interferon causes bactericidal activity by macrophages. D) Alpha interferon acts against specific viruses. E) Beta interferon attacks invading viruses.

C) Gamma interferon causes bactericidal activity by macrophages.

DNA made from an RNA template will be incorporated into the virus capsid of A) Retroviridae. B) Herpesviridae. C) Hepadnaviridae. D) bacteriophage families. E) influenzavirus.

C) Hepadnaviridae.

The antibodies found in mucus, saliva, and tears are A) IgG. B) IgM. C) IgA. D) IgD. E) IgE.

C) IgA.

Proof that a microbe could cause disease was provided by A) Pasteur. B) Lister. C) Koch. D) Wasserman. E) Semmelweis.

C) Koch.

Which of the following recognizes antigens displayed on host cells with MHC II? A) TCcell B) B cell C) THcell D) natural killer cell E) basophil

C) THcell

Which of the following statements about nosocomial infections is FALSE? A) They may be caused by drug-resistant bacteria. B) They may be caused by opportunists. C) The patient was infected before hospitalization. D) They may be caused by normal microbiota. E) They occur in compromised patients.

C) The patient was infected before hospitalization.

How can specialized transduction contribute to the transfer of antibiotic resistance genes in a bacterial population? A) The phage lyses the bacterium and releases resistance genes into the local environment, which can then be taken up by recipient cells. B) The phage causes the destruction of any antibiotic present during the specialized transduction process. C) The prophage takes an antibiotic resistance gene with it and is packaged with the newly synthesized viral DNA.

C) The prophage takes an antibiotic resistance gene with it and is packaged with the newly synthesized viral DNA.

Which of the following statements about exotoxins is generally FALSE? A) They are more potent than endotoxins. B) They are composed of proteins. C) They are resistant to heat. D) They have specific methods of action. E) They are produced by gram-positive bacteria.

C) They are resistant to heat.

Which of the following statements provides the most significant support for the idea that viruses are nonliving chemicals? A) They are not composed of cells. B) They are filterable. C) They cannot reproduce themselves outside a host. D) They cause diseases similar to those caused by chemicals. E) They are chemically simple.

C) They cannot reproduce themselves outside a host.

Margination refers to A) the adherence of phagocytes to microorganisms. B) the chemotactic response of phagocytes. C) adherence of phagocytes to the lining of blood vessels. D) dilation of blood vessels. E) the movement of phagocytes through walls of blood vessels.

C) adherence of phagocytes to the lining of blood vessels.

A Treg cell deficiency could result in A) increased number of viral infections. B) increased number of bacterial infections. C) autoimmunity. D) increased severity of bacterial infections. E) transplant rejection.

C) autoimmunity.

Which of the following disinfectants acts by disrupting the plasma membrane? A) soaps B) aldehydes C) bisphenols D) halogens E) heavy metals

C) bisphenols

Assume a patient has influenza. During which time on the graph in Figure 13.2 would the patient show the symptoms of the illness? A) a B) b C) c D) d E) e

C) c

The most conclusive evidence that viruses cause cancers is provided by A) finding oncogenes in viruses. B) the presence of antibodies against viruses in cancer patients. C) cancer following injection of cell-free filtrates. D) treating cancer with antibodies. E) some liver cancer patients having had hepatitis.

C) cancer following injection of cell-free filtrates.

Classification of organisms into three domains is based on A) the presence of a cell wall. B) the number of cells in the organism. C) cellular organization. D) nutritional requirements. E) cellular proteins.

C) cellular organization.

Which of the following pairs is mismatched? A) endoplasmic reticulum internal transport B) Golgi complex secretion C) centrosome food storage D) lysosome digestive enzymes E) mitochondria ATP production

C) centrosome food storage

Which of the following substances is used for surgical hand scrubs? A) phenol B) chlorine bleach C) chlorhexidine D) soap E) glutaraldehyde

C) chlorhexidine

Which of the following structures is NOT found in some prokaryotic cells? A) pilus B) flagellum C) cilium D) peritrichous flagella E) axial filament

C) cilium

Haemophilus capsule polysaccharide plus diphtheria toxoid is a(n) A) inactivated whole-agent vaccine. B) attenuated whole-agent vaccine. C) conjugated vaccine. D) subunit vaccine. E) toxoid vaccine.

C) conjugated vaccine.

Superantigens produce intense immune responses by stimulating lymphocytes to produce A) endotoxins. B) exotoxins. C) cytokines. D) leukocidins. E) interferons.

C) cytokines.

A researcher has performed a prospective study on the disease tetanus. To which specific kind of epidemiological study is this referring? A) prodromal B) analytical C) descriptive D) case control E) experimental

C) descriptive

The preservation of beef jerky from microbial growth relies on which method of microbial control? A) filtration B) lyophilization C) desiccation D) ionizing radiation E) supercritical CO2

C) desiccation

Which microscope is used to see detail of a 300-nm virus? A) darkfield microscope B) phase-contrast microscope C) electron microscope D) DIC microscope E) fluorescence microscope

C) electron microscope

Patients developed inflammation a few hours following eye surgery. Instruments and solutions were sterile, and the Limulus assay was positive. The patients' inflammation was due to A) bacterial infection. B) viral infection. C) endotoxin. D) exotoxin. E) The answer cannot be determined based on the information provided.

C) endotoxin.

Bacteria such as E. coli and Salmonella produce invasins that bind host cells, thus causing the cells to A) release TNF. B) produce iron-binding proteins. C) engulf the bacteria. D) destroy the bacteria. E) release cytokines.

C) engulf the bacteria.

Most pathogens that gain access through the skin A) can penetrate intact skin. B) just infect the skin itself. C) enter through hair follicles and sweat ducts. D) must adhere first while their invasive factors allow them to penetrate. E) must be injected.

C) enter through hair follicles and sweat ducts.

Which of the following definitions is INCORRECT? A) endemic: a disease that is constantly present in a population B) pandemic: a disease that affects a large number of people in the world in a short time C) epidemic: a disease that is endemic across the world D) sporadic: a disease that affects a population occasionally E) incidence: number of new cases of a disease

C) epidemic: a disease that is endemic across the world

Which of the following chemical agents is used for sterilization? A) alcohol B) phenolics C) ethylene oxide D) chlorine E) soap

C) ethylene oxide

The data in Table 6.2 indicate that S. aureus is a(n) A) mesophile. B) facultative anaerobe. C) facultative halophile. D) aerobe. E) halophile.

C) facultative halophile.

Which of the following methods is used to preserve food by slowing the metabolic processes of foodborne microbes? A) lyophilization B) nonionizing radiation C) freezing D) ionizing radiation E) pasteurization

C) freezing

An indirect version of which test using antihuman globulin may be used to detect patient's antibodies against Treponema pallidum? A) agglutination reaction. B) complement fixation. C) immunofluorescence. D) neutralization reaction. E) precipitation reaction

C) immunofluorescence.

Vector transmission is not responsible for: A) Lyme disease. B) dengue. C) influenza. D) malaria. E) Rocky Mountain Spotted Fever.

C) influenza.

Activation of C5-C9 results in A) activation of C3. B) fixation of complement. C) lysis of microbial cells. D) phagocytosis. E) inflammation.

C) lysis of microbial cells.

A commensal bacterium: A) is beneficial to its host. B) does not receive any benefit from its host. C) may also be an opportunistic pathogen. D) does not infect its host. E) is beneficial to, and does not infect, its host.

C) may also be an opportunistic pathogen.

Which of the following is the best method to sterilize heat-labile solutions? A) dry heat B) autoclave C) membrane filtration D) pasteurization E) freezing

C) membrane filtration

Polio is transmitted by ingestion of water contaminated with feces containing polio virus. What portal of entry does polio virus use? A) skin only B) parenteral only C) mucous membranes only D) skin and parenteral E) skin, parenteral, and mucous membranes

C) mucous membranes only

Which one of the following is most resistant to chemical biocides? A) gram-negative bacteria B) gram-positive bacteria C) mycobacteria D) protozoan cysts E) viruses with lipid envelopes

C) mycobacteria

Newborns' immunity due to the transfer of antibodies across the placenta is an example of A) innate immunity. B) naturally acquired active immunity. C) naturally acquired passive immunity. D) artificially acquired active immunity. E) artificially acquired passive immunity.

C) naturally acquired passive immunity.

A DNA plasmid encoding a protein antigen from West Nile virus is injected into muscle cells of a horse. This is an example of a(n) A) subunit vaccine. B) conjugated vaccine. C) nucleic acid vaccine. D) attenuated whole-agent vaccine. E) live whole-agent vaccine.

C) nucleic acid vaccine.

Antibiotics that target cell wall synthesis ultimately cause bacterial cell death as a result of A) plasmolysis. B) inhibition of molecular transport. C) osmotic lysis. D) decreased synthesis of peptidoglycan. E) cell shrinkage.

C) osmotic lysis.

The source of nutrients in nutrient agar is A) agar. B) gelatin. C) peptone and beef extract. D) peptone and NaCl. E) agar and NaCl.

C) peptone and beef extract.

Which enzyme catalyzes the reaction: H2O2 + 2H+ → 2H2O? A) catalase B) oxidase C) peroxidase D) superoxide dismutase

C) peroxidase

A child falls and suffers a deep cut on her leg. The cut went through her skin and she is bleeding. Which of the following defense mechanisms will participate in eliminating contaminating microbes? A) mucociliary escalator B) normal skin flora C) phagocytosis in the inflammatory response D) acidic skin secretions E) lysozyme

C) phagocytosis in the inflammatory response

Which microscope is used to see internal structures of cells in a natural state? A) darkfield microscope B) fluorescence microscope C) phase-contrast microscope D) electron microscope E) compound light microscope

C) phase-contrast microscope

Which of the following is an advantage of the direct microscopic count? A) can readily count organisms that are motile B) can easily distinguish live from dead cells C) requires no incubation time D) sample volume is unknown E) requires a large number of cells

C) requires no incubation time

Which microscope can be used to visualize DNA or botulinum toxin? A) scanning electron microscope B) compound light microscope C) scanning tunneling microscope D) confocal microscope E) phase-contrast microscope

C) scanning tunneling microscope

Which of the following pairs is mismatched? A) darkfield microscope —uses visible light B) fluorescence microscope —uses a fluorescent light C) scanning tunneling microscope —allows visualization of atoms D) scanning electron microscope —produces a three-dimensional image E) confocal microscope —produces a three-dimensional image

C) scanning tunneling microscope —allows visualization of atoms

The following steps occur during biosynthesis of a + strand RNA virus. What is the third step? A) attachment B) penetration and uncoating C) synthesis of - strand RNA D) synthesis of + strand RNA E) synthesis of viral proteins

C) synthesis of - strand RNA

Which of the following is NOT a communicable diseases? A) AIDS B) typhoid fever C) tetanus D) tuberculosis E) malaria

C) tetanus

Where are phospholipids most likely found in a prokaryotic cell? A) flagella B) around organelles C) the plasma membrane D) ribosomes E) the plasma membrane and around organelles

C) the plasma membrane

Assume you inoculated 100 cells, with a generation time of 20 minutes, into 100 ml of nutrient broth. You then inoculated 100 cells of the same species into 200 ml of nutrient broth. After incubation for 4 hours, you can reasonably expect to have A) more cells in the 100 ml. B) more cells in the 200 ml. C) the same number of cells in both. D) The answer cannot be determined based on the information provided.

C) the same number of cells in both.

The following is true of quarternary ammonium compounds EXCEPT A) they are non-toxic at lower concentrations. B) they are tasteless. C) they are effective when combined with soaps. D) they are stable. E) they may be an ingredient in mouthwash.

C) they are effective when combined with soaps.

Which microscope is used to see intracellular detail in a living cell? A) brightfield microscope B) fluorescence microscope C) two-photon microscope D) atomic force microscope E) transmission electron microscope

C) two-photon microscope

Which of the following would be an example of disease transmission via INDIRECT contact? A. A tick that is infected with Lyme disease bites a hiker, and the hiker gets Lyme disease. B. Mr. Smith sneezes on an airplane, and the person sitting beside him catches his cold. C. A student sneezes on her test booklet. The instructor grades it and catches her cold. D. A restaurant worker has diarrhea caused by Norovirus. He fails to wash his hands adequately before preparing the salad, and his customers get sick.

C. A student sneezes on her test booklet. The instructor grades it and catches her cold.

An example of a contagious disease is A. anthrax B. tetanus C. chickenpox D. rabies

C. chickenpox

The following choices list several types of diseases, along with factors that may contribute to their emergence. Which disease and associated factor do NOT match? A. Increased incidence of Lyme disease: increases in deer populations B. spread of cholera to new regions: modern travel C. emergence of avian influenza A (H5N1): use of antibiotics D. increased incidence of malaria: global warming

C. emergence of avian influenza A (H5N1): use of antibiotics

Malaria is an infectious disease caused by infection with a protozoan. In certain tropical regions, malaria is constantly present. We would say that malaria is a(n) _________ disease in these regions. A. sporadic B. epidemic C. endemic D. pandemic

C. endemic

A nurse is working in southeast Asia, where the incidence of Mycobacterium tuberculosis is very high. Why is it necessary to select a specific method of disinfection for this organism? a. The presence of an envelope makes this organism relatively resistant to disinfectants. b. The waxy, lipid-rich components of the cell wall make this organism relatively resistant to disinfectants. c. The presence of a capsule makes this organism more resistant to disinfectants. d. Bacterial endospores made by this organism are resistant to disinfectants.

b. The waxy, lipid-rich components of the cell wall make this organism relatively resistant to disinfectants.

The cellular immune response can seem a bit more complex than the humoral response because there are more cell types involved and more than one outcome for many of these cell types. Let's focus on the different T cell subtypes and their role in the cell-mediated response. Identify the following statements regarding cell-mediated immunity as either correct or incorrect.

Correct: - The cellular immune response is mediated by T cells. - CD8+ T cells are T cytotoxic (TC) cells that bind to MHC class I molecules and can differentiate into an effector cytotoxic T lymphocyte (CTL). - The recognition of antigens by a T cell requires that an antigen-presenting cell (APC) first process them. - Cytotoxic T lymphocytes (CTLs) can use perforin, a pore-forming protein, to kill self cells that have been altered by infection with a pathogen. - T cells, like B cells, are specific for a particular antigen. - T helper cells aid in both the humoral and cellular immune responses. - CD4+ T cells are helper cells that bind to the major histocompatibility complex (MHC) II class molecules on B cells and antigen-presenting cells (APCs). Incorrect: - T helper (TH) cells differentiate primarily into two different subsets, TH1 and TH10. - The cell-mediated response functions to target and effectively remove freely circulating pathogens where antibodies can come in contact with them. - T cells are classified by their clusters of differentiation (CD), which serve as receptors. The most important CD classes for cell-mediated immunity are CD4 and CD6.

Thirty-six colonies grew in nutrient agar from 1.0 ml of undiluted sample in a standard plate count. How many cells were in the original sample? A) 4 per milliliter B) 9 per milliliter C) 18 per milliliter D) 36 per milliliter E) 72 per milliliter

D) 36 per milliliter

A sample of milk is tested for its bacterial content in a plate count assay. A one-milliliter sample of the milk is diluted in a 1:10 dilution series. One milliliter of the third dilution tube is plated in a pour plate. After incubation, the plate has 54 colonies, indicating that the original milk sample contained A) 54 cells per milliliter. B) 540 cells per milliliter. C) 5,400 cells per milliliter. D) 54,000 cells per milliliter. E) 540,000 cells per milliliter.

D) 54,000 cells per milliliter.

Which of the following statements is TRUE? A) Symbiosis refers to different organisms living together and benefiting from each other. B) Members of a symbiotic relationship cannot live without each other. C) At least one member must not benefit in a symbiotic relationship. D) At least one member must benefit in a symbiotic relationship. E) A parasite is not in symbiosis with its host.

D) At least one member must benefit in a symbiotic relationship.

Which of the following is NOT a characteristic of cellular immunity? A) The cells originate in bone marrow. B) Cells are processed in the thymus gland. C) It can inhibit the immune response. D) B cells make antibodies. E) T cells react with antigens.

D) B cells make antibodies.

The formal system for classifying and naming organisms was developed by A) Robert Koch. B) Ignaz Semmelweis. C) Aristotle. D) Carolus Linnaeus. E) Louis Pasteur.

D) Carolus Linnaeus.

If cells are grown in media containing amino acids labeled with radioactive nitrogen (15N), most of the radioactivity will be found in the cells' A) DNA. B) proteins. C) phospholipids. D) DNA and proteins. E) DNA and phospholipids.

D) DNA and proteins.

In Figure 14.2, when is the prevalence the highest? A) July B) January C) March D) February E) The answer cannot be determined based on the information provided.

D) February

All of the following protect the skin and mucous membranes from infection EXCEPT A) multiple layers of cells. B) tears. C) saliva. D) HCl. E) the "ciliary escalator."

D) HCl.

Which of the following is a limitation of the autoclave? A) It requires an excessively long time to achieve sterilization. B) It cannot inactivate viruses. C) It cannot kill endospores. D) It cannot be used with heat-labile materials. E) It cannot be used with glassware.

D) It cannot be used with heat-labile materials.

Which of the following statements about IL-12 is FALSE? A) It activates macrophages. B) It inhibits some tumor cells. C) It activates the TH1 pathway. D) It causes autoimmune diseases. E) It causes THcells to respond to HIV.

D) It causes autoimmune diseases.

Which of the following statements about staphylococcal enterotoxin is FALSE? A) It causes vomiting. B) It causes diarrhea. C) It is an exotoxin. D) It is produced by Staphylococcus aureus growing in the host's intestines. E) It is a superantigen.

D) It is produced by Staphylococcus aureus growing in the host's intestines.

Which of the following statements about M protein is FALSE? A) It is found on Streptococcus pyogenes. B) It is found on fimbriae. C) It is heat- and acid-resistant. D) It is readily digested by phagocytes. E) It is a protein.

D) It is readily digested by phagocytes.

Gram-negative septic shock results from the following events. What is the second step? A) Body temperature is reset in the hypothalamus. B) Fever occurs. C) IL-1 is released. D) LPS is released from gram-negative bacteria. E) Phagocytes ingest gram-negative bacteria.

D) LPS is released from gram-negative bacteria.

Which of the following WBCs are NOT lymphocytes? A) cytotoxic T cells B) helper T cells C) NK cells D) M cells E) B cells

D) M cells

Which of the following is necessary for replication of a prion? A) DNA B) DNA polymerase C) lysozyme D) PrPSc E) RNA

D) PrPSc

How do spirochetes and spirilla differ? A) Spirochetes have a rigid, corkscrew shape while spirilla are helical and more flexible. B) Spirochetes and spirilla are basically the same organisms and the terms can be used interchangeably. C) Spirilla are found in chains of cells whereas spirochetes exist as individual cells. D) Spirilla have an external flagella but spirochetes have axial filaments. E) Spirochetes do not have a cell wall but spirilla do.

D) Spirilla have an external flagella but spirochetes have axial filaments.

The following events elicit an antibody response. What is the third step? A) Antigen-digest goes to surface of APC. B) APC phagocytizes antigen. C) B cell is activated. D) THcell recognizes antigen-digest and MHC II. E) THcell produces cytokines.

D) THcell recognizes antigen-digest and MHC II.

What will happen if a bacterial cell is placed in distilled water with lysozyme? A) The cell will plasmolyze. B) Lysozyme will diffuse into the cell. C) Water will leave the cell. D) The cell will undergo osmotic lysis. E) No change will result; the solution is isotonic.

D) The cell will undergo osmotic lysis.

Which of the following statements is FALSE? A) The variable region of a heavy chain is partially responsible for binding with antigen. B) The variable region of a light chain is partially responsible for binding with antigen. C) The Fc region attaches to a host cell. D) The constant region of a heavy chain is the same for all antibodies. E) All of the answers are correct.

D) The constant region of a heavy chain is the same for all antibodies.

Which of the following statements about measles is FALSE? A) It is a serious disease. B) It is preventable by vaccination. C) Annually, it kills thousands of children worldwide. D) The disease has been eradicated in the United States. E) Complications include pneumonia, encephalitis, and death.

D) The disease has been eradicated in the United States.

Which of the following treatments is the most effective for controlling microbial growth? A) 63°C for 30 minutes B) 72°C for 15 seconds C) 140°C for 4 seconds D) They are equivalent treatments. E) None of the answers is correct.

D) They are equivalent treatments.

Antibodies for serological testing can be obtained from all of the following except A) Vaccinated humans B) Vaccinated animals C) Monoclonal antibodies D) Viral cultures E) None of the above

D) Viral cultures

Which of the following statements about viruses is FALSE? A) Viruses contain DNA or RNA but never both. B) Viruses contain a protein coat. C) Viruses use the anabolic machinery of the cell. D) Viruses use their own catabolic enzymes. E) Viruses have genes.

D) Viruses use their own catabolic enzymes.

Which of the following statements best describes what happens when a bacterial cell is placed in a solution containing 5 percent NaCl? A) Water will move into the cell. B) Sucrose will move into the cell from a higher to a lower concentration. C) The cell will undergo osmotic lysis. D) Water will move out of the cell. E) No change will result; the solution is isotonic.

D) Water will move out of the cell.

The fimbriae of Neisseria gonorrhea and enteropathogenic E. coli are examples of A) adhesins. B) ligands. C) receptors. D) adhesins and ligands. E) adhesins, ligands, and receptors.

D) adhesins and ligands.

Toxoid vaccines, such as the vaccines against diphtheria and tetanus, elicit a(n) A) TCcell response. B) immune complex. C) dendritic cell proliferation. D) antibody response against these bacterial toxins. E) antibody response against gram-positive bacteria.

D) antibody response against these bacterial toxins.

Monoclonal antibodies are used in diagnostic tests and disease treatments because they A) are highly specific. B) can be produced in large quantities. C) contain a mixture of antibodies. D) are highly specific and they can be produced in large quantities. E) are highly specific, they can be produced in large quantities, and they contain a mixture of antibodies.

D) are highly specific and they can be produced in large quantities.

What type of vaccine is the live, weakened measles virus? A) conjugated vaccine B) subunit vaccine C) nucleic acid vaccine D) attenuated whole-agent vaccine E) toxoid vaccine

D) attenuated whole-agent vaccine

Patients with indwelling catheters (long-term tubes inserted into body orifices for drainage, such as through the urethra and into the urinary bladder) are susceptible to infections because A) injected solutions are contaminated. B) their immune systems are weakened. C) infections can be transmitted from other people. D) biofilms develop on catheters. E) bacteria cause infections.

D) biofilms develop on catheters.

Bacteriophages and animal viruses do NOT differ significantly in which one of the following steps? A) attachment B) penetration C) uncoating D) biosynthesis E) release

D) biosynthesis

Which of the following does NOT contribute to the symptoms of a fungal disease? A) capsules B) toxins C) allergic response of the host D) cell walls E) metabolic products

D) cell walls

Simple staining is often necessary to improve contrast in which microscope? A) phase-contrast microscope B) fluorescence microscope C) darkfield microscope D) compound light microscope E) electron microscope

D) compound light microscope

Continuous cell lines differ from primary cell lines in that A) viruses can be grown in continuous cell lines. B) continuous cell lines always have to be re-isolated from animal tissues. C) continuous cell lines are derived from primary cell lines. D) continuous cell lines can be maintained through an indefinite number of generations. E) continuous cell lines are from human embryos.

D) continuous cell lines can be maintained through an indefinite number of generations.

Twenty-five people developed symptoms of nausea, vomiting, and diarrhea three to six hours after attending a church picnic where they ate a ham and green bean casserole with cream sauce. The most likely cause of this case of food intoxication is A) botulinum toxin. B) aflatoxin. C) staphylococcal enterotoxin. D) erythrogenic toxin. E) cholera toxin.

D) erythrogenic toxin.

Which of the following uses fluorescent-labeled antibodies? A) agglutination B) complement fixation C) precipitation D) flow cytometry E) neutralization

D) flow cytometry

All of the following are examples of entry via the parenteral route EXCEPT A) injection. B) bite. C) surgery. D) hair follicle. E) skin cut.

D) hair follicle.

You have isolated a motile, gram-positive cell with no visible nucleus. You can safely assume that the cell A) lives in an extreme environment. B) has 9 pairs + 2 flagella. C) has a mitochondrion. D) has a cell wall. E) has cilia.

D) has a cell wall.

Salts and sugars work to preserve foods by creating a A) depletion of nutrients. B) hypotonic environment. C) lower osmotic pressure. D) hypertonic environment. E) lower pH.

D) hypertonic environment.

Which of the following tests is MOST useful in determining the presence of AIDS antibodies? A) agglutination B) complement fixation C) neutralization D) indirect ELISA E) direct fluorescent-antibody

D) indirect ELISA

Which of the following mechanisms is used by gram-negative bacteria to cross the blood-brain barrier? A) producing fimbriae B) inducing endocytosis C) producing toxins D) inducing TNF E) antigenic variation

D) inducing TNF

Bacteriophage replication differs from animal virus replication because only bacteriophage replication involves A) adsorption to specific receptors. B) assembly of viral components. C) replication of viral nucleic acid. D) injection of naked nucleic acid into the host cell. E) lysis of the host cell.

D) injection of naked nucleic acid into the host cell.

Ethylene oxide A) is a good antiseptic. B) is not sporicidal. C) requires high heat to be effective. D) is a sterilizing agent. E) is the active chemical in household bleach.

D) is a sterilizing agent.

A disease in which the causative agent remains inactive for a time before producing symptoms is referred to as A) acute. B) subacute. C) subclinical. D) latent. E) zoonotic.

D) latent.

Phagocytes utilize all of the following to optimize interaction with (getting to and getting hold of) microorganisms EXCEPT A) trapping a bacterium against a rough surface. B) opsonization. C) chemotaxis. D) lysozyme. E) complement.

D) lysozyme.

Which of the following is NOT a direct method to measure microbial growth? A) direct microscopic count B) standard plate count C) filtration on a support membrane followed by incubation on medium D) metabolic activity E) most probable number (MPN)

D) metabolic activity

All of the following are methods of food preservation EXCEPT A) desiccation. B) high pressure. C) ionizing radiation. D) microwaves. E) osmotic pressure.

D) microwaves.

Which of the following organelles most closely resembles a prokaryotic cell? A) Golgi complex B) cell wall C) nucleus D) mitochondrion E) vacuole

D) mitochondrion

A clear area against a confluent "lawn" of bacteria is called a A) phage. B) pock. C) cell lysis. D) plaque. E) rash.

D) plaque.

7) The purpose of a mordant in the Gram stain is to A) remove the simple stain. B) make gram-negative cells visible. C) make the flagella visible. D) prevent the crystal violet from leaving the cells. E) make the bacterial cells larger.

D) prevent the crystal violet from leaving the cells.

Which disease has been eliminated through the use of vaccines? A) tuberculosis B) measles C) rubella D) smallpox E) influenza

D) smallpox

Isolated and purified hepatitis B virus surface antigen can be used in a(n) A) inactivated whole-agent vaccine. B) attenuated whole-agent vaccine. C) conjugated vaccine. D) subunit vaccine. E) toxoid vaccine.

D) subunit vaccine.

Symptoms of intense inflammation and shock occur in some gram-positive bacterial infections due to A) A-B toxins. B) lipid A. C) membrane-disrupting toxins. D) superantigens. E) erythrogenic toxin.

D) superantigens.

All of the following are true regarding NK cells EXCEPT A) they are a type of lymphocyte. B) they are found in tissues of the lymphatic system. C) they have the ability to kill infected body cells and some tumor cells. D) they destroy infected body cells by phagocytosis. E) they release toxic substances that cause cell lysis or apoptosis.

D) they destroy infected body cells by phagocytosis.

Oxygen crosses a plasma membrane A) by osmosis. B) through facilitated diffusion. C) through porins. D) through simple diffusion. E) with the help of a nonspecific transporter.

D) through simple diffusion.

Assume you have isolated an unknown virus. This virus has a single, positive sense strand of RNA, and possesses an envelope. To which group does it most likely belong? A) herpesvirus B) picornavirus C) retrovirus D) togavirus E) papovavirus

D) togavirus

A gram-positive bacteria suddenly acquires resistance to the antibiotic methicillin. This trait most likely occurred due to A) binary fission. B) meisosis. C) conjugation. D) transformation. E) transduction.

D) transformation.

Which of the following are sources of antibodies for serological testing? A) vaccinated animals B) cells producing monoclonal antibodies C) viral cultures D) vaccinated animals and cells producing monoclonal antibodies E) vaccinated animals, cells producing monoclonal antibodies, and viral cultures

D) vaccinated animals and cells producing monoclonal antibodies

Which of the following is true regarding cultivation and isolation of animal viruses? A. Viruses can be easily grown in liquid culture without any other organisms present. B. Mouse models are available for virtually all human viral infections and can be routinely used for vaccine development. C. The preferred and widely used method of viral isolation and growth is via growth in an embryonated egg. D. Diploid cell culture lines, developed from human embryos, are widely used for culturing viruses that require a human host.

D. Diploid cell culture lines, developed from human embryos, are widely used for culturing viruses that require a human host.

How can health care workers reduce the occurrence of nosocomial infections? A. Reduce the number of times they visit a patient B. Limit the number of visitors who can see the patient C. Administer all medications orally instead of through injections D. Practice more stringent aseptic techniques

D. Practice more stringent aseptic techniques

The normal function of the PrP protein in mammals is believed to be: A. assisting in normal membrane development and function. B. assisting proteins in forming beta-pleated sheets. C. assisting proteins in forming alpha-helices. D. assisting in normal synaptic development and function.

D. assisting in normal synaptic development and function.

Which of the following is classified as a latent disease? A. infectious mononucleosis B. tuberculosis C. influenza D. shingles

D. shingles

__________ bacteria may survive pasteurization. a. Psychotroph b. Thermoduric c. Mesophiles d. Psychrophile

b. Thermoduric

16) A virus measures 100 nm in length. What is its length in μm? A) 0.01 μm B) 1 μm C) 0.001 μm D) 10 μm E) .1 μm

E) .1 μm

Which of the following regarding antimicrobial control agents is FALSE? A) Contaminating organic debris such as blood or sputum will decrease effectiveness. B) Some agents kill by denaturing microbial cell proteins. C) Some agents affect microbial cell membranes by dissolving lipids. D) Some agents are utilized as both an antiseptic and a disinfectant. E) A true antimicrobial control agent is equally effective against both bacteria and viruses.

E) A true antimicrobial control agent is equally effective against both bacteria and viruses.

All of the following are used by bacteria to attach to host cells EXCEPT A) M protein. B) ligands. C) fimbriae. D) capsules. E) A-B toxins.

E) A-B toxins.

Which of the following statements is FALSE? A) Leukocidins destroy neutrophils. B) Hemolysins lyse red blood cells. C) Hyaluronidase breaks down substances between cells. D) Kinase destroys fibrin clots. E) Coagulase destroys blood clots.

E) Coagulase destroys blood clots.

Which of the following statements is NOT true of lysogeny? A) It can give infected pathogens the genetic information for toxin production. B) Prophage is inserted into the host genome. C) Lytic cycle may follow lysogeny. D) It is a "silent" infection; the virus does not replicate. E) It causes lysis of host cells.

E) It causes lysis of host cells.

A patient's serum, Mycobacterium, guinea pig complement, sheep red blood cells, and anti-sheep red blood cell antibodies are mixed in a test tube. What happens if the patient has antibodies to Mycobacterium? A) Bacteria fluoresce. B) Hemagglutination occurs. C) Hemagglutination-inhibition occurs. D) Hemolysis occurs. E) No hemolysis occurs.

E) No hemolysis occurs.

Which of the following tests is not correctly matched to its positive reaction? A) Hemagglutination - clumping of red blood cells B) Complement fixation - no hemolysis C) Neutralization - no tissue/animal death D) ELISA - enzyme-substrate reaction E) None of the above

E) None of the above

Which one of the following is NOT a zoonosis? A) Hantavirus pulmonary syndrome (rodents) B) rabies (from a bite) C) cat-scratch disease (scratched, licked, or bitten by cat) D) tapeworm (worm) E) None of the answers is correct; all of these are zoonoses.

E) None of the answers is correct; all of these are zoonoses.

Which of the following is NOT a reservoir of infection? A) a hospital B) a healthy person C) a sick person D) a sick animal E) None of the answers is correct; all of these can be reservoirs of infection.

E) None of the answers is correct; all of these can be reservoirs of infection.

The following events occur in cellular immunity, leading to a response from TH cells. What is the third step? A) Antibodies are produced. B) Dendritic cell takes up antigen. C) Antigen enters M cell. D) THcell produces cytokines. E) THcells proliferate.

E) THcells proliferate.

A patient shows the presence of antibodies against diphtheria toxin. Which of the following statements is FALSE? A) The patient may have the disease. B) The patient may have had the disease and has recovered. C) The patient may have been vaccinated. D) A recent transfusion may have passively introduced the antibodies. E) The patient was near someone who had the disease.

E) The patient was near someone who had the disease.

Which of the following statements is FALSE? A) A prophage is phage DNA inserted into a bacterial chromosome. B) A prophage can "pop" out of the chromosome. C) Prophage genes are repressed by a repressor protein coded for by the prophage. D) A prophage may result in new properties of the host cell. E) The prophage makes the host cell immune to infection by other phages.

E) The prophage makes the host cell immune to infection by other phages.

Which of the following statements about viral spikes is FALSE? A) They are composed of carbohydrate-protein complexes. B) They are used for attachment. C) They may cause hemagglutination. D) They bind to receptors on the host cell surface. E) They are found only on nonenveloped viruses.

E) They are found only on nonenveloped viruses.

Which of the following statements about gram-negative cell walls is FALSE? A) They are toxic to humans. B) They have an extra outer layer composed of lipoproteins, lipopolysaccharides, and phospholipids. C) Their Gram reaction is due to the outer membrane. D) They protect the cell in a hypotonic environment. E) They are sensitive to penicillin.

E) They are sensitive to penicillin.

Which of the following is a fomite? A) insects B) droplets from a sneeze C) pus D) water E) a hypodermic needle

E) a hypodermic needle

You find colorless areas in cells in a gram-stained smear. What should you apply next? A) an acid-fast stain B) a capsule stain C) a flagella stain D) a simple stain E) an endospore stain

E) an endospore stain

All of the following are generally used in vaccines EXCEPT A) toxoids. B) parts of bacterial cells. C) live, attenuated viruses. D) inactivated viruses. E) antibodies.

E) antibodies.

What type of immunity results from transfer of antibodies from one individual to a susceptible individual by means of injection? A) innate immunity B) naturally acquired active immunity C) naturally acquired passive immunity D) artificially acquired active immunity E) artificially acquired passive immunity

E) artificially acquired passive immunity

Which of the following are NOT energy reserves? A) polysaccharide granules B) sulfur granules C) metachromatic granules D) lipid inclusions E) carboxysomes

E) carboxysomes

This microscope produces an image of a light cell against a dark background; internal structures are NOT visible. A) electron microscope B) compound light microscope C) phase-contrast microscope D) fluorescence microscope E) darkfield microscope

E) darkfield microscope

Which microscope uses visible light? A) scanning acoustic microscope B) confocal microscope C) fluorescence microscope D) scanning electron microscope E) differential interference contrast microscope

E) differential interference contrast microscope

Assume a patient had chickenpox (human herpesvirus 3) as a child. Which line on the graph in Figure 13.2 would show the number of viruses present in this person as a 60-year-old with shingles (human herpesvirus 3)? A) a B) b C) c D) d E) e

E) e

In Figure 17.1, which letter on the graph indicates the highest antibody titer during the patient's response to a second and distinct/different antigen? A) a B) b C) c D) d E) e

E) e

In Figure 17.2, what portion will typically attach to a host cell? A) a and c B) b C) b and c D) a and d E) e

E) e

Which microscope is used to observe a specimen that emits light when illuminated with an ultraviolet light? A) electron microscope B) darkfield microscope C) compound light microscope D) phase-contrast microscope E) fluorescence microscope

E) fluorescence microscope

A needlestick is an example of A) direct contact. B) vector. C) droplet transmission. D) vehicle transmission. E) fomite.

E) fomite.

Which of the following results in lethal damage to nucleic acids? A) heat B) radiation C) certain chemicals D) heat and radiation E) heat, radiation, and some chemicals

E) heat, radiation, and some chemicals

Patient's serum, influenza virus, and red blood cells are mixed in a tube. What happens if the patient has antibodies against influenza virus? A) agglutination B) hemagglutination C) complement fixation D) hemolysis E) hemagglutination-inhibition (none of the above)

E) hemagglutination-inhibition (none of the above)

In which of the following patterns of disease does the patient experience no signs or symptoms? A) prodromal B) decline C) convalescence D) incubation E) incubation and convalescence

E) incubation and convalescence

The antimicrobial effects of AMPs include all of the following EXCEPT A) inhibition of cell wall synthesis. B) lysis of bacterial cells. C) destruction of nucleic acids. D) pore formation in bacterial membranes. E) inhibition of phagocytosis.

E) inhibition of phagocytosis.

An iodophor is a(n) A) phenol. B) agent that reduces oxygen. C) quaternary ammonium compound. D) form of formaldehyde. E) iodine mixed with a surfactant.

E) iodine mixed with a surfactant.

The DNA found in most bacterial cells A) utilizes histones for chromosomal packaging. B) is found in multiple copies. C) is surrounded by a nuclear membrane. D) is linear in structure. E) is circular in structure.

E) is circular in structure.

Focal infections initially start out as A) sepsis. B) systemic infections. C) bacteremia. D) septicemia. E) local infections.

E) local infections.

Bacteria are a commonly used organism for studies of genetic material in the research laboratory. The nucleic acids must first be isolated from the cells for these studies. Which of the following would most likely be used to lyse the bacterial cells for nucleic acid isolation? A) polymixins B) water C) alcohol D) mycolic acid E) lysozyme

E) lysozyme

Which one of the following causes transmembrane channels in target cells? A) granzymes B) hapten C) IL-1 D) IL-2 E) perforin

E) perforin

Which of the following is a test to determine the presence of soluble antigens in a patient's saliva? A) direct agglutination reaction B) passive agglutination reaction C) immunofluorescence D) neutralization reaction E) precipitation reaction

E) precipitation reaction

A reaction between an antibody and soluble antigen-forming lattices is called a(n) A) agglutination reaction. B) complement fixation. C) immunofluorescence. D) neutralization reaction. E) precipitation reaction.

E) precipitation reaction.

If you were preparing nutrient agar at home and did not have an autoclave, what could you use to sterilize the nutrient agar? A) bleach B) boiling for one hour C) hydrogen peroxide D) oven at 121°C for one hour E) pressure cooker at 121°C for 15 minutes

E) pressure cooker at 121°C for 15 minutes

All of the following are effects of histamine EXCEPT A) destruction of an injurious agent. B) removal of an injurious agent. C) isolation of an injurious agent. D) repair of damaged tissue. E) production of antibodies.

E) production of antibodies.

Which microscope is most useful for visualizing a biofilm? A) phase-contrast microscope B) compound light microscope C) fluorescence microscope D) transmission electron microscope E) scanning acoustic microscope

E) scanning acoustic microscope

Microorganisms are involved in each of the following processes EXCEPT A) infection. B) decomposition of organic material. C) O2 production. D) food production. E) smog production.

E) smog production.

Where are phospholipids most likely found in a eukaryotic cell? A) ribosomes B) the plasma membrane C) surrounding flagella D) around organelles E) the plasma membrane, around organelles, and surrounding flagella

E) the plasma membrane, around organelles, and surrounding flagella

Which of the following is NOT a cytopathic effect of viruses? A) cell death B) host cells fusing to form multinucleated syncytia C) inclusion bodies forming in the cytoplasm or nucleus D) increased cell growth E) toxin production

E) toxin production

Inactivated tetanus toxin is a(n) A) conjugated vaccine. B) subunit vaccine. C) nucleic acid vaccine. D) inactivated whole-agent vaccine. E) toxoid vaccine.

E) toxoid vaccine.

Which microscope is used to observe viruses and the internal structure of thinly sectioned cells? A) brightfield microscope B) fluorescence microscope C) darkfield microscope D) scanning electron microscope E) transmission electron microscope

E) transmission electron microscope

The following steps occur during multiplication of herpesviruses. Which is the third step? A) attachment B) biosynthesis C) penetration D) release E) uncoating

E) uncoating

All of the following occur during inflammation. What is the first step? A) diapedesis B) margination C) phagocyte migration D) repair E) vasodilation

E) vasodilation

The clumping of test red blood cells indicates a negative test result (no antibodies against the virus in the patient's serum) in the A) direct agglutination test. B) indirect agglutination test. C) complement-fixation test. D) precipitation test. E) viral hemagglutination inhibition test.

E) viral hemagglutination inhibition test.

Starch is readily metabolized by many cells, but a starch molecule is too large to cross the plasma membrane. How does a cell obtain the glucose molecules from a starch polymer? How does the cell tranposrt these glucose molecules across the plasma membrane?

Endocytosis occurs to engulf and take in a starch polymer. The cell uses active transport to move the glucose molecules across the plasma membrane

Consider the following five infectious diseases: Ebola hemorrhagic fever avian influenza SARS West Nile encephalitis mad cow disease What do these diseases have in common? a. They are all caused by viruses. b. They are all emerging infectious diseases. c. They are all transmitted by arthropod vectors. d. They all exhibit airborne transmission.

b. They are all emerging infectious diseases.

As you can see from this activity, the immune system is very complex and is made of many components. In this section, place each immune component in the appropriate bin.

First Line of Defense: - sebum (oil) - sweat Second Line of Defense: - fever - NK cells Third Line of Defense: - T lymphocytes - B lymphocytes

Which of the following is an accurate statement about A-B toxins? a. They are a type of endotoxin. b. They are proteins. c. The A component binds to a receptor on the host cell. d. The A and B components must remain attached in order to exert their effects on the cell.

b. They are proteins.

Antiviral Action of Alpha and Beta Interferons: 1. Viral RNA from an infecting virus enters the cell. 2. The virus induces the host cell to produce interferon mRNA, which is translated into alpha and beta interferons. 3. Interferons make contact with uninfected neighboring host cells, where they bind either to the plasma membrane or to nuclear receptors. Interferons induce the cells to synthesize antiviral proteins (AVPs). 4. AVPs degrade viral mRNA and inhibit protein synthesis - and this interfere with viral replication.

GO OVER STEPS

Clonal Selection and Differentiation of B cells: 1. Stem cells differentiate into mature B cells; each bearing surface Igs against a specific antigen. 2. B cell II encounters its specific antigen and proliferates. 3. Some B cells proliferate into long-lived memory cells, which at a later date can be stimulated to become antibody-producing plasma cells. 4. Other B cells proliferate into antibody-producing plasma cells. 5. Plasma cells secrete antibodies into circulation.

GO OVER THE STEPS

How do superantigens enable pathogens to hide from the immune system if they actually stimulate the immune system? a. They cause the immune system to destroy IgA antibodies. b. They cause the immune system to produce an exaggerated response, distracting it from the actual pathogen. c. They cause fever, which destroys the complement proteins. d. They cause the immune system to turn on itself.

b. They cause the immune system to produce an exaggerated response, distracting it from the actual pathogen.

After the attachment and entry of a virus into a host cell, what is the next step in the multiplication of animal viruses? a. Transcription of "early" genes b. Uncoating c. Synthesis of capsid proteins d. Transcription of "late" genes e. Viral DNA is degraded by enzymes

b. Uncoating

Which of the following is NOT characteristic of viruses? a. Viruses lack an ATP-generating mechanism. b. Viral nucleic acid is surrounded by a plasma membrane. c. Viruses have either DNA or RNA but not both. d. Viruses are unaffected by antibiotics.

b. Viral nucleic acid is surrounded by a plasma membrane.

How do all viruses differ from bacteria? a. Viruses do not reproduce. b. Viruses are not composed of cells. c. Viruses do not have any nucleic acid. d. Viruses are filterable. e. Viruses are obligate intracellular parasites.

b. Viruses are not composed of cells.

In one hospital, Pseudomonas aeruginosa serotype 10 infected the biliary tract of 10 percent of 1300 patients who underwent gastrointestinal endoscopic procedures. After each use, endoscopes were washed with an automatic reprocessor that flushed detergent and glutaraldehyde through the endoscopes, followed by a tap water rinse. P. aeruginosa serotype 10 was not isolated from the detergent, glutaraldehyde, or tap water. What was the source of the infections? a. fecal contamination of the bile ducts b. a biofilm in the reprocessor c. bacterial cell walls in the water d. contaminated disinfectant e. None of the answers is correct.

b. a biofilm in the reprocessor

in an immunodiffusion test, a strip of filter paper containing diphtheria antitoxin is placed on a solid culture medium. Then bacteria are streaked perpendicular to the filter paper. If the bacteria are toxigenic: a. the filter paper would turn red b. a line of antigen-antibody precipitate will form c. the cells will lyse d. the cells will fluoresce e. none of the above

b. a line of antigen-antibody precipitate will form

A population of bacterial cells has been placed in a very nutrient-poor environment with extremely low concentrations of sugars and amino acids. Which kind of membrane transport becomes crucial in this environment? a. facilitated diffusion b. active transport c. pinocytosis d. osmosis e. simple diffusion

b. active transport

An organism that has peroxidase and superoxide dismutase but lacks catalase is most likely an a. aerobe. b. aerotolerant anaerobe. c. obligate anaerobe.

b. aerotolerant anaerobe.

Which immunoglobulin class, found in blood, lymph, and the intestine, can cross the placenta and help to protect a fetus?

IgG

If a patient has been exposed to an antigen for the first time, which class of immunoglobulin appears first?

IgM

What is the purpose of a decolorizer in the Gram stain? In the acid-fast stain?

In the Gram stain, the decolorizer removes the color from gram-negative cells. In the acid-fast stain, the decolorizer removes the color from the non-acid-fast cells.

Activation of CD4+ T Helper Cells: 1. An APC encounters and ingests a microorganism. The antigen is enzymatically processed into short peptides, which combine with MHC class II molecules and are displayed on the surface of the APC. 2. A receptor (TCR) on the surface of the CD4+ T helper cell binds to the MHC-antigen complex. This includes a toll-like receptor. The TH cell or APC is stimulated to secrete a costimulatory molecule. These two signals activate the TH cell, which produces cytokines. 3. The cytokines cause the TH cell (which recognize a dendritic cell that is producing costimulatory molecules) to become activated.

JUST KNOW AN OVERVIEW OF THE STEPS

Activation of B cells to Produce Antibodies: 1. B cell receptors recognize and attach to extraceullar antigens. 2. Antigen is internalized into the B cell. 3. Fragments of the antigen are presented on MHC proteins on the surface of the cell. (Class II) 4. A T helper cell that recognizes this antigen fragment is activated and releases cytokines, activating the B cell. 5. The activated B cell begins clonal expansion, producing an array of antibody-producing plasma cells and memory cells.

KNOW THE STEPS!

Principle Vaccines used for Viral Diseases: - Chickenpox = attenuated virus - Hepatitis A = inactivated virus - Hepatitis B = antigenic fragments of virus - Herpes zoster = attenuated virus - Human papillomavirus = antigenic fragments of virus - Influenza = injected vaccine, inactivated virus (nasally administered vaccine with attenuated virus is now available for some). - Measles = attenuated virus - Mumps = attenuated virus - Poliomyelitis = killed virus - Rabies = killed virus - Rotavirus = Rota Teq, modified rotavirus, Rotarix vaccine, attenuated strain - Rubella = attenuated virus - Smallpox = live vaccinia virus

KNOW what type of vaccine goes with which disease!

GRAPH: - Incubation period: initial exposure/ subclinical stage of disease = no signs or symptoms. - Prodromal period: mild signs or symptoms. - Period of illness: exponential growth = overall health affected = most severe signs and symptoms. - Period of decline: adaptive immune system caught up = wipe out pathogen in exponential rate = start to feel better. - Period of convalescence: return to normal self.

LOOK AT GRAPH = ON EXAM

A Precipitation Curve: - Zone of antibody excess - Zone of equivalence = precipitate formed (lattice) - Zone of antigen excess

LOOK AT PICTURE

Reviewing the overall microbial mechanisms of pathogenicity (Figure 15.9), predict the ability of the pathogen to cause infection in each of the following scenarios.

Likely to Cause Infection: - A population of microbes less than the infectious dose is introduced in a compromised human host. - A pathogen that causes gastrointestinal infections is accidentally ingested in contaminated food. - A healthy individual inhales droplets from a person infected with a respiratory virus. - A pathogen in quantities more than double its infectious dose is introduced at the appropriate portal of entry. Not Likely to Cause Infection: - A pathogen with multiple virulence factors is introduced in a healthy host, but in quantities far below its infectious dose. - A pathogen that causes urinary tract infections is accidentally ingested in contaminated water. - A population of microbes greater than the infectious dose is introduced in a healthy individual, but these microbes are unable to adhere to host tissues.

Sort each item based on which one of the following portals of entry it best represents: mucous membrane, skin, or parenteral route.

Mucous Membrane: - conjunctiva - respiratory tract - gastrointestinal tract - genitourinary tract Skin: - hair follicles - sweat gland ducts Parenteral route: - deep wounds - injections - surgery

Each of the following scenarios describes factors that influence infection at the portal of entry. For each scenario, determine whether the pathogen's ability to cause infection relates to the number of invading microbes or adherence to the host tissue.

Number of Invading Microbes: - For cutaneous anthrax, the infectious dose is 10 to 50 endospores, whereas for inhalation anthrax and gastrointestinal anthrax, the infectious doses are 10,000 to 20,000 and 250,000 to 1,000,000 endospores, respectively. - For Vibrio cholerae, the infectious dose is 108 cells, but if stomach acid is neutralized with bicarbonate, this number decreases significantly. Adherence to the Host Tissue: - Enteropathogenic strains of Escherichia coli have fimbriae that bind to specific regions of the small intestine. - Neisseria gonorrhoeae uses fimbriae to attach to cells in the genitourinary tract, eyes, and pharynx. - Staphylococcus aureus binds and infects skin by a mechanism that resembles viral attachment. - Treponema pallidum uses its tapered end as a hook to attach to host cells during a syphilis infection.

The ELISA Method: - Postive Direct ELISA: 1. Antibody is adsorbed to well. 2. Patient sample is added; complementary antigen binds to antibody. 3. Enzyme-linked antibody specific for test antigen is added and binds to antigen, forming sandwich. 4. enzyme's substrate is added, and reaction produces a product that causes a visible color change. - Positive Indirect ELISA: 1. Antigen is adsorbed to well. 2. Patient serum is added; complementary antibody binds to antigen. 3. Enzyme-linked anti-HISG is added and binds to bound antibody. 4. Enzyme's substrate is added, and reaction produces a product that causes a visible color change.

OVERVIEW OF STEPS

The Production of Monoclonal Antibodies: 1. A mouse is injected with a specific antigen that will induce production of antibodies against that antigen. 2. The spleen of the mouse is removed and homogenized into a cell suspension. The suspension includes B cells that produce antibodies against the injected antigen. 3. The spleen cells are then mixed with myeloma cells that are capable of continuous growth in culture but have lost the ability to produce antibodies. Some of the antibody-producing spleen cells and myeloma cells fuse to form hybrid cells. These hybrid cells are now capable of growing continuously in culture while producing antibodies. 4. The mixture of cells is placed in a selective medium that allows only hybrid cells to grow. 5. Hybrid cells proliferate into clones called hybridomas. The hybridomas are screened for production of the desired antibody. 6. The selected hybridomas are then cultured to produce large quantities of monoclonal antibodies. Isolated antibodies are used for treating and diagnosing disease.

OVERVIEW OF STEPS

The Use of Monoclonal Antibodies in a Home Pregnancy Test: - Sandwich ELISA: 1. Free monoclonal antibody specific for hCG, a hormone produced during pregnancy. 2. Capture monoclonal antibody bound to substrate. 3. Sandwich formed by combination of capture antibody and free antibody when hCG is present, creating a color change - Human chorionic gonadotropin (hCG)

OVERVIEW OF STEPS

CHAPTER 16: Innate Immunity: Nonspecific Defenses of the Host

STARTS HERE

CHAPTER 17: Adaptive Immunity: Specific Defenses of the Host

STARTS HERE!

CHAPTER 15: Microbial Mechanisms of Pathogenicity

STARTS HERE! - Picture = overview of whole chapter!

CHAPTER 18: Practical Applications of Immunology

STARTS HERE!!!!

The Lytic Cycle: _______ Bacteriophages - 1. __________: phage attaches by the tail fibers (spikes, envelope, glycocalyx, etc.) to the host cell. - 2. _________: phage lysozyme opens the cell wall; tail sheath contracts to force the tail core and DNA into the cell. - 3. _________: production of phage DNA and proteins. - 4. _________: assembly of phage particles. - 5. _________: phage lysozyme breaks the cell wall. - Without ________, there would be no infection. - In the lytic cycle, the ______ of the bacteriophage never enters the cell, just the DNA or RNA will enter.

T-even - 1. attachment - 2. penetration - 3. biosynthesis - 4. maturation - 5. release (lytic part of the cycle) - attachment - capsid

Drag each of the following descriptions of essential humoral and cellular immune factors to its corresponding event number on the diagram above.

Th Cell: - The helping function of this cell is activated by two signals: the first signal occurs with the binding of the T cell receptor (TCR) to a processed antigen, and the second signal is a costimulatory cytokine. Memory Cell: - This cell is responsible for the enhanced secondary response to an antigen and is produced via clonal selection and differentiation of B cells. B cell: - This cell becomes activated when its immunoglobulins bind to its specific epitope, and in order to be activated, it may require assistance via helper cells. Antibodies: - This molecule is made up of protein chains that form a complex with antigens. This complex serves to tag foreign cells and molecules for destruction by phagocytes and complement. Cytotoxic T Lymphocyte: - This cell is an effector cell that has the ability to recognize and kill target cells that are considered nonself cells.

Which of the following statements concerning immunological memory is true?

The memory response is due to production of long-lived cells that can respond very rapidly upon second exposure.

What would happen if the septum did not form during binary fission?

The parent cell would now have two copies of the chromosome.

Which of the following organisms have eukaryotic cells that do NOT contain a cell wall? a. algae b. animals c. fungi d. plants

b. animals

Which of the following pairs is correctly matched? a. transmission electron microscope and ticks b. atomic force microscopes and DNA c. light microscope and viruses d. scanned-probe microscope and internal bacterial structures

b. atomic force microscopes and DNA

A pathogen that is capable of antigenic variation can ________________. a. adhere tightly to host tissues b. avoid host immune defenses c. cause fever, low blood pressure, and bleeding problems d. lyse host cells

b. avoid host immune defenses

Which of the following is NOT an example of microbial antagonism? a. acid production by bacteria b. bacteria causing disease c. bacteria producing vitamin K d. bacteriocin production e. bacteria occupying host receptors

b. bacteria causing disease

All of the following are examples of biotechnology EXCEPT __________. a. enzyme production by bacteria and fungi b. bacteria that cause disease or serve as normal microbiota c. antibiotic production by bacteria d. bread making with bacteria and fungi

b. bacteria that cause disease or serve as normal microbiota

Which of the following pairs is mismatched? a. nigrosin - negative stain b. basic dye - negative stain c. methylene blue - simple stain d. acidic dye - capsule stain e. crystal violet - simple stain

b. basic dye - negative stain

Arsenic is a relatively common hazardous waste generated by smelting processes and can contaminate soil and water surrounding smelting facilities. A bioprocess using naturally occurring bacteria to remove arsenic has been developed. This process is an example of __________. a. biotechnology b. bioremediation c. gene therapy d. fermentation e. nitrogen fixation

b. bioremediation

Normal microbiota a. almost always cause disease in the host. b. both indefinitely colonize the body and take up residence in sites such as the colon and mouth. c. take up residence in sites such as the colon and mouth. d. indefinitely colonize the body. e. are normally found in blood.

b. both indefinitely colonize the body and take up residence in sites such as the colon and mouth.

Many pathogenic (disease-producing) bacteria produce a(n) __________ that protects them from phagocytosis by host cells. a. flagellum b. capsule c. pilus d. endospore

b. capsule

In some cases, viral infections may __________. a. cause cells to produce endotoxin b. cause cells to lose contact inhibition c. cause cells to produce invasive enzymes such as hyaluronidase and collagenase d. cause cells to form capsules

b. cause cells to lose contact inhibition

Oncogenic viruses a. cause acute infections. b. cause tumors to develop. c. are lytic viruses that kill the host cell. d. are genetically unstable. e. have no effect on the host cell.

b. cause tumors to develop.

Robert Hooke's observation of the individual units in thin layers of cork tissue led to the development of the __________ theory. a. biogenesis b. cell c. spontaneous generation d. germ

b. cell

During the lag phase, __________. a. nutrients are depleted b. cells are engaged in intense enzymatic activity c. cells are decreasing in number d. changes in pH occur e. no cellular activity can be detected

b. cells are engaged in intense enzymatic activity

Which of the following is NOT a functionally analogous pair? a. mitochondria - prokaryotic plasma membrane b. cilia - pili c. nucleus -nucleoid region d. 9+2 flagella - bacterial flagella e. chloroplasts - thylakoids

b. cilia - pili

Which one of these pairs is NOT correctly matched? a. leukocidin; lyses WBC membranes b. coagulase; lyses fibrin clots c. collagenase; breaks down connective tissue d. IgA protease; digest antibodies e. siderophore; traps iron

b. coagulase; lyses fibrin clots

Gene therapy is currently used to treat all of the following diseases EXCEPT ______ A) LDL-receptor deficiency. B) colon cancer. C) Duchenne's muscular dystrophy. D) severe combined immunodeficiency disease (SCID). E) cystic fibrosis.

b. colon cancer

To look for bacteria that degrade petroleum, a culture medium containing crude oil, sodium nitrate, phosphate buffer, and magnesium sulfate is inoculated with soil. This medium is __________. a. reduced b. complex c. differential d. chemically defined

b. complex

The __________ of a compound light microscope focuses light from the light source on the specimen. a. objective b. condenser c. ocular lense d. stage

b. condenser

Which of the following statements concerning antigen-presenting cells is true?

They are involved in activating T cells.

Through which structure would the light pass before it reaches the specimen affixed to the stage? a. ocular lens b. condenser c. objective d. illuminator

b. condenser

For the Hib vaccine, Haemophilus influenzae type b capsular polysaccharide is attached to a protein such as diphtheria toxoid. This attachment to the protein causes a better immune response against H. influenzae type b in infants. This Hib vaccine is a(n) __________. a. attenuated whole-agent vaccine b. conjugated vaccine c. toxoid vaccine d. subunit vaccine

b. conjugated vaccine

The ID50 for cutaneous anthrax due to Bacillus anthracis is 10 to 50 endospores, while the ID50 for inhalation anthrax is 10,000 to 20,000 endospores. This means that __________. a. inhalation anthrax is easier to acquire than cutaneous anthrax b. cutaneous anthrax is easier to acquire than inhalation anthrax c. both cutaneous and inhalation anthrax can easily be acquired d. neither cutaneous or inhalation anthrax can easily be acquired e. not enough information is available to answer this question

b. cutaneous anthrax is easier to acquire than inhalation anthrax

Blood agar used to observe hemolysis or clearing around Streptococcus pyogenes colonies is an example of a(n) __________. a. reducing media b. differential medium c. selective media d. enrichment media e. isolation media

b. differential medium

One remarkable finding on a patient's laboratory workup is a marked eosinophilia. This might be suggestive of __________. a. a parasitic infection b. either a parasitic infection or an allergic (hypersensitivity) reaction c. a viral infection d. a bacterial infection e. an allergic (hypersensitivity) reaction

b. either a parasitic infection or an allergic (hypersensitivity) reaction

Which of the following membrane transport mechanisms takes place in eukaryotic cells but NOT in prokaryotic cells? a. facilitated diffusion b. endocytosis c. osmosis d. active transport

b. endocytosis

Spirochetes and spirilla are both spiral-shaped bacteria. However, they differ in that spirochetes have __________ and spirilla do not. a. fimbriae b. endoflagella (axial filaments) c. peritrichous flagella d. N-acetylglucosamine

b. endoflagella (axial filaments)

Some viruses have a membranelike structure on their surface, composed of lipids, proteins, and carbohydrates. This is called a(n) __________. a. capsid b. envelope c. capsule d. core

b. envelope

Which of the following are NOT a major target for action of antimicrobials? a. proteins b. flagella c. enzymes d. nucleic acids e. cell membranes

b. flagella

Which of the following bacterial structures is necessary for chemotaxis? a. capsules b. flagella c. plasmids d. metachromatic granules

b. flagella

Which of the following types of microscopy can be used to specifically identify pathogens (such as Mycobacterium tuberculosis or rabies virus) in specimens? a. Brightfield b. Fluorescence c. Phase-contrast d. Darkfield e. Transmission electron

b. fluorescence

Which of the following is an example of a metabolic activity that could be used to measure microbial growth? a. most probable number (MPN) b. glucose consumption c. direct microscopic count d. standard plate count e. turbidity

b. glucose consumption

__________ is important in medical microbiology because the results help physicians select appropriate antibiotics for treatment. a. simple staining b. gram staining c. the observation of morphology of bacteria d. the observation of the size of bacteria

b. gram staining

Carbolfuchsin can be used as a simple stain and a negative stain. As a simple stain, the pH is: a. 2 b. higher than the negative stain c. lower than the negative stain d. the same as the negative stain

b. higher than the negative stain

Which one of the following physical methods of microbial control is bactericidal? a. refrigeration b. incineration c. desiccation d. deep-freezing

b. incineration

Which of the following is an effect of opsonization? a. increased diapedesis of phagocytes b. increased adherence of phagocytes to microorganisms c. inflammation d. increased margination of phagocytes e. cytolysis

b. increased adherence of phagocytes to microorganisms

Julie is preparing vegetables for dinner and accidentally cuts her finger. She washes it with soap and water before covering it with a bandage. A short while later, Julie notices that her finger is red, swollen, warm to the touch, and very painful. Which of the following describes the process going on in Julie's finger? a. complement b. inflammation c. phagocytosis d. fever

b. inflammation

Each of the following is an effect of complement activation EXCEPT a. opsonization. b. interference with viral replication. c. bacterial cell lysis. d. increased blood vessel permeability. e. increased phagocytic activity.

b. interference with viral replication.

Antibody-dependent cell-mediated cytotoxicity __________. a. involves antibodies binding to a microbe in the same orientation that IgE antibodies bind to basophils b. is particularly important for killing microbes that are too large be destroyed by phagocytosis c. involves eosinophils binding directly to a microbe d. damages, but rarely lyses, a microbe e. involves NK cells binding directly to a microbe

b. is particularly important for killing microbes that are too large be destroyed by phagocytosis

A __________ solution is a medium in which the overall concentration of solutes equals that found inside the cell. Water enters and leaves the cell at the same rate. a. hypertonic b. isotonic c. hypotonic d. ionic

b. isotonic

The plasma membrane is considered a barrier to the environment because __________. a. no molecules may pass through it b. it controls the passage of molecules into the cell c. it is located outside of the cell wall d. only water and small molecules may penetrate the membrane e. it contains rigid cellulose and steroid molecules

b. it controls the passage of molecules into the cell

Which of the following body sites typically does NOT have normal microbiota? a. nasopharynx b. kidney c. skin d. vagina

b. kidney

Emergence of infectious diseases can be caused by a variety of factors. Which factor listed below is related to human behavior? a. mutations in microbes b. lack of vaccination c. floods d. earthquakes

b. lack of vaccination

An example of lysogeny in animals could be a. slow viral infections. b. latent viral infections. c. T-even bacteriophages. d. infections resulting in cell death. e. none of the above

b. latent viral infections.

Place these structures of the compound light microscope in the order that light passes through them on the way to the observer's eyes: (1) condenser, (2) ocular lens, (3) light source, (4) specimen, (5) objective lens. a. ocular lens → condenser → specimen → light source → objective lens b. light source → condenser → specimen → objective lens → ocular lens c. light source → condenser → objective lens → specimen → ocular lens d. light source → specimen → condenser → objective lens → ocular lens

b. light source → condenser → specimen → objective lens → ocular lens

During __________, the phage remains latent. a. the lytic cycle b. lysogeny c. budding d. assembly

b. lysogeny

Which of the following is NOT an Emerging Infectious Disease? a. MERS (Middle East Respiratory Syndrome) b. Measles c. H1N1 influenza (swine flu) d. West Nile Encephalitis e. VRSA (vancomycin-resistant Staphylococcus aureus)

b. measles

A(n) __________ is equal to 0.000001 m. a. nanometer b. micrometer c. millimeter d. centimeter

b. micrometer

Which of the following is NOT a chemical requirement of all bacteria? a. carbon b. molecular oxygen c. nitrogen d. sulfur e. mineral elements

b. molecular oxygen

Chemotaxis refers to the ability of microorganisms to __________. a. survive under adverse conditions b. move toward or away from chemical stimuli c. attach to solid surfaces d. move in a wavelike motion e. escape phagocytosis

b. move toward or away from chemical stimuli

The ability of microbes to utilize N2 as a nitrogen source is called __________. a. nitrate fixation b. nitrogen fixation c. nitrite fixation d. nitrogen oxidation

b. nitrogen fixation

Following coronary artery bypass surgery, seven patients develop Rhodococcus bronchialis infections. Cultures of the operating rooms, Nurse A, and Nurse B are taken. R. bronchialis grows from the hand and nasal swabs of Nurse A. The patients' infections are an example of a/an __________. a. vector-borne infection b. nosocomial infection c. endemic infection d. epidemic infection e. emerging infectious disease

b. nosocomial infection

Hooke and van Leeuwenhoek helped to create the basis of the cell theory by __________. a. developing industrial products using microorganisms b. observing and documenting cells, and using microscopes to visualize the invisible structure of living things c. clarifying the connection between disease and microorganisms d. proving that living organisms originate spontaneously from nonliving materials

b. observing and documenting cells, and using microscopes to visualize the invisible structure of living things

The process of coating bacteria with serum proteins to promote attachment of phagocytes is called __________. a. adherence b. opsonization c. chemotaxis d. inflammation

b. opsonization

Which of the following types of microscopy is most useful for viewing the internal structures of unstained specimens? a. brightfield b. phase-contrast c. confocal d. electron e. fluorescence

b. phase-contrast

This activity asks you to consider various methods of counting or estimating the size of bacterial populations and to select the method that will provide the most accurate data for plotting a bacterial growth curve. Which of the following methods would be most appropriate for gathering data to plot a bacterial growth curve throughout the four phases? a. direct microscopic count b. plate count c. indirect estimate based on turbidity d. electronic cell counter e. Any of these methods will provide reliable data for plotting a growth curve.

b. plate count

All of the following describe a major feature used to classify organisms into three domains (bacteria, archaea, and eukarya) EXCEPT _____________. a. presence or absence of peptidoglycan b. presence or absence of DNA c. nutritional and metabolic factors d. presence or absence of a nucleus

b. presence of absence of DNA

Immersion oil is used with the 100 × objective lens of the compound light microscope to __________. a. increase contrast by bending light. b. prevent the loss of resolution due to refraction c. separate the light into different wavelengths d. condense the light

b. prevent the loss of resolution due to refraction

__________ organisms are often responsible for food spoilage in refrigerated foods. a. Thermophile b. Psychrotroph c. Halophile d. Psychrophile

b. psychrotroph

Which of the following types of media would not be used to cultureaerobes? a. selective media b. reducing media c. enrichment media d. differential media e. complex media

b. reducing media

The endoplasmic reticulum that has ribosomes attached to its outer surface is referred to as __________. a. smooth ER b. rough ER c. wavy ER d. golgi

b. rough ER

A medium containing chemicals to inhibit the growth of gram-positive bacteria would be called __________. a. enriched b. selective c. reduced d. differential

b. selective

A medium containing lauryl sulfate inhibits the growth of gram-positive bacteria. This medium is __________. a. differential b. selective c. enriched d. reduced

b. selective

Which one of these bacterial proteins can take iron from human lactoferrin? a. kinase b. siderophore c. hyaluronidase d. protease

b. siderophore

An antiseptic is used to remove microbes from __________. a. foods before canning them b. skin before an injection c. restaurant glassware d. toilet surfaces e. food preparation areas

b. skin before an injection

Which of the following is the best definition of epitope? a. specific regions on antigens that interact with haptens b. specific regions on antigens that interact with antibodies c. specific regions on antigens that interact with MHC class molecules d. specific regions on antigens that interact with T-cell receptors e. specific regions on antigens that interact with perforins

b. specific regions on antigens that interact with antibodies

All of the following can be used to preserve bacterial cultures EXCEPT __________. a. refrigeration b. storing cultures at room temperature (25°C) c. lyophilization (freeze-drying) d. freezing quickly and storing at -70°C

b. storing cultures at room temperature (25°C)

Vaccinations are available for all of the following diseases EXCEPT ______ A) measles. B) strep throat. C) mumps. D) rubella. E) hepatitis B.

b. strep throat

An infection that does NOT cause any signs of disease is a(n) __________. a. endemic infection b. subclinical infection c. secondary infection d. sporadic infection

b. subclinical infection

A virus with RNA-dependent RNA polymerase a. synthesizes DNA from an RNA template. b. synthesizes double-stranded RNA from an RNA template. c. synthesizes double-stranded RNA from a DNA template. d. transcribes mRNA from DNA. e. none of the above

b. synthesizes double-stranded RNA from an RNA template.

All of the following are true of the plate count method EXCEPT that it __________. a. is dependable b. takes less than 12 hours to complete c. measures number of viable cells d. involves cell plating and growth e. is the most commonly used method for assay of bacterial cell number

b. takes less than 12 hours to complete

A persistent infection is one in which a. the virus remains in equilibrium with the host without causing a disease. b. the disease process occurs gradually over a long period. c. host cells are gradually lysed. d. host cells are transformed. e. viral replication is unusually slow.

b. the disease process occurs gradually over a long period.

The morbidity rate is best defined as __________. a. the percentage of infectious diseases that are nationally notifiable b. the number of individuals, relative to the population, affected by a particular disease in a period of time c. the immunization rate for notifiable diseases that can be prevented through vaccination programs d. the number of individuals, relative to the population, who have died as a result of a particular disease in a period of time e. the decrease in death rate following effective vaccination

b. the number of individuals, relative to the population, affected by a particular disease in a period of time

You are viewing a sputum smear that has been stained with an acid-fast stain. On this smear you see 5-micrometer-long red cells. You can conclude that __________. a. human cells are acid-fast b. there are acid-fast bacteria in the specimen c. normal microbiota are acid-fast d. normal microbiota are gram-negative e. there are no acid-fast bacteria in the specimen

b. there are acid-fast bacteria in the specimen

You inoculate two tubes of liquid culture medium with 100 bacterial cells and incubate one tube at 37°C and the other at 55°C. After 48 hours, there are 20,000 bacteria per milliliter in the 37°C tube and 1,568,000 bacteria per milliliter in the 55°C tube. You conclude that this species is a __________. a. mesophile b. thermophile c. hyperthermophile d. psychrophile

b. thermophile

An individual with a high concentration of a particular antibody is said to have a high __________. a. agglutination b. titer c. serum d. viral load

b. titer

Jenner's work with vaccinations depended on which of the following? a. Exposing patients to the exact pathogen he was trying to protect them from b. Trusting that the body produced agents that effectively overcame a disease-causing microbe c. Selecting a microbe that could easily evade the immune system d. Selecting a pathogen that was a bacterium

b. trusting that the body produced agents that effectively overcame a disease-causing microbe

Assume that you are viewing a Gram-stained field of red cocci and blue bacilli through the microscope. You can safely conclude that you have: a. made a mistake in staining b. two different species c. old bacterial cells d. young bacterial cells e. none of the above

b. two different species

Bacterial cells have an overall negative charge on their surface. If a bacterial smear is stained by a pink, acidic, water-soluble stain and then washed with water, which of the following describes the result that will be observed when viewing the stained slide under a microscope? a. pink cells b. Unstained cells with no background c. Unstained cells in a pink background d. Pink cells in a pink background e. Purple cells

b. unstained cells with no background

In a negative staining procedure, the bacterial cells would appear __________ when viewed under a microscope. a. brightly colored in a contrasting background b. unstained in a colored background c. stained in an unstained background d. attracted to the dye used in the staining procedure e. the same color as the background and thus not easily observed

b. unstained in a colored background

Which of the following best describes gene therapy? a. use of microbes for bioremediation b. use of a harmless virus to insert a gene in a host cell c. use of microbes to control pests d. use of microbes to recycle nitrogen from the air into the soil

b. use of a harmless virus to insert a gene in a host cell

Cells produce alpha interferon and beta interferon in response to __________. a. helminthic infections b. viral infections c. bacterial infections d. protozoan infections

b. viral infections

Pasteurization was first used by Pasteur to control spoilage of __________. a. milk b. wine c. yogurt d. cheese e. canned foods

b. wine

For the three types of media in Table 6.1, which medium (or media) is/are chemically defined? a b c a and b a and c

a

In Table, 6.1, in which medium (or media) would an autotroph grow? a b c a and b a and c

a

which of the following statements is not true? a) E. coli never causes disease b) E. coli provides vitamin K for its host c) E. coli often exists in a mutualistic relationship with humans d) E.coli gets nutrients from intestinal contents

a) E. coli never causes disease

Which statement best describes what happens when a gram-positive bacterium is placed in an aqueous solution of lysozyme and 10% sucrose? a) No change will result; the solution is isotonic b) Water will move into the cell c) Water will move out of the cell d) the cell will undergo osmotic lysis e) Sucrose will move into the cell from an area of higher concentration to one of lower concentration

a) No change will result; the solution is isotonic

Which statement regarding endotoxins is true? a) One consequence of endotoxins is the activation of blood-clotting proteins. b) The effects of endotoxins vary greatly, depending on the specific bacterium the produces them. c) Endotoxins are part of the outer portion of the cell wall of gram-positive bacteria. d) Endotoxins induce host cells to produce effective antitoxins that help to protect them against the toxin's effects.

a) One consequence of endotoxins is the activation of blood-clotting proteins.

How are superantigens different from other types of exotoxins? a) Superantigens cause an overstimulation of the host immune system. b) Superantigens are comprised of two functional domains. c) Superantigens only act against host neurons. d) Superantigens must be endocytosed into a target cell before becoming active.

a) Superantigens cause an overstimulation of the host immune system.

You have isolated a motile, gram-positive cell with no visible nucleus. You can assume this cell has: a) ribosomes b) mitochondria c) an endoplasmic reticulum d) a Golgi complex e) all of the above

a) ribosomes

A person who attended a picnic early in the day develops a very high fever and is unresponsive by the evening. This person most likely has been exposed to a(n) a) superantigen b) cytotoxin. c) membrane disrupting toxin. d) enterotoxin.

a) superantigen

the emergence of a new infectious disease is probably due to all of the following except: a) the need of bacteria to cause disease b) the ability of humans to travel by air c) changing environments (flood, drought, pollution) d) a pathogen crossing the species barrier e) the increasing human population

a) the need of bacteria to cause disease

Which of the following is NOT visible through a compound light microscope? a. 0.02-µm ribosome b. 1.5-mm tick c. 5-µm nucleus d. 15-µm white blood cell

a. 0.02-µm ribosome

Most bacteria are in the __________ size range. a. 1-10 micrometer b. 1-10 nanometer c. 2-5 millimeter d. 0.5-1 centimeter

a. 1-10 micrometers

If you use a compound light microscope, a 2-µm bacterial cell is best seen at which magnification? a. 1000× b. 10× c. 100× d. 400×

a. 1000x

The maximum magnification of a compound microscope is (a)_____; that of an electron microscope, (b)______. The maximum resolution of a compound microscope is (c)______; that of an electron microscope, (d)______. One advantage of a scanning electron microscope over a transmission electron microscope is (e)______.

a. 2,000X b. 100,000X c. 0.2μm d. 0.0025μm e. Seeing three-dimensional detail.

In an agglutination assay in which serum is diluted by half in each successive well (1:2; 1:4; 1:8; 1:16, etc.), if the 1:64 well is the last well to show agglutination, the titer is __________. a. 64 b. 32 c. 128 d. 1:32 e. 1:64

a. 64

Which of the following is true of biofilms? a. A biofilm is a complex aggregation of microbes. b. A biofilm is the layer of material examined on a microscope slide. c. Biofilms are never found in humans. d. Biofilms are weak aggregations of microbes that are easily removed from surfaces.

a. A biofilm is a complex aggregation of microbes.

Consider the following situations, and determine when a simple stain should be used instead of a Gram stain. a. A microbiologist is trying to determine whether an organism has flagella. b. An unknown species of bacteria is being characterized and identified. c. A wound specimen is being examined. d. A specimen is being examined in which the presence of multiple species of bacteria is suspected.

a. A microbiologist is trying to determine whether an organism has flagella.

Which of the following is an example of a healthcare-associated infection (HAI)? a. A patient develops a Pseudomonas infection during treatment in a burn unit. b. A child visits a clinic with a suspected case of strep throat. c. A man enters the hospital with a suspected rabies infection due an animal bite. d. A cancer patient is admitted to the hospital with complications.

a. A patient develops a Pseudomonas infection during treatment in a burn unit.

Which of the following results in comparatively long-lasting immunity? a. A person surviving an infectious disease b. A baby receiving antibodies to chicken pox across the placenta c. A baby receiving antibodies against many pathogens through breastfeeding d. An adult receiving gamma globulin e. An adult receiving antiserum

a. A person surviving an infectious disease

A newly identified bacterial pathogen has been shown to cause disease in humans, disrupting the production of some proteins by interfering with the function of the endoplasmic reticulum of liver cells. Under specific growth conditions, this bacterial strain secretes a protein that appears to be responsible for the pathology of the disease. This protein is composed of two polypeptide chains, one of which binds to a receptor on the surface of liver cells, stimulating the uptake of the protein by endocytosis. Once inside the cell, the other polypeptide component interferes with the activity of ribosomes bound to the endoplasmic reticulum. This bacterial protein will most likely be classified as a/an __________. a. A-B toxin b. superantigen c. invasin d. bacterial kinase e. membrane-disrupting toxin

a. A-B toxin

TLRs attach to all of the following EXCEPT a. AMPs. b. flagellin. c. LPS. d. peptidoglycan. e. PAMPs.

a. AMPs.

Which of the following processes occurs in bacterial plasma membranes but NOT in eukaryotic plasma membranes? a. ATP synthesis b. Active transport c. Protein synthesis d. Facilitated diffusion e. RNA synthesis

a. ATP synthesis

To test a patient's serum for evidence of a salmonella infection, you are looking for antibodies. You mix the serum sample with bacteria. What would a positive test show? a. Agglutination b. Fluorescence c. Cell lysis d. Hemagglutination

a. Agglutination

While on safari in Serengeti National Park, Tanzania, your friend ventures away from camp and is bitten by a black widow spider. Fortunately, you are prepared and administer artificially acquired passive immunity. With what did you inject your friend? a. Antivenin b. Snake venom c. Penicillin d. Antigen

a. Antivenin

As a pediatric oncology nurse, you work with children who are receiving chemotherapy and are frequently immunosuppressed. Which of these types of vaccines are generally NOT appropriate for this patient population? a. Attenuated whole-agent vaccines b. Inactivated whole-agent vaccines c. Toxoid vaccines d. Subunit vaccines e. Conjugated vaccines

a. Attenuated whole-agent vaccines

The biosafety level (BSL) for most introductory microbiology laboratories is a. BSL-1. b. BSL-2. c. BSL-3. d. BSL-4.

a. BSL-1.

Which level of biosafety (BSL) is appropriate for handling organisms that present the highest risk level for infection and the lowest success rate for prevention and treatment? a. BSL-4 b. BSL-1 c. BSL-2 d. BSL-3

a. BSL-4

Which of the following statements concerning phagocytosis is true? a. Bacteria are digested when the phagosome fuses with a lysosome. b. Phagocytes cannot ingest microorganisms unless they are coated with antibodies. c. Phagocytes ingest microorganisms by using protein transporters that are specific to the bacteria. d. Adherence always requires opsonization.

a. Bacteria are digested when the phagosome fuses with a lysosome.

Which of the following is true of bacteria that have atypical cell walls, such as those in the genus Mycoplasma? a. Bacteria with atypical cell walls are generally among the smallest bacteria that can reproduce outside a host cell. b. These atypical bacteria have the same plasma membrane typical of bacteria. c. These atypical bacteria usually stain gram-negative. d. Although small, these atypical bacteria cannot pass through most filters.

a. Bacteria with atypical cell walls are generally among the smallest bacteria that can reproduce outside a host cell.

Which one of the following substances is effective against bacterial endospores? a. Chlorine b. Mercury c. Bisphenols d. Quaternary compounds

a. Chlorine

Which of these organisms does NOT produce an enterotoxin? a. Clostridium botulinum b. Staphylococcus aureus c. Vibrio cholerae d. Shigella spp.

a. Clostridium botulinum

Which type of microscope would be best to use to observe each of the following? a. a stained bacterial smear b. unstained bacterial cells: the cells are small, and no detail is needed c. unstained live tissue when it is desirable to see some intracellular detail d. a sample that emits light when illuminated with ultraviolet light e. intracellular detail of a cell that is 1μm long f. unstained live cells in which intracellular structures are shown in color

a. Compound Light Microscope b. Darkfield Microscope c. Phase-Contrast Microscope d. Fluorescence Microscope e. Electron Microscope f. Differential Interference Contrast Microscope

Given the following LD50 values for Bacillus anthracis, through which portal of entry is it easiest to get anthrax? a. Cutaneous: 50 endospores b. Inhalation: 20,000 endospores c. Ingestion: 1 million endospores d. All of the listed choices are equally easy portals of entry.

a. Cutaneous: 50 endospores

Which of the following staining procedures is correctly matched with the principal use for that particular stain? a. Endospore stain to distinguish dormant structures formed during adverse environmental conditions b. Acid-fast stain for microbes with capsules c. Negative stain used to differentiate cell wall components d. Flagella stain used to detect capsules

a. Endospore stain to distinguish dormant structures formed during adverse environmental conditions

Which of the following statements is true? a. Endospores are extremely durable structures that can survive high temperatures. b. Endospores allow a cell to attach to solid surfaces and to surfaces within the host. c. Endospores are reproductive structures. d. One bacterial cell produces many endospores.

a. Endospores are extremely durable structures that can survive high temperatures.

Which of these events leads to all of the others in a pyrogenic (fever) response? a. Endotoxin is released from the cell wall of gram-negative bacteria. b. IL-1 is released by macrophages. c. IL-1 travels via the blood to the hypothalamus. d. The hypothalamus releases prostaglandins. e. The body's thermostat is set to a higher level, and fever occurs.

a. Endotoxin is released from the cell wall of gram-negative bacteria.

Which one of these chemical treatments kills cells by causing cross-linking of nucleic acids and proteins and inhibits vital cellular functions? a. Ethylene oxide b. Alcohol c. Heavy metals d. Soaps and detergents

a. Ethylene oxide

A rapid test for Bordetella pertussis uses fluorescent dye-labeled antibodies to stain a smear made from a nasopharyngeal swab. What would a positive test show? a. Fluorescent cells b. Agglutination c. Purple cells d. Enzyme activity

a. Fluorescent cells

Which of the following is the preferred method for cultivating many animal viruses? a. Growing them in animal cell cultures b. Growing them on highly enriched agar media c. Inoculating appropriate laboratory animals d. Growing them in bacteria

a. Growing them in animal cell cultures

You contaminate the kitchen counters with Salmonella enterica in chicken blood and expose it to a variety of treatments to study the survival of the bacteria on kitchen surfaces. The following DRT values were obtained: 2.0 minutes at 52°C; 3.7 minutes at pH 2.6; 13.3 minutes with 10 mM hydrogen peroxide; and > 35 days without treatment. Which treatment is most effective against S. enterica? a. Heat (52ºC) b. 10 mM hydrogen peroxide c. Acid (pH 2.6) d. These are equivalent treatments. e. The most effective treatment cannot be determined from the given data.

a. Heat (52ºC)

Which of the following are NOT antigen-presenting cells ? a. Helper T cells b. Dendritic cells c. Macrophages d. B cells e. All of the listed choices are antigen-presenting cells.

a. Helper T cells

Which two virus families make DNA from an RNA template? a. Hepadnaviridae and Retroviridae b. Togaviridae and Picornaviridae c. Adenoviridae and Poxviridae d. Reoviridae and Herpesviridae

a. Hepadnaviridae and Retroviridae

Which of the following is a correct match? a. IgA: found in secretions such as colostrum, tears, and mucus b. IgM: readily crosses the placenta to protect the fetus/newborn c. IgE: typically structured as a pentamer d. IgG: first immunoglobulin produced in a primary immune response

a. IgA: found in secretions such as colostrum, tears, and mucus

Which immunoglobulin class, found in blood, lymph, and the intestine, can cross the placenta and help to protect a fetus? a. IgG b. IgA c. IgM d. IgE

a. IgG

Which of these antibodies provides evidence of an active infection when present? a. IgM b. IgG c. IgE d. IgD

a. IgM

Which of the following events does NOT occur in diseases transmitted by vectors? a. In biological transmission, the vector multiplies in the human host. b. In biological transmission, pathogens multiply in the vector, which can transmit the pathogens by injecting saliva directly into the host. c. In biological transmission, a vector transmits pathogens by defecating while taking a blood meal. d. In mechanical transmission, insects contact feces containing pathogenic bacteria and transfer it to food. e. In biological transmission, vectors can spread pathogens by vomiting.

a. In biological transmission, the vector multiplies in the human host.

A decrease in the production of C3 would result in: a. Increased susceptibility to infection. b. Increased numbers of white blood cells. c. Increased phagocytosis. d. Activation of C5 through C9. e. None of the above.

a. Increased susceptibility to infection.

Which of the following statements about a gram-negative cell wall is FALSE? a. It contains teichoic acid. b. It includes a thin layer of peptidoglycan. c. It functions as an endotoxin. d. It maintains the shape of the cell.

a. It contains teichoic acid.

Which of the following statements regarding latent viral infections is true? a. Latent infections can persist for years in an individual without causing any symptoms. b. Latent viral infections are caused by the slow conversion of cellular glycoproteins from normal to infectious form. c. Latent viral infections are almost always fatal. d. During latent infections, small amounts of virus are produced, and virus numbers build up over time.

a. Latent infections can persist for years in an individual without causing any symptoms.

__________ cells are important in facilitating immune responses against pathogens that enter the body via the digestive system. a. M b. B c. T

a. M

The adaptive immune response, or third line of defense, is activated when the first and second lines of defense do not succeed in containing an infection. It is adaptive because the cells in this part of the immune system are capable of responding to specific pathogens. Another important feature of the third line is the ability of these cells to create memory cells. Why are memory cells an important product of an immune response? a. Memory cells allow subsequent immune responses against the same antigen to occur more strongly and more quickly. b. Memory cells phagocytize foreign antigens as part of the subsequent immune response. c. Memory cells release chemical mediators that cause inflammation after recognizing a pathogen for the second time. d. Memory cells directly produce antibodies that will aid in subsequent immune responses.

a. Memory cells allow subsequent immune responses against the same antigen to occur more strongly and more quickly.

Which answer is true for bacterial destruction by phagocytosis? a. Myeloperoxidase in lysosomes is involved in the formation of HOCl. b. Lipids and proteins, but not nucleic acids, can be digested inside lysosomes. c. Phagolysosomes have a neutral pH. d. Toxic oxygen products, such as hydrogen peroxide, are removed. e. Listeria monocytogenes is killed within the phagolysosome.

a. Myeloperoxidase in lysosomes is involved in the formation of HOCl.

On October 29, Barbara participated in a study group for her microbiology class. On November 1, Barbara had a "scratchy throat" when she swallowed. On November 2, Barbara had a headache, runny nose, and watery eyes. She was fully recovered on November 7. During which time was Barbara in the prodromal period? a. November 1 b. October 30 c. November 2 d. November 5

a. November 1

Which of the following is used to control microbial growth in foods? a. Organic acids b. Alcohols c. Aldehydes d. Heavy metals e. All of the above.

a. Organic acids

Which of the following is NOT a notifiable infectious disease? a. Pneumonia b. Pertussis c. Salmonellosis d. Botulism

a. Pneumonia

Legionella uses C3b receptors to enter monocytes. This: a. Prevents phagocytosis b. Degrades complement. c. Inactivates complement. d. Prevents inflammation. e. Prevents cytolysis.

a. Prevents phagocytosis

When properly subjected to an acid fast stain protocol, bacteria in the genus Mycobacterium appear __________. When using a 10× ocular lens and a 40× objective, the total magnification achieved is __________. a. Red; 400X b. Red; 1000X c. Blue; 1000X d. Purple; 400X

a. Red; 400X

Which of the following statements accurately describes the culture medium necessary for growing an obligate anaerobe, such as Clostridium tetani? a. Reducing media are complex media containing chemicals, such as thioglycolate, that combine with oxygen, creating an anaerobic environment. b. A complex medium is one whose exact chemical composition is known. c. A chemically defined medium is one made up of extracts such as those from yeasts, meat, or plants whose exact chemical composition may vary slightly. d. Nutrient agar contains ingredients that combine with oxygen and remove it, creating an anaerobic environment.

a. Reducing media are complex media containing chemicals, such as thioglycolate, that combine with oxygen, creating an anaerobic environment.

Which of the following correctly describes antisepsis? a. Removal of vegetative microorganisms from living tissue b. Destruction of vegetative microorganisms on inanimate surfaces c. Destruction of all organisms on any surface d. Reduction of microbial cells on eating or drinking utensils

a. Removal of vegetative microorganisms from living tissue

An encapsulated bacterium can be virulent because the capsule: a. Resists phagocytosis. b. Is an endotoxin. c. Destroys host tissues. d. Interferes with physiological processes. e. Has no effect; because many pathogens do not have capsules, capsules do not contribute to virulence.

a. Resists phagocytosis.

Which of the following is NOT an oncolytic virus? a. Retrovirus b. Adenovirus c. Vaccinia Virus d. Simplexvirus

a. Retrovirus

A microscopist is studying the surface pores on a bacterial specimen. Which microscope provides the best three-dimensional surface structure image? a. Scanning electron microscope b. Transmission electron microscope c. Fluorescence microscope d. Brightfield microscope

a. Scanning electron microscope

The earliest smallpox vaccines were infected tissue rubbed into the skin of a healthy person. The recipient of such a vaccine usually developed a mild case of smallpox, recovered, and was immune thereafter. What is the most likely reason this vaccine did not kill more people? a. Skin is the wrong portal of entry for smallpox. b. The vaccine consisted of a mild form of the virus. c. Smallpox is normally transmitted by skin-to-skin contact. d. Smallpox is a virus. e. The virus mutated.

a. Skin is the wrong portal of entry for smallpox.

The fungus Coccidioides immitis is found in the soil. Humans contract coccidioidomycosis by inhaling fungal spores. Which of these descriptions applies to the role of soil in transmission of coccidioidomycosis? a. Soil is the reservoir. b. Soil is the vector. c. Soil is the carrier. d. Soil is the predisposing factor.

a. Soil is the reservoir.

Which of these lists is in the correct order of differentiation? a. Stem cells to B cells to plasma cells b. Stem cells to plasma cells to B cells c. B cells to plasma cells to stem cells d. B cells to stem cells to plasma cells e. Plasma cells to B cells to stem cells

a. Stem cells to B cells to plasma cells

What is the name for a vaccine that is made from an antigenic fragment of a pathogen? a. Subunit vaccine b. Attenuated whole-agent vaccine c. Toxoid d. Inactivated whole-agent vaccine

a. Subunit vaccine

Which of the following statements is FALSE? a. The M in MRSA stands for mannitol. b. S. aureus is differentiated from other mannitol+ cocci by the coagulase test. c. The USA300 strain accounts for most community-acquired MRSA. d. The USA100 strain accounts for most hospital-acquired MRSA. e. Antimicrobial therapy for hemodialysis-associated infections increases antibiotic resistance.

a. The M in MRSA stands for mannitol.

What is quorum sensing? a. The ability of bacteria in a biofilm to communicate with each other and coordinate their activities b. The ability of bacteria in a biofilm to sense O2 c. The ability of bacteria in a biofilm to use inorganic nitrogen sources d. The ability of bacteria in a biofilm to grow on selective media

a. The ability of bacteria in a biofilm to communicate with each other and coordinate their activities

Which of these conditions would NOT affect the ability of Streptococcus mutans to attach to teeth? a. The absence of Actinomyces, a bacterium that can be part of dental plaque b. The inability to form dextran c. The lack of a glycocalyx d. The lack of the enzyme glucosyltransferase e. The lack of sucrose

a. The absence of Actinomyces, a bacterium that can be part of dental plaque

What will occur if a bacterial cell with a weakened or damaged cell wall is placed in pure water? a. The cell will swell and burst. b. The cell will remain the same size. c. The cell will shrink. d. The cell will form endospores. e. The cell will form a new cell wall.

a. The cell will swell and burst.

Which of the following statements about cytokines is FALSE? a. There are 10 types. b. They are produced by immune cells in response to a stimulus. c. They are chemical communication between cells. d. They are soluble proteins or glycoproteins. e. Some have multiple functions.

a. There are 10 types.

Which of the following statements regarding infections involving biofilms is FALSE? a. There is no way to prevent biofilms from developing on surfaces. b. The majority of hospital-acquired infections (nosocomial infections) are related to biofilms. c. Microbes in biofilms are more resistant to antimicrobials than are independent organisms. d. It is estimated that nearly 70% of all infections are caused by biofilms.

a. There is no way to prevent biofilms from developing on surfaces.

Which of the following statements about natural killer cells is FALSE? a. They are stimulated by an antigen. b. They destroy virus-infected cells. c. They destroy tumor cells. d. They destroy cells lacking MHC I. e. None of the answers are correct; all of these statements are true.

a. They are stimulated by an antigen.

Which of the following is NOT an advantage of antimicrobial peptides? a. They have a narrow spectrum of activity, so they are very specific. b. They are very stable. c. They exhibit synergy when used with other antimicrobial compounds. d. Microorganisms do not seem to develop resistance to them.

a. They have a narrow spectrum of activity, so they are very specific.

Which of the following statements is NOT true for T helper cells? a. They lyse target cells. b. They activate macrophages. c. They have CD4 molecules on the cell surface. d. They activate B cells. e. They recognize antigen presented by class II MHC molecules.

a. They lyse target cells.

What is the purpose of a complement fixation test? a. To detect low levels of antibodies b. To test for viral antigens c. To test for bacterial antigens d. To test for viral hemagglutination

a. To detect low levels of antibodies

In the Gram stain, what is the purpose of the decolorizer? a. To selectively remove stain from cells b. To clean off excess stain c. To wash slides d. To stain gram-negative cells

a. To selectively remove stain from cells

Which of the following is INCORRECT about nucleic acid vaccines? a. Two DNA vaccines for humans have been approved. b. Experiments with animals show that plasmids of "naked" DNA injected into muscle results in the production of the protein antigen encoded in the DNA. c. A "gene gun" method delivers the vaccine into many skin cell nuclei. d. Nucleic acid vaccines, often called DNA vaccines, are among the newest and most promising vaccines.

a. Two DNA vaccines for humans have been approved.

All of the following can occur during bacterial infection. Which would prevent all of the others? a. Vaccination against fimbriae. b. Phagocytosis. c. Inhibition of phagocytic digestion. d. Destruction of adhesins. e. Alteration of cytoskeleton.

a. Vaccination against fimbriae.

Which statement about viruses is FALSE? a. Viruses will usually infect any available cell, regardless of the cell type. b. The genome of a virus can be either DNA or RNA, depending on the particular virus. c. Viruses are active only when inside a cell. d. Viruses always have a protein coat.

a. Viruses will usually infect any available cell, regardless of the cell type.

The bacterium Aeromonas hydrophila is a common inhabitant of freshwater lakes and rivers. When this bacterium enters an open wound in humans, it can cause a form of necrotizing fasciitis. Which of the following statements is an accurate representation of the role of water in the infection? a. Water is the reservoir. b. Water is the carrier. c. Water is the vector. d. Water is the aerosol source

a. Water is the reservoir.

In the figure, which letter on the graph indicates the epidemic level? a. a b. b c. c d. d

a. a

In the figure, which line best illustrates the growth of a facultative anaerobe incubated aerobically? a. a b. b c. c

a. a

Which of the following is the best definition of antigen? a. a chemical that elicits an antibody response and can combine with these antibodies b. a pathogen c. a protein that combines with antibodies d. a chemical that combines with antibodies e. something foreign in the body

a. a chemical that elicits an antibody response and can combine with these antibodies

In response to the presence of endotoxin, phagocytes secrete tumor necrosis factor. This causes a. a decrease in blood pressure. b. the disease to subside. c. a gram-negative infection. d. an increase in red blood cells. e. a fever.

a. a decrease in blood pressure.

To detect the presence of antibody to a pathogen in a person through use of the indirect fluorescent-antibody technique, __________. a. a known antigen is fixed to a slide, patient serum is added, and labeled antibodies that are directed against human antibodies are added b. a known antigen is fixed to a slide, and patient serum labeled with a fluorescent tag is added c. a known antigen is fixed to a slide, patient serum is added, and labeled anti-mouse immune serum globulin is added d. a slide is coated with patient serum, and labeled antibodies against human antibodies are added e. a slide is coated with patient serum, and microbes dyed with a fluorescent stain are added

a. a known antigen is fixed to a slide, patient serum is added, and labeled antibodies that are directed against human antibodies are added

Two patients developed endophthalmitis (infection of the internal layers of the eye) within 24 hours of corneal transplant surgery. Streptococcus pneumoniae was isolated from conjunctival swabs of the affected eyes of both patients and from the supply of packing that was used to cover the eyes after surgery. This is an example of __________. a. a nosocomial infection b. contact transmission c. vehicle transmission d. airborne transmission

a. a nosocomial infection

In a prokaryotic cell, all of the following are functions of either fimbriae or pili EXCEPT __________. a. protection from phagocytosis or determining the shape of the cell b. transfer of DNA c. formation of biofilms d. attachment to a surface

a. a. protection from phagocytosis or determining the shape of the cell

Which of the following processes requires cellular energy in the form of ATP? a. active transport b. simple diffusion c. osmosis d. facilitated diffusion

a. active transport

Cells may frequently find themselves in an environment with very low extracellular concentrations of substances needed in higher amounts inside the cell. To obtain these needed items, such cells would be most likely to engage in a. active transport. b. facilitated diffusion. c. osmosis. d. simple diffusion.

a. active transport.

The resistance to reinfection with measles virus following recovery from measles infection is called __________. a. adaptive immunity b. innate immunity c. passive immunity d. artificial immunity e. natural selection

a. adaptive immunity

Which type of organism produces the toxin responsible for paralytic shellfish poisoning? a. an alga b. a bacterium c. a helminth d. a protozoan

a. an alga

An injection of pooled human gamma globulin may provide passive immunity to humans from hepatitis A because it contains __________. a. antibodies b. T cytotoxic cells c. B cells d. T helper cells

a. antibodies

Microbes are NOT responsible for __________. a. antibody production b. breaking down wastes c. the synthesis of vitamins d. the synthesis of acetone and alcohol e. oxygen generation via photosynthesis

a. antibody production

Programmed cell death is referred to as __________. a. apoptosis b. phagocytosis c. agglutination

a. apoptosis

Interferons ___________. a. are host-specific but not virus-specific b. are virus-specific but not host-specific c. are both host-specific and virus-specific d. are useful only for treating viral infections e. can protect any host against any virus

a. are host-specific but not virus-specific

Which type of microscope uses a metal and diamond probe that is gently forced down along the surface of a specimen? a. Atomic force b. Confocal c. Transmission electron d. Scanning electron e. Scanning tunneling

a. atomic force

Some antimicrobial treatments kill microbes; some inhibit growth. Which term refers to an agent that acts by killing bacteria? a. bactericidal b. homeostatic c. germicidal d. bacteriostatic e. fungicidal

a. bactericidal

Which of the following physical methods of microbial control denatures proteins as a mechanism of action? a. boiling or autoclaving b. incineration c. osmotic pressure d. radiation

a. boiling or autoclaving

Which of the following properties are true of both bacteria and archaea? a. both are characterized as prokaryotes b. both have cell walls composed of peptidoglycan c. both are known to cause disease in humans d. both contain either DNA or RNA e. both bacteria and archaea contain methanogens, extreme halophiles, and extreme thermophiles

a. both are characterized as prokaryotes

In 1996, scientists described a new tapework parasite that had killed at least one person. The initial examination of the patient's abdominal mass was most likely made using: a. brightfield microscopy b. darkfield microscopy c. electron microscopy d. phase-contrast microscopy e. fluorescence microscopy

a. brightfield microscopy

Which enzyme catalyzes the following reaction? 2H2O2 → 2H2O + O2 a. catalase b. oxidase c. peroxidase d. superoxide dismutase

a. catalase

Looking at the cell of a photosynthetic microorganism, you observe that the chloroplasts are green in brightfield microscopy and red in fluorescent microscopy. You conclude that: a. chlorophyll is fluorescent b. the magnification has distorted the image c. you're not looking at the same structure in both microscopes d. the stain masked the green color e. none of the above

a. chlorophyll is fluorescent

Protozoan motility structures include a. cilia, flagella, and pseudopods. b. cilia c. pseudopods d. cilia and pseudopods only e. flagella

a. cilia, flagella, and pseudopods.

What is the general term for observable changes in cells that occur as a result of viral infection? a. cytopathic effects b. transformation bodies c. inclusion bodies d. syncytium

a. cytopathic effects

Which of the following types of microscopy provides advantages for viewing live images? a. darkfield microscopy b. fluorescent microscopy c. scanning electron microscopy d. transmission electron microscopy

a. darkfield microscopy

Application of heat to living cells can result in all of the following EXCEPT a. decreased thermal death time. b. breaking of hydrogen bonds. c. damage to nucleic acids. d. alteration of membrane permeability. e. denaturation of enzymes.

a. decreased thermal death time.

Several inherited deficiencies in the complement system occur in humans. Which of the following would be the MOST severe? a. deficiency of C3 b. deficiency of C5 c. deficiency of C6 d. deficiency of C7 e. deficiency of C8

a. deficiency of C3

The negative stain is used to a. determine cell size. b. visualize endospores. c. visualize fimbriae. d. determine Gram reaction. e. determine flagella arrangement.

a. determine cell size.

During log phase, bacteria are __________. a. dividing at the fastest possible rate b. preparing to divide c. dying and dividing in equal numbers d. dying exponentially

a. dividing at the fastest possible rate

Which of the following findings was essential for Edward Jenner's vaccination process? A) Exposure to a milder disease form may produce immunity. B) Disease is caused by viruses. C) Someone who recovers from a disease will not acquire that disease again. D) Pathogenic microorganisms infect all humans and animals in the same manner. E) A weakened microorganism will not cause disease.

a. exposure to a milder disease form may produce immunity

Neutrophils, basophils, and eosinophils are collectively referred to as __________. a. granulocytes b. agranulocytes c. plasma cells d. red blood cells

a. granulocytes

The presence of a capsule and the M protein of Streptococcus pyogenes are both involved in __________. a. helping bacteria resist phagocytosis b. helping bacteria kill phagocytes c. helping a virus change its surface antigens d. helping bacteria survive inside a phagocyte

a. helping bacteria resist phagocytosis

The __________ controls normal body temperature; it is stimulated to reset the body to a higher temperature in response to some infections. a. hypothalamus b. lacrimal apparatus c. complement cascade d. thymus

a. hypothalamus

In the Gram stain, crystal violet remains in gram-positive cells after treatment with alcohol because crystal violet--iodine (CV--I) complexes are trapped __________. a. in thick layers of peptidoglycan b. in the outer membrane c. by endospores d. by flagella e. in mycolic acid

a. in thick layers of peptidoglycan

An infection transmitted by a hypodermic needle is transmitted by __________. a. indirect contact b. direct contact c. common vehicle transmission d. a vector e. droplet transmission

a. indirect contact

Which of the following diseases is NOT correctly matched to it's reservoir? a. influenza - animal b. rabies - animal c. botulism - nonliving d. anthrax - nonliving e. toxoplasmosis - cats

a. influenza - animal

A cell undergoing apoptosis a. is employed as an infection-fighting mechanism. b. will likely damage nearby cells. c. bursts and releases intracellular contents. d. is a malfunction of the immune system. e. was necessarily bound by antibodies.

a. is employed as an infection-fighting mechanism.

Which of the following is not characteristic of a protozoan? a. lacks organelles such as a mitochondrion. b. may be autotrophic or heterotrophic c. contains ribosomes in its cytoplasm d. may utilize a flagellum for locomotion

a. lacks organelles such as a mitochondrion

In which phase of the growth curve is the population-doubling time fastest? a. log phase b. lag phase c. logarithmic decline phase d. death phase e. stationary phase

a. log phase

Which of these cells do NOT have phagocytic activity? a. lymphocytes b. eosinophils c. macrophages d. neutrophils

a. lymphocytes

Assume you mix red blood cells, antibodies against the red blood cells, and complement in a test tube. What would you expect to see? a. lysis of the red blood cells b. opsonization of the red blood cells c. agglutination of the red blood cells d. phagocytosis e. shrinkage (crenation) of the red blood cells

a. lysis of the red blood cells

Microbial control methods usually target any of the following microbial structuresEXCEPT which one? a. mitochondria b. microbial proteins c. plasma membrane d. permeability e. nucleic acids

a. mitochondria

Which type of adaptive immunity does the following statement describe?This type of immunity is acquired when antigens enter the body and an infection occurs. The immune system works to fight the infection via the innate and adaptive immune responses and creates an immunological memory of that particular antigen. a. naturally acquired active immunity b. artificially acquired passive immunity c. naturally acquired passive immunity d. artificially acquired active immunity

a. naturally acquired active immunity

Your lab partner slipped on his way to class and scraped his arm on the concrete. You make a smear of the fluid from his scrape and observe large nucleated cells. These cells are most likely __________. a. neutrophils b. lymphocytes c. erythrocytes d. bacteria

a. neutrophils

Which of the following is not a functionally analogous pair of stains? a. nigrosin and malachite green b. crystal violet and carbolfuchsin c. safranin and methylene blue d. ethanol-acetone and acid-alcohol e. none of the above

a. nigrosin and malachite green

Bacteria that CANNOT grow in the presence of oxygen (O2) are called __________. a. obligate anaerobes b. facultative anaerobe c. aerotolerant anaerobe d. microaerophile

a. obligate anaerobes

If a patient has a deep tissue infection as the result of an animal bite on the arm, the portal of entry is described as the __________. a. parenteral route b. skin c. connective tissue d. gastrointestinal route e. respiratory route

a. parenteral route

The potential use of viruses that infect bacteria to treat bacterial infections in humans is known as __________. a. phage therapy b. latent infection c. oncolytic therapy d. persistent therapy

a. phage therapy

The fate of E. coli O157:H7 in apple cider held at 8°C for 2 weeks, with and without preservatives, is shown below: - Cider only: 2.2 - Cider with potassium sorbate: 2.0 - Cider with sodium benzoate: 0.5 - Cider with potassium sorbate + sodium benzoate: 0 In the table, which preservative is most effective? a. potassium sorbate + sodium benzoate b. potassium sorbate c. no preservative d. sodium benzoate

a. potassium sorbate + sodium benzoate

In order to understand the full scope of a disease, we take its occurrence into account. The __________ of a disease is the number of people in a population who develop a disease at a specified time. a. prevalence b. incidence c. sporadic infection d. endemic infection

a. prevalence

Which is NOT specifically employed by pathogens to avoid destruction by phagocytosis? a. producing superantigens b. possessing ability to remain dormant within a phagocyte c. possessing ability to replicate within a phagolysosome d. forming biofilms e. producing a capsule

a. producing superantigens

Innate immunity includes all of the following EXCEPT a. production of antibody. b. inflammation. c. production of interferon. d. phagocytosis. e. activation of complement.

a. production of antibody.

You are performing a Gram stain on gram-negative bacteria and you stop after the addition of the mordant. What is the appearance of the bacteria at this point? a. purple b. red c. colorless d. brown

a. purple

Clostridium can be cultured in an anaerobic incubator or in the presence of atmospheric oxygen if thioglycolate is added to the nutrient broth. The addition of thioglycolate makes the medium __________. a. reduced b. selective c. differential d. enriched

a. reduced

Retroviridae use an RNA-dependent DNA polymerase called __________ to transcribe DNA from an RNA strand. a. reverse transcriptase b. provirus c. complimentary polymerase d. primase

a. reverse transcriptase

Which of the following types of microscopy is used to study the structure of the slimy accumulation of bacteria on an IV catheter tip? a. Scanning acoustic b. Atomic force c. Scanning electron d. Phase-contrast e. Confocal

a. scanning acoustic

On September 6, a 6 year old boy experienced chills, fever, and vomiting. On Sep 7, he was hospitalized with diarrhea and swollen lymph nodes under both arms. On Sep 3, the boy had been scratched by a cat. The cat was found dead on Sep 5, and Yersinia was isolated from the cat. Chloro was administered to the boy on Sep 7, when Yersinia was isolated from him. they boys temp returned to normal on Sep 17 and Sep 22 he was released from the hospital. identify the incubation period for this case of bubonic plague: a. sep 3-5 b. sep 3-6 c. sep 6-7 d. sep 6-17 identify the prodromal phase for this disease: a. sep 3-5 b. sep 3-6 c. sep 6-7 d. sep 6-17

a. sep 3-5 c. sep 6-7

Pasteurization was first developed to kill __________ in wine. a. spoilage bacteria b. all bacteria c. disease-causing bacteria d. antibiotic-producing bacteria e. probiotic bacteria

a. spoilage bacteria

Measles viruses are capable of inactivating host defenses by a. suppressing the immune system. b. producing superantigens. c. destroying complement proteins. d. producing leukocidins.

a. suppressing the immune system.

How would you recognize an antibiotic-producing soil bacterium on a plate crowded with other bacteria? The bacterial colony producing the antibiotic would be __________. a. surrounded by a clear area b. red or yellow c. star-shaped d. visible as massed called mycelia e. characterized by pseudopods

a. surrounded by a clear area

Members of the Adenoviridae cause ________________. a. the common cold b. hepatitis c. influenza d. cold sores e. smallpox

a. the common cold

Pasteurized milk in an unopened container spoils in the refrigerator. A sample reveals the presence of microorganisms. The most likely explanation is __________. a. the microbes that survived pasteurization were able to grow at 4ºC b. the microbes are pathogens c. the pasteurization process was ineffective d. pasteurization-resistant mutants developed after pasteurization e. the milk was pasteurized too long

a. the microbes that survived pasteurization were able to grow at 4ºC

Which of the following represents a chemical that is NOT an oxidizing agent and also would NOT be effective in killing the organism paired with it? a. triclosan; E. coli b. hypochlorite; cysts of protozoa c. hydrogen peroxide; Clostridium endospores d. chlorine; Mycobacterium tuberculosis e. ozone; Pseudomonas aeruginosa

a. triclosan; E. coli

Depending on the disease, individuals can be contagious during the incubation period, prodromal period, period of illness, period of decline, and period of convalescence. A) True B) False

a. true

Which cytokines are believed to contribute to autoimmune disease, such as rheumatoid arthritis? a. tumor necrosis factor (TNF) b. interferons c. chemokines d. hematopoietic cytokines

a. tumor necrosis factor (TNF)

Which type of nosocomial infection is the most common? a. urinary tract infection b. blood infection c. skin infection d. intestinal infection

a. urinary tract infection

If you were setting up an experiment to disprove spontaneous generation in a liquid medium, which of the following would be essential to the experiment? A) using a sterile liquid and eliminating exposure to microorganisms B) adding antibiotics to the liquid C) supplying the liquid with nutrients D) starting with a liquid that contains microorganisms E) adding carbon dioxide to the liquid

a. using a sterile liquid and eliminating exposure to microorganisms

The number of regions on an antibody molecule that can bind to an antigen is referred to as the __________ of that antibody. a. valence b. epitope c. hapten

a. valence

The practice dating back many hundreds of years in which dried material from smallpox lesions was used to protect against smallpox was called __________. a. variolation b. subunit vaccine c. attenuated vaccine d. nucleic acid vaccine

a. variolation

Ehrlichiosis is transmitted by ticks. This is an example of __________. a. vector transmission b. vehicle transmission c. indirect contact transmission d. airborne transmission

a. vector transmission

A maryland woman was hospitalized with dehydration V. cholera and P. shigelloides were isolated from the patient. she had attended a party 2 days before her hospitalization. two other people at the party had acute diarrheal illness and elevated levels of serum antibodies against virbio. everyone at the party ate crabs and rice pudding with milk. crabs left over from the party were served at a second party. one of the 20 people at the second party were served and had an onset of mild diarrhea ; specimens from 14 other people were negative for virbio antibodies this is an example of: a. vehicle transmission b. airborne transmission c. transmission by fomites d. direct contact transmission e. nosocomial transmission the etiologic agent of the disease is: a. P. shigelloides b. crabs c. virbio cholerae d. coconut milk e. rice pudding the source of the disease was: a. P. shigelloides b. crabs c. V. cholerae d. coconut milk e. rice pudding

a. vehicle transmission c. virbio cholerae b. crabs

The infectious agent that causes AIDS is a ______ A) virus. B) mold. C) bacterium. D) protozoan. E) yeast.

a. virus

As a health care worker, you are keenly aware of how important it is to avoid harming patients. You worry about inadvertently transmitting an infectious disease to an already compromised individual. According to the CDC, what is the most important thing you can do to avoid this? a. wash my hands before and after interacting with any patient b. wear a lab coat c. wear shoe covers d. shave my head

a. wash my hands before and after interacting with any patient

The figure shows a typical bacterial growth curve with the y-axis indicating the log of the number of bacteria and the x-axis indicating time in culture. In the figure, which section (or sections) shows a growth phase where the number of cells dying equals the number of cells dividing? a b c d a and c

c

Which domain of the A-B toxin binds to cell surface receptors on the host cell? a) Both the A and B domains have the ability to bind to cell surface receptors. b) A-B toxins do not bind to cell surfaces. c) B domain d) A domain

c) B domain

Which of the following would be the first sign of an infection that resulted in the release of endotoxin? a) Weakness b) Pain c) Fever d) Nausea

c) Fever

Why is a release of endotoxin into the bloodstream potentially deadly? a) It causes necrosis of the liver. b) It results in dehydration of the patient. c) It can lower blood pressure and cause the patient to go into shock. d) Endotoxin can quickly enter the brain from the bloodstream, causing brain damage.

c) It can lower blood pressure and cause the patient to go into shock.

In mice, the LD50 for staphylococcal enterotoxin is 1350 ng/kg, and the LD50 for Shiga toxin is 250 ng/kg. Which of the following statements is true? a) More organisms of Staphylococcal bacteria must be ingested to cause infection, as compared to Shigella bacteria. b) Staphylococcal enterotoxin is the more lethal of the two toxins. c) Shiga toxin is more lethal than staphylococcal enterotoxin. d) The parenteral route is the preferred portal entry for Shigella bacteria.

c) Shiga toxin is more lethal than staphylococcal enterotoxin.

Phagocytosis is a process for engulfing large particles (>1μm). Which feature of antibodies will help to make particles larger, therefore enhancing phagocytosis?

agglutination

A microorganism measures 5 μm in length. Its length in mm would be a. 500 mm. b. 0.05 mm. c. 0.005 mm. d. 0.5 mm. e. 50 mm.

c. 0.005 mm.

If a single bacterium replicated every 30 minutes, how many bacteria would be present in 2 hours? a. 4 b. 8 c. 16 d. 32 e. 64

c. 16

The most resistant of the bacterial endospores can withstand boiling for up to a. 10 min. b. 5 hours. c. 20 hours. d. 1 hour. e. 30 min.

c. 20 hours.

in an agglutination test, eight serial dilutions to determine antibody titer were set up. Tube 1 contained a 1:2 dilution, tube 2 contained a 1:4 dilution and so on. If tube 5 is the last tube showing agglutination, what is the antibody titer? a. 5 b. 1:5 c. 32 d. 1:32

c. 32

A kidney-transplant patient experienced a cytotoxic rejection of his new kidney. Place the following in order for that rejection:1. Apoptosis occurs; 2. CD8+ T cell becomes CTL; 3. Granzymes released; 4. MHC class I activates CD8+ T cell; 5. Perforin released. a. 1,2,3,4,5 b. 5,4,3,2,1 c. 4,2,5,3,1 d. 3,4,5,1,2 e. 2,3,4,1,5

c. 4,2,5,3,1

When using a 10× ocular lens and a 40× objective, the total magnification achieved is __________. a. 10 b. 40 c. 400 d. 50

c. 400

Assume that you inoculated potato salad with 10 bacterial cells and stored it at room temperature. After 3 hours, there are 640 cells. How many generations did the cells go through? a. 1 generation b. 3 generations c. 6 generations d. 12 generations

c. 6 generations

Alcohol is most effective when used as a __________ solution. a. 95% b. 100% c. 70% d. 25% e. 60%

c. 70%

Which of the following would be the first step in the biosynthesis of a virus with reverse transcriptase? a. A complementary strand of RNA must be synthesized. b. Double-stranded RNA must be synthesized. c. A complementary strand of DNA must be synthesized from an RNA template. d. A complementary strand of DNA must be synthesized from a DNA template. e. none of the above

c. A complementary strand of DNA must be synthesized from an RNA template.

Which of these effects is most likely to occur if a pathogen enters the body by a portal of entry other than the preferred one? a. Pathogens cannot enter by alternate routes. b. The exact same disease will result. c. A milder disease will result. d. A more severe disease will result.

c. A milder disease will result.

Which of the following statements is true? a.The primary goal of a pathogen is to kill its host. b. Evolution selects for the most virulent pathogens. c. A successful pathogen doesn't kill its host before it is transmitted. d. A successful pathogen never kills its host.

c. A successful pathogen doesn't kill its host before it is transmitted.

Which of the following is an INCORRECT statement about antimicrobial peptides (AMPs)? a. The modes of action of AMPs include inhibiting cell wall synthesis; forming pores in the plasma membrane, resulting in lysis; and destroying DNA and RNA. b. AMPs are also very stable over a wide range of pHs. c. AMPs have a narrow spectrum of antimicrobial activities. d. AMPs are on the most important components of the innate immune system.

c. AMPs have a narrow spectrum of antimicrobial activities.

Which of the following is true of an antimicrobial substance with a microbial death rate of 90% after 1 minute? a. All of the microbes would be killed instantly upon application of the agent. b. After the first 2 minutes of application of the agent, 10% of the original microbial count survives. c. After the first minute of application of the agent, 10% of the original microbes are still left on the surface. d. After 1 minute of application of the agent, 75% of the microbes would be killed.

c. After the first minute of application of the agent, 10% of the original microbes are still left on the surface.

Which of the following regarding antimicrobial control agents is FALSE? a. Contaminating organic debris such as blood or sputum decrease effectiveness. b. Some agents affect microbial cell membranes by dissolving lipids. c. Alcohols effectively inactivate nonenveloped viruses by attacking lipids. d. Some agents kill by denaturing microbial cell proteins. e. Silver-impregnated dressings are used for treating antibiotic-resistant bacteria.

c. Alcohols effectively inactivate nonenveloped viruses by attacking lipids.

To perform an indirect ELISA test to identify the strain of influenza in a patient, you would mix the following ingredients. Which do you add FIRST? a. Enzyme-linked antibody b. Enzyme substrate c. Antigen d. Patient's serum

c. Antigen

The CDC is located in a. Chicago, IL. b. Washington, DC. c. Atlanta, GA. d. Los Angeles, CA. e. New York City, NY.

c. Atlanta, GA.

Which of these statements is NOT true? a. Uncoating can occur because of host cell lysosome action. b. Penetration of enveloped viruses can occur by a process called fusion. c. Attachment of animal viruses to host cells is random and nonspecific. d. Animal viral DNA that is integrated into the host chromosome is called a provirus. e. Enveloped viruses are released from the cell by budding.

c. Attachment of animal viruses to host cells is random and nonspecific.

__________ are only involved in the humoral immune response. a. T helper cells b. NK cells c. B cells

c. B cells

Which of the following statements is NOT true of the classical pathway of complement activation? a. Activated C2a and C4b activate C3. b. C5b joins C6, C7, C8, and C9 to form the membrane attack complex. c. C3 is the first component to be activated. d. Activated C1 activates C2 and C4. e. C1 is activated by an antigen-antibody complex

c. C3 is the first component to be activated.

Which of the following describes the correct chronological order of events in phagocytosis? a. Ingestion, adherence, digestion, chemotaxis b. Chemotaxis, ingestion, adherence, digestion c. Chemotaxis, adherence, ingestion, digestion d. Ingestion, digestion, adherence, chemotaxis

c. Chemotaxis, adherence, ingestion, digestion

Which term best describes the symbiotic relationship between humans and most of the normal microbiota that live on our human skin? a. Pathological b. Parasitism c. Commensalism d. Mutualism

c. Commensalism

The ID50 for Campylobacter sp. is 500 cells; the ID50 for Cryptosporidium sp. is 100 cells. Which of the following statements is false? a. Both microbes are pathogens. b. Both microbes produce infections in 50% of the inoculated hosts. c. Cryptosporidium is more virulent than Campylobacter. d. Campylobacter and Cryptosporidium are equally virulent; they cause infections in the same number of test animals. e. Cryptosporidium infections are more severe than Campylobacter infections.

c. Cryptosporidium is more virulent than Campylobacter.

What type of molecule activates cytotoxic T lymphocytes? a. Antigen presented with MHC I b. Antibodies c. Cytokines released from T helper cells d. Perforin

c. Cytokines released from T helper cells

Which of the following toxins is NOT produced by a bacterium as a result of lysogenic conversion? a. Botulinum neurotoxin b. Staphylococcal enterotoxin c. Endotoxin d. Diphtheria toxin

c. Endotoxin

If the following are placed in the order of occurrence, which would be the third step? a. Diapedesis. b. Digestion. c. Formation of a phagosome. d. Formation of a phagolysosome. e. Margination.

c. Formation of a phagosome.

The most abundant Ig in the blood serum is __________. a. E b. M c. G d. A e. D

c. G

Operating rooms may receive ventilation from air that has passed through __________ filters to remove microorganisms. a. air b. UV c. HEPA d. dust

c. HEPA

A viral species is not defined on the basis of the disease symptoms it causes. The best example of this is: a. Polio. b. Rabies. c. Hepatitis. d. Chickenpox and shingles. e. Measles.

c. Hepatitis.

Which of the following pairs is NOT correctly matched? a. siderophore; traps iron b. fibrinolysin; lyses fibrin clots c. IgA protease; lyses fibrin clots d. leukocidin; lyses WBC membranes e. collagenase; breaks down connective tissue

c. IgA protease; lyses fibrin clots

In addition to IgG, the antibodies that can fix complement are a. IgA. b. IgD. c. IgM. d. IgE. e. None of the answers is correct.

c. IgM.

__________ acts to decrease refraction of light rays and thus increase resolution. a. The condenser b. The glass slide c. Immersion oil d. The specimen

c. Immersion oil

In humans, where do B cells mature? a. In the bursa b. In the lymph nodes c. In the bone marrow d. In the blood

c. In the bone marrow

Edema is defined as a collection of fluid in an area of the body. What is the physiological change that causes edema? a. Activation of complement b. Fever c. Increased permeability of blood vessels d. Constriction of blood vessels

c. Increased permeability of blood vessels

Which of the following would you use to detect antibodies against HIV in a patient? a. Direct fluorescent-antibody test b. Direct agglutination test c. Indirect ELISA d. Fluorescence-activated cell sorter

c. Indirect ELISA

Which one of these is NOT an example of pathogen entry via the parenteral route? a. Injection of the pathogen via a contaminated needle b. Injection via the bite of an infected insect c. Infection of a hair follicle d. Entry through a break in the skin caused by a cut

c. Infection of a hair follicle

Which of the following is NOT true of the gram-negative outer membrane? a. It is a part of the gram-negative cell wall. b. It contains lipopolysaccharide. c. It contains enzymes for energy synthesis. d. It has polysaccharide antigens that are useful in bacterial identification. e. It contains lipids also known as endotoxins.

c. It contains enzymes for energy synthesis.

Which of the following statements concerning adaptive immunity is FALSE? a. It can be stimulated by vaccination. b. It requires specific recognition of microbes and antigens. c. It is always present and instantly protects against infection. d. It can form memory responses.

c. It is always present and instantly protects against infection.

Which of the following statements concerning lysozyme is FALSE? a. It is an enzyme. b. It is found in many different body fluids. c. It is an organelle in white blood cells. d. It breaks down peptidoglycan.

c. It is an organelle in white blood cells.

A student is observing a Gram stain with the 100 × (oil immersion) lens. The image appears cloudy and dark. Which of the following actions will NOT increase the clarity of the image? a. Increasing the light using the diaphragm b. Using lens paper to clean any dirt or old oil off the lenses c. Lowering the stage to increase the distance between the object and the objective lens d. Adjusting the condenser to better focus light through the specimen

c. Lowering the stage to increase the distance between the object and the objective lens

Which of these is a cell wall component that contributes to invasiveness? a. Hemolysin b. Coagulase c. M protein d. Endotoxin

c. M protein

What is the typical mechanism of action for chemical food preservatives? a. Disruption of plasma membranes b. Oxidation c. Metabolic inhibition d. Surface-active agents for removal of microbes

c. Metabolic inhibition

Which type of radiation is LEAST effective in killing microbes? a. High-energy electron beams b. Ultraviolet rays c. Microwaves d. X rays e. Gamma rays

c. Microwaves

Meningitis and gonorrhea are caused by a. Pseudomonas species. b. measles virus. c. Neisseria species.

c. Neisseria species.

Barbara is worried about spending time with her grandchildren while being treated for shingles. Can her grandchildren contract chickenpox or shingles from spending time with their grandmother? a. Yes. Because they are young, the grandchildren are susceptible to both chickenpox and shingles. b. Yes. The grandchildren are susceptible because they do not have natural immunity against chickenpox or shingles. c. No. Because of their ages, the grandchildren have most likely been vaccinated against the chickenpox. They are also safe from contracting shingles because they are young. d. No. The children are safe from contracting the virus because Barbara is not contagious.

c. No. Because of their ages, the grandchildren have most likely been vaccinated against the chickenpox. They are also safe from contracting shingles because they are young.

Which of the following statements about culture media is FALSE? a. Complex media are formulated from nutrient-rich whole food sources whose exact ingredients may not be known. b. Reducing media are complex media containing chemicals, such as thioglycolate, that combine with oxygen, creating an anaerobic environment. c. Nutrient agar contains ingredients that combine with oxygen and remove it. d. A chemically defined medium is specifically formulated, and each ingredient is known.

c. Nutrient agar contains ingredients that combine with oxygen and remove it.

Hepatitis B virus transmitted by a finger-stick device enters the host via which portal of entry? a. Adherence b. Mucous membrane c. Parenteral d. Skin

c. Parenteral

In 1884, Elie Metchnikoff observed blood cells collected around a splinter inserted in a sea star embryo. This was the discovery of: a. Blood cells. b. Sea stars. c. Phagocytosis. d. Immunity. e. None of the above.

c. Phagocytosis.

Which of the following is true about antibiotics? a. Antibiotics are strong chemicals that resist microbial enzymes, retaining antimicrobial activity at all times. b. Antibiotics are chemotherapeutic agents that are always produced in the laboratory. c. Previously treatable microbes have become resistant to some antibiotics. d. Antibiotics kill a wide range of organisms, including viruses.

c. Previously treatable microbes have become resistant to some antibiotics.

Which of these microbes is the most difficult to destroy using either chemical or physical methods? a. MRSA (methicillin-resistant Staphylococcus aureus) b. Bacterial endospores c. Prions d. HIV

c. Prions

__________ ammonium compounds are less effective against gram-negative bacteria and more effective against gram-positive bacteria. a. Secondary b. Tertiary c. Quaternary d. Primary

c. Quaternary

P. aeruginosa and other gram-negative bacteria tend to be difficult to control with antibiotics because of their __________, which carry genes that determine resistance to antibiotics. a. F factors b. DNA c. R factors d. genomes

c. R factors

Martian soil is inoculated into a glucose-containing medium. The radioactive form of carbon, 14C, is used in the glucose. After incubation for five days, which of the following would provide evidence suggesting that there is life on Mars? a. No radioactivity b. Radioactive soil c. Radioactive carbon dioxide d. Radioactive glucose

c. Radioactive carbon dioxide

Which of these processes of viral multiplication is most likely to damage the host cell? a. Release of enveloped viruses b. Viral entry into host cells by fusion c. Release of nonenveloped viruses d. Uncoating e. Reverse transcription of retroviral RNA

c. Release of nonenveloped viruses

Which of the following diseases has been eliminated worldwide by vaccination? a. Measles b. Mumps c. Smallpox d. Polio

c. Smallpox

Consider a virus whose genome is composed of minus (-) sense RNA (for example, the rabies virus). What would be the first step in the biosynthesis of this virus? a. Synthesize DNA from the RNA template b. Synthesize proteins from the mRNA template c. Synthesize mRNA from the - sense RNA genome d. Synthesize mRNA from a DNA template

c. Synthesize mRNA from the - sense RNA genome

Which of these diseases does not have a human reservoir? a. Diphtheria b. Candida albicans c. Tetanus d. HIV e. Gonorrhea

c. Tetanus

What do all of these bacteria have in common? - Bacteria in the rumen of cattle and sheep - Bacteria in a sewage treatment plant - Bacteria growing in the middle ear in chronic otitis media - Bacteria growing on the teeth in dental plaque a. The bacteria are all gram-positive. b. The bacteria are all thermophilic. c. The bacteria are most likely growing in biofilms. d. The bacteria are all fermenters. e. The bacteria are all obligate aerobes.

c. The bacteria are most likely growing in biofilms.

When bacteria are inoculated into a new sterile nutrient broth, their numbers don't begin to increase immediately. Instead, there is a lag phase that may last for an hour or even several days. Why don't bacterial numbers increase immediately? a. The bacteria have to establish a biofilm before their numbers can increase. b. There are not enough nutrients for the bacteria to grow, and growth is delayed until there are some dead cells to cannibalize. c. The bacteria must adjust to the nutrient content in the new medium, synthesizing necessary amino acids, growth factors, and enzymes. d. The medium contains inhibitors that prohibit rapid growth of the bacteria, and these must be inactivated before bacterial numbers will increase.

c. The bacteria must adjust to the nutrient content in the new medium, synthesizing necessary amino acids, growth factors, and enzymes.

A student completes a Gram stain on a bacterial smear that has a mix of gram-negative and gram-positive organisms. The student stops the procedure without adding safranin. What would be the outcome of the Gram stain? a. All organisms would appear pink (gram-negative). b. The Gram stain would show pink (gram-negative) and purple (gram-positive) bacteria. c. The gram-positive organisms would appear purple, and the gram-negative organisms would be colorless. d. All organisms would appear colorless.

c. The gram-positive organisms would appear purple, and the gram-negative organisms would be colorless.

Which of the following is NOT necessary to satisfy Koch's postulates? a. The organism must cause the disease when introduced from pure culture into a healthy host. b. The organism must be re-isolated from the experimentally infected diseased animal and shown to be identical to the original isolate. c. The organism must cause disease through toxin production. d. The organism must be present in every case of the disease. e. The organism must be grown in pure culture outside the diseased host.

c. The organism must cause disease through toxin production.

Knowing that rubella virus causes hemagglutination, you mix red blood cells, rubella virus, and a patient's serum in a tube. Hemagglutination does NOT occur. What do you conclude? a. The patient has rubella. b. The patient does not have antibodies against rubella. c. The patient has antibodies against rubella virus. d. Rubella virus does not agglutinate red blood cells.

c. The patient has antibodies against rubella virus.

Which of the following are FALSE concerning microbes? a. Microbes produce vitamins in the intestines. b. Microbes generate oxygen through photosynthesis. c. The primary role of microbes on the planet involves causing disease in animals and humans. d. Microbes play a role in breaking down waste and decomposing dead organisms.

c. The primary role of microbes on the planet involves causing disease in animals and humans.

Which of the following best defines the term pathology? a. The invasion and colonization of a host by a microorganism b. Pathogenesis c. The structural and functional changes in an individual that are brought about by disease d. Etiology

c. The structural and functional changes in an individual that are brought about by disease

Which of these answers is a potential concern of using T-independent antigens as vaccines? a. The lack of macrophage participation will lead to a weak response. b. Many individuals have impaired responses to T-independent antigens because of an inherited T-cell disorder. c. These antigens will be ineffective in producing an immune response in infants. d. These antigens can induce autoimmune reactions. e. These antigens can lead to an aggressive cell-mediated immune response.

c. These antigens will be ineffective in producing an immune response in infants.

Which statement is NOT true of endotoxins? a. They are more heat-resistant than exotoxins are. b. They can induce chills, fever, aches, clotting, shock, and miscarriage. c. They are eliminated from the body as a result of antitoxin production. d. They can lyse amebocytes found in crab hemolymph. e. Endotoxins are produced by Neisseria meningitidis and E. coli.

c. They are eliminated from the body as a result of antitoxin production.

Which of the following statements is NOT true of A-B exotoxins? a. The B portion of the toxin binds to surface receptors on host cells. b. Many exotoxins are A-B toxins. c. They are produced only by gram-negative bacteria. d. They consist of two polypeptide components. e. The A portion of the toxin is the active component.

c. They are produced only by gram-negative bacteria.

Which of the following statements is INCORRECT regarding prokaryotic cells? a. They typically have a circular chromosome. b. They lack membrane-enclosed organelles. c. They lack a plasma membrane. d. They reproduce by binary fission. e. Their DNA is not enclosed within a membrane.

c. They lack a plasma membrane.

Which of the following statements is NOT true of toxoid vaccines? a. They usually require booster shots for maximum efficacy. b. They are used, for example, to prevent diphtheria. c. They may cause infection. d. They are derived from toxins. e. They are used, for example, to prevent symptoms of tetanus.

c. They may cause infection.

A direct agglutination test could be used for which of the following? a. To detect antibodies against a virus b. To detect viruses c. To detect antibodies against a bacterium d. To detect bacterial pathogens

c. To detect antibodies against a bacterium

What is the function of T cytotoxic cells? a. To produce antibodies b. To phagocytize pathogens c. To induce apoptosis of target cells d. To activate B cells

c. To induce apoptosis of target cells

In the Gram stain, what is the purpose of the counterstain? a. To decolorize gram-positive cells b. To make gram-positive cells visible c. To make gram-negative cells visible d. To decolorize gram-negative cells

c. To make gram-negative cells visible

Which of these chemicals inhibits an enzyme required for synthesis of membrane lipids? a. Hydrogen peroxide b. Chlorine c. Triclosan d. Ozone

c. Triclosan

Which one of the following is an example of bioremediation? a. Use of cowpox virus to vaccinate against smallpox b. Use of Bacillus thuringiensis to kill crop-eating insects c. Use of Pseudomonas to remove uranium from soil at weapons depots d. Use of Saccharomy cescerevisiae to make wine

c. Use of Pseudomonas to remove uranium from soil at weapons depots

Which of the following is NOT an oncogenic virus? a. Hepatitis B virus b. Human papillomavirus c. Varicellovirus d. HTLV-1

c. Varicellovirus

Which of the following pairs is NOT correctly matched? a. Persistent infection: an infection lasting months or years b. Prion: infectious protein c. Viroid: infectious DNA d. Latent viral infection: an inactive virus

c. Viroid: infectious DNA

Which of the following statements is false? a. Viruses contain DNA or RNA. b. The nucleic acid of a virus is surrounded by a protein coat. c. Viruses multiply inside living cells using viral mRNA, tRNA, and ribosomes. d. Viruses cause the synthesis of specialized infectious elements. e. Viruses multiply inside living cells.

c. Viruses multiply inside living cells using viral mRNA, tRNA, and ribosomes.

Which type of solution would cause a bacterium with a weak or damaged cell wall to burst as water moves into the cell? a. an isotonic solution b. a hypertonic solution c. a hypotonic solution d. either a hypotonic or an isotonic solution

c. a hypotonic solution

During a six-month period, 239 cases of pneumonia occurred in a town of 300 people. A clinical case was defined as fever ≥ 39°C lasting >2 days with three or more symptoms (i.e., chills, sweats, severe headache, cough, aching muscles/joints, fatigue, or feeling ill). A laboratory-confirmed case was defined as a positive result for antibodies against Coxiella burnetii. Before the outbreak, 2000 sheep were kept northwest of the town. Of the 20 sheep tested from the flock, 15 were positive for C. burnetii antibodies. Wind blew from the northwest, and rainfall was 0.5 cm compared with 7 to 10 cm during each of the previous three years. The situation is an example of: a. a nonliving reservoir. b. a vector. c. a zoonosis. d. a focal infection. e. human reservoirs. The etiologic agent of the disease in Situation 14.1 is A) sheep. B) Coxiella burnetii. C) wind. D) pneumonia. E) soil. The method of transmission of the disease in Situation 14.1 was A) direct contact. B) vehicle. C) vector-borne. D) indirect contact. E) droplet.

c. a zoonosis. B) Coxiella burnetii. B) vehicle.

When stained, bacteria in the genus Mycobacterium are __________. a. gram positive b. gram negative c. acid fast d. all of the listed responses are correct e. none of the listed responses are correct

c. acid fast

The stage of phagocytosis in which the phagocyte's plasma membrane attaches to the surface of the microbe is called __________. a. cytolysis b. ingestion c. adherence d. chemotaxis

c. adherence

The reaction of antibody with particulate antigen is called a(n) __________ reaction. a. immunodiffusion b. hemeagglutination c. agglutination d. precipitation reaction

c. agglutination

Florence Nightingale compared disease rates and other statistics for soldiers versus civilian populations. This is an example of __________. a. descriptive epidemiology b. controlled experimentation c. analytical epidemiology d. case reporting

c. analytical epidemiology

The yeast Candida albicans does NOT normally cause disease because of a. commensal bacteria. b. symbiotic bacteria. c. antagonistic bacteria. d. other fungi. e. parasitic bacteria.

c. antagonistic bacteria.

29) Robert Koch identified the cause of ______ A) diphtheria. B) tuberculosis. C) anthrax. D) smallpox. E) AIDS.

c. anthrax

Bacillus anthraciscauses the deadly disease anthrax. Organisms of the genus Bacillus may form endospores. This bacterium would be suitable for biological warfare because endospores __________. a. are multilayered structures b. are difficult to stain c. are resistant to high temperatures, UV light, and desiccation d. are centrally located within the bacterial cells e. contain a high concentration of toxic enzymes

c. are resistant to high temperatures, UV light, and desiccation

A vaccine consisting of a live avirulent strain of mumps virus is called a(n) __________. a. subunit vaccine b. conjugated vaccine c. attenuated vaccine d. toxoid

c. attenuated vaccine

A new microorganism has been isolated from hot springs in Yellowstone National Park. It consists of single cells, which appear to lack a nucleus. Chemical analysis shows the presence of both DNA and RNA in the cytoplasm and peptidoglycan in the cell wall. In which of the following groups will this organism be classified? a. archaea b. fungi c. bacteria d. plants e. protists

c. bacteria

Bacteria reproduce by __________. a. lysogeny b. mitosis c. binary fission d. meiosis

c. binary fission

The scum that builds up on shower doors, the formation of dental plaque on teeth, and the algae growth on the walls of swimming pools are all examples of __________. a. fimbriae b. capsules c. biofilms d. glycocalyces

c. biofilms

The following steps occur during bacteriophage replication. During which of these steps does nucleic acid replication occur? a. penetration b. lysis c. biosynthesis d. attachment

c. biosynthesis

Normal microbiota are typically found in and on all the following body locations EXCEPT the ______ A) mouth. B) upper respiratory system. C) blood. D) colon. E) skin.

c. blood

Some viruses leave a cell by pushing through the cell membrane (rather than lysing the cell). When this happens, a portion of the membrane wraps around the viral capsid, becoming the envelope. What is the name for this process? a. Conjugation b. Lysogeny c. Budding d. Biosynthesis

c. budding

Monoclonal antibodies are produced __________. a. in one animal b. by pooled sera from several humans c. by hybrid cells d. by one human

c. by hybrid cells

In the figure, which letter on the graph indicates the patient's secondary response to a repeated exposure with the identical antigen? a. a b. b c. c d. d e. e

c. c

In the figure, which line best depicts a psychrotroph incubated at 0°C? a. a b. b c. c

c. c

The viral hemagglutination inhibition test __________. a. involves antiviral antibodies enhancing binding of viral particles to erythrocytes b. involves antibodies against erythrocytes blocking viral binding sites c. can be used to detect the presence of antibody to the measles virus in patient serum d. can be used to detect the presence of the measles virus in patient serum e. is used to determine blood type

c. can be used to detect the presence of antibody to the measles virus in patient serum

Only a small percentage of microbes a. maintain balance between chemicals in the environment b. are pests that cause devastating crop diseases c. cause disease d. produce important products for living organisms, such as oxygen and vitamins

c. cause disease

Lysozyme and the antibiotic penicillin have similar mechanisms of action in that they both cause damage to the bacterial a. cell membrane. b. ribosomes. c. cell wall. d. capsule. e. DNA.

c. cell wall.

A new chemical messenger has been discovered that enhances the chemotaxis of macrophages and neutrophils toward sites of infection. It would be specifically classified as a(n) __________. a. interferon b. tumor necrosis factor c. chemokine d. hormone

c. chemokine

Evaluate the following chemical agents in regard to the effective use against endospores and mycobacteria. In a clinical situation where it is essential to control microbial growth that includes both mycobacteria and endospores, which chemical agent would be the most effective to guarantee the broadest disinfection? a. alcohols b. phenolics c. chlorines d. iodine

c. chlorines

What disease does the human herpesvirus-1 cause? a. canker sores b. chancres c. cold sores or fever blisters d. infectious mononucleosis

c. cold sores or fever blisters

Cell lines derived from transformed (cancerous) cells are called __________. a. plaques b. embryonated c. continuous cell lines d. primary cell lines e. monolayers

c. continuous cell lines

An exposure to _________ protects against infection with smallpox. a. penicillin b. chemotherapy c. cowpox d. bacteria e. normal microbiota

c. cowpox

What is the correct order for the Gram stain process? a. crystal violet → safranin → decolorizer → iodine b. iodine → crystal violet → decolorizer → safranin c. crystal violet → iodine → decolorizer → safranin d. malachite green → crystal violet → decolorizer → safranin

c. crystal violet → iodine → decolorizer → safranin

It has been said hat bacteria are essential for the existence of life on Earth. Which of the following is the essential function performed by bacteria? a. control insect populations b. directly provide food for humans c. decompose organic material and recycle elements d. cause disease e. produce hormones such as insulin

c. decompose organic material and recycle elements

You note that the body temperature of one of your patients is starting to increase. As a result, you can infer that all of the following may be occurring in this patient EXCEPT __________. a. increased metabolic rate b. shivering c. dilation of blood vessels d. constriction of blood vessels

c. dilation of blood vessels

Lister knew that carbolic acid (phenol) kills bacteria. He used it as the first __________. a. antibiotic b. vaccine c. disinfectant d. gene therapy

c. disinfectant

Which type of microscope is needed to view a virus in the laboratory? a. Compound b. Darkfield c. Electron d. Brightfield e. Fluorescent

c. electron

Which of the following is NOT a type of a compound microscope? a. brightfield microscope b. fluorescent microscope c. electron microscope d. darkfield microscope

c. electron microscope

Assume you stain Bacillus by applying malachite green with heat and then counterstain with safranin. Through the microscope, the green structures are: a. cell walls b. capsules c. endospores d. flagella e. impossible to identify

c. endospores

__________ is used to sterilize medical equipment that might be damaged by exposure to the heat of autoclaving. a. bleach b. iodine c. ethylene oxide gas d. alcohol

c. ethylene oxide gas

An organism that grows both in the presence and the absence of oxygen and uses oxygen when it is available is called a(n) __________. a. aerobe b. microaerophile c. facultative anaerobe d. anaerobe e. aerotolerant anaerobe

c. facultative anaerobe

You are testing the number of coliforms in a drinking water source. Which of the following test methods would be best to use? a. turbidity b. dry weight c. filtration d. direct microscopic count

c. filtration

__________ are too small to be seen with the light microscope without adding a mordant and carbolfuchsin. a. ribosomes b. nuclei c. flagella d. mitochondria

c. flagella

Which of the following could be used to sterilize plastic Petri plates in a plastic wrapper? a. microwaves b. autoclave c. gamma radiation d. sunlight e. ultraviolet radiation

c. gamma radiation

Antibiotics can lead to septic shock if used to treat a. viral infections. b. helminth infestations. c. gram-negative bacterial infections. d. protozoan infections. e. gram-positive bacterial infections.

c. gram-negative bacterial infections.

Which one of the following does NOT contribute to the incidence of healthcare-associated infections? a. antibiotic resistance b. lack of handwashing c. gram-negative cell walls d. lack of disinfecting surfaces e. lapse in aseptic techniques

c. gram-negative cell walls

A Gram stain of a wound reveals purple, spherical bacteria that divide and remain attached in grapelike clusters. These are referred to as __________. a. gram-positive streptococci b. gram-positive tetrads c. gram-positive staphylococci d. gram-negative spirochetes

c. gram-positive staphylococci

Surface-active agents act by decreasing surface tension so that microbes can more easily be removed. All of the following agents use this mechanism of action EXCEPT __________. a. acid-anionic sanitizers b. quaternary ammonium compounds (quats) c. halogens d. soaps and detergents

c. halogens

Which of the following is not a characteristic of bacteria? a. are prokaryotic b. have peptidoglycan cell walls c. have the same shape d. grow by binary fission e. have the ability to move

c. have the same shape

The protection from infection received when individuals susceptible to a particular disease live in a population where many individuals are immune is referred to as __________. a. disease prevention b. latent immunity c. herd immunity d. vaccination

c. herd immunity

Polyphosphate (volutin), carboxysomes, and magnetosomes are examples of __________. a. chloroplasts b. capsules c. inclusion bodies d. fimbriae e. mesosomes

c. inclusion bodies

Within the cytoplasm of bacterial cells are reserve deposits where certain nutrients accumulate. These are called __________. a. vacuoles b. ribosomes c. inclusions d. lysosomes

c. inclusions

Microorganisms are essential to our life. Each of the following is an example of a beneficial function of microorganisms EXCEPT a. agriculture b. bioremediation c. increased number of illnesses d. gene therapy e. alternative fuel production

c. increased number of illnesses

The rise in temperature that causes a fever is due to the hypothalamus responding to __________. a. intact gram-negative bacteria b. complement c. interleukin-1 d. interleukin-2

c. interleukin-1

Cell-mediated immunity in part protects against __________. a. toxins b. extracellular viruses c. intracellular bacteria and viruses d. viruses free in body fluids e. bacteria free in body fluids

c. intracellular bacteria and viruses

The following steps occur during binary fission. What is the third step? a. cell elongation b. DNA replication c. inward growth of membrane d. cross-wall formation

c. inward growth of membrane

Which of the following is not a characteristic of biofilms? a. antibiotic resistance b. hydrogel c. iron deficiency d. quorum sensing

c. iron deficiency

The term describing bacteria with flagella that are arranged as a tuft coming from one pole is __________. a. amphytrichous b. peritrichous c. lophotrichous d. monotrichous

c. lophotrichous

Which of these viral cytopathic effects is most likely to be associated with the development of cancer? a. stimulation of interferon production b. cell death c. loss of contact inhibition d. inclusion bodies e. cell fusion

c. loss of contact inhibition

Ehrlich searched for a/an __________ that would destroy a pathogen without harming the infected host. a. vital force b. vaccine c. magic bullet d. herbal remedy e. pathogen

c. magic bullet

Which of the following pathogens does NOT belong with the others? a. Hantavirus b. Brucella c. measles virus d. Yersinia pestis

c. measles virus

Complement components C5 through C9 form plasma membrane channels in cellular microbes referred to as the __________. a. inflammatory complex b. membrane channel complex c. membrane attack complex d. Cytolytic complex

c. membrane attack complex

Microbes have very narrow optimum temperature ranges. Which of the following classifications of microbes are most likely to cause human disease, based on temperature requirements? a. psychrophiles b. thermophiles c. mesophiles d. hyperthermophiles

c. mesophiles

Which of the following is the most convenient and appropriate unit for expressing the size of an average bacterial cell? a. millimeter b. centimeter c. micrometer d. nanometer

c. micrometer

Which of the following cells is NOT an APC? a. macrophages b. mature B cells c. natural killer cells d. dentritic cells e. None of the answers is correct; all of these are APCs.

c. natural killer cells

An individual may be exposed to a pathogen and become infected without actually getting sick. This is known as a subclinical infection. Even in subclinical infections, the individual's adaptive immune system can generate memory for the pathogen. What type of adaptive immunity is this? a. artificially acquired active immunity b. artificially acquired passive immunity c. naturally acquired active immunity d. naturally acquired passive immunity

c. naturally acquired active immunity

Influenza viruses are classified according to their hemagglutin and __________ proteins. a. arabinose b. polymorphic c. neuraminidase d. amphiphilic e. nucleic

c. neuraminidase

Microorganisms that typically colonize a host without causing disease are referred to as the __________. a. pathogens b. opportunistic pathogens c. normal microbiota d. transient microbes

c. normal microbiota

Health care professionals who fail to use aseptic techniques can cause __________. a. zoonoses b. pathogens c. nosocomial infections d. herd immunity e. predisposing factors

c. nosocomial infections

Which of the following are NOT typically destroyed byantibacterial agents? a. nucleic acids b. plasma membranes c. nuclei d. proteins

c. nuclei

Which of the following contributes to the virulence of a pathogen? a. numbers of microorganisms that gain access to a host and evasion of host defenses b. evasion of host defenses c. numbers of microorganisms that gain access to a host, evasion of host defenses, and toxin production d. numbers of microorganisms that gain access to a host e. toxin production

c. numbers of microorganisms that gain access to a host, evasion of host defenses, and toxin production

The term used to describe a disease-causing microorganism is: a. microbe b. infection c. pathogen d. virus e. bacterium

c. pathogen

Fimbriae and pili differ in that a. there are only one or two pili per cell. b. pili are used to transfer DNA. c. pili are used for transfer of DNA and motility. d. pili are used for motility. e. pili are used for attachment to surfaces.

c. pili are used for transfer of DNA and motility.

In the Gram stain, crystal violet is the __________. a. counterstain b. fixative c. primary stain d. mordant e. decolorizing agent

c. primary stain

Infectious agents known as __________ cause Creutzfeldt-Jakob disease (CJD). a. viroids b. plaques c. prions d. phages

c. prions

Vaccination eliminates diseases such as smallpox and polio by __________. a. killing pathogens b. using all of the listed methods c. producing non-susceptible hosts d. curing infected people

c. producing non-susceptible hosts

Bacteria require nitrogen for the synthesis of __________. a. lipids b. carbohydrates c. proteins d. fatty acids e. sugars

c. proteins

Consider a culture medium on which only gram-positive organisms such as Staphylococcus aureus colonies can grow due to an elevated NaCl level. A yellow halo surrounds the growth, indicating the bacterium fermented a sugar in the medium, decreasing the pH as a result and changing the color of a pH indicator chemical. This type of medium would be referred to as a(n) a. differential and enrichment culture. b. selective medium. c. selective and differential medium. d. differential medium. e. enrichment culture.

c. selective and differential medium.

Martin Lewis agar is an enriched media (containing heated blood) designed for the growth of Neisseria gonorrhea. Antibiotics are added to suppress the growth of normal microbiota that may be found in patient specimens, yet permit the growth of Neisseria gonorrhea. This medium would best be described as __________. a. differential media b. broad spectrum media c. selective media d. nutrient agar e. reduced media

c. selective media

When blood cells are removed from blood, the remaining liquid is referred to as __________. a. buffy coat b. leukocytic fluid c. serum d. red blood cells

c. serum

Normal microbiota provide resistance to disease in all of the following ways except __________________. a. microbial antagonism b. competitive exclusion c. serving as prebiotics d. promoting the development and maturation of the immune system

c. serving as prebiotics

An antiseptic is used to remove microbes from __________. a. toilet surfaces b. food preparation areas c. skin, before an injection d. foods, before canning them e. restaurant glassware

c. skin, before an injection

Which of the following pairs is matched correctly? a. L form—a wall-less bacterium created by the action of lysozyme; the plasma membrane remains intact, and the cell carries on metabolism b. Mycoplasma spp.—bacteria that spontaneously, or in response to penicillin or lysozyme, partially lose their cell walls and swell into irregularly shaped bacteria that divide and metabolize and may regain their cell wall c. spheroplast—gram-negative bacteria that are exposed to lysozyme but retain some of the outer membrane d. protoplast—a genus of bacteria that typically do not have cell walls

c. spheroplast—gram-negative bacteria that are exposed to lysozyme but retain some of the outer membrane

Spherical bacteria that divide and remain attached in chainlike patterns are called __________. a. staphylococci b. tetrads c. streptococci d. spirochetes

c. streptococci

All of the following are found in the cell walls of gram-negative bacteria EXCEPT __________. a. lipid A b. porins c. teichoic acid d. N-acetylglucosamine e. tetrapeptide chains

c. teichoic acid

Assume you inoculated 100 facultatively anaerobic cells onto nutrient agar and incubated the plate aerobically. You then inoculated the second plate anaerobically. After incubation for 24 hours, you should have.... a. more colonies on the aerobic plate b. more colonies on the anaerobic plate c. the same number of colonies on both plates

c. the same number of colonies on both plates

Molecular biology is the study of ______ A) DNA synthesis. B) protein synthesis. C) the structure and function of macromolecules essential to life. D) RNA replication. E) enzyme function.

c. the structure and function of macromolecules essential to life

Why are viruses not classified as prokaryotes or eukaryotes? a. they lack a cell wall b. they lack peptidoglycan c. they need a host to survive, thus are not considered alive d. they are so small, an electron microscope is needed to observe them

c. they need a host to survive, thus are not considered alive

Normal microbiota provide protection from infection in each of the following ways EXCEPT a. they make the chemical environment unsuitable for nonresident bacteria. b. they change the pH of the environment. c. they produce lysozyme. d. they produce antibacterial chemicals. e. they compete with pathogens for nutrients.

c. they produce lysozyme.

A vaccine consisting of an inactivated diphtheria toxin is called a(n) __________. a. subunit vaccine b. attenuated whole-agent vaccine c. toxoid d. conjugated vaccine

c. toxoid

The motility of bacteria with flagella occurs through a series of "runs" and __________. a. phototaxis b. chemotaxis c. tumbles d. turns

c. tumbles

Which of the following pairs is mismatched? a. plasma sterilization - free radicals b. ozone - takes electrons from substances c. ultraviolet radiation - desiccation d. supercritical fluids - CO2 e. ionizing radiation - hydroxyl radicals

c. ultraviolet radiation - desiccation

The rise in herd immunity amongst a population can be directly attributed to a. improved handwashing. b. antibiotic-resistant microorganisms. c. vaccinations. d. increased use of antibiotics. e. None of the answers is correct.

c. vaccinations.

Cytopathic effects are changes in host cells due to a. fungal infections. b. protozoan infections. c. viral infections. d. helminthic infections. e. bacterial infections.

c. viral infections.

Acid-fast Mycobacteria differ from non-acid-fast bacteria by the presence of __________. a. capsules b. endospores c. waxy material in their cell walls d. flagella e. peptidoglycan

c. waxy material in their cell walls

Antigenic stimulation of a particular B cell that results in the production of a large number of plasma and memory cells, all capable of responding to that antigen, is referred to as _______

clonal selection

When would endotoxins be released from a bacterial cell? a) When the cell attaches to a host cell in the human body b) When the cell moves toward a energy source c) During bacterial conjugation d) When the cell dies

d) When the cell dies

Which of the following statements best describes what happens to a cell exposed to polymyxins that destroy phospholipids? a) in an isotonic solution, nothing will happen b) in a hypotonic solution, the cell will lyse c) water will move into the cell d) intracellular contents will leak from the cell e) any of the above might happen

d) intracellular contents will leak from the cell

A patient who has been hospitalized with uncontrolled muscle spasms has probably been infected with bacteria that secrete a(n) a) membrane disrupting toxin. b) superantigen c) enterotoxin. d) neurotoxin.

d) neurotoxin.

Which statement best describes what happens when a gram-positive bacterium is placed in distilled water and penicillin? a) No change will result; the solution is isotonic b) Water will move into the cell c) Water will move out of the cell d) the cell will undergo osmotic lysis e) Sucrose will move into the cell from an area of higher concentration to one of lower concentration

d) the cell will undergo osmotic lysis

A paramecium is approximately 150 micrometers in length. What is this measurement expressed in millimeters (mm)? a. 1.5 mm b. 0.015 mm c. 1500 mm d. 0.15 mm e. 15 mm

d. 0.15 mm

Assume you stain Clostridium by applying a basic stain, carbolfuchsin, with heat, decolorizing with acid-alcohol, and counterstaining with an acid stain, nigrosin. Through the microscope, the endospores are (1)____, and the cells are stained (2)____. a. 1 - red; 2 - black b. 1 - black; 2 - colorless c. 1 - colorless; 2 - black d. 1 - red; 2 - colorless e. 1 - black; 2 - red

d. 1 - red; 2 - colorless

At a minimum, the human immune system is capable of recognizing approximately how many different antigens? a. 10^20 b. 10^10 c. 10^25 d. 10^15 e. 10^5

d. 10^15

Put the following in the correct sequence to elicit an antibody response: 1. Th cell recognizes B cell; 2. APC contacts antigen; 3. antigen fragment goes to surface of APC; 4. Th recognizes antigen digest and MHC; 5. B cell proliferates. a. 1,2,3,4,5 b. 5,4,3,2,1 c. 3,4,5,1,2 d. 2,3,4,1,5 e. 4,5,3,1,2

d. 2,3,4,1,5

Which of the following places these items in the correct order for DNA-virus replication?1. Maturation2. DNA synthesis3. Transcription4. Translation a. 1; 2; 3; 4 b. 3; 4; 1; 2 c. 4; 1; 2; 3 d. 2; 3; 4; 1 e. 4; 3; 2; 1

d. 2; 3; 4; 1

Place the following in the order in which they are found in a host cell: (1) capsid proteins; (2) infective phage particles; (3) phage nucleic acid. a. 1, 2, 3 b. 3, 2, 1 c. 2, 1, 3 d. 3, 1, 2 e. 1, 3, 2

d. 3, 1, 2

What is the usual size range of viruses? a. 20 to 300 micrometers b. 20 to 300 millimeters c. 20 to 300 centimeters d. 30 to 300 nanometers

d. 30 to 300 nanometers

If 52 bacterial colonies grow on a nutrient agar plate inoculated with 1 ml of a 1:1000 dilution of hamburger, how many bacteria are in the original hamburger sample? a. 52 bacteria per gram b. 520 bacteria per gram c. 5200 bacteria per gram d. 52,000 bacteria per gram

d. 52,000 bacteria per gram

Decimal reduction time is the time in minutes in which __________ of the population at a given temperature will be killed. a. 10% b. 75% c. 100% d. 90%

d. 90%

What is a plasma cell? a. A cell that can directly kill other cells b. A cell that has phagocytic capability c. A cell that produces inflammatory chemicals d. A cell that produces antibodies

d. A cell that produces antibodies

Which of the following inanimate objects could ordinarily be treated with a low-level disinfectant? a. An endoscope used to examine the colon b. An endospore-contaminated rectal thermometer c. A surgical scalpel d. A hospital scale

d. A hospital scale

What does refraction mean? a. The ability of a microscope to stay in focus when the objective lenses are moved from low to high magnification b. The ability of a microscope to distinguish fine details and differentiate between two very close objects c. The total magnification of the visual image seen d. A measure of the light-bending ability of a medium

d. A measure of the light-bending ability of a medium

The acid-fast stain differs from the Gram stain procedure in the decolorizing agent used. What is the reagent used in an acid-fast stain, and why is it necessary? a. Acetone-alcohol is used as the decolorizer in order to enhance the primary stain. b. Acetone is used as the decolorizer to penetrate the endospore. c. Alcohol is used as the decolorizer in order to penetrate the bacterial capsule. d. Acid-alcohol is used to remove the primary stain from bacteria that do not have waxy cell walls.

d. Acid-alcohol is used to remove the primary stain from bacteria that do not have waxy cell walls.

If the following are placed in the order of occurrence, which would be the third step? a. Activation of C5 through C9. b. Cell lysis. c. Antigen-antibody reaction. d. Activation of C3. e. Activation of C2 through C4.

d. Activation of C3.

Which of the following statements about T-dependent antigens is true? a. A TH cell acts as an antigen-presenting cell for the T-dependent antigen. b. T-dependent antigens are usually polysaccharides. c. CTL cells are required for the activation process. d. Activation of a B cell by a T-dependent antigen requires cytokines secreted by a TH cell.

d. Activation of a B cell by a T-dependent antigen requires cytokines secreted by a TH cell.

__________ are molecules on bacterial cell surfaces that enable them to adhere to the surface of host cells. a. Coagulases b. Invasins c. Siderophores d. Adhesins

d. Adhesins

Which of the following factors contributes to the frequency of healthcare-associated infections (HAIs)? a. Transmission from healthcare professionals to patients. b. The presence of patients with weakened immune systems. c. The presence of a wide range of pathogens in the healthcare environment. d. All of the above

d. All of the above

Which of the following is NOT a cytopathic effect (CPE) of viruses? a. chromosomal changes in the host cell b. formation of inclusion bodies c. the formation of a syncytium d. All of the listed choices are possible cytopathic effects of viruses.

d. All of the listed choices are possible cytopathic effects of viruses.

Which of the following would be an example of an infection initiated via the parenteral route? a. An individual contracts a hookworm infection as a result of walking around outside barefoot. b. An individual contracts gonorrhea as a result of unprotected sex. c. An individual contracts a gastrointestinal infection by consuming contaminated water. d. An individual contracts hepatitis B from an accidental stick with a contaminated needle.

d. An individual contracts hepatitis B from an accidental stick with a contaminated needle.

Which of the following statements about ribosomes is FALSE? a. Eukaryotic ribosomes are larger (80S) than prokaryotic ribosomes (70S). b. Cells with a high rate of protein synthesis have large numbers of ribosomes. c. Antibiotics that interfere with protein synthesis harm prokaryotic ribosomes, but not eukaryotic ribosomes. d. Antibiotics that interfere with protein synthesis harm both prokaryotic and eukaryotic ribosomes.

d. Antibiotics that interfere with protein synthesis harm both prokaryotic and eukaryotic ribosomes.

Which of these molecules or structures is/are NOT associated with innate immunity? a. Mucous membranes b. Lysozyme c. Phagocytes d. Antibodies e. Macrophages

d. Antibodies

Cyclosporine is a drug sometimes used to prevent transplant rejection after organ transplant surgery. This drug specifically disrupts cell-mediated immunity by cytotoxic T cells. Which of these events can be predicted based on this information? a. Antigen presentation by macrophages and dendritic cells will be impaired. b. The recipient will be susceptible to repeated infections with common bacteria such as staphylococci and streptococci. c. Autoimmune disorders will be a potential side effect. d. Antibody production will NOT be disrupted in the recipient. e. T helper cells will no longer be produced.

d. Antibody production will NOT be disrupted in the recipient.

Which of the following cannot be used to sterilize a heat- labile solution stored in a plastic container? a. Gamma radiation b. Ethylene oxide c. Supercritical fluids d. Autoclaving e. Short- wave length radiation.

d. Autoclaving

Which of the following treatments achieves sterilization? a. Freezing b. Pasteurization c. Bringing water to a boil d. Autoclaving

d. Autoclaving

Chlamydia can prevent the formation of phagolysosomes and therefore can: a. Avoid being phagocytosed. b. Avoid destruction by complement. c. Prevent adherence. d. Avoid being digested. e. None of the above.

d. Avoid being digested.

Which biosafety level features open laboratory bench tops, gloves, lab coat, and face and eye protection? a. BSL-1 b. BSL-3 c. BSL-4 d. BSL-2

d. BSL-2

Which term refers to an agent that inhibits bacterial growth? a. Fungicidal b. Germicidal c. Homeostatic d. Bacteriostatic e. Bactericidal

d. Bacteriostatic

Which of these pathogens would most likely be attacked by antibody-dependent cell-mediated cytotoxicity? a. Varicella-zoster virus b. Salmonella typhi c. Staphylococcus aureus d. Blood flukes (schistosomes)

d. Blood flukes (schistosomes)

Which of the following is not a portal of entry for pathogens? a. Mucous membranes of the respiratory tract. b. Mucous membranes of the gastrointestinal tract. c. Skin. d. Blood. e. Parenteral route.

d. Blood.

Which one of these diseases is NOT communicable? a. HIV b. Influenza c. Streptococcal sore throat d. Botulism

d. Botulism

Activation of which of these complement proteins causes cytolysis? a. C1 b. C3 c. C5 d. C9

d. C9

Which of the following statements concerning cellular immunity is FALSE? a. T cells respond to antigens when the antigens bind to receptors on their surface. b. The thymus is necessary for development of cells involved in cellular immunity. c. The lymphocytes involved with cellular immunity are found primarily in lymphoid organs and blood. d. Cellular immunity involves cells that recognize antigens and make specific antibodies against them.

d. Cellular immunity involves cells that recognize antigens and make specific antibodies against them.

Which one of these halogen compounds would be considered an antiseptic? a. Chlorine gas b. Hypochlorous acid c. Calcium hypochlorite d. Chlorine dioxide

d. Chlorine dioxide

Which of these immunological assays for serum antibody requires cells as the indicator? a. Precipitin ring b. Immunodiffusion c. Indirect agglutination d. Complement fixation e. ELISA

d. Complement fixation

When attracted to an infected area, macrophages can leave the bloodstream by squeezing through the endothelial cells lining a blood vessel. What is this process called? a. Adherence b. Margination c. Opsonization d. Diapedesis

d. Diapedesis

To perform a direct ELISA test for rotavirus, you would mix the following ingredients. Which do you add LAST? a. Anti-rotavirus antibodies b. Patient sample c. Enzyme-linked antibody d. Enzyme substrate

d. Enzyme substrate

Which one of these products is most useful for disinfecting medical instruments? a. Iodine b. Soap and detergents c. Alcohols d. Glutaraldehyde

d. Glutaraldehyde

Which one of the following pairs is correctly matched? a. lysosomes—secretion b. mitochondrion—cell division c. centrioles—cellular respiration d. Golgi complex—membrane formation and secretion of proteins

d. Golgi complex—membrane formation and secretion of proteins

Which of these statements about toxoids is INCORRECT? a. Exotoxins can be altered by heat or chemicals to form toxoids. b. Diphtheria and tetanus can be prevented by toxoid vaccinations. c. Toxoids stimulate the body to produce antitoxins without causing disease. d. Gram-negative septic shock is commonly prevented by toxoids.

d. Gram-negative septic shock is commonly prevented by toxoids.

Penicillin specifically interferes with peptidoglycan synthesis. Which of the following cells is most likely to be damaged by penicillin? a. Cells of archaea b. Yeast cells c. Human cells d. Gram-positive bacterial cells

d. Gram-positive bacterial cells

You are observing a Gram stain of rod-shaped microorganisms that are linked in a chain and stain purple. How would you describe these bacteria using the correct terminology for the cell shape and arrangement? a. Gram-positive tetrads b. Gram-negative staphylococci c. Gram-positive coccobacilli d. Gram-positive streptobacilli

d. Gram-positive streptobacilli

Which of the following terms are mismatched? a. Extreme thermophiles; 100°C b. Extreme halophiles; 30% salt c. Psychrophiles; 0°C d. Hyperthermophiles; 0°C e. Acidophiles; low pH

d. Hyperthermophiles; 0°C

Which of the following statements is true? a. Quats are sporicidal, while hypochlorite-based disinfectants are not. b. Quats are halogen-based disinfectants, while hypochlorite-based disinfectants are cationic detergents. c. Quats are considered high-level disinfectants, while hypochlorite-based disinfectants are not. d. Hypochlorite-based disinfectants are effective against a wider range of pathogens than quats.

d. Hypochlorite-based disinfectants are effective against a wider range of pathogens than quats.

Which statement regarding the structure or function of ribosomes is correct? a. The ribosomes in the prokaryote are slightly larger than those found in the eukaryotic endoplasmic reticulum. b. Ribosomes are found both free-floating and attached to the rough endoplasmic reticulum in eukaryotes and in prokaryotes. c. Ribosomes are the sites of lipid biosynthesis in both prokaryotes and eukaryotes. d. In eukaryotes, the ribosomes found in chloroplasts and mitochondria are 70S ribosomes, which are similar in size to prokaryotic ribosomes.

d. In eukaryotes, the ribosomes found in chloroplasts and mitochondria are 70S ribosomes, which are similar in size to prokaryotic ribosomes.

What is the role of iodine in the Gram stain process? a. Iodine is a basic purple dye, which acts as the primary stain in the Gram stain technique. b. Iodine is the decolorizer removing the primary stain after the first step in the Gram stain technique. c. Iodine is a pink dye that acts as the primary stain in the Gram stain technique. d. Iodine is a mordant in the Gram stain technique, which functions to intensify the primary stain.

d. Iodine is a mordant in the Gram stain technique, which functions to intensify the primary stain.

Which of the following statements is correct about passive diffusion? a. It is a process in which molecules move from a region of lower concentration to one of higher concentration (or up a concentration gradient). b. It requires an expenditure of energy by the cell. c. It may require a transport protein. d. It involves movement of molecules down a concentration gradient and may require a transport protein. e. It is a process in which molecules move from a region of higher concentration to a region of lower concentration (or down a concentration gradient).

d. It involves movement of molecules down a concentration gradient and may require a transport protein.

Which of the following statements about IFN- (alpha) is false? a. It interferes with viral replication. b. It is host-cell-specific. c. It is released by fibroblasts. d. It is virus-specific. e. It is released by lymphocytes.

d. It is virus-specific.

A strain of Neisseria gonorrhoeae has been genetically altered and can no longer produce fimbriae. What is a likely outcome? a. It will be readily phagocytized upon entering the host b. It will be unable to carry out aerobic respiration c. It will no longer be able to secrete exotoxins. d. It will be unable to adhere to host tissue and establish infection e. It will demonstrate increased resistance to antibiotics

d. It will be unable to adhere to host tissue and establish infection

________ is the physician first associated with vaccination. A) Koch B) Escherich C) Pasteur D) Jenner E) Lister

d. Jenner

__________ are inflammatory molecules that are usually found in blood in an inactive form. Once activated, they help to attract neutrophils to the injured area. a. Leukotrienes b. Histamines c. Prostaglandins d. Kinins

d. Kinins

Which kind of vaccine may confer lifelong immunity against a pathogen? a. Inactivated killed vaccines b. Toxoids c. Subunit vaccines d. Live attenuated vaccines

d. Live attenuated vaccines

Which of the following is NOT an effect an antibody might have on a target cell? a. Agglutination b. Neutralization c. Opsonization d. Lysis

d. Lysis

Which of the following statements is NOT true of inflammation? a. Inflammation can be triggered by microbial infection, burns, exposure to chemicals, or trauma. b. Histamine released by damaged host cells can result in vasodilation. c. Vasodilation causes redness in affected tissues. d. Many neutrophils can be found at the site of chronic inflammation. e. Granulocytes that have died are commonly engulfed by macrophages.

d. Many neutrophils can be found at the site of chronic inflammation.

Which of the following regarding antimicrobial control agents is FALSE? a. Some agents affect microbial cell membranes by dissolving lipids. b. Some agents kill by denaturing microbial cell proteins. c. Contaminating organic debris such as blood or sputum will decrease effectiveness. d. Most chemical agents can achieve sterility. e. Some agents are utilized as both an antiseptic and a disinfectant.

d. Most chemical agents can achieve sterility.

Patients with Chediak-Higashi syndrome suffer from various types of cancer. These patients are most likely lacking which of the following: a. Tr cells b. Th1 cells c. B Cells d. NK cells e. Th2 Cells

d. NK cells

What is the difference between normal and transient microbiota? a. Normal microbiota are opportunistic pathogens. b. Normal microbiota cause disease. c. Normal microbiota are on the human body. d. Normal microbiota are permanently present.

d. Normal microbiota are permanently present.

Which of the following statements is NOT true of toxoid vaccines? a. They are used, for example, to prevent symptoms of tetanus. b. They are used, for example, to prevent diphtheria. c. They require booster shots for maximum efficacy. d. Only one dose is generally enough to infer a lasting immune response. e. They are derived from toxins.

d. Only one dose is generally enough to infer a lasting immune response.

__________ pathogens are those that do NOT cause disease in their normal habitat but may do so in a different environment. a. Nosocomial b. Resident c. Transient d. Opportunistic

d. Opportunistic

Which of the following statements concerning pathology, infection, and disease is true? a. The term infection is synonymous with the term disease. b. Microorganisms that make up the normal microbiota of an individual never cause disease. c. The majority of microorganisms are pathogenic. d. Pathology refers to the study of structural and functional changes that occur in the body as a result of a disease.

d. Pathology refers to the study of structural and functional changes that occur in the body as a result of a disease.

Which of these viruses can incorporate the molecule serving as mRNA into its capsid? a. Rhabdovirus b. Herpesvirus c. Poxvirus d. Picornavirus

d. Picornavirus

Which of the following is NOT a functionally analogous pair? a. Bacterial flagella; 9 + 2 flagella b. Peptidoglycan; cellulose c. 70s ribosomes; 80S ribosomes d. Pili; centrioles e. Circular bacterial chromosome; linear eukaryotic chromosomes

d. Pili; centrioles

Which of the following statements does NOT provide evidence for the endosymbiotic theory? a. The ribosomes contained within mitochondria and chloroplasts are very similar to prokaryotic ribosomes. b. Mitochondria and chloroplasts contain circular DNA, similar to the DNA in prokaryotes. c. The same antibiotics that inhibit protein synthesis in prokaryotes also inhibit protein synthesis within mitochondria and chloroplasts. d. Prokaryotes contain peptidoglycan in their cell walls.

d. Prokaryotes contain peptidoglycan in their cell walls.

Which of the following would most likely resist dessication and high pressure? a. Viruses b. Protozoans c. Molds d. Spore-forming bacteria

d. Spore-forming bacteria

The removal of plasmids reduces virulence in which of the following organisms: a. Clostridium tetani b. Escherichia coli c. Salmonella enterica d. Streptococcus mutans e. Clostridium botulinum

d. Streptococcus mutans

__________ vaccines are made from antigenic fragments of microbes rather than whole microbes. a. Attenuated b. Nucleic acid c. Conjugated d. Subunit

d. Subunit

Which of the following does not represent the same mechanism for avoiding host defenses as the others? a. Rabies virus attaches to the receptor for the neurotransmitter acetylcholine. b. Salmonella attaches to the receptor for epidermal growth factor. c. Epstein-Barr (EB) virus binds to the host receptor for complement. d. Surface protein genes in Neisseria gonorrhoeae mutate frequently. e. None of the above.

d. Surface protein genes in Neisseria gonorrhoeae mutate frequently.

Which of the following CANNOT function as an antigen-presenting cell? a. dendritic cell b. macrophage c. B cell d. T cell

d. T cell

For pathogen below, choose the type of cell that would be used in the adaptive immune response. * Regarding to picture! a. B cells b. TH17 cells c. TH1 cells d. TH2 cells

d. TH2 cells

Pneumococcal vaccines are made using the polysaccharide molecules that make up the capsule of the bacterial cell. How would the vaccine given to adults differ from the one given to infants? a. The adult vaccine is an attenuated whole-agent vaccine; the vaccine for infants is an inactivated whole-agent vaccine. b. The adult vaccine is a DNA vaccine; the vaccine for infants is a subunit vaccine. c. The adult vaccine is a toxoid; the vaccine for infants is a subunit vaccine. d. The adult vaccine consists of purified capsular polysaccharide; the vaccine for infants is a conjugated vaccine in which the polysaccharide is attached to a protein.

d. The adult vaccine consists of purified capsular polysaccharide; the vaccine for infants is a conjugated vaccine in which the polysaccharide is attached to a protein.

After receiving a tick bite, Jenny worries that she may have become infected with Lyme disease. Her doctor immediately orders a Lyme titer, which comes back as negative. Her doctor advises her to come back in three weeks and have the titer repeated. Why? a. The doctor will send the second titer to a different laboratory as a backup. b. No antibodies are produced by the immune system until at least 3 weeks following infection. c. T cell stimulation is delayed during Lyme disease incubation. d. The initial titer may have been done so early in the infection that detectable levels of antibody were not yet present. e. The laboratory may have made an error, but it is unlikely to make the same error twice.

d. The initial titer may have been done so early in the infection that detectable levels of antibody were not yet present.

Which of the following statements regarding biofilms is true? a. Biofilms occur in nature but rarely occur in human infections. b. Fortunately, biofilms are not an issue that impacts human health. c. The close proximity of microbes within a biofilm probably inhibits conjugation. d. The microbes in biofilms can work cooperatively to carry out complex tasks.

d. The microbes in biofilms can work cooperatively to carry out complex tasks.

Which of the following statements about the development of infectious diseases is correct? a. The prodromal period is characterized by very severe symptoms. b. The period of decline is the time when the infected individual's health rapidly deteriorates. c. During the incubation period, the infected individual exhibits obvious signs of sickness. d. The period of convalescence is the time during which the person regains health and fully recovers (back to the pre-disease state).

d. The period of convalescence is the time during which the person regains health and fully recovers (back to the pre-disease state).

How would you know that viruses were multiplying in a confluent lawn of E. coli on a solid culture medium? a. There would be small blue spots on the bacterial culture. b. The bacterial culture would grow faster. c. The bacterial colonies would swell. d. There would be small zones of clearing in the bacterial culture.

d. There would be small zones of clearing in the bacterial culture.

Which of the following is NOT a type of vaccine? a. Subunit vaccines b. Conjugated vaccine c. Nucleic acid vaccines d. They are all considered types of vaccines.

d. They are all considered types of vaccines.

Which statement is true of endotoxins? a. They increase blood pressure. b. They are disease specific. c. They are proteins. d. They are released upon cell lysis. e. They are produced by gram-positive bacteria.

d. They are released upon cell lysis.

Which of these statements is true for conjugated vaccines? a. They contain attenuated pathogens. b. The antigens are made using recombinant DNA technology. c. They contain only T-dependent antigens. d. They are useful because infants respond poorly to polysaccharide-based antigens. e. They contain only protein subunits.

d. They are useful because infants respond poorly to polysaccharide-based antigens.

__________ are involved in detecting foreign invaders. They do so by binding to pathogen- associated molecular patterns (PAMPs) on the surface of the pathogen. a. Inflammatory molecules b. Granzymes c. Mucous membranes d. Toll-like receptors

d. Toll-like receptors

Which of the following occurs first, setting in motion the remaining events? a. Antimicrobial peptides (AMPs) are produced and damage microbes in a variety of ways. b. Additional dendritic cells are attracted to the infection site by AMPs. c. Adaptive immune responses are initiated. d. Toll-like receptors (TLRs) on macrophages and dendritic cells attach to pathogen-associated microbial patterns (PAMPS) on invading microorganisms. e. The macrophages and dendritic cells release cytokines.

d. Toll-like receptors (TLRs) on macrophages and dendritic cells attach to pathogen-associated microbial patterns (PAMPS) on invading microorganisms.

While waiting for your meal at a restaurant, you read the information on the creamer containers at your table and notice that they do not need to be refrigerated. You are surprised, because dairy products typically must be kept cold to prevent spoilage. How were these products treated to eliminate microbial growth and prevent spoilage? a. Chemical preservatives b. Pasteurization c. Gamma irradiation d. Ultra-high-temperature (UHT) treatment

d. Ultra-high-temperature (UHT) treatment

All members of the seven families for whom Mary Mallon prepared meals developed typhoid fever. Mary did not get sick. This is an example of __________. a. sporadic disease b. an opportunistic infection c. a nosocomial infection d. a carrier

d. a carrier

Which of the following does not initiate DNA synthesis? a. a double-stranded DNA virus (Poxviridae) b. a DNA virus with reverse transcriptase (Hepadnaviridae) c. an RNA virus with reverse transcriptase (Retroviridae) d. a single-stranded RNA virus (Togaviridae) e. none of the above

d. a single-stranded RNA virus (Togaviridae)

A property of T cells, but not B cells, is their __________. a. ability to differentiate into memory cells b. ability to undergo clonal selection c. recognition of specific antigens d. ability to form cells that directly kill virus-infected host cells e. development from stem cells in the bone marrow

d. ability to form cells that directly kill virus-infected host cells

Prions cause disease by __________. a. causing transcription and translation of abnormal proteins b. altering genes c. the activity of a reverse transcriptase d. altering normal proteins

d. altering normal proteins

While on safari in Serengeti National Park, Tanzania, your friend ventures away from camp and is bitten by a boomslang snake. The venom will cause hemorrhaging and death within hours. Fortunately, you are prepared and administer artificially acquired passive immunity. With what did you inject your friend? a. antigen b. penicillin c. snake venom d. antibodies

d. antibodies

All of the following are components of the inflammatory process EXCEPT a. chemotaxis. b. diapedesis. c. release of histamines and prostaglandins. d. antibody synthesis. e. dilation of blood vessels.

d. antibody synthesis.

Prior to drawing blood for a blood donation, the nurse will clean the arm with a Betadine solution. This form of antimicrobial control is called __________. a. disinfection b. sterilization c. sanitization d. antisepsis

d. antisepsis

An organism is found in a deep-sea vent where the temperature is very high and the mineral content very unusual. The organism is unicellular, has no apparent nucleus, and has a cell wall but no peptidoglycan. This organism would be classified as which of the following types of organisms? a. bacteria b. algae c. virus d. archaea

d. archaea

What type of immunity results from vaccination? a. innate immunity b. naturally acquired active immunity c. naturally acquired passive immunity d. artificially acquired active immunity e. artificially acquired passive immunity

d. artificially acquired active immunity

Which type of adaptive immunity does the following statement describe?This type of immunity is acquired when a person is vaccinated for a specific type of infection via the introduction of antigens. These antigens normally have undergone some type of modification and may not confer the same type of long-lasting memory that would occur with unmodified antigens. a. artificially acquired passive immunity b. naturally acquired active immunity c. naturally acquired passive immunity d. artificially acquired active immunity

d. artificially acquired active immunity

Which type of adaptive immunity does the following statement describe?This type of immunity is acquired via injection of antibodies from an individual or host that has immunological memory to the specific pathogen or antigen. a. artificially acquired active immunity b. naturally acquired active immunity c. naturally acquired passive immunity d. artificially acquired passive immunity

d. artificially acquired passive immunity

NK cells do all of the following EXCEPT a. bind to Fc regions of bound antibodies. b. participate in antibody dependent cell cytotoxicity. c. kill cells not expressing MHCI. d. become activated by TH-2 cells. e. comprise 10-15% of circulating lymphocytes.

d. become activated by TH-2 cells.

Functions of the glycocalyx include all of the following EXCEPT a. source of nutrition. b. protection against dehydration. c. biofilm formation. d. binary fission. e. increased virulence.

d. binary fission.

All of the following are effective for destroying prions EXCEPT a. proteases. b. incineration. c. NaOH + autoclaving at 134°C. d. boiling. e. None of the answers are correct; each of these will destroy prions.

d. boiling.

A bacterial medium contains chemicals that inhibit gram-positive bacteria and indicators so that bacteria that ferment lactose produce red colonies, and bacteria that do not ferment lactose produce colorless colonies. Such a medium is called __________. a. selective b. enriched c. differential d. both selective and differential

d. both selective and differential

Which type of light microscopy is used to visualize stained specimens? a. darkfield b. phase-contrast c. compound d. brightfield e. binocular

d. brightfield

Cytokines that induce migration of leukocytes into areas of infection or tissue damage are known as __________. a. interleukins b. interferons c. TNF d. chemokines

d. chemokines

In a complement-fixation test, for a serum sample containing antibody against the target antigen, __________. a. complement will not be fixed, and sheep red blood cells will lyse b. complement will not be fixed, and sheep red blood cells will not agglutinate c. complement will be fixed, and the sheep red blood cells will agglutinate d. complement will be fixed, and sheep red blood cells will not lyse e. complement will be fixed, and sheep red blood cells will lyse

d. complement will be fixed, and sheep red blood cells will not lyse

Three-dimensional images of live cells can be produced with: a. darkfield microscopy b. fluorescence microscopy c. transmission electron microscopy d. confocal microscopy e. phase-contrast microscopy

d. confocal microscopy

Which of the following is NOT a typical characteristic of most bacterial plasma membranes? a. contains proteins b. site of energy production c. is selectively permeable d. contains cholesterol e. composed of a phospholipid bilayer

d. contains cholesterol

Figure 14.1 shows the incidence of influenza during a typical year. Which letter on the graph indicates the endemic level? a. a b. b c. c d. d e. e

d. d

John Snow analyzed information about cholera victims, including where the victims lived and where they got water. This is an example of __________. a. analytical epidemiology b. controlled experimentation c. case reporting d. descriptive epidemiology

d. descriptive epidemiology

Which of the following is an advantage of the standard plate count? a. provides immediate results b. can be performed on very dilute samples, such as lake water c. can be used to count heat-sensitive bacteria d. determines the number of viable cells e. can readily count cells that form aggregates

d. determines the number of viable cells

The __________ is the part of the microscope that controls the amount of light entering the condenser. a. objectives b. ocular lenses c. condenser d. diaphragm

d. diaphragm

Which of these factors is NOT used in classifying viruses? a. Host range b. Morphology c. Genome d. Disease symptoms

d. disease symptoms

What is the best definition of sterilization? a. removal of unwanted microorganisms b. elimination of bacteria c. removal of viruses d. elimination of all microorganisms

d. elimination of all microorganisms

Commercial utilization of microbial products has become increasingly popular due to their environmentally friendly nature. Removal of these products from the environment typically utilizes a) organic solvents b) soap c) alcohol d) enzymes e) organic acids

d. enzymes

The graph in the figure shows the incidence of polio in the United States. The period between 1945 and 1955 indicates a(n) a. endemic level. b. communicable disease. c. sporadic infection. d. epidemic level. e. pandemic.

d. epidemic level.

The science that deals with when diseases occur and how they are transmitted is called a. ecology. b. morbidity and mortality. c. public health. d. epidemiology. e. communicable disease.

d. epidemiology.

Which group of microorganisms is most likely to spoil a freshwater trout preserved with salt? a. thermophiles b. anaerobes c. psychrophiles d. facultative halophiles e. hyperthermophiles

d. facultative halophiles

Each of the following can be used for the detection and/or identification of viruses except _______________. a. polymerase chain reaction b. observation of cytopathic effects c. serologic (immunologic) tests d. fermentation tests

d. fermentation tests

Which one of these substances would NOT be produced at the same time as coagulase? a. capsules b. fimbriae c. hemolysin d. fibrinolysin

d. fibrinolysin

Which of the following types of microscopy allows organisms to be identified based on the presence of antigens, even if the organisms are inside a cell? a. scanning acoustic microscopy b. darkfield microscopy c. phase-contrast microscopy d. fluorescent microscopy

d. fluorescent microscopy

The chemical requirements for microbial growth include carbon, nitrogen, phosphorus, and sulfur. Why do all cells need nitrogen? a. as an energy source b. as part of the phospholipid bilayers that form the cell membranes c. for maintaining a favorable pH d. for building amino acids, proteins, and nucleic acids

d. for building amino acids, proteins, and nucleic acids

Consider organisms that are eukaryotic, are found in both unicellular and multicellular forms, may resemble plants but are not photosynthetic, have chitin in their cell walls, and absorb organic nutrients from decomposing material in the environment. These organisms would be classified as which of the following types of organisms? a. archaea b. bacteria c. protozoa d. fungi

d. fungi

A chemical that reduces spoilage in fruit by destroying fungi but does NOT appear to affect other microbes would be called a __________. a. fungistatic agent b. bactericidal agent c. sterilant d. fungicidal agent

d. fungicidal agent

Archaea differ from bacteria in that archaea ______ A) reproduce by binary fission. B) use organic compounds for food. C) lack nuclei. D) have diverse cell wall compositions. E) are prokaryotic.

d. have diverse cell wall compositions

Which of the following virulence factors is specifically involved in helping an organism to physically spread throughout the body? a. hemolysin b. siderophores c. endotoxins d. hyaluronidase

d. hyaluronidase

Which one of the following virulence factors contributes most to the spread of disease in the body? a. siderophore production b. endotoxin production c. hemolysin d. hyaluronidase

d. hyaluronidase

Lysogeny can result in all of the following EXCEPT a. acquisition of new characteristics by the host cell. b. phage conversion. c. specialized transduction. d. immunity to reinfection by any phage. e. immunity to reinfection by the same phage.

d. immunity to reinfection by any phage.

Biological transmission differs from mechanical transmission in that biological transmission a. works only with noncommunicable diseases. b. involves fomites. c. requires direct contact. d. involves reproduction of a pathogen in an arthropod vector prior to transmission. e. occurs when a pathogen is carried on the feet of an insect.

d. involves reproduction of a pathogen in an arthropod vector prior to transmission.

Each of the following statements concerning the gram-positive cell wall is true EXCEPT a. it is sensitive to penicillin. b. it contains teichoic acids. c. it maintains the shape of the cell. d. it protects the cell in a hypertonic environment. e. it is sensitive to lysozyme.

d. it protects the cell in a hypertonic environment.

Shingles is a medical condition that usually occurs years after chickenpox, even though no illness is present in the intervening period of time. This occurs because human herpes virus-3 (HHV-3) is capable of __________. a. lytic infection b. persistent infection c. lysogenic infection d. latent infection

d. latent infection

The antibiotic polymyxin B combines with phospholipids. This will cause __________. a. stoppage of protein synthesis b. sporulation c. loss of motility d. leakage of intracellular contents e. binary fission

d. leakage of intracellular contents

What phase of the cell cycle is extended in a chemostat? a. death phase b. stationary phase c. lag phase d. log phase e. logarithmic decline phase

d. log phase

Which of the following phases of the bacterial growth curve is matched with the correct definition? a. stationary phase--the phase in which cells are not dividing very rapidly as they acclimatize to a new environment b. lag phase--the phase in which the death of organisms exceeds the production of new cells and the curve declines c. death phase--the phase in which the curve is flat; microbial deaths balance the number of new cells d. log phase--the phase in which organisms are actively dividing and the generation time is constant

d. log phase--the phase in which organisms are actively dividing and the generation time is constant

All of the following increase blood vessel permeability EXCEPT a. kinins. b. histamine. c. leukotrienes. d. lysozymes. e. prostaglandins.

d. lysozymes.

All of the following are methods of avoiding host antibodies EXCEPT a. inducing endocytosis. b. IgA proteases. c. antigenic changes. d. membrane-disrupting toxins. e. invasins.

d. membrane-disrupting toxins.

The ID50 for many pathogens is significantly smaller when testing with gnotobiotic animals compared to animals with normal microbiota. This is likely because of __________. a. commensalism b. complement inactivation c. impaired phagocytosis d. microbial antagonism e. parasitism

d. microbial antagonism

The most frequently used portal of entry for pathogens is the a. mucous membranes of the gastrointestinal tract. b. skin. c. parenteral route. d. mucous membranes of the respiratory tract. e. All of these portals are used equally.

d. mucous membranes of the respiratory tract.

Which type of adaptive immunity does the following statement describe?This type of immunity is acquired via the passing of antibodies from a mother to a child to give immunity during the development of the child's immune system. a. naturally acquired active immunity b. artificially acquired active immunity c. artificially acquired passive immunity d. naturally acquired passive immunity

d. naturally acquired passive immunity

__________ is an example of an acidic dye; __________ is an example of a basic dye. a. safranin; methylene blue b. malachite green; eosin c. acid fuchsin; ink d. nigrosin; crystal violet

d. nigrosin; crystal violet

Which method CANNOT be used to culture viruses in a laboratory? a. live animals b. embryonate eggs c. tissue culture d. nutrient agar culture medium e. primates

d. nutrient agar culture medium

An unknown organism grows at the top of a tube of thioglycolate broth. This organism is best described as a(n) __________ for its oxygen requirements. a. facultative anaerobe b. obligate anaerobe c. microaerophile d. obligate aerobe e. aerotolerant anaerobe

d. obligate aerobe

The first step for directly linking a microbe to a specific disease according to Koch's postulates is to ______ A) compare the blood of a sick animal to blood obtained from a healthy animal. B) isolate microbes from the blood of healthy animals. C) inject a sample of blood or other body fluid from a diseased animal into a healthy animal. D) obtain a sample of blood or other body fluid from a diseased animal. E) culture the blood or other body fluid from a diseased animal using nutrient medium.

d. obtain a sample of blood or other body fluid from a diseased animal.

Recently, silver has been considered in new applications for microbial control, such as impregnating surgical dressings and infusing food containers with silver. These new applications are important because silver is __________. a. useful as a surface-active agent b. effective as an oxidizing agent c. effective in dissolving lipids in the plasma membrane d. oligodynamic and unaffected by antimicrobial resistance

d. oligodynamic and unaffected by antimicrobial resistance

Niacin, when added to a medium, would be considered a(n) __________. a. reducing agent b. electron carrier c. carbon source d. organic growth factor e. enzyme cofactor

d. organic growth factor

Endotoxins are a. associated with gram-positive bacteria. b. molecules that bind nerve cells. c. A-B toxins. d. part of the gram-negative cell wall. e. excreted from the cell.

d. part of the gram-negative cell wall.

The term describing bacteria with flagella distributed over the entire surface of the cell is __________. a. monotrichous b. amphytrichous c. lophotrichous d. peritrichous

d. peritrichous

A eukaryotic cell can ingest a prokaryotic cell by __________. a. pinocytosis b. digestive vacuolization c. active transport d. phagocytosis e. osmosis

d. phagocytosis

As mentioned previously, the second line of defense is activated when the first line of defense fails. In Part A, you identified one of these defenses as inflammation, which occurs when the body responds to a foreign antigen and increases fluid accumulation in a specific area. An added benefit of this excess fluid is that it also brings immune cells to the area to aid clearance of foreign antigens. In Julie's situation, that added fluid would bring immune cells to prevent any microbial infections that could be caused if microbes were introduced when she cut her finger. Which of the following cell types is NOT part of the second line of defense? a. basophils b. mast cells c. eosinophils d. plasma cells

d. plasma cells

The __________ charge of a basic dye adheres to the __________ charge of bacterial cell surfaces. a. negative; acidic b. negative; positive c. positive; acidic d. positive; negative e. positive; basic

d. positive; negative

Ingesting lactic acid bacteria to prevent colonization by intestinal pathogens such as Salmonella enterica during antibiotic therapy is an example of __________. a. opportunism b. vaccination c. parasitism d. probiotics e. chemotherapy

d. probiotics

Koch's postulates are a set of guidelines to follow if you want to __________. a. determine how a disease is transmitted b. cure a disease c. identify the reservoir for a disease d. prove that a specific infectious disease is caused by a specific microorganism

d. prove that a specific infectious disease is caused by a specific microorganism

An organism displays some growth at 4°C and at 25°C. However, it grows best at 20°C. This organism would be classified as a __________. a. thermophile b. halophile c. mesophile d. psychrotroph e. psychrophile

d. psychrotroph

In the Gram stain, if the decolorizing step is deleted, gram-negative cells will appear __________ at the completion of the staining procedure. a. pink b. unstained c. green d. purple e. blue

d. purple

What factors contribute to the rising incidence of antibiotic resistance? a. random mutations in bacterial genomes b. overuse of the specific drugs c. overuse and misuse of specific drugs d. random mutations, overuse and misuse of specific drugs e. misuse of the specific drugs

d. random mutations, overuse and misuse of specific drugs

Viruses that use RNA as a template for transcribing DNA include: a. Picornaviridae b. Togaviridae c. Herpesviridae d. Retroviridae e. Rhabdoviridae

d. retroviridae

The occurrence of streptococcal bronchopneumonia in an individual recovering from influenza is an example of a __________. a. latent infection b. subacute infection c. sporadic infection d. secondary infection e. chronic infection

d. secondary infection

Which of the following definitions is INCORRECT? a. chronic: a disease that develops slowly and lasts for months b. inapparent: infection characteristic of a carrier state c. primary infection: an initial illness d. secondary infection: a long-lasting illness e. acute: a short-lasting primary infection

d. secondary infection: a long-lasting illness

In polio virus replication, the function of the antisense (- strand) RNA is to __________. a. become incorporated into the capsid proteins b. be translated for the production of capsid proteins c. allow a virus to enter the nucleus d. serve as a template for the production of sense (+ strand) RNA e. enhance the pathogenicity of the viral toxins

d. serve as a template for the production of sense (+ strand) RNA

Inflammation is part of the second line of defense that is activated when the first line of defense is breached. Julie's first-line defenses were damaged when her injury occurred. Which of the following represents the first line of defense? a. platelets b. white blood cells c. red blood cells d. skin and mucous membranes

d. skin and mucous membranes

Before bacteria can be identified, mixed cultures must be separated. What technique is used to separate bacteria and get individual colonies? a. differential media b. enriched media c. selective media d. streak plate

d. streak plate

Which of the following is NOT one of Koch's Postulates? a. the same pathogen must be present in every case of the disease. b. the pathogen must be isolated and grown in pure culture from the diseased host. c. the pathogen from pure culture must cause the disease when inoculated into a healthy, susceptible laboratory animal. d. the disease must be transmitted from a diseased animal to a healthy, susceptible animal by direct contact. e. the pathogen must be isolated in pure culture from an experimentally infected lab animal.

d. the disease must be transmitted from a diseased animal to a healthy, susceptible animal by direct contact.

When microbial control methods are used, microbes are destroyed at a constant rate; there is no instantaneous death of all the microbes present. That microbial death rate is influenced by all of the following factors EXCEPT __________. a. the microbial characteristics b. the number of microbes c. the time of exposure d. toxins produced by the microbe

d. toxins produced by the microbe

Packages of milk and coffee creamers may be stored without refrigeration if they have been sterilized by __________. a. autoclaving b. treatment with phenol c. boiling d. ultra-high-temperature (UHT) treatment e. high-temperature short-time (HTST) pasteurization

d. ultra-high-temperature (UHT) treatment

Alfalfa sprouts were responsible for transmitting Salmonella to 32 people in four states. This is an example of __________. a. indirect contact transmission b. contact transmission c. droplet transmission d. vehicle transmission

d. vehicle transmission

Acid-fast mycobacteria differ from non-acid-fast bacteria by the presence of __________. In the decolorizing step of the acid-fast stain, __________ is used. a. peptidoglycan; carbolfuchsin b. capsules; acetone-alcohol c. endospores; methylene blue d. waxy material in their cell walls; acid-alcohol

d. waxy material in their cell walls; acid-alcohol

In Figure 6.3, which tube shows the expected growth pattern for a microaerophile? a b c d e

e

Which of the following pairs is mismatched? a) metachromatic granules -- stored phosphates b) polysaccharide granules -- stored starch c) lipid inclusions -- poly-β-hydroxybutyric acid d) sulfur granules -- energy reserve e) ribosomes -- protein storage

e) ribosomes -- protein storage

Which of the following is not a distinguishing characteristic of prokaryotic cells? a) they usually have a single, circular chromosome b) they lack membrane-enclosed organelles c) they have cell walls containing preptidoglycan d) their DNA is not associated with histones e) they lack a plasma membrane

e) they lack a plasma membrane

Which of the following is false about fimbriae? a) they are composed of protein b) they may be used for attachment c) they are found on gram-negative cells d) they are composed of pilin e) they may be used for motility

e) they may be used for motility

HEPA filters are effective because they are designed to remove all airborne particulates, including microorganisms down to a. 10 nanometers. b. 1 micrometer. c. 0.01 micrometers. d. 10 micrometers. e. 0.3 micrometers.

e. 0.3 micrometers.

Place the following in the most likely order for biosynthesis of a bacteriophage:(1) phage lysozyme; (2) mRNA; (3) DNA; (4) viral proteins; (5) DNA polymerase. a. 5,4,3,2,1 b. 1,2,3,4,5 c. 5,3,4,2,1 d. 3,5,2,4,1 e. 2,5,3,4,1

e. 2,5,3,4,1

Which one of the following temperatures would most likely kill amesophile? a. −50°C b. 0°C c. 9°C d. 37°C e. 60°C

e. 60°C

The DRT for a particular bacterial species at 60°C is 30 minutes. How long would it take at this temperature to remove 99.9% of this bacterial population? a. 10 minutes b. 30 minutes c. 60 minutes d. 120 minutes e. 90 minutes

e. 90 minutes

To what does the term viral species refer? a. A group of viruses sharing the same disease symptoms b. A group of viruses that are reproductively isolated c. Viruses grouped according to their susceptibility to antibiotics d. Viruses grouped according to growth on selective media e. A group of viruses sharing the same genetic information and structure

e. A group of viruses sharing the same genetic information and structure

Which of these events is an example of contact transmission? a. Tuberculosis is spread from one person to another after traveling through the air for several meters. b. Vacationers develop shigellosis, caused by the intestinal pathogen Shigella spp. after drinking contaminated water. c. Several people become infected with Listeria after consuming contaminated ice cream. d. A person develops plague from a flea bite. e. A person drinks from a cup used by another individual.

e. A person drinks from a cup used by another individual.

Which of these eukaryotic molecules/structures can be responsible for movement of bacteria within host cells? a. Flagella b. Cilia c. Invasins d. Pseudopods e. Actin molecules

e. Actin molecules

Which of these substances are most important in the establishment of biofilms? a. Siderophores b. Invasins c. Exotoxins d. Hemolysins e. Adhesins

e. Adhesins

Which of the following can contribute to postoperative infections? a. antibiotic resistance b. using syringes more than once c. errors in aseptic technique d. normal microbiota on the operating room staff e. All of the answers are correct.

e. All of the answers are correct.

Which of the following is a true statement about the major histocompatibility complex (MHC)? a. The MHC is a collection of genes that encode molecules of genetically diverse glycoproteins. b. Class I MHC are found on the plasma membranes of mammalian nucleated cells. c. Class II MHC molecules exist only on the surface of antigen-presenting molecules (APCs). d. Thymic selection will rid the body of T cells that will not recognize MHC molecules of the host. e. All of the answers are correct.

e. All of the answers are correct.

Cocci may be arranged as __________. a. tetrads b. streptococci c. diplococcic d. staphylococci e. All of the listed answers.

e. All of the listed answers.

Which of the following is NOT a predisposing factor of disease? a. Lifestyle b. Occupation c. Gender d. Climate e. All of the listed choices can be predisposing factors of disease.

e. All of the listed choices can be predisposing factors of disease.

Which of the following cell wall components do/does NOT contribute to virulence? a. M protein b. fimbriae c. Opa d. mycolic acids e. All of the listed choices contribute to virulence.

e. All of the listed choices contribute to virulence.

Which of the following diseases does NOT share a commonality with the others? a. Brucella b. rabies c. Hantavirus d. Yersinia pestis e. All of the listed diseases share a commonality; they are all zoonoses.

e. All of the listed diseases share a commonality; they are all zoonoses.

Which of the following does NOT describe the innate immune response? a. Innate immunity includes your first and second lines of defense. b. The innate immune response is always available to provide rapid responses to protect against disease. c. The innate immune response does not confer immunologic memory. d. The innate immune response is nonspecific. e. All of the listed statements describe the innate immune response.

e. All of the listed statements describe the innate immune response.

Which of these statements is NOT true of antibody molecules? a. Antibody molecules can enhance phagocytosis of the antigen. b. Cell-bound antibody molecules can initiate a process that results in cell lysis. c. Cell-bound antibody molecules can bind complement, triggering the complement cascade. d. Cell-bound antibody molecules can bind cells that in turn release chemical compounds that damage parasitic worms. e. Antibody molecules can directly destroy antigens.

e. Antibody molecules can directly destroy antigens.

Which of the following would be a possible consequence of a disorder that selectively destroys the T regulatory cells in a patient? a. Impaired antibody production b. Frequent infections with viral and fungal pathogens c. Increased incidence of certain cancers d. frequent infections with parasitic pathogens e. Autoimmune diseases

e. Autoimmune diseases

Members of the genus Clostridium display the following properties: - Gram-positive bacilli - Endospore formation - Anaerobic growth Which of the following would be appropriate for the culture of members of this genus? a. Blood agar plate in a candle jar b. Blood agar plate in an anaerobe jar c. Nutrient broth in a standard incubator d. Sodium thioglycolate broth e. Both a blood agar plate in an anaerobe jar and sodium thioglycolate broth

e. Both a blood agar plate in an anaerobe jar and sodium thioglycolate broth

The pathogenicity of which of the following is NOT the result of lysogeny? a. Clostridium botulinum b. Corynebacterium diphtheriae c. Vibrio cholerae d. Streptococcus pyogenes e. Clostridium tetani

e. Clostridium tetani

What cell structures does Neisseria gonorrhoeae use to attach and enter host epithelial cells? a. M proteins b. Cell wall waxes c. Flagella d. Capsules e. Fimbriae

e. Fimbriae

Which of the following steps is common to both the Gram stain and the acid-fast stain? a. Steam exposure for stain penetration b. Acid-alcohol for decolorizing c. The application of iodine as a mordant d. The use of methylene blue as a counterstain e. Heat fixation of the smear before staining

e. Heat fixation of the smear before staining

Viruses that utilize reverse transcriptase belong to the virus families a. Rhabdoviridae and Herpesviridae. b. Retroviridae and Picornaviridae. c. Herpesviridae and Retroviridae. d. Herpesviridae and Poxviridae. e. Hepadnaviridae and Retroviridae.

e. Hepadnaviridae and Retroviridae.

Which of the following does not stimulate phagocytes? a. Cytokines. b. IFN-y (gamma) c. C3b d. Lipid A. e. Histamine.

e. Histamine.

Which of these processes is in the proper sequence? a. IgE is formed; antigen binds IgE; IgE binds to mast cells and basophils; histamine is released. b. IgE is formed; antigen binds IgE; histamine is released; IgE binds to mast cells and basophils. c. IgE is formed; IgE binds to mast cells and basophils; histamine is released; antigen binds IgE. d. IgE is formed; antigen binds IgE; IgE binds to mast cells; mast cells bind to basophils; histamine is released. e. IgE is formed; IgE binds to mast cells and basophils; antigen binds IgE; histamine is released.

e. IgE is formed; IgE binds to mast cells and basophils; antigen binds IgE; histamine is released.

Which of these statements is NOT true for bacterial capsules? a. Capsules related to virulence are produced by the causative agents of anthrax and bubonic plague. b. Pathogenic and nonpathogenic bacteria can produce capsules. c. The importance of the capsule to virulence for Streptococcus pneumoniae can be determined because there are strains both with and without the capsule. d. For Streptococcus pneumoniae, the encapsulated strain is more virulent. e. Immune system antibodies are not produced against a capsule.

e. Immune system antibodies are not produced against a capsule.

Which of these disease stages is most likely to be altered in length if the number of infecting organisms at the start of the infection is very high? a. Prodromal period b. Period of illness c. Period of decline d. Period of convalescence e. Incubation period

e. Incubation period

Which of these is NOT a property of the fluorescence-activated cell sorter (FACS)? a. It can detect fluorescence. b. It can separate cells based on charge. c. It can modify cell charge. d. It uses a laser beam. e. It can detect serum antibody levels.

e. It can detect serum antibody levels.

Which of the following statements about sebum is NOT true? a. It has antimicrobial properties. b. Its metabolism can result in acne. c. It is secreted by sebaceous glands. d. Accutane limits acne by preventing its formation. e. It raises the pH of skin.

e. It raises the pH of skin.

Which of these structures are NOT part of the mononuclear phagocytic system? a. Kupffer's cells b. Microglial cells c. Wandering macrophages d. Alveolar macrophages e. Lymphocytes

e. Lymphocytes

Which of the following is NOT an example of microbial antagonism (also known as competitive exclusion)? a. Microbes producing acidic compounds that limit the growth of many bacteria b. Microbes competing with other microbes for nutrients c. Microbes producing compounds inhibitory to other microbes d. Microbes utilizing oxygen necessary for the growth of other microbes e. Microbes producing vitamins and growth factors that can be utilized by the host

e. Microbes producing vitamins and growth factors that can be utilized by the host

Germ-free (gnotobiotic) animals often are more susceptible to infections and serious disease than are animals with a typical complement of normal microbiota. Based on this observation, which of the following would be an appropriate conclusion? a. Normal microbiota provide supplemental nutrition to the host. b. Normal microbiota are parasitic. c. Normal microbiota are incapable of causing disease. d. Normal microbiota always result in opportunistic infections. e. Normal microbiota stimulate the development of the immune system.

e. Normal microbiota stimulate the development of the immune system.

With a light microscope, which of the following would allow discrimination between eukaryotes and prokaryotes? a. Observation of the plasma membrane b. Observation of the ribosomes c. Observation of flagella d. Observation of photosynthetic pigment e. Observation of nuclei

e. Observation of nuclei

Why is penicillin selectively toxic to bacterial cells but harmless to human cells? a. Penicillin is rapidly metabolized and destroyed by human cells. b. The glycocalyx of an animal cell provides a barrier through which penicillin cannot pass. c. Human cells actively transport penicillin out of the cell. d. Penicillin is inactivated within lysosomes of human cells. e. Penicillin specifically weakens peptidoglycan, which is found only in bacterial cells.

e. Penicillin specifically weakens peptidoglycan, which is found only in bacterial cells.

Which of these cell types is NOT involved in cell-mediated immunity? a. Natural killer cells b. Cytotoxic T lymphocytes c. T regulatory cells d. TH1 cells e. Plasma cells

e. Plasma cells

Which disease is correctly matched with the common portal of entry? a. Chlamydia; skin b. Measles; parenteral route c. Influenza; mucous membranes of genitourinary tract d. Hookworm; mucous membranes of genitourinary tract e. Poliomyelitis; mucous membranes of gastrointestinal tract

e. Poliomyelitis; mucous membranes of gastrointestinal tract

Which of the following terms are NOT correctly matched? a. Herpesviridae; infectious mononucleosis b. Picornaviridae; polio c. Picornaviridae; common cold d. Flaviviridae; hepatitis e. Poxviridae; chickenpox

e. Poxviridae; chickenpox

You are preparing a medium for growing fastidious bacteria and must add several heat-labile solutions of growth factors. Which of the following is an appropriate strategy for preparing and sterilizing this medium? a. Use dry heat to sterilize the medium. b. Use UV radiation to sterilize this medium. c. Prepare the medium, add the growth factors, and autoclave. d. Pasteurize the medium. e. Prepare and autoclave the medium before adding the growth factors.

e. Prepare and autoclave the medium before adding the growth factors.

Which of the following infectious agents challenge the current sterilization strategies that have been accepted and in use for decades? a. Endospores b. Mycobacteria c. HIV d. Methicillin-Resistant Staphylococcus aureus (MRSA) e. Prions

e. Prions

Which of the following is LEAST likely to be damaged by exposure to gamma radiation? a. Bacteria b. Viruses c. Fungi d. Protozoa e. Prions

e. Prions

Which of the following pairs is MISMATCHED? a. Endoplasmic reticulum; internal transport b. Golgi complex; secretion c. Mitochondria; ATP production d. Lysosome; digestive enzymes e. Ribosomes; storage

e. Ribosomes; storage

Who is credited with first observing cells? a. Robert Koch b. Carolus Linnaeus c. Louis Pasteur d. Anton van Leeuwenhoek e. Robert Hooke

e. Robert Hooke

How might a virus pick up a human oncogene? a. Transformation b. Reverse transcription c. Biosynthesis d. Viral conversion e. Specialized transduction

e. Specialized transduction

Which of these answers describes the best way to diagnose congenital toxoplasmosis in a newborn infant? a. Test the mother's serum for the presence of IgM antibodies against Toxoplasma. b. Test the infant's serum for the presence of IgG antibodies against Toxoplasma. c. Test the mother's serum for the presence of IgG antibodies against Toxoplasma. d. Test the infant's blood for the presence of Toxoplasma protozoa. e. Test the infant's serum for the presence of IgM antibodies against Toxoplasma.

e. Test the infant's serum for the presence of IgM antibodies against Toxoplasma.

Which answer is NOT true of the inflammatory process? a. Swelling occurs because of vasodilation and increased capillary permeability. b. Edema occurs. c. Kinins cause increased capillary permeability. d. Leukotrienes cause increased capillary permeability. e. The area becomes red because of a decrease in capillary diameter.

e. The area becomes red because of a decrease in capillary diameter.

Emergence of infectious diseases can be attributed to all of the following EXCEPT a. ease of travel. b. antibiotic resistance. c. new strains of previously known agents. d. climatic changes. e. The emergence of infectious diseases can be attributed to all of these.

e. The emergence of infectious diseases can be attributed to all of these.

Which of the following statements is NOT true for both TEM and SEM? a. The illuminating source is an electron beam. b. Both can be used to view specimens smaller than 0.2 micrometers. c. Black-and-white images are produced. d. The microscope is focused using electromagnetic lenses. e. The specimen must be sectioned before viewing.

e. The specimen must be sectioned before viewing

The results of a titer for varicella antibodies show that Sarah is nonimmune to varicella virus. She decides to get the varicella vaccine, but her physician will not vaccinate her at this time because she is currently pregnant. Why should this vaccine NOT be given to pregnant women? a. This vaccine will not produce an immune response in pregnant women. b. This vaccine may cause pregnant women to develop shingles. c. This vaccine may cause chicken pox in pregnant women. d. Vaccinating pregnant women is linked to the development of lung and kidney disorders in children. e. There is a risk of the attenuated vaccine strain crossing the placenta and infecting the fetus.

e. There is a risk of the attenuated vaccine strain crossing the placenta and infecting the fetus.

Which of the following is part of a plan established by the CDC, NIH, and WHO to address and prioritize issues relating to EIDs? a. Detect infectious pathogens, the diseases they cause, and factors that influence their emergence b. Expand basic and applied research on ecological and environmental factors that influence EIDs c. Enhance the communication of public health information and the prompt implementation of prevention strategies regarding EIDs d. Establish plans to monitor and control EIDs worldwide e. These are all part of the plan related to EIDs.

e. These are all part of the plan related to EIDs.

Monoclonal antibodies do NOT have which of the following properties? a. They are produced by a hybrid cell formed by combining a myeloma cell with a normal B cell. b. They can be produced in large quantities. c. They can be combined with toxins and used therapeutically to destroy cancer cells. d. They can be used to suppress T cell activity and prevent transplant rejection. e. They consist of antibodies that react with many different antigens.

e. They consist of antibodies that react with many different antigens.

Which of the following statements is NOT true of antigens? a. They are often surface molecules on microbes. b. They are often proteins or polysaccharides. c. They are typically nonself molecules. d. They can include nonmicrobial molecules, such as pollen, egg white, and blood cell surface molecules. e. They often have a molecular weight of less than 10,000.

e. They often have a molecular weight of less than 10,000.

Which of the following is NOT a characteristic of B cells? a. They are responsible for antibody formation. b. They have antibodies on their surfaces. c. They originate in bone marrow. d. They are responsible for the memory response. e. They recognize antigens associated with MHC I.

e. They recognize antigens associated with MHC I.

The following may be listed on the ingredient list of your favorite snack food. Which of these products is NOT antimicrobial? a. Calcium propionate b. Potassium sorbate c. Sorbic acid d. Sodium benzoate e. Xanthan

e. Xanthan

Microbes are responsible for __________. a. breaking down wastes b. oxygen generation via photosynthesis c. synthesis of vitamins d. synthesis of acetone and alcohol e. all of the above

e. all of the above

The ability of a virus to infect an organism is regulated by a. the host species. b. the type of cells. c. the availability of an attachment site. d. cell factors necessary for viral replication. e. all of the above

e. all of the above

Regarding Louis Pasteur's experiments with the S-neck flask, which of the following statements is TRUE? A) The possibility of contamination was removed. B) All preexisting microorganisms were killed. C) A food source was provided. D) Air exchange was involved. E) All of the answers are correct.

e. all of the answers are correct

Common commercial benefits of microorganisms include synthesis of a) aspirin b) antibiotics c) insulin d) antibiotics and aspirin e) antibiotics and insulin

e. antibiotic and insulin

The classical pathway for complement activation is initiated by a. factors released from phagocytes. b. C5-C9. c. lipid-carbohydrate complexes and C3. d. factors released from damaged tissues. e. antigen-antibody reactions.

e. antigen-antibody reactions.

A new microorganism has been isolated from hot springs in Yellowstone National Park. It consists of single cells, which appear to lack a nucleus. Chemical analysis shows the presence of both DNA and RNA in the cytoplasm and pseudomurein in the cell wall. In which of the following groups will this organism be classified? a. fungi b. bacteria c. protists d. plants e. archaea

e. archaea

The presence of bacteria in the bloodstream is referred to as __________. a. cytokine storm b. symbiosis c. edema d. bacterial dimorphism e. bacteremia

e. bacteremia

Antibiotics are produced by ______ A) bacteria. B) protozoa. C) fungi. D) viruses. E) bacteria and fungi.

e. bacteria and fungi

All of the following are examples of microbial control using heavy metals EXCEPT __________. a. burn treatment using silver-sulfadiazine ointment b. algae control in swimming pools using copper sulfate c. mouthwash solution containing zinc chloride d. antiseptic solution containing mercurochrome e. benzoyl peroxide used for acne treatment

e. benzoyl peroxide used for acne treatment

Through metabolism, pathogens often produce acids that interfere with their own growth. __________ are/is added to media to control pH changes. a. pH indicators b. growth inhibitors c. saline d. vinegar e. buffers

e. buffers

Activation of the complement cascade __________. a. typically reduces the ability of phagocytes to engulf microbes b. reduces swelling in affected tissues c. prevents cleavage of complement proteins, such as C3 and C5 d. can reduce inflammation e. can cause the infecting microbe to be killed by lysis

e. can cause the infecting microbe to be killed by lysis

The toxin neutralization assay __________. a. is designed so that, in the presence of toxin and corresponding antibodies, toxin molecules bind to target cells and then to antibodies directed against that toxin b. is designed to show the presence of a particular toxin when the cells used in the assay are damaged by the toxin c. requires antibody molecules to be labeled with a fluorescent dye d. is designed to neutralize toxins by cross-linking the antibody molecules directed against them e. can detect the presence of antitoxin

e. can detect the presence of antitoxin

Which of the following statements is NOT a possible outcome of antigen-antibody reaction? a. activation of complement b. agglutination c. opsonization d. ADCC e. clonal deletion

e. clonal deletion

A patient's serum is tested using the complement fixation test. If hemolysis occurs, __________. a. it can be concluded that the patient had antibody to sheep red blood cells b. it can be concluded that the patient had antibody to the test antigen c. complement bound to neither the antibody directed against the target antigen nor to the antibody directed against sheep red blood cells d. the patient suffers from a disease in which complement is lacking or in low concentration e. complement was not fixed by antibodies in the patient's serum and was available to the antibodies, thus inducing lysis of sheep red blood cells

e. complement was not fixed by antibodies in the patient's serum and was available to the antibodies, thus inducing lysis of sheep red blood cells

A culture medium consisting of agar, peptone, and beef-heart extract is a(n) __________. a. differential medium b. chemically defined medium c. selective medium d. enrichment medium e. complex medium

e. complex medium

The nucleoid (nuclear area) of the bacterial cell __________. a. is enclosed in a nuclear membrane b. has abundant histone proteins c. contains many linear chromosomes d. is connected to the endoplasmic reticulum e. contains the bacterial chromosome

e. contains the bacterial chromosome

The epidermis __________. a. is below the dermis b. is composed of loosely packed cells c. serves as one of the more common portals of entry for pathogens d. is composed largely of epidermal cells, all of which are alive e. contains the protein keratin

e. contains the protein keratin

Saxitoxin is produced by __________. a. protozoa b. bacteria c. helminths d. fungi e. dinoflagellates

e. dinoflagellates

All of the following methods are used for food preservation EXCEPT __________. a. deep freezing b. dessication c. osmotic pressure d. commercial canning e. direct flaming

e. direct flaming

Which microscope achieves the highest magnification and greatest resolution? a. darkfield microscope b. phase-contrast microscope c. compound light microscope d. fluorescence microscope e. electron microscope

e. electron microscope

In the figure, the arrow at time (d) indicates a. the secondary response. b. the T-cell response. c. the time of exposure to the same antigen as at time (a). d. the primary response. e. exposure to a new antigen.

e. exposure to a new antigen.

Which of the following pairs is mismatched? a. darkfield microscope - uses visible light b. scanning electron microscope - produces a three-dimensional image c. confocal microscope - produces a three-dimensional image d. scanning tunneling microscope - allows visualization of atoms e. fluorescence microscope - uses a fluorescent light

e. fluorescence microscope - uses a fluorescent light

Viral vaccines have been produced using viruses grown by all of these methods EXCEPT being __________. a. grown in bird embryos b. grown in animal cell culture c. isolated directly from humans d. grown in animals e. grown on agar plates

e. grown on agar plates

Your lab partner tells you the bacteria are moving in his Gram-stained smear. You can conclude that __________. a. the bacteria are gram-negative b. the bacteria have capsules c. he did a flagella stain, not a Gram stain d. the bacteria are acid-fast e. he didn't properly fix the smear

e. he didn't properly fix the smear

An infection transmitted by a handkerchief or tissue is transmitted by __________. a. direct contact b. a vector c. droplet transmission d. common vehicle transmission e. indirect contact

e. indirect contact

Hemagglutination in an immunoassay __________. a. involves blood clotting b. requires complement to be fixed c. is not truly agglutination d. must indicate the presence of antibody e. involves antibodies cross-linking erythrocytes

e. involves antibodies cross-linking erythrocytes

All of the following are found in the cell walls of gram-positive bacteria EXCEPT __________. a. peptidoglycan b. teichoic acid c. N-acetylglucosamine d. lipoteichoic acid e. lipid A

e. lipid A

Which of the following is NOT a step in binary fission? a. invagination of the plasma membrane b. replication of chromosomal DNA c. cell elongation d. cross-wall formation e. lysis of the existing cell wall

e. lysis of the existing cell wall

A feature that may be found in viruses but never in bacteria is a. an ability to infect more than one type of host. b. a sensitivity to antibiotics. c. they cannot reproduce themselves outside a host. d. the ability to pass through 0.22 micrometer pore filters. e. may contain an RNA genome.

e. may contain an RNA genome.

Bacteria growing in and on the human body, including normal microbiota as well as pathogens, are classified as __________. a. thermophilic and acidophilic b. mesophilic and halophilic c. thermophilic and halophilic d. mesophilic and acidophilic e. mesophilic and heterotrophic

e. mesophilic and heterotrophic

Mycology is the study of ______ A) molds. B) mycoplasma. C) mushrooms. D) protozoa. E) molds, yeast, and mushrooms.

e. molds, yeast, and mushrooms

Phototaxis refers to the ability of microorganisms to __________. a. move toward or away from chemical stimuli b. move in a wavelike motion c. attach to solid surfaces d. survive under adverse conditions e. move toward a source of light.

e. move toward a source of light.

Each of the following organisms would be considered a microbe EXCEPT: a. Bacterium b. Virus c. Yeast d. Protozoan e. Mushroom

e. mushroom

Which of the following pairs is mismatched? a. capsule - negative stain b. cell arrangement - simple stain c. cell size - negative stain d. Gram stain - bacterial identification e. none of the above

e. none of the above

Which of the following pairs is mismatched? a. influenza - droplet infection b. syphilis - direct contact c. malaria - vector d. salmonellosis - vehicle transmission e. none of the pairs is mismatched

e. none of the pairs is mismatched

The morphological types of viruses illustrated in the figure are ultimately determined by the a. envelope. b. membrane spikes. c. capsomeres. d. viroids. e. nucleic acid.

e. nucleic acid

During the bacteriophage lysogenic cycle, __________. a. no attachment occurs b. the host cell lyses, releasing new virions c. the burst time is shortened d. new phage DNA is synthesized e. phage DNA is inserted into the host chromosome

e. phage DNA is inserted into the host chromosome

A virus may contain any of any of the following EXCEPT a a. capsid proteins b. ssRNA c. lipid envelope d. spike proteins e. ribosomes

e. ribosomes

Which would be the most UNLIKELY location to find adhesin molecules on a newly discovered bacterium? a. Glycocalyx b. Fimbriae c. Cell wall d. Capsule e. Ribosomes

e. ribosomes

An example of a latent virus infection is __________. a. measles b. polio c. influenza d. smallpox e. shingles

e. shingles

For both the direct and the indirect ELISA, __________. a. agglutination of antigen is observed b. the purpose is to detect antigen c. the purpose is to detect antibody d. hemagglutination is the desired outcome e. substrate is added as a final step

e. substrate is added as a final step

Bone and tendons for transplant are decontaminated by a. glutaraldehyde. b. ethylene oxide. c. peroxygens. d. plasma sterilization. e. supercritical fluids.

e. supercritical fluids.

Recombinant DNA technology has become an increasingly important part of our life. It is used for all of the following EXCEPT ______ A) vaccine production. B) increasing the nutritional value of food. C) drug production. D) enhancing food longevity. E) synthesis of water.

e. synthesis of water

Assume you are growing a bacterial culture in a glucose medium. Your lab partner adds more glucose in hopes of speeding up the experiment. The bacteria suddenly die. The best explanation for this result is that __________. a. glucose is toxic to bacteria b. the bacteria could not use glucose c. osmotic lysis occurred d. the cells underwent sporulation e. the cells plasmolyzed

e. the cells plasmolyzed

You discover a compound that you suspect reduces spoilage in fruit. You treat apples with your compound and inoculate them with Botrytis, a common spoilage fungus. After 7 days, the dry weight of fungi is 5 mg on the treated apples and 10 mg on the untreated apples. You conclude from these results that __________. a. apple spoilage is caused by fungi b. the compound is toxic to humans c. the compound kills fungi d. the fungus probably will have no effect on the apples e. the compound is fungistatic

e. the compound is fungistatic

The ID50 is a. the dose that will kill 50 percent of the test population. b. the dose that will cause an infection in some of the test population. c. the dose that will kill some of the test population. d. a measure of pathogenicity. e. the dose that will cause an infection in 50 percent of the test population.

e. the dose that will cause an infection in 50 percent of the test population.

Complement can be activated by all of the following EXCEPT __________. a. antigen-antibody binding b. contact with a pathogen c. opsonization d. mannose-binding lectins e. the presence of host tissue

e. the presence of host tissue

The specificity of an antibody is due to a. the L chains. b. the H chains. c. the constant portions of the H and L chains. d. its valence. e. the variable portions of the H and L chains.

e. the variable portions of the H and L chains.

HIV selectively destroys CD4 cells and as a result, a person with AIDS is susceptible to life-threatening viral infections. Knowing this, you can conclude that __________. a. these viruses have T-independent antigens b. regulatory T cells are killed by the virus c. B cells do not recognize viral antigens d. CD4 T cells need help from B cells to recognize viral antigens e. these viruses have T-dependent antigens

e. these viruses have T-dependent antigens

Subunit vaccines tend to be safer than whole-cell vaccines because __________. a. they contain intact but dead microbes b. they contain avirulent strains c. they are antigenic d. they produce a stronger immune response e. they cannot reproduce in the host

e. they cannot reproduce in the host

Symptoms of protozoan and helminthic diseases are due to a. tissue damage due to growth of the parasite on the tissues and waste products excreted by the parasite. b. products released from damaged tissues. c. waste products excreted by the parasite. d. tissue damage due to growth of the parasite on the tissues. e. tissue damage due to growth of the parasite on the tissues, waste products excreted by the parasite, and products released from damaged tissues.

e. tissue damage due to growth of the parasite on the tissues, waste products excreted by the parasite, and products released from damaged tissues.

Which of the following kinds of microscopy would be most appropriate for viewing the presence and distribution of mitochondria in a protozoan cell? a. darkfield microscopy b. phase-contrast microscopy c. brightfield microscopy d. scanning electron microscopy e. transmission electron microscopy

e. transmission electron microscopy

The function of the "ciliary escalator" is to a. remove microorganisms from the lower respiratory tract. b. propel inhaled dust and microorganisms away from the mouth, toward the lower respiratory tract. c. remove microorganisms from the gastrointestinal tract. d. trap microorganisms in mucus in the upper respiratory tract. e. trap inhaled dust and microorganisms in mucus and propel it away from the lower respiratory tract.

e. trap inhaled dust and microorganisms in mucus and propel it away from the lower respiratory tract.

Which of the following is NOT a method used for the direct measurement of microbial growth? a. filtration b. serial dilutions c. the most probable number (MPN) method d. spread plates e. turbidity

e. turbidity

Resolution is very good when using an electron microscope because the __________ of the electron beam is much shorter than that of visible light. a. frequency b. penetration c. absorption d. reflection e. wavelength

e. wavelength

- __________: the study of where and when diseases occur and how they are transmitted in populations. - Epidemiologists: * determine _______ of a disease. * identify other important factors concerning the _______ of disease. * develop methods for _______ a disease. * assemble data and graphs to outline ________ of disease.

epidemiology * etiology * spread * controlling * incidence

Drag the images into the corresponding boxes to indicate the correct order of events illustrating the mechanism of antibody-mediated immunity.

extracellular antigen -> B-cell -> plasma cell -> antibodies

A researcher only needs to select a cohort group when implementing an analytical epidemiological study. true or false

false

A viroid is a completely developed infectious agent composed of nucleic acid and surrounded by a capsid. true or false

false

Acid-fast bacteria demonstrate unique staining properties because of a special protein layer found in their cell walls. true or false

false

Aerotolerant organisms can grow in the presence of oxygen because their cell wall protects them from toxic products of oxygen. true or false

false

An example of a TLR would be peptidoglycan found in the cell wall of gram-positive bacteria. true or false

false

An injection of "naked" DNA into muscle cells to induce an immune response against the proteins encoded by the DNA is an example of a subunit vaccine. true or false

false

Any process that destroys the non-spore forming contaminants on inanimate objects is sterilization. true or false

false

Bacterial motility and flagellar movement can readily be viewed using transmission electron microscopy. true or false

false

Binomial nomenclature is used to name viruses. true or false

false

Bovine spongiform encephalitis (BSE, or "mad cow disease") is caused by a virus: - true or false

false

Cells placed in a hypotonic solution tend to lose water due to osmotic pressure. true of false

false

Desiccation is a reliable form of sterilization. true or false

false

Digestion of microorganisms occurs in phagosomes. true or false

false

Diseases that are referred to as EIDs have only been discovered in the past fifty years. true of false

false

Endospores are a reproductive structure. true or false

false

For a single antibody molecule, the two light chains have an identical amino acid sequence in the constant regions but different amino acid sequences in the variable regions. true of false

false

For the complement cascade, only the classical pathway can result in cytolysis. true or false

false

Helical and icosahedral are terms used to describe the shapes of a virus envelope. true or false

false

Herd immunity is a term primarily used to describe the protection of cattle from tuberculosis. true or false

false

If acid-fast bacteria are stained with the Gram stain, they will stain gram-negative. true or false

false

If bacteria A and B have LD50 values of 109 and 1010, respectively, more cells of A than B will be needed to kill the host. true or false

false

If bacteria attach to host tissue, they will cause disease. true or false

false

If phagocytic cells are found in tissues, they will be macrophages. true or false

false

In A-B exotoxins, the A component binds to the host cell receptor so that the B component can enter the cell. true or false

false

In a direct microscopic count, dead cells are easily differentiated from live cells. true or false

false

In a vaccine preparation, the term "attenuated" means that the agent does NOT replicate. true or false

false

In general, the LD50 for exotoxins is much greater than the LD50 for endotoxins. true or false

false

Infections that alter the differential white blood cell count always do so by increasing the number of leukocytes. true or false

false

Iodine is one of the LEAST effective antimicrobial chemicals, effective only against certain very sensitive strains of bacteria. true or false

false

Lifelong immunity without the need for booster immunizations can often be achieved with toxoid vaccines. true or false

false

Live cultures of bacteria that can be ingested and are intended to produce a beneficial effect in humans are known as opportunists. true or false

false

Magnification is the quality of the microscope that allows one to distinguish between two points that are very close together. true or false

false

Many enzymes in both prokaryotic and eukaryotic cells are compartmentalized within organelles. true or false

false

Memory cells do not require B cell receptors. true or false

false

Microorganisms placed in high concentrations of salts and sugars undergo lysis. true or false

false

Most pathogenic bacteria are thermophiles. true or false

false

Only dendritic cells produce interleukins. true or false

false

Penicillin is more effective against gram-negative bacteria than against gram-positive bacteria because it specifically interferes with the synthesis of lipopolysaccharides. true or false

false

Plasma cells will eventually differentiate into memory cells. true or false

false

Pure cultures can easily be obtained on streak plates, even if the desired bacteria are present in very low concentrations in the initial culture broth. true of false

false

Reservoirs of infections are always animate objects. true or false

false

Soap is ineffective in killing bacteria. Therefore, handwashing is a rather ineffective measure in preventing disease transmission. true or false

false

Some viruses can contain both DNA and RNA: - true or false

false

The H5N1 avian influenza (bird flu) virus is readily spread from human to human. true or false

false

The capsules and flagella of bacteria can be observed in gram-stained smears. true or false

false

The dermis is composed of connective tissue and serves the primary purpose of providing direct protection from the external environment. true or false

false

The greater resolution of the electron microscope compared to the compound microscope is due to the longer wavelengths of the electrons used to examine specimens. true or false

false

The inflammatory response can only be triggered by an infection. true or false

false

The pH of the medium has no effect on the activity of the disinfectant being applied. true or false

false

The process of eliminating potentially self-reactive T cells in the thymus is called clonal selection. true or false

false

The process of pasteurization to reduce food spoilage utilizes high heat to kill all bacteria present: - true or false

false

The production of interferons at an infection site is critical for chemotaxis. true or false

false

The variable region of the antibody is solely responsible the significant diversity of antigen targets. true or false

false

The vast majority of bacteria are harmful to humans or plants: - true or false

false

Transmission of Plasmodium spp. (malaria) through a mosquito bite is an example of vehicle transmission. true or false

false

Viruses are the only known infectious agents that are obligatory intracellular parasites. true or false

false

Water can serve as a human reservoir of Vibrio cholerae. true or false

false

Each of the three graphs shown below includes data collected during exponential growth of a species of bacteria grown in three different growth conditions. Which growth condition resulted in the longest generation time?

growth condition C

The Lysogenic Cycle: Bacteriophage _______: - Lysogeny: the phage remains _______. - Phage DNA _________ into host cell DNA. * inserted phage DNA is known as a _________. * when the host cell replicates its chromosome, it also replicated prophage DNA. * results in ____________ - the host cell exhibits new properties. - Difference between bacteriophage and animal virus: * bacteriophage: _______ never enters the cell/ animal virus: entire _____ enters the cell through endocytosis. - The lysogenic cycle is often results in ______ infections because the host cells keep multiplying and not rupturing.

lambda - latent - incorporates * prophage * phage conversion * capsid x2 - chronic

The flu virus mutates fairly frequently. Its adhesive proteins change such that we have different "strains" of influenza each year. When a particular flu virus mutates such that its adhesive proteins change, which function of antibodies is disrupted?

neutralization

During the lysogenic cycle of the bacteriophage lambda, integrated phage DNA is known as the ________.

prophage

Immunological Memory: - _________ or __________ occurs after the second exposure to an antigen. * more ______, lasts many days, and greater in magnitude. * ______ cells produced in response to the initial exposure are activated by the secondary response. - _________: is the relative amount of antibody in the serum. * reflects _______ of the humoral response. * _____ is produced first, followed later by ______.

secondary memory or anamnestic response * rapid * memory - antibody titer * intensity * IgM/ IgG

The release of endotoxins as bacteria are destroyed by phagocytes causes the phagocytes to release tumor necrosis factor (TNF). The life-threatening loss of blood pressure due to the action of TNF is called ________

septic shock

A highly specific diagnostic test will be unlikely to indicate a positive result if a specimen being tested is a true negative. true or false

true

A host is not considered diseased until an infection changes one's state of health. true or false

true

A nosocomial infection is one that develops as a result of a stay in a health care facility. true of false

true

A segmented genome can result in antigenic shift. true or false

true

A student has obtained a sample of pond water for study. Using the high-power lens, he observes several cells with nuclei. He can conclude that the cells are NOT bacteria: - true or false

true

Accumulations of viral proteins and nucleic acids inside infected host cells are called inclusion bodies. true or false

true

Adding salts to a solution increases osmotic pressure and is used to preserve food. true or false

true

Adjuvants such as aluminum salts are used as additives in vaccines to enhance immune responses. true or false

true

Agar is used as a solidifying agent in microbiological media since few bacteria can degrade it. true or false

true

Agglutination tests use particulate antigens while precipitation tests use soluble antigens. true or false

true

An isolated colony on a streak plate contains millions (or even billions) of identical cells all arising from one initial cell. true or false

true

An unknown organism grows at the top of a tube of thioglycolate broth. This organism is best described as an obligate aerobe for its oxygen requirements. true or false

true

Anton van Leeuwenhoek was the first to observe microbes: - true or false

true

Aspirin alleviates pain, inflammation, and fever by inhibiting prostaglandins. true or false

true

Autoclaving is the most effective method of moist heat sterilization. true or false

true

Bacterial growth refers to an increase in the numbers of cells in a bacterial culture. true or false

true

Bacterial smears should be fixed to the slide before all staining procedures. true or false

true

Biofilms provide pathogens with an adhesion mechanism and aid in resistance to antimicrobial agents. true or false

true

Blood typing tests are examples of hemagglutination reactions. true or false

true

Both normal and transient flora can become opportunistic pathogens true or false

true

Cells viewed in darkfield microscopy appear similar to those stained with the negative stain. true or false

true

Cytopathic effects, such as inclusion bodies and syncytium formation, are the visible signs of viral infections. true or false

true

Dogs do not get measles because their cells lack the correct receptor sites for that virus. true or false

true

During lysogeny, the phage remains latent. true or false

true

Ergot and aflatoxin are toxins sometimes found in grains contaminated with fungi. true or false

true

Ethylene oxide gas is used to sterilize medical equipment that might be damaged by exposure to the heat of autoclaving. true or false

true

Filtration methods, followed by growth of the bacteria trapped on the filters in growth media, are used to count bacteria present in very low concentrations, such as in lakes and streams. true or false

true

Flagella are appendages that allow locomotion in bacteria: - true and false

true

Flagella are too small to be seen with the light microscope when stained by the Gram stain protocol. true or false

true

For a particular disease at a specific time period, morbidity rates should always be equal or greater than mortality rates. true or false

true

Helper T cells are involved in both the humoral and the cellular immune responses. true or false

true

If you observe rod-shaped red cells after the Gram stain, you can assume their cell walls contain endotoxin. true or false

true

Immersion oil acts to decrease refraction of light rays and thus increase resolution. true or false

true

In a completed Gram stain, gram-positive bacteria are purple. true or false

true

In biological transmission, pathogens reproduce in the vector. true of false

true

Infections with some viruses may induce chromosomal changes that alter the growth properties of host cells. true or false

true

Laboratory cultivation of obligate anaerobes requires reducing media or special growth chambers filled with inert gases. true or false

true

Many pathogens use the same portal for entry and exit from the body. true or false

true

Maturation of stem cells into mature blood cells occurs in the red bone marrow. true or false

true

Microbes are associated with life-sustaining benefits as well as life-threatening diseases: - true or false

true

Microorganisms that are members of the normal microbiota are also known to cause disease. true or false

true

Moist heat destroys organisms by denaturing proteins. true or false

true

Most drugs that interfere with viral multiplication also interfere with host cell function. true or false

true

Positive sense RNA strands of viruses are treated like mRNA inside the host cell. true or false

true

Pseudomonas has been found growing in quaternary ammonium compounds (quats). true or false

true

Psychrotrophic organisms are often responsible for food spoilage in refrigerated foods. true or false

true

Small, hydrophobic molecules pass through a cell membrane most easily by diffusion. true of false

true

Some microorganisms escape from the phagosome before it fuses with the lysosome and are able to survive and grow within phagocytes. true or false

true

Spheroplasts, protoplasts, and mycoplasms are bacterial cells without cell walls. true or false

true

T cells react to antigens on the surface of APCs only when those antigens are associated with proteins of the major histocompatibility complex. true or false

true

Testing the effectiveness of a new drug for anthrax would be best performed as an experimental study. true or false

true

Tetanus, which humans acquire from wounds that become infected with the soil microbe Clostridium tetani, is considered noncommunicable. true or false

true

The Gram stain is important in medical microbiology because the results help physicians select appropriate antibiotics for treatment. true or false

true

The M protein enhances the virulence of Streptococcus by preventing phagocytosis. true or false

true

The cell membrane is a fluid structure that allows membrane proteins to move freely. true or false

true

The cell walls of bacteria are responsible for the shape of the bacteria and the difference in the Gram stain reaction. true of false

true

The counterstain used in the Gram stain is a basic dye. true or false

true

The home pregnancy test kit is an example of a direct ELISA. true or false

true

The implementation of vaccinations occurred prior to experimental support for the germ theory of disease. true or false

true

The internal structure of eukaryotic cilia and flagella are the same. true or false

true

The limit of resolution of the compound microscope illuminated with visible light is approximately 0.2 μm. true or false

true

The presence of organic matter, such as blood or saliva, may impair the action of antimicrobial chemicals. true or false

true

The simple presence of antibody against a particular pathogen is insufficient for disease diagnosis. true or false

true

The thermal death time is the time needed to kill all the bacteria in a particular culture at a certain temperature. true or false

true

Turbidity is an indirect measurement of bacterial growth that can be measured using a spectrophotometer. true or false

true

Viruses that infect bacteria can be used to treat bacterial infections in humans. true or false

true

Western blotting uses antibodies to detect specific proteins in a mixture of proteins. true or false

true

Endotoxins are also known as a) cytokines. b) prostaglandins. c) interleukin-1. d) Lipid A.

d) Lipid A.

Plasma cells are activated by a(n) A) antigen. B) T cell. C) B cell. D) memory cell. E) APC.

A) antigen.

All of the following contribute to a pathogen's invasiveness EXCEPT A) toxins. B) capsules. C) cell wall components. D) hyaluronidase. E) coagulases

A) toxins.

What type of vaccine os Streptococcus pygenes capsule? A) Conjugated vaccine B) Subunit vaccine C) Nucleic acid vaccine D) Attenuated whole-agent vaccine E) Toxoid vaccine

B) Subunit vaccine

How does specialized transduction differ from regular lysogeny? A) The resulting bacteriophage from specialized transduction does not contain any viral DNA. B) The prophage in specialized transduction carries with it pieces of the host chromosomal DNA. C) The resulting bacteriophage from specialized transduction cannot infect a new host cell. D) The prophage is not excised during specialized transduction.

B) The prophage in specialized transduction carries with it pieces of the host chromosomal DNA.

__________ is the quality of the microscope that allows one to distinguish between two points that are very close together. a. refractive index b. magnification c. resolution d. oil immersion

c. resolution

Salt agar tends to inhibit the growth of most organisms, except Staphylococci. This is an example of a(n) __________ medium. a. enriched b. all-purpose c. selective d. differential

c. selective

Consider a gram-positive cell in a hypertonic medium. If the peptidoglycan were damaged, the cell would __________. a. swell b. remain the same size c. shrink and then swell d. shrink

d. shrink

The term trace elements refers to a. the elements CHONPS. b. vitamins. c. nitrogen, phosphorus, and sulfur. d. small mineral requirements. e. toxic substances.

d. small mineral requirements.

Urinary tract infections are the most common forms of nosocomial infections. true or false

true

Useful vaccines for fungal, protozoan, or helminthic diseases are not widely available for use in humans. true or false

true

Vaccines are preparations of organisms or fractions of organisms that are used to induce protective immune responses. true or false

true

Vaccines were invented by Alexander Fleming in 1920: - true or false

true

Virus spikes are used for attachment to the host cell. true or false

true

If you begin with 6 bacterial cells how many cells will you have after 3 rounds of division?

48 cells

A reaction between antibody and particulate antigen is called a(n) A) agglutination reaction. B) complement fixation. C) immunofluorescence. D) neutralization reaction. E) precipitation reaction.

A) agglutination reaction.

In the figure, which areas are different for all IgM antibodies? a. a and b b. b and c c. a and c d. c and d

a. a and b

Some antimicrobial chemicals are considered to be disinfectants and antiseptics. true or false

true

Clonal Selection of Antibody-Producing Cells: - Clonal selection differentiates activated B cells into: * _________ & * ________ (remember antigen = secondary response). - __________ eliminates harmful B cells. - B cells combat _________ pathogens.

* antibody-producing plasma cells * memory cells - clonal deletion - extracellular

Most bacteria grow best at pH A) 1. B) 5. C) 7. D) 9. E) 14.

C) 7.

CHAPTER 1: THE MICROBIAL WORLD AND YOU

BEGINS HERE

- Scientific names: * are ______ or _________ * the ______ is capitalized and the ________ is lowercase. * are ________ and used worldwide * may be ________ or honor a scientist - Escherichia coli: * honors the discoverer, ___________ * describes the bacterium's habitat - ____________ - Staphylococcus aureus * describes the ________ (staphylo-) __________ (coccus) cells * describes the _________ (aureus) colonies * this is normal biota of the ___________. - What is the study of shapes?

* italicized or underlined * genus/ specific epithet * latinized * descriptive * Theodor Escherich * large intestine or colon * clustered/ spherical * gold-colored * respiratory system - morphology

Three Domains: - Bacteria: * no ________ or ________. * _________ cell walls * 1 _____ polymer * F-methionine * _________ rare * no __________ * ________ chromosomes - Archaea: * no ______ or _______ * no peptidoglycan/ several polymers/ methionine/ introns present * has ________ * _______ chromosomes - Eukarya: * has a ______ and _______ * no peptidoglycan/ several polymers/ methionine/ introns are common. * has ________ * ______ chromosomes

* nucleus or organelles * peptidoglycan * RNA * introns * histamines * circular * nucleus or organelles * histamines * circular * nucleus or organelles * histamines * linear

- E. coli O157:H7: * _________ producing strain of E. coli. * first seen in 1982; causes ________ diarrhea (leading cause of diarrhea worldwide) ... death from dehydration. - Middle East Respiratory Syndrome (MERS): * caused by the Middle East respiratory syndrome ________ (MERS-CoV). * common to SARS: causes severe _____________. * 100 deaths in the Middle East from 2012 to 2014. - Avian Influenza A (H5N1) * Influenza A virus * primarily in _______ and _______ * sustained human-to-human transmission has not yet occurred. - Methicillin-Resistant Staphylococcus aureus (MRSA): * Is a type of ________. * 1950s: ________ resistance developed * 1980s: ________ resistance * 1990s: MRSA resistance to ________ reported (VISA = vancomycin-intermediate S. aureus/ VRSA = vancomycin-resistant S. aureus) - West Nile Encephalitis (WNE): * caused by the _______. * first diagnosed in the West Nile region of Uganda in 1937 * appeared in New York City in 1999 * transmitted between birds and to horses and humans by _______. - Bovine Spongiform Encephalopathy (________): * caused by a ________ (an infectious ________ that also causes _________) * new variant of CJD in humans is related to cattle that have been given feed made from prion-infected ________. - Ebola Hemorrhagic Fever (EHF): * Ebola ______ * causes fever, hemorrhaging and _________ * transmitted via contact with infected _____ or _______. * first identified near Ebola River, Congo * 2014 outbreak in Guinea, hundreds killed - Crytosporidiosis: * Crytosporidium ____________ * first reported in 1976 * causes 30% of ________ illness in developing countries. * in the United States, transmitted via _______. - AIDS (Acquired Immunodeficiency Syndrome): * caused by ___________. * first identified in 1981 * _______ affecting males and females * worldwide epidemic infecting 35 million people; 6000 new infections every day * HIV/AIDS in the United States: ____% are female, and ____% are African American

* shiga toxin * bloody * coronavirus * acute respiratory syndrome * waterfowl and poultry * bacteria (big deal in hospital settings) * penicillin * methicillin * vancomycin (last resort because of bad side effects = messes with liver) * West Nile virus * mosquitoes - mad cow * prion/ (mis-folded protein/ Creutzfeldt-Jakob disease (CJD) * sheep * virus * blood clotting * blood or body fluids * protozoa * diarrheal * water * human immunodeficiency virus (HIV) * STI * 26/ 49

The Microbiome: - The Human Microbiome Project (begun in 2007) * goal of determining the makeup of __________ of various areas of the body. * secondary goal of understanding relationship between changes in _______ and __________. - The National Microbiome Initiative (NMI, begun 2016) * explores the role microbes play in different _________.

* typical microbiota * microbiome and human diseases * ecosystems

A Fortunate Accident: Antibiotics: - 1928: ________ discovered the first antibiotic (by accident). - He observed that Penicillium fungus made an antibiotic, __________, that killed __________. * Explain what the fungus/antibiotic does to bacteria! - 1940s: this antibiotic was tested clinically and mass-produced. - Before antibiotics, what did doctors use to treat infections?

- Alexander Fleming - penicillin/ S. aureus * attacks/inhibits the cell wall of bacteria and has no hinder on our own cells (b/c we don't have cell walls). Fungus evolved as a way of survival. - bacteriophages (type of virus) = attack only the bacteria and leave our cells alone

Classification of Microorganisms: - Developed by __________ in 1977. - Three domains based on ___________. - What are the three domains? - What are the four subcategories (kingdoms) of eukarya? - Explain endosymbiosis! * mitochondria are related to __________. * chloroplasts are related to _________. **** MITOCHONDRIA CAN NOT GROW ON ITS OWN IF PULLED OUT OF CELLS BECAUSE ITS GENOME IS EVOLVING!

- Carl Woese - cellular organization (structure) - Bacteria (pro.), Archaea (pro.), and Eukarya (euk.) - protist (catch all), fungi, plants, and animals - mitochondria and chloroplasts have their own genome = used to be bacteria (divide by binary fission) and at some point became part of eukaryotic cells. * alpha-proteio bacteria * cyanobacteria

- ___________ established the system of scientific naming of organisms in 1735. * naming known as __________. * He is known as the ____________. - Each organism has _____ names: * the _______ (first) * the _______ or ________ (last) - This allows us to show that there is a relationship between all living things. - What is the order of naming (going from broad to more specific)

- Carolus Linnaeus * binomial nomenclature * father of modern taxonomy - two * genus * specific epithet or species - domain -> kingdom -> phylum -> class -> order -> family -> genus -> species

Vaccination: - 1796: _________ inoculated a person with cowpox virus, who was then immune from _________ (first person to vaccinate a person and exposed him to the actual disease). - Vaccination is derived from the Latin word vacca, meaning _____. - This protection is called _______. - 1717: _________: did work in vaccination by exposing people to active forms of the cowpox virus (variolation - similar to vaccination).

- Edward Jenner/ smallpox - cow - immunity - Lady Mary Montagu

- Who developed a vaccine against smallpox? - Who discovered how DNA controls protein synthesis in a cell? - Who discovered penicillin? - Who discovered that DNA can be transferred from one bacterium to another? - Who disproved spontaneous generation? - Who was the first to characterize a virus? - Who was the first to use disinfectants in surgical procedures? - Who was the first to observe bacteria? - Who was the first to observe cells in plant material and name them? - Who observed that viruses are filterable material? - Who proved that DNA is the hereditary material? - Who proved that microorganisms can cause disease? - Who said living cells arise from preexisting living cells? - Who showed that genes code for enzymes? - Who spliced animal DNA to bacterial DNA? - Who used bacteria to produce acetone? - Who used the first synthetic chemotherapeutic agent? - Who proposed a classification system for streptococci based on antigens in their cells walls?

- Jenner - Jacob and Monad - Fleming - Lederberg and Tatum - Pasteur - Stanley - Lister - Van Leeuwenhoek - Hooke - Iwanowski - Avery, MacLeod, and McCarty - Koch - Virchow - Beadle and Tatum - Berg - Weizmann - Elrlich -Lancefield

- _________ discovered a cholera outbreak at a water pump in London.

- John Snow

The Germ Theory of Disease: - 1860s: Applying Pasteur's work showing that microbes are in the air, can spoil food, and cause animal disease, _________ used a chemical antiseptic (_______) to prevent surgical wound infections. * ______ is a good primary germ barrier. - 1876: _________ discovered that a bacterium causes anthrax and provided the experimental steps, ____________, to demonstrate that a specific microbe causes a specific disease. * anthrax is caused from a _______ bacteria known as __________. * use this process to determine the _______ of a disease even to this day (where it comes from).

- Joseph Lister/ (phenol) * skin - Robert Koch/ Koch's postulates * soil/ Bacillus anthrasis * etiology

Which of the following conclusions can be drawn from Pasteur's experiment involving the flasks with S-shaped necks? Select all that apply! - Microbial life can arise from nonliving matter. - Microbial life can be destroyed by heat. - Pasteur's observations support the theory of spontaneous generation. - Microbes can be blocked from accessing favorable growth environments. - Microorganisms can be present in nonliving matter, such as air, liquids, and solids. - Beef broth is an unsuitable environment for growing microorganisms. - Pasteur's observations support the theory of biogenesis.

- Microbial life can be destroyed by heat. - Microbes can be blocked from accessing favorable growth environments. - Microorganisms can be present in nonliving matter, such as air, liquids, and solids. - Pasteur's observations support the theory of biogenesis.

Theory of Biogenesis: - What was the landmark for this theory? * provided paradime shift - Pasteur _______ flask keeps microbes out but allows air in. - Broth in the flask showed no __________. - Neck of the flask traps microbes. - Microorganisms originate in ______ or _______, not mystical forces.

- Pasteur and his S-shaped flask - swan neck - signs of life - air or fluids

The First Observations: - 1665: ________ reported that living things are composed of little boxes, or cells (coined the term cells). * Observed a piece of cork. * Marked the beginning of the ________: all living things are composed of cells. - The first microbes were observed from 1623-1676 by __________. * He called them ________ when viewing them through the magnifying lenses (saw the first microorganisms). * Created what type of microscope and what were its magnification powers (human blood cells and sperm through original microscope.

- Robert Hooke * cell theory - Anton van Leeuwenhoek * Animalcules * simple microscope/ 70x - 250x

- _________: is the use of microbes for practical applications, such as producing food and chemicals.

- biotechnology

- An adult human is composed of 10^13 __________. * harbors another 10^14 _________ (10x more) = these help provide _________ immunity. - The ________ is a community of microbes that live stably on/in the human body. * help to maintain ________. * can prevent growth of ________ microbes. * may help train the ________ to discriminate threats. - Microbes normally present in/on the human body are called _________.

- body cells * bacterial cells/ innate - microbiome * good health * pathogenic * immune system - normal microbiota

The Birth of Modern Chemotherapy: Dreams of a Magic Bullet: - Treatment of disease with chemicals is called ________. - Hurdle: scientists can develop a drug that kills a bacteria but also the host = _________; looking for the magic bullet! - Chemotherapeutic agents used to treat infectious disease can be __________ or _________. - _______ are chemicals produced by bacteria and fungi that inhibit or kill other microbes (do not work against _________). * Why would doctors give penicillin (for bacteria) if you have a virus (viral infections do not react to penicillin)?

- chemotherapy - cross reactivity - synthetic drugs or antibiotics - antibiotics (viruses) - to prevent a second infection (bacterial) because the immune system is already weak from the viral infection.

- What are the three major morphologies? - What does red around the type of microscope mean? - What is the reproductive part of fungi? - Amoeba use ______ for movement and engulfing food particles .... macrophages use these too.

- coccus (round)/ bacillus (rod)/ and spiral (corkscrew) - the specimen has been artificially colored - sporangia - pseudopods

- Pasteur showed that microbes are responsible for __________. - Fermentation is the microbial conversion of ______ to ________ in the absence of air. - Microbial growth is also responsible for _______ of food and beverages. - Bacteria that use air spoil wine by turning it to _______ (_________) - Fermentation for animals creates ________. - Lower temperatures in fridge slows down metabolism of bacteria = extends lifetime of food.

- fermentation - sugar/ alcohol - spoilage - vinegar (acetic acid) - lactic acid

- Pasteur demonstrated that spoilage bacteria could be killed by _______ that was not hot enough to evaporate the alcohol in wine. - __________ is the application of a high heat for a short time to kill harmful bacteria in beverages = reduce the number of bacteria and extend the lifetime of the product (these bacteria were ________). **** Pasteurization is not _________ *****

- heat - pasteurization/ (heat labile) - sterilization

- _______: is the study of immunity. * _______ and _______ are used to prevent and cure viral diseases. - A major advance in immunology occurred in 1933 when ________ classified streptococci based on their cell wall components. * The grouping of streptococci into distinct groups is based on its _________ and macrophages identify these for destruction.

- immunology * vaccines and interferons - Rebecca Lancefield * cell wall

Recycling Vital Elements: - _________: is the study of the relationship between microorganisms and their environment. - Bacteria convert ______, _____, ______, ______, and ______ into forms used by plants and animals (are crucial to recycle the energy from producers and consumers = _________)

- microbial ecology - carbon, oxygen, nitrogen, sulfur, and phosphorus/ (decomposers)

- __________: are organisms that are too small to be seen with the unaided eye. - __________: include bacteria, fungi, protozoa, microscopic algae, and viruses.

- microorganisms - microbes

- _________: is the collection of acquired microorganisms on/in a healthy human being. * being to be acquired as newborns (explain exposure of bacteria at birth!) * may colonize the body indefinitely (_________) * may colonize the body fleetingly (making them _________ microbiota) = most pathogens are this type. - __________ can only occur at body sites that provide nutrients and the right environment for the microbes to flourish.

- normal microbiota * amnionic sac is sterile, and at birth they are exposed to numerous amounts of bacteria * resident * transient - colonization

Microbes in Our Lives: - A few are _________ (disease-producing), while most are beneficial (have a ________ relationship). - Some ________ organic waste. - Some generate oxygen by _____________ (ex. __________). - Some produce ____________ such as ethanol, acetone, and vitamins. - Some produce _________ such as vinegar, cheese, and bread. - Some produce products used in _____________ (cellulase) and ______________ (insulin).

- pathogenic/ (synergist) - decompose - photosynthesis (cyanobacteria) - chemical products - fermented foods - manufacturing/ disease treatment

Normal Microbiota: - Normal microbiota prevent growth of _________. - Normal microbiota produce growth factors such as ______ and ______. - _________ is the ability of the body to ward off disease. * resistance factors include ______, ________, and _________. * provide an immunological benefit called __________. ***** First thing pathogens have to do for infection to occur is to _________ ******

- pathogens - vitamin B and K - resistance * skin, stomach acid (helicobacter neutralize stomach acid) , and antimicrobial chemicals * bacterial antagonism ***** attach *****

Bioremediation: - Using microbes to clean up ________. - Ex. Bacteria degrade organic matter in _______. - Ex. Bacteria degrade or detoxify pollutants such as _____ and ______. - Ex. landfill = bacteria convert waste into ___________ - Ex. bacteria who help in oil spills and in wells.

- pollutants - sewage - oil and mercury - methane

Archaea Characteristics: - ____________ - Lack _________ cell walls * May have ___________ (unusual for single celled organisms) - Often live in _________ environments = called __________. - Include: * _________ (CO2 -> CH4) * ___________ (salt) * ___________ (temperature) - Generally not known to cause ________ in humans. - What is the normal salt content for humans? * most pathogens have coevolved with us = why we get sick.

- prokaryotes - peptidoglycan * no cell wall - extreme/ extremophiles * methanogens * extreme halophiles * extreme thermophiles - disease - .85%

Bacteria Characteristics: - ____________ (pre-nucleus) = no ________. The DNA is ________. - ________ celled - ________ cell walls (carbohydrate polymer matrix = protects the bacteria against ____________) - Divide via ________ (genetically identical offspring) - Derive nutrition from ______ or _______ chemicals or __________. - May swim by using moving appendages called ________. - E. coli divide every _____ minutes, meaning they have __________ rates.

- prokaryotes/ no membrane bound organelles or nucleus/ floating around freely - single - peptidoglycan/ (osmotic pressure changes) - binary fission - organic or inorganic/ photosynthesis - flagella - 20/ exponential growth

The First Synthetic Drugs: - ________ from tree bark was long used to treat malaria (is a ________ that lives in salivary mucosa). - _________ speculated about a magic bullet that could destroy a pathogen without harming the host. * 1910: He developed a synthetic ______ drug, ________, to treat syphilis. - 1930s: _________ were synthesized.

- quinine (plasmodium) - Paul Ehrlich * arsenic/ salvarsan - sulfonamides

- ____________: enables bacteria and fungi to produce a variety of proteins, vaccines, and enzymes. * Missing or defective genes in human cells can be replaced in __________. * Genetically modified bacteria are used to protect crops from _______ and from ________. * Is an example of _________. * Two examples of this type of DNA are _______ and _______, which allow us to purify and produce large amounts.

- recombinant DNA technology * gene therapy * insects/ freezing * biotechnology * insulin and Bt endotoxin gene (plants)

- When a pathogen invades a host and overcomes the host's _________, disease results. - ____________: new diseases and diseases increasing in incidence. - What are the main EIDs we need to know?

- resistance - emerging infectious diseases (EIDs) - E coli. O157:H7, Middle East respiratory syndrome (MERS), avian influenza A (H5N1), methicillin-resistant Staphylococcus aureus (MRSA), west nile encephalitis (WNE), bovine spongiform encephalopathy (mad cow), ebola hemorrhagic fever (EHF), crytosporidiosis, acquired immunodeficiency syndrome (AIDS)

Identify the picture and determine the scenario(s) in which airborne microbial contamination of the sterile broth is most likely to occur: - scenario a: break near the mouth of the flask - scenario b: break before the first curve in the S-shaped neck - scenario c: break between the first and second curve in the S-shaped neck - scenario d: break after the second curve in the S-shaped neck

- scenario a: break near the mouth of the flask - scenario b: break before the first curve in the S-shaped neck

The Debate over Spontaneous Generation: - Occurred at the same time as Hooke and Leeuwenhoek discovers. - __________: the hypothesis that life arises from nonliving matter; a "vital force" is necessary for life. - _________: the hypothesis that living cells arise only from preexisting living cells.

- spontaneous generation - biogenesis

Given what you have learned from the design of Pasteur's experiment, which of the following scenarios would be considered an aseptic technique? Select all that apply! - using a Bunsen burner (flame) to heat a wire inoculating loop prior to use - allowing sterile solutions to be openly exposed to the air - wearing gloves when handling sterile laboratory equipment - blowing on heated, sterile surfaces with your mouth in order to facilitate cooling - taking steps to prevent contaminants from entering sterile solutions

- using a Bunsen burner (flame) to heat a wire inoculating loop prior to use - wearing gloves when handling sterile laboratory equipment - taking steps to prevent contaminants from entering sterile solutions

What was instrumental in DISPROVING spontaneous generation?

Pasteur's experiments using a swan-necked flask that prevented microbes from entering flasks open to the air.

Which of the following statements about E.coli is false? a. E.coli was the first disease causing bacterium identified by Koch b. E.coli is part of the normal microbiota of humans c. E.coli is beneficial in human intestines d. E. coli gets nutrients from intestinal contents. e. None of the above

a. E. coli was the first disease causing bacterium identified by Koch

Pasteur discovered that microbes can cause food spoilage. His solution to protect wine from spoiling was a process called pasteurization, which is based on what principle? a. Heating beer and wine just enough to kill those organisms causing the spoilage b. The destruction of all microbes in the wine c. Filtration of the wine to remove all microbes d. Addition of specific disinfectants and food preservatives that prevent the spoilage

a. Heating beer and wine just enough to kill those organisms causing the spoilage

Which of the following is an example of bioremediation? a. application of oil degrading bacteria to an oil spill b. application of bacteria to a crop to prevent frost damage c. fixation of gaseous nitrogen into usable nitrogen d. production by bacteria of a human protein such as interferon e. all of the above

a. application of oil degrading bacteria to an oil spill

Which of the following properties are true of both bacteria and viruses? a. both use a molecule of nucleic acid to determine heredity b. both have cell walls composed of peptidoglycan c. both can replicate using biogenesis d. both are surrounded by a protein coat e. both contain DNA and RNA

a. both use a molecule of nucleic acid to determine heredity

Which of the following types of microorganisms and its description are NOT correctly matched? a. protozoa - small animals b. bacteria - prokaryotic c. fungi - eukaryotic d. helminths - multicellular animals

a. protozoa - small animals

All of the following factors contribute to the threat of emerging infectious diseases (EIDs) EXCEPT __________. a. vaccinations against various types of childhood diseases b. spread of known diseases to new geographic regions or populations by modern transportation c. increased human exposure to new, unusual infectious agents in areas that are undergoing ecological changes such as deforestation and construction d. evolutionary changes to existing organisms through genetic changes

a. vaccinations against various types of childhood diseases

Which of the following names is incorrectly matched with their discovery? a. Lancefield—Classification of streptococci b. Beijerinck and Winogradsky—Gene therapy c. Watson and Crick—Structure and replication of DNA d. Avery, MacLeod, McCarthy—DNA was the hereditary material e. Lederberg and Tatum—Transfer of genetic material via conjugation

b. Beijerinck and Winogradsky—Gene therapy

Which of the following statements is the best definition of biogenesis? a. Nonliving matter gives rise to living organisms b. Living cells can only arise from preexisting cells c. A vital force is necessary for life d. Air is necessary for living organisms e. Microorganisms can be generated from nonliving matter

b. living cells can only arise from preexisting cells

Which of the following are correctly matched? a. Pasteur—antisepsis b. Erhlich—vaccinations c. Fleming—antimicrobials d. Koch—microscopy

c. Fleming—antimicrobials

Which of the following is characteristic of bacteria? a. bacteria live in extreme environments including pH, temperature, and pressure extremes b. bacteria have DNA and RNA, but not both c. bacteria lack a membrane-bound nucleus d. bacteria reproduce by a cell-division process known as mitosis e. bacteria have cellulose or chitin in their cell wall

c. bacteria lack a membrane-bound nucleus

Increased human exposure to new and unusual infectious agents in areas that are undergoing ecologic changes accounts for the __________. a. debate over the use of vaccines b. increased incidence of smallpox c. emergence of new infectious diseases d. lack of natural resistance to infectious disease e. development of antibiotic-resistant bacteria

c. emergence of new infectious diseases

Spallanzani's conclusion about spontaneous generation was challenged because Lavoisier had just shown that oxygen was the vital component of air. Which of the following statements is true? a. All life requires air b. Only disease-causing organisms require air c. Some microbes do not require air d. Pasteur kept air out of his biogenesis experiment e. Lavoisier was mistaken

c. some microbes do not require air

Recombinant DNA is? a. DNA in bacteria b. the study of how genes work c. the DNA resulting when genes of two different organisms are mixed d. the use of bacteria in the production of foods e. the production of proteins by genes

c. the DNA resulting when genes of two different organisms are mixed

You suggest to use a eukaryotic microorganism for a genetic experiment in lab but your classmate thinks is better to use bacteria. Why is your classmate correct? a. Bacteria are less complex than plants and animals. b. Bacterial life cycles can be very short. c. Some bacteria are very well studied and much is known about their genetic makeup. d. all of the above

d. all of the above

Which of the following processes are beneficial activities of microorganisms? a. recycling elements in the environment b. decomposing sewage c. producing foods and chemicals d. all of the listed responses are correct

d. all of the listed responses are correct

Robert Koch's studies on Bacillus anthracis established a sequence of experimental steps to prove that microbes __________. a. recycle elements such as nitrogen b. produce antiviral compounds c. control insect pests d. cause disease e. can be altered to produce products such as human insulin

d. cause disease

Aseptic technique is a standard lab practice that includes all of the following EXCEPT __________. a. using careful methods to sterilize equipment used to transfer and isolate colonies b. preventing airborne contaminants from accessing nutrient-rich environments in cultures c. preventing contamination of unwanted microbes present on solids, in liquids, and in the air d. elimination of all oxygen during any processes to prevent contamination

d. elimination of all oxygen during any processes to prevent contamination

Algae are ____________. a. classified as prokaryotes b. photosynthetic c. classified as eukaryotes d. photosynthetic and classified as eukaryotes e. photosynthetic and classified as prokaryotes

d. photosynthetic and classified as eukaryotes

Overuse and misuse of antibiotics has __________. a. caused viruses to become resistant to a variety of antibiotics b. caused bacteria to acquire virulence factors and become more deadly c. caused bacteria to become resistant to household disinfectants, such as chlorine bleach d. selected for antibiotic-resistant strains of bacteria and increased their frequency in both the hospital environment and the community e. directly caused mutations in humans that make them allergic to the antibiotic

d. selected for antibiotic-resistant strains of bacteria and increased their frequency in both the hospital environment and the community

In Pasteur's swan-neck flask experiment, the importance of the S-shaped curves in the flasks was twofold. The curves allowed entry of air but excluded ___________. a. oxygen b. viruses c. heat d. nutrients e. airborne microorganisms

e. airborne microorganisms

Biofilms are commonly found in which of the following environments? a. a rock in a lake b. teeth c. water pipes d. medical devices e. all of the above

e. all of the above

The usefulness of antibiotics is hampered by ___________. a. their inability to kill gram negative bacteria b. difficulties encountered in mass production c. the limited types of antibiotics available d. an antibiotic's lack of specificity for bacterial versus animal cells e. the emergence of antibiotic-resistant bacteria

e. the emergence of antibiotic-resistant bacteria


Related study sets

transcription, translation and replication

View Set

Logic: Chapter 1- What Logic Studies

View Set

Unit 4 (Chapter 20)- The 20th Century Early Years

View Set

Chapter 10: Substance related & impulse control

View Set

ATI Neuro/Musculoskeletal Focused

View Set

Kitchener's Five Moral Principles

View Set

Urbanization & Services AP Human Geography

View Set